You are on page 1of 284

Readers Reviews of the Big Fat Genius Guide to LSAT Logic Games

It is the best written, most concise, and most coherent book about the LSAT on the market.
It reinforces the concepts taught, and I found it very engaging (which I cannot say about
most LSAT books). I received a 173, but more importantly I scored a perfect games
section. I tell everyone I can to get this book.
M. P.
This book helped me a lot. I used to finish only 3 games, but Brian's method helped me
make it through all of them. After reading this book, I took the LSAT in October (for the
second time) and got a 175 (which was 9 points higher than my first score). I liked the way
this book really broke down the questions by category and guided me through the process
of studying. Games was by far my worst section before I began to use the Big Fat Genius
Guide, but I got all of the questions in the games section correct on the day of the actual
test. I would recommend the book to anyone; it was better than all of the other books I
used.
M. M.
The Big Fat Genius Guide to the logic games was the best $40 I have ever spent. When I
bought the book, I was running out of time on every practice logic games section I did, and
on average getting about half of the questions wrong. After reading it and practicing with
it, I was finishing on time and averaging one question wrong per game. I took the October
LSAT, I answered every single question right, AND I finished an insane 12 minutes ahead of
time!!! Thanks to this book, the answers in the logic games just pop out at me. Logic
games were the one section I had trouble with before studying from this book, and on the
actual test it was my best section! I am in love with this book.
C. J.
I cannot express how great a favor Brian Talbot has done for the luckless souls like myself
who've gone through Stanley Kaplan courses and come out the other side convinced they
were never going to get more than half of the Logic Games questions right. In my wildest
dreams I never thought that my strongest section would be the games, but it was, and I
scored a 168 on the LSAT. The efficacy and clarity of the method, and the depth to which it
is thought through, are nothing short of remarkable. The book teaches a method of solving
logic games based on simple principles applied efficiently and quickly. The prose style is
friendly and reassuring. This method is utterly different from anything Id learned before.
You can really get a high score on the Logic Games section. Not just a "getting-by score" or
a "better than I thought I'd do score," but a high score. For people like me, for whom the
Logic Games were the most daunting section, this book is incredibly useful.
M. J.
Turn the page for more reader reviews
More Readers Reviews of the Big Fat Genius Guide to LSAT Logic Games
This book is simply outstanding. It is clear in its explanations and concise in its methods.
With this combination, it offers you a powerful weapon to score well on the games section.
I improved 14 points, and I know that this book was key to that success.
J. Z.
I was absolutely "stuck" on the logic games section it was what handicapped me the first
time I took the LSAT, when I missed roughly half the logic games questions. I got every
other prep book out there, and they only made my problem worse infinite game types and
setups to memorize, etc. By a few weeks before my second LSAT, I was at a point of
extreme desperation. This book was akin to divine revelation it's concise and
conversational, and the answer to all logic games hang-ups. Brian Talbot tells you that all
logic games are basically THE SAME, and that they can all be solved using one
straightforward method. I went from missing half the questions on logic games to missing
only three I improved my LSAT score to a 166, and I honestly owe almost all of that
improvement to this book. If you can only buy one, get this one no BS, it's the best book
out there.
L. H.
I cannot say enough good things about this book. I studied for almost two months, and my
scores jumped more than 15 points! When I took the exam, I was so relaxed that I flew
through the questions. I started to worry that I was doing something wrong, but took
Brian's advice and relaxed. He said not to stress over the answers, they had to be right,
because I knew how to take the exam and beat it. I did. I only missed two questions in the
entire section, a personal best! I have recommended this book to everyone I know and will
continue to do so!
L. E.
The book is exceptional Brian's manner and conversation-like writing style are a great way
to learn a difficult subject (perhaps the most challenging part of the LSAT). He analyzes a
selection of games in sequence from easy to (very) difficult and, along the way, teaches you
principles and methods that will help you attack games that you have never seen. He
explains what you should have achieved at each stopping point and makes suggestions
about what to do and what to not do. I found it a comfortable way to learn. If you trust
the methods, they will work for you and you will come out of the test knowing you did
great. I made quantum leaps in my skill with this book it worked for me. If I had it to do
over again, I'd use this book again. It's worth every cent!
J. R.
THE
BIG FAT GENIUS
GUIDE TO LSAT LOGIC GAMES
BY BRIAN TALBOT
THE BIG FAT GENIUS GUIDE TO LSAT LOGIC GAMES
Copyright 2004 by Brian Talbot.
All rights reserved.
Cover design copyright 2004 by Mariya Ishkhanova.
Published by Big Fat Genius.
WWW.BIGFATGENIUS.COM
Big Fat Genius is a trademark of Brian Talbot.
This book and its author is not affiliated with the Law School Admissions Council, Inc., who
are the creators of the LSAT. LSAC does not review or endorse specific test preparation
materials or services, and the reference to LSAT questions within this work does not imply
the review or endorsement of LSAC. LSAT is a registered trademark of LSAC.
ISBN: 0-9763959-0-8
For their invaluable assistance in making this book what it is, I would like to thank
Jeremy Moreno-Gershman
and
Valeri Williams.
For his support and belief in my ability, I would like to thank my grandfather
Hersey Steinwinter
Table of Contents
Introduction 1
How to Use This Book 3
Studying Timeline 7
Studying Guide 9
How to Take the LSAT Logic Games Section 21
The Fundamentals 29
Game One 52
"If then" Rules and Negations 64
Game Two 76
Game Three 96
The Five Most Important Skills 112
Intermediate Techniques 115
Game Four 120
Game Five 134
Game Six 146
Game Seven 162
Game Eight 176
Advanced Techniques 192
Game Nine 204
Game Ten 220
Game Eleven 234
Game Twelve 246
Timing for the Section 261
Before the Test 266
During the Test 268
Glossary 273
Introduction 1
Introduction
My name is Brian Talbot, and I am going to teach you how to answer LSAT logic games
questions faster and more accurately than you do now.
Before we get started, I'd like to address a couple of basic questions: first, what are LSAT
logic games, and second, why should you listen to me?
Every LSAT consists of six sections. Two of these are Logical Reasoning these sections
consist of paragraphs followed by single questions, such as "What does the above argument
assume?" or "Which of the following best supports the author's conclusion?" Another
section is called Reading Comprehension. In this section you will have to read four longer
passages (generally four or five paragraphs) and answer questions about the contents of
each passage. There is also a section called Analytical Reasoning, where you will be
presented with four odd situations, governed by seemingly arbitrary rules, and asked
questions about how the situation may or may not be arranged, given the rules. This is the
section I, and most LSAT teachers, call logic games. The LSAT also has an Experimental
section, which will be the same format as one of the three sections I just described. In the
Experimental section the makers of the LSAT test out new questions; you will not be told
when you are working on the Experimental section. Finally, at the end of the test, you will
be required to write a short Writing Sample.
This book is only about the Analytical Reasoning section (which I will call the "logic games"
section from now on). If you've never studied for the LSAT before, you may wonder why I
devote an entire book to this one section. For the vast majority of people, logic games are
the single most difficult aspect of the LSAT. Yet this section can be made the most
straightforward section of the test if you follow a series of fairly simple rules. The goal of
this book is to teach you those rules.
Now to answer the second question: why should you listen to what I have to say about
logic games?
I took the LSAT in June of 1994. On the first sample LSAT I ever took before I started
studying, I got in the upper 150s, which I wasn't happy about. Logic games was the section
that held me back. I studied smart and got a 176 on the actual test. I got my law degree
from UC Berkeley in 1998, took and passed the California Bar in 1999, and then realized I
liked teaching better than the law. In other words, I've been where you are now, I
struggled and succeeded, and (except for the deciding not to practice part), I've been where
you are going.
I've been teaching the LSAT since 1996, originally for a large national test preparation
company. In January of 2001, I started Big Fat Genius, a company dedicated to teaching
students my system for approaching the LSAT (and GMAT). In the time I've been teaching,
I've seen more students than I can count, and I've worked individually with a large number
of them. With Big Fat Genius I work almost exclusively as a one-on-one tutor. This has
allowed me to study in a very focused way the types of problems students have on LSAT
logic games, to work out strategies to deal with the games, and to see how students are
able to use these strategies. I create all the lessons for my students, I develop all the
techniques, and I get to see first-hand those techniques applied by students. This has
allowed me to refine and hone my approach to the test.
2 Introduction
What you have in your hands, then, is the result of seven years of constant improvement in
my approach to logic games, and in my teaching. All of these techniques have been tested
by me and by students. These techniques are designed not only to be as efficient and
accurate as possible, but also to be learnable and apply-able by practically anyone it does
me no good to have a perfect system if no one but me can use it. This book is written with
my teaching experiences in mind. I have done my best to make it readable, somewhat
enjoyable, clear, and thorough.
My students often come to me after going through entire courses with other companies.
Thus, I have been able to compare my system to those taught by every major test
preparation company Kaplan, Princeton Review, Test Masters, Power Score. My system is
significantly different from each of theirs; it is simpler, easier to learn, easier to apply, and
more effective in more LSAT situations.
The final proof of the system is up to you. Read this book, apply what it says, and
experience the score improvement.
How to Use This Book 3
How to Use This Book
Before We Begin
There are two things you need to do before you start using this book. First, you need to
look at at least a couple of logic games. It will be much easier for you to understand what it
is you are supposed to be learning if you have a bit of context. I expect that most of you
have at least a little logic games experience you wouldn't be buying a book just on logic
games if you weren't a little bit worried about them, and you wouldn't be worried about
them if you hadn't done them before.
For those of you who haven't seen an LSAT logic game before, go to www.lsac.org (not
.com, which is a yachting club). You can download a sample LSAT there (at the time of this
writing, if you put your mouse over the "Downloadable Forms and Documents" button on
the left, a new menu will appear; click on "Sample LSAT." Of course, knowing the internet,
the page will change ten times this year). Download it (wait don't download it until you've
read the next paragraph). This test is the same test given in October of 1996 (minus the
Experimental section); section 3 of the test is logic games. Work through that section; time
yourself, but don't stop when your time is up. Timing yourself at this point is just to get a
feeling for how long these things take. Now we all know what LSAT logic games are.
The other thing you need to do before you start using this book is to make sure that you
have access to some LSATs to study from. This book contains twelve logic games, all of
which come from actual, previously administered LSATs. But you will need more games to
practice on, and you should only practice from real LSATs (if you have a book with logic
games in it, but you don't know if they are actual real LSATs, check the beginning of the
book where the copyright information is; if the book doesn't attribute its questions to LSAC,
it doesn't contain real LSATs). You can order LSATs at www.lsac.org. I recommend
ordering at least ten LSATs to study from. You should always try to get the newest LSATs
available (the higher the number of the LSAT, the newer it is December 2003 is 42, for
example), because the logic games have changed a bit over the years (that's why in this
book I only use games from 1999 and later). LSAC sells a book called The Next Ten Actual,
Official LSAT PrepTests which contains ten of the more recent tests (tests 29 through 38),
although not the most recent ones. Ordering this book and whatever more recent (higher
numbered) LSATs are currently available would give you a good number of tests to work
from without forcing you to spend too much money. If you want more exams than that I
also recommend 10 More Actual, Official LSAT PrepTests from LSAC. It contains LSATs 19
through 28 in one book, and is fairly economical (note the title is "More actual"). Don't
bother ordering 10 Actual, Official LSAT PrepTests (without the "more"), or any of the Triple
Prep books; the tests in these books are old and the games are somewhat outdated. If you
don't have access to LSATs to study from (or you don't have enough), order them now, so
that they will have arrived by the time you need them. You can still study from this book
while you wait for them to arrive, but by the time you finish the chapter called Game Three
you need to have some LSATs on hand.
Some of you may have access to LSATs that are not numbered; I know that some test prep
companies give their students old LSATs but put the test administration date on them rather
than the number that LSAC uses. To help you out, I've put a list of the recent LSATs (19
and on) and the date they were administered at the end of this chapter.
OK, those are the basics out of the way.
4 How to Use This Book
Using This Book
Everyone who uses this book should use it in the same way. It doesn't matter how much
LSAT experience you have. Begin at the beginning of the book and go through it word by
word, chapter by chapter. Chapters will tell you if you should reread previous chapters, or if
you should do work outside the book (and they will give you suggestions on what to do).
Some of you who have studied logic games extensively before this may want to skip around
in the book, or jump to the more "advanced" techniques given at the end. Don't do this.
As I said in the Introduction, the techniques in this book are on the cutting edge of LSAT
thought. They are different from anything you are likely to have seen before; even the
most basic things I teach, like diagramming games and writing down rules, are very
different from the way other companies approach games. You may find that, while you are
pretty good at the games now, the fundamental techniques I teach suddenly help you see
the games in a whole new way and that you make a large improvement without using any
of the "advanced" techniques. In fact, I will argue in the course of the book that the
fundamental, "basic" techniques are much more crucial to your score than the "advanced"
ones.
I give two possible schedules for how to use this book one for people who have only two
weeks to study logic games, one for people who have more than two weeks in the chapter
entitled Studying Timeline. The two-week plan is very rushed, so don't use it if you can
avoid it.
This book is written a little idiosyncratically. First, I write in a sort of "chatty" style. I like
to ask you questions, as if you were really sitting with me studying. I do this for a very
good reason. Human beings have been talking for tens of thousands of years. We have
only been reading and writing for a couple thousand, and it has only been in the last
century that anything like a majority of us have been literate. We have evolved as talkers,
not as readers. Thus, we understand things better by talking them out than by reading
them. I can't talk to each of you individually, but I can write a book that simulates the way
we learn through dialogue. So, when I ask you a question in the text, I expect you to stop
reading, think about it, and give your best answer. Then keep on reading and see how your
answer compares to mine; if it isn't close, you haven't been understanding what you have
been reading, so you might want to go back and reread it. Why do I do this? Because you
think better by talking things through; answering questions allows you to activate those
dialogue centers of your mind. Plus, it keeps you awake and engaged. Just to review, you
should have answered that question ("Why do I do this?") when I asked it. However, the
second answer I gave (about keeping you awake) wasn't in the text, so don't worry if you
didn't give that as an answer.
Second, I like to underline things. If I underline something, it is something I think you need
to remember. There are lots of things in this book you should remember, that you will want
to remember. I won't underline all of them the book would be 90% underlined. But I will
underline the most crucial concepts, the ones that are most essential to remember. You
should be taking notes as you read; write down anything that you consider important. But
you must write down and remember the stuff I underline, although you should write it in
your own words (that forces you to process it and understand it). If you start getting too
lazy or tired to do that, take a break from reading for a while.
How to Use This Book 5
Third, I will occasionally put things in a box.
Here is an example of something in a box.
Often there will be quite a bit of text in the box, unlike here.
This is generally stuff I consider important or interesting, but not necessary for you to read.
Sometimes it is an inspiring story, sometimes an interesting example, sometimes a little
joke. I think you should read it, but I put it in the box so as not to interrupt the flow of the
main text. In a few chapters I will use text in boxes to show alternate ways of answering
the questions; in these chapters I will tell you that this is what I am going to do.
Fourth, I use boldface in a certain way. I use it for chapter and section titles. I also use it
when I refer to chapter titles in the text; so if I mention Game One, or the Advanced
Techniques chapter, I'll put them in bold. Finally, when I use a word for the first time in a
chapter that is defined in the Glossary, I put that in bold (the first time only, otherwise it
just gets obnoxious). So if you see a word in boldface and you don't know what it means,
look it up in the Glossary.
Fifth, the numbering of the questions in each game might seem a little odd. For example,
the first question in Game Three is number 6, but the last question in Game Two is
number 12. There is a reason for this. Each of these games comes from a previously
administered LSAT, but the games don't all come from the same LSATs. I have preserved
the numbering that was used on the actual LSATs. So Game Two and Game Three, for
example, were both the second game on their respective LSATs, and so use the same
question numbers.
One last note. For each game I go through, I reproduce the game, as it was on the actual
LSAT, at the beginning of the chapter. Unfortunately, LSAC's policy on licensing LSAT
questions prevents me from printing the questions, or their answers, more than once in the
book. You may have to flip back to the beginning of the chapter periodically to see what a
question or answer says. I do my best to reduce this necessity, but there's no way I can
eliminate it (legally).
Let me recap: Answer when I ask questions. Take notes as you read, and especially note
anything underlined. Stuff in boxes is side commentary. Boldfaced words are chapter or
chapter section titles, or vocabulary from the Glossary. Got it? Good.
A Final Note Before You Begin
Students generally want to spend the most time studying what scares them the most about
a test. For the majority of LSAT students this is the logic games section. But the logic
games section is only one quarter of your LSAT score. Spending more time on it than on
Logical Reasoning or on Reading Comprehension may not be the best way to improve your
LSAT score. How do you know how much time to devote to one section versus another?
This involves balancing two considerations. First, how many questions are you missing in
each section? You have to double this for Logical Reasoning, because there are two of these
sections. Second, how easy will it be to improve your performance? For most people, logic
games is easier to improve. This will also change as time goes on; as you get better and
better at a section, you reach a point of diminishing returns, where you see less
improvement for more time put in.
6 How to Use This Book
For example, a student who misses ten questions total on both Logical Reasoning sections
(five per section), and ten questions on logic games, should study logic games more than
Logical Reasoning. They are pretty good on Logical Reasoning missing only five questions
per section so it will be harder (but not impossible) for them to improve that than to
improve logic games.
Evaluate where you stand in LSAT performance, and what score you want to achieve. Then
ask yourself how best to achieve that. Is spending more time on Logical Reasoning going to
give you better score gains than spending more time on logic games or Reading
Comprehension? Don't focus disproportionately on logic games just because they make you
nervous.
LSAT Number to Date Conversion Chart
This chart lists the LSATs from 19 to 48 (48 is the most recent released LSAT as of the
writing of this book) and gives the dates they were administered. I only list from 19 on
because the LSATs from earlier tests are slightly different and not as good to study from.
19 June 1996
20 October 1996
21 December 1996
22 June 1997
23 October 1997
24 December 1997
25 June 1998
26 September 1998
27 December 1998
28 June 1999
29 October 1999
30 December 1999
31 June 2000
32 October 2000
33 December 2000
34 June 2001
35 October 2001
36 December 2001
37 June 2002
38 October 2002
39 December 2002
40 June 2003
41 October 2003
42 December 2003
43 June 2004
44 October 2004
45 December 2004
46 June 2005
47 October 2005
48 December 2005
Studying Timeline 7
Studying Timeline
Here are two possible timelines for studying LSAT logic games. These are just for logic
games, not the whole test. Both are guidelines, intended to give you a general idea of how
you might spend your time; you should tailor your studying timeline to your needs and
situation.
Short Plan (Approximately Two Weeks)
This plan is for people who have only two weeks to study logic games. I highly recommend
devoting more than two weeks to studying logic games. If you really feel that you have to
take the LSAT now, and you have only two weeks, consider the difference going to a better
law school will make in your life. If you consider that difference minimal, or not worth
putting off applications for some period of time, go ahead with the test. If not, take a later
LSAT.
If you have exactly two weeks before the LSAT, this plan is not quite for you. You need to
spend at least the last three days before the test reviewing your strategies for the entire
test, not just logic games. Adjust this plan accordingly give yourself those three days at
the end to review the whole test. Since you have embarked on a two-week crash course, I
will assume you are willing to spend all 14 days of those two weeks studying.
Week One
The goal of this week is to learn the fundamental steps of logic games, and understand how
to apply these steps to any game. No work you do in this week should be timed.
Our first goal is to learn the steps. Read all the chapters up to (but not including)
Intermediate Techniques. Follow all instructions in the chapters, including those at the
end under the heading "What Do I Do Next?" Make sure you apply all the techniques
discussed in the Studying Guide when you study (not this chapter, but the one coming up
about how to study). Read the chapters in order, but, since the chapters involve a lot of
information and many of the chapters require you to work along with them, getting through
them may take you a couple of days. Don't redo the games after you've read the chapters,
as they tell you to, because you don't have the time. Make flash cards and memorize the
steps as you go through this week (see the end of The Fundamentals for a quick how-to
on flash cards). Spend the next two days practicing games from the "Recommended
Games" list at the end of Game Three.
Our next goal is to learn how to apply these steps to any game, including the more difficult
ones. Once you have memorized the steps (including how to approach every type of
question), read the chapters starting with Intermediate Techniques and up to (but not
including) Advanced Techniques. Follow all instructions in these chapters. This will take
you a day or two. Week one is almost up. Spend the rest of the week working on games
from the "Recommended Games" lists at the end of Game Three and Game Eight.
Week Two
The goal of this week is to get used to applying what you know about logic games under
timed conditions.
Skip the Advanced Techniques chapter. Read Game Nine through Game Twelve; read
the chapter on Timing. From this point forward you will time yourself on every logic game
8 Studying Timeline
you do. Follow the instructions in the "What Do I Do Next?" section at the end of the
Timing chapter.
On the second to last day of week two, read the chapters Before the Test and During the
Test and follow the instructions therein. This gives you one extra day to review your notes
and make sure you have internalized everything you need to.
Normal Plan (At Least Four Weeks)
This plan is for people who have more than two weeks to study. If you have less than four
weeks, spend less time doing what I describe in "Week Three." The instructions in the
"What Do I Do Next?" sections at the end of the chapters assume you are following this
plan. This plan assumes that you spend four to five days per week, 2 hours per day,
studying.
Week One
The goal of this week is to get comfortable with the fundamental skills. Read and work
through the chapters up to (but not including) Intermediate Techniques. Follow all
instructions in the chapters, including those at the end under the heading "What Do I Do
Next?" Make sure you apply all the techniques discussed in the Studying Guide when you
study (not this chapter, but the one coming up about how to study). Reading through the
chapters may take you several days, especially if you work through each of Game One
through Three on your own after reading their chapters. Make sure you give yourself a
couple of days at the end of the week to practice; spend these days working on games from
the "Recommended Games" list at the end of Game Three.
Week Two
The goal of this week is to cement your comfort with the fundamental skills and start
building mastery of them.
If you don't yet feel like you understand the fundamentals, or don't have them memorized,
feel free to continue working on games from the "Recommended Games" list at the end of
Game Three for the first day or two of this week. Once you have reached the point where
you do feel comfortable with them, read from Intermediate Techniques up to (but not
including) Advanced Technique (so stop after Game Eight). Spend the rest of the week
working on games from the "Recommended Games" lists at the end of Game Three and
Game Eight. Make sure you use the studying techniques I talk about in the Studying
Guide.
Week Three, Week Four
The goal of these weeks is to learn to apply what you know quickly and efficiently.
If you don't feel comfortable applying your skills to more complex games, continue to work
on games untimed for a few days. Don't do this beyond midweek of week three, though.
When you are ready, read Advanced Techniques through (and including) Timing. You
may or may not want to apply the advanced techniques, but you should be aware of them.
Follow the instructions in the "What Do I Do Next?" section of the Timing chapter.
Three or four days before the end of week four, read Before the Test and do what the
chapter tells you. At some point in these days you should also read During the Test. You
have now learned everything I have to teach you.
Studying Guide 9
Studying Guide
The Why, What, How, When, and Where of Studying
Most of us have only a limited amount of LSAT studying time (or energy). Nevertheless, we
want to get a good score. This means that we absolutely must maximize the little time that
we have. Simply plugging through questions will not get you a good LSAT score, because it
reinforces your bad habits as well as your good ones. This section provides you with a clear
and proven effective system for studying for the LSAT logic games (although it can easily be
generalized to the other sections of the test, as well). It won't be easy to use at first;
sometimes it may seem overly burdensome. But, used correctly and consistently, it is the
only way to guarantee improvements. Read this section, think about what it says, and then
read it again and take notes. If you really want to improve your LSAT score, begin
implementing these strategies today. You'll end up studying less than you might expect,
yet learning more every time you study.
Why Study?
The answer to this question is so obvious that you were probably tempted to skip this
section. But an understanding of why you study underpins the what, how, when, and
where.
You study because you want to get a good LSAT score. This is the only thing you should be
concerned with when you study. If you ever wonder whether or not you are doing the right
thing when you study, ask yourself, "Is this going to help me get a better LSAT score?" If it
is, right on. If not, stop.
This may seem obvious, but most people don't act like this is their goal when they study.
For example, one thing I commonly see people doing is studying when they are exhausted,
such as immediately after getting home from work. Now, is studying when you can't focus
on what you are doing helping you get better at the LSAT? Of course not, because you are
going to not do what you should, being too tired to practice new skills.
Most people act like their goal is to do a lot of practice games before they get to the LSAT.
That is, their entire studying plan is to do game after game, without really thinking about
what they can learn from each game. Doing a lot of games can be helpful, but it is only
helpful if it improves your skills. If you continue to make the same mistakes over and over,
doing those games is doing nothing but building bad habits. The point of doing practice
games is to build up the skills that you need for the LSAT. Every practice question is a
chance to practice at least one useful LSAT skill; if you don't use the right skills on the
question, you have wasted your opportunity. If you don't learn from the question, and
figure out how you should do similar questions in the future, you cannot improve. When we
practice, we want to make sure we are practicing skills that we need to learn; after we
practice, we want to review what we have done to determine what we now need to work on.
Every time you sit down to study, ask yourself, "What is my goal? What can I do now to
improve my LSAT skills?"
Here's a little story about how important it is to know your goals when you study.
Once upon a time, back when I briefly practiced law, I went to the UCLA law library to do
some research. I was sitting outside the library thinking about such and such and I
overheard a conversation between two 1
st
year law students (this was during the middle of
their second quarter of school).
10 Studying Guide
their second quarter of school).
One student was complaining to another about her contracts teacher. Every week, she said,
he assigned them tons of reading. She could barely get all of it done and briefed (which is a
way law students often take notes on the cases that they read) before class, and it
consumed more time than any of her other classes. Yet, every week, the teacher never
once talked about any of the reading he had assigned. This upset her quite a bit.
This, clearly, was a student who didn't know what her goals were. I could see three
possible goals she might have: 1) learn the material, 2) impress the teacher, or 3) do well
in the class. If she wanted to accomplish 1), then she should do exactly what she was
doing and stop complaining, because she was doing what she wanted to do and it didn't
matter what the teacher asked or didn't ask. If she wanted to do 2), she should stop
bothering to read the material, since the teacher clearly didn't care whether the students
read it or not. Likewise, if she wanted to do 3), she probably didn't have to read all the
material either, because in my experience most cases you are assigned to read in law
school are not relevant to the final exam. She should have been well aware of that, having
already been through one series of final exams. If the cases were necessary to do well in
the class, though, she should also stop complaining, because, again, she was working
towards her goal.
The reason she was so unhappy was either because she didn't realize that she was doing
exactly what she wanted to do, or because she was doing work which was irrelevant to her
goals. If you are aware of your goals, necessary work isn't always pleasant, but it is easier
to accept it and be satisfied that you are doing it; as far as unnecessary work goes, you just
shouldn't bother doing it.
What to Study?
Your goal in studying is to improve skills. Before you can do this, you need to determine
what the skills are you are trying to improve.
Every time you sit down to study the first thing you should do is decide what it is you are
going to work on. Never study without first identifying the specific skills or goals you will
work on in that session.
Making a "long list"
As you work through each chapter in this book, take notes on what you learn. Everything
underlined should definitely go in your notes, but don't limit yourself just to that. Your
notes should include enough information that you can understand them without having to
go back to the chapter itself; don't just jot down a phrase from the book, but rather explain
the concept to yourself. Each chapter will briefly summarize itself at the end; use this to
check your notes and make sure you covered everything. This summary won't include
every single concept, just the key ones, and it won't explain the concepts to you, which
your notes should (otherwise you can't use the concepts they contain). Don't be lazy and
wait to take notes until the end of the chapter; the act of taking notes forces you to think
about what you are reading while you are reading it, which will cause you to understand
things better.
After working through a chapter in this book, you should review the notes you took and the
questions you did during that chapter. You should do this no more than 24 hours after
finishing the chapter. However, don't start reviewing immediately after finishing; give
Studying Guide 11
yourself at least an hour of rest after the session before you start reviewing. Reviewing
means reading through your notes and making sure you understand them; add anything
you need to clarify what you wrote.
Again, write your notes with the intent that, when you review them later, you will
understand exactly what they mean and be able to implement what they say without a lot
of head-scratching.
After you write everything down, you may want to type it up and organize it in some way
that makes sense to you. Organizing this stuff is helpful in that it forces you to think further
about how everything fits together, rather than viewing every little tidbit as a completely
separate idea to be memorized on its own. Use an organizational scheme that makes sense
to you; you may want to put all the techniques about a certain type of question together, or
group techniques by aspect of the test (timing, eliminating answers, etc.).
These notes are your long list. The long list will contain everything that you want to know
for the LSAT.
Prioritize: make a "short list"
As you learn more and more, your list of things you want to learn is going to get rather
long. You won't be able to learn everything on it at once. You will also need a shorter list
of things you want to focus on right now. This is the short list.
Typically, your short list will contain issues of procedure and technique, such as a list of
steps to follow for certain question types. The short list should be easy to read and
understand at a glance. It should only have a few points on it, just the number of things
you can focus on learning in one sitting (generally 3-5). Put this short list right next to you
when you study, so you can refer to it easily.
The point of the short list is that you will refer to it constantly as you work through practice
questions. The point of doing practice questions is to practice new skills; if you don't
actually use your new skills as you practice, you won't learn them. The short list reminds
you what these skills are and how to use them. By referring to it constantly (at least before
every question) you will keep these skills at the forefront of your mind, which helps to
guarantee that each question brings you closer to mastering and habituating them.
Making a short list be specific
This is crucial. The notes you take are meant to help you. They must give you instructions
you can implement without too much thought. Your notes, both the long and short lists,
should contain ideas that you can look at in the middle of doing a problem, read,
understand, and implement immediately.
Bad notes example
! Be careful - don't make stupid mistakes
! Learn question types!
The above notes (either as a short list or a piece of a long list) give important goals.
However, there are no instructions on how to actually accomplish those goals. These notes
are the equivalent of writing yourself a note to "Get answers right!" They are useless.
12 Studying Guide
Good notes example
! Be careful:
! Read every word in a sentence
! Write down any information given
! If I need to test something out, make a new diagram
! Go through all the rules
! Learn question types:
! On "Can be true" questions, test each answer and see if it can be true
! On "Must be true" questions
! [and so forth]
Notice that the good list expands on the goals of the bad list in very concrete ways. Every
time the list gives a more complicated concept, it breaks it down into smaller steps. These
notes give thought processes that can be used on the test. These notes answer the question
"What should I do in a given situation?"
How to Study
Now that we've talked a bit about how to figure out what to study, lets talk about how to
study it. When you first start studying you will be learning new skills. You can't worry
about how fast you are at this stage. You must learn a skill before you can do it quickly,
and if you try to get fast too early you will become sloppy. In this section I will talk only
about studying without timing yourself.
Studying is actually harder than taking the test it requires more effort because you are
learning new skills, whereas taking the test is just following a path you've already set out
for yourself. Taking the LSAT is just a matter of showing up; all the real work is done
beforehand, when you study. The right approach to studying will make the real LSAT a
breeze (relatively speaking).
Studying consists of three stages. You must do each of these stages every single time you
sit down to study.
Stage 1: Preparing what will you practice?
As discussed above, before you can learn a skill, you need to know what the skill is you are
trying to learn. The first step in studying is reviewing your long list and putting together
your short list.
You will be creating your long list as you read through this book; your short list will either
be created from scratch when you finish a new chapter or will be based on material you've
been working on already.
Once you have a short list, you must spend some time reviewing it before you do any
problems. Make sure you know what each item on the short list means, and how to go
about implementing it.
Before you go on to stage 2, visualize doing a game or two. What this means is closing
your eyes and imagining working through a game. This may sound bizarre, but it is
extremely useful. Walk yourself, in your mind, through all the things you should be thinking
about when approaching a game, from the setup stage to answering questions. Think about
the skills to employ, the tricks to watch out for, the possible mistakes you might make.
Why is this useful? When you are actually doing a game, you typically focus on the game
Studying Guide 13
itself, which means you aren't thinking so much about the skills you are trying to practice.
This means that it will be hard to use these skills. By visualizing, you get a chance to
practice using the skills without distractions; this gives you an opportunity to practice
(rehearse) the skills themselves before you have to use them. If you do this correctly, when
you actually do problems you will simply be following a plan you have already laid out for
yourself. Visualization is a technique useful in any situation where you have to employ a
complex skill under pressure, from cooking to archery, and will really help your LSAT
practice. Always visualize using your new skills before you do any set of questions.
Stage 2: Practicing
Once you have identified what you want to work on, you must actually work on it. This
means you must do practice questions.
The point of doing practice questions is to practice your new skills so that they can become
habits. Habits are things you do without having to think about them much. If you develop
good habits, when you take the actual LSAT you'll be able to focus on doing the questions,
not on trying to remember how to do them.
As you do each question, make sure you do everything exactly the way you are supposed
to. Constantly refer to your short list to make sure you are working on the skills identified
there. This is absolutely crucial because it leads to forming proper habits, so let's repeat.
When you are doing practice questions, you must make sure that you are following the
correct procedure; that is, that you are doing the right steps in the right order, without
skipping steps or doing anything extra. This is the only way to form habits. Start this from
the very beginning. The more you do this in practicing, the better you will do on the actual
test.
If you come across something you have done before, but forget how to do it, stop and look
it up in your notes. It is better to take the time to do this and be slow than it is to do the
question the wrong way. Don't worry about memorizing things yet; you'll end up
memorizing most skills through use (I'll note in the chapters where you should take extra,
non-practice, time to memorize things).
Every time you skip a step, get sloppy, or allow yourself to lose concentration, you are
reinforcing bad habits which will cost you points on the test. Conversely, every time you do
the right thing on a question you are reinforcing good habits which will help you get
questions right on the test. People fall back on their habits when tired or under pressure. If
you have the right habits, you will do the right thing when things get tough on the test.
Follow all the steps you have learned, even when you feel that they are unnecessary, or
that a question is easy without them. Practicing good habits even on easy questions means
that you will have these habits on hard questions, which will make them easier.
Initially, do only one game at a time. At this point don't worry about how long each game
takes you. You only do one game at a time because there is a chance that you will do some
questions the wrong way. If you only do a few questions, you can catch your mistakes
before you repeat them and make them a habit. In addition, doing only a few questions
allows you to devote all your attention to the questions you do; do too many games in a
row and you will get tired and sloppy, creating or reinforcing bad habits. Once you finish a
game, stop and take a break. You may do more games that day, so don't worry that you'll
only be doing one game a day, but you have to do step 3 before you do any more questions
(and then step 1 again, as well).
14 Studying Guide
Remember, no one cares about how many games you did when you practiced for the LSAT.
No one will ever know, unless you tell them, and this fact will never make a difference in
getting into law school. What matters is your actual LSAT score. This studying method puts
a lot of emphasis on thinking about questions before and after you practice them, not on
cranking through as many questions as possible. This may be frustrating at first, but the
only way to improve what you do (and eventually improve your LSAT score) is to know what
you are supposed to do and make sure you do it.
Stage 3: Review
After you have finished your game you must check your work. This doesn't just mean
seeing if you got the answers right; this means checking your work to see if you used the
right approach to each question. No one cares how many questions you get right when you
practice; all that matters is how you do on the LSAT. If you get questions right, but use
skills that won't work on the real LSAT, then you aren't improving. When you review
questions, you are checking to see if you used the skills you will need for the LSAT.
In the beginning, review questions you got right as well as those you got wrong. See if you
did everything you were supposed to do, or if you did work that you shouldn't have. If you
did get questions wrong, try to reconstruct why, and ask yourself what you could have done
to avoid your mistakes. Try to see how the general principles in this book apply to the
games you do; don't try to construct overly specific ideas that will only apply to games
exactly like the one you just worked through.
If you find you are making mistakes, look back through this book to find where I talk about
similar issues. Here you will find concepts and skills you will want to use in the future to
avoid these mistakes. Add these to your notes. Notice patterns of mistakes you make.
Think about how you can avoid making these mistakes in the future. Don't just tell
yourself, "Don't do this," but think about what you should do to avoid that mistake. For
example, you might say to yourself, "Don't read too fast!" A better note would be, "Read
carefully by stopping after every sentence and writing all the information down." Notice
that the second note is something you can do to solve your problem, rather than simply an
instruction not to have a problem.
If an issue comes up often enough, put the solution on your short list, or (if it's already
there) highlight it to make sure that you work harder on it in the future. Anything you learn
about the test or yourself through this review should be added to your notes (at least to
your long list).
You should also notice what you did well. People are usually good at beating themselves
up, but don't notice what they are improving at. Spend at least a few minutes every time
you study thinking about what you are getting better at what you do better now than you
did before. There are two reasons for this. One, it will make you feel good, which will help
you continue to work hard. Two, you will eventually move some things off your short list as
you get good at them.
One quick note: don't laugh off stupid or sloppy mistakes. Making careless errors is the
result of improper technique. People who miss their score goals on tests often do so not
because they didn't understand the material, but because they deviate from what they have
learned that is, they get sloppy under the pressure of the test. The techniques in this
book are designed to make it very hard to make dumb mistakes; learn and use them. If
you keep making dumb mistakes, figure out what you are not doing. If you don't learn how
to avoid sloppy mistakes now, then you will be more likely to make them on the test when
Studying Guide 15
you are under more pressure. It is crucial that you build good habits now, when you are
studying, so that they will be present on the actual test. If you make sloppy mistakes you
must force yourself to stick to the techniques you have learned and slow down.
This is why you only do one game at a time, so you can constantly assess yourself and re-
prioritize. Doing 100 questions in a row does no good if they are all done wrong. In fact, it
will probably hurt you more than anything else, since you will have built up some really bad
habits. Stopping frequently and checking your work allows you to nip problems in the bud.
It also keeps you from dying of overwork.
Return to Stage 1: Set new goals and start over
After having done a game you will know more about what you should be working on. You
may continue on the same types of problems, or move on to something else. You may have
changed your short list of things to focus on. You may be feeling confident about a lot of
things which troubled you up to then.
Your self-assessment, performed during stage 3, should help you to change your short list
periodically. If you feel that you have reached a point where you automatically use one of
the skills on your short list, take it off the list. Add new high-priority skills to the short list
as you master old ones.
Take a break. Think about whether or not you have the energy and focus to do more work
this day. Maybe you need a significant, 20 minute to several hour break, maybe you can
start right up, or maybe you need to stop for the day. If you are going to study more this
session, take some time to think about what the questions you have done have shown you,
and what you should be working on. Then repeat the process: review or remake your short
list, visualize using the new skills you want to learn, do another game, and review it.
All studying should consist of these three steps preparing by making or reviewing your
short list, doing some questions to practice your skills, and assessing your progress. You
cannot eliminate any of these steps if you want to study efficiently.
Studying under timed conditions
Studying under timed conditions is a lot like normal studying, which I just described. I'll let
you know later in this book when I think you should start timing yourself. When you do,
you will still go through steps 1, 2, and 3 as I describe above. However, in step 2, you will
generally do whole test sections four games in 35 minutes, or sometimes do just one
game, allowing 8 minutes to do it. Your goal is to answer as many questions as possible in
the time given. Don't allow yourself to go over time; stop doing questions when your time
is up. You should be practicing everything you normally practice, but paying extra attention
to watching if you do extra stuff you shouldn't be. The key to speed is to eliminate excess
work (anything I don't tell you to do in this book), not to rush through necessary work.
When you review the game (step 3), you can do all the extra stuff you didn't do in your 35
or 8 minutes answer questions you didn't get to, try alternate diagrams for the game, try
to make deductions you missed. The goal during your timed practice is to simulate the
LSAT, which means not doing anything you haven't already mastered. When you review
you can think about what else you would like to learn and practice.
16 Studying Guide
Take frequent breaks
It is important to be able to concentrate when you study. Remember, you are trying to
build habits for the test. Being sloppy and unfocused is a bad habit. When you start to feel
fuzzy, take a break. The best kind of break involves some minor exercise a little walking,
for example. If you allow yourself to recharge often you will get more out of the time you
spend studying.
Take a significant break every half hour or 45 minutes, even if you feel fine. This break
should be at least a few minutes long. A few minutes of break time is very helpful. Of
course, you won't be taking breaks this long when you take the LSAT, but remember,
studying is actually harder than taking the test. Thus, you need more energy when you
study, which means you need to take more and longer breaks.
You should also take shorter breaks more frequently; take a 30 second to 2 minute break
every 8 minutes. This is the same frequency at which you should take breaks during the
actual test (you'll take a break after finishing each game); thus, taking these breaks is a
way of practicing for the LSAT. These breaks help maintain concentration at peak levels,
break up the tedium of questions, allow your blood to move more actively, and combat eye
strain. During this break you should breathe deeply while not moving, move your head and
shoulders to stimulate blood flow, focus away from your book or computer (preferably at
something far away), and adjust the way you sit to be more comfortable.
When to Study?
The goal of studying is to learn. You should only study when you are capable of learning.
Following the three-stage study process requires that you be alert, focused, and somewhat
energetic. Do not study when you don't feel alert, focused, and somewhat energetic.
Studying when tired or distracted leads to studying wrong. Studying wrong leads either to
wasted time or to the learning of bad habits, which leads to bad test scores.
What time of day is best for studying?
If you work, you should probably study before work on a regular basis. If you can't, then
you should not study immediately after work. Allow yourself an hour or two of rest and
have a light meal, and then study.
Don't watch TV or listen to music when studying.
In addition, the times you study should reflect your natural cycles of attention. Every
human body follows a 25 hour cycle called the circadian rhythm. Although this rhythm is
not exactly as long as our 24 hour day, it is reset every morning by exposure to bright light,
usually sunlight. The circadian rhythm causes humans to experience two major alert and
two major sleepy phases. The sleepy phases are (generally) from 2 to 7 am and 1 to 5 pm.
The alert phases are roughly 9 to 11 am and 7 to 9 pm. Thus, try to avoid studying
between 1 and 5 pm.
In addition to the circadian rhythms, humans also have natural ultradian rhythms. These
rhythms are a cycle that re-occurs through the day; we experience approximately 90 to 120
minutes of alertness, followed by 20 minutes of "recovery," during which thinking and
concentration are more difficult. Because of this, you should not expect to be able to study
for more than 90 minutes without a significant break (remember, you should be taking
Studying Guide 17
short breaks periodically every 8 minutes, or at the end of every game). This significant
break should be 20 minutes long; during these 20 minutes you should not study or think
about the test at all.
Because the exact length and effects of these two cycles can vary from person to person,
you should spend some time trying to figure out when your two main circadian sleepy times
are, as well as when your 20 minute ultradian down times occur. In addition, don't consider
these hard and fast rules; in general, try not to study during these down times, but
sometimes it may be unavoidable.
It is OK to break up your studying so that you do different stages of studying at separate
parts of the day. For example, you might do practice questions in the morning (stage 2),
and then review them and prepare for the next set (stages 1 and 3) in the evening. This is
fine, as long as you always precede practice questions with stage 1, and follow it with stage
3. If you do break up your study times, make sure you do some visualization right before
doing practice questions.
One note: the practice stage of studying (stage 2) requires more concentration than the
other stages. Try to time your studying so that you are doing this when most focused. If
you have to do one stage in the morning and the rest in the evening, do most of your
practice questions in the morning.
You won't be able to follow these guidelines all the time no one lives an ideal LSAT
studying life. But do your best to make your LSAT studying productive. If you never have
time to study when you are alert, you aren't going to improve much. If this is the case, you
need to reconsider your priorities.
How often should I study?
You should study at least three or four times a week, but generally not more than five. It is
more important to study often than it is to study for a long time each session. Preparing for
the LSAT is all about building good habits. Habits can only be built by consistent repetition,
over the course of days and weeks; it can't be built by cramming. This is for two reasons:
first, your ability to acquire the habit diminishes as you do something for consecutive hours
(you need to recharge); second, if you wait between studying sessions, you begin to lose
your habit.
Sports, or other difficult physical activities, are good analogues to preparing for the LSAT.
If you want to improve your free throws, your golf swing, or the amount you can bench
press, you have to practice consistently. If you play golf once a month, you will get better
only slowly, if at all. This is because your muscles build up in certain ways when you use
them, so that next time you use them in that way they function more efficiently; if too
much time passes between these types of use they break back down, and any gains you
made are lost again.
At the same time, practicing too often is bad for you as well. Weightlifting is a great
example. If you go into the gym every day and work out your biceps, let's say, you'll make
less progress than someone who went in half as much. This is because you aren't giving
your muscles time to recuperate and build up in between uses.
Your brain is like a lumpy gray muscle. If you use it in the same way consistently, certain
neural pathways get reinforced, which makes it easier to use these pathways in the future;
with time and disuse, these paths return to normal. At the same time, if you overwork your
brain, it won't respond as well until it gets a chance to rest.
18 Studying Guide
If you want to be good at the LSAT, you have to train properly practice consistently, don't
over-train, and always use the right form.
Each of these studying sessions should be 1 1/2 to 2 hours long (although you can break it
up over the course of your day). Studying for much less than an hour is a waste. You need
at least 20 minutes to prepare to do questions (reviewing notes and making a list of what to
practice), at least 20 minutes to do questions, and at least 20-30 minutes to review the
questions. If you study for less than an hour you will have to eliminate one of these steps.
This means you won't learn anything from that time.
Studying for more than a few hours in a row means you are going to be studying when you
are very tired. No one can focus properly at a high level for this long. If you need to study
for several hours in a day, take at least a 20 minute break every 1 to 1 1/2 hours.
Studying consistently is much more important and useful than studying a lot. If you study 1
1/2 hours five times a week (total = 7 1/2 hours per week) you will learn more than
studying 8 hours a day every weekend (total = 16 hours per week). Here, less than half
the studying will result in more learning.
1
Don't put off your studying until the weekend;
you'll learn less in a weekend than you would in a week (even studying the same total
amount of time) and you'll lose a lot of what you learned by the next weekend. Anyone can
fit studying into their weekly schedule. You may have to make some sacrifices, but a month
or two months of unhappiness is better than spending three years at an inadequate school,
followed by several years in a lower-paying job.
In summary, study consistently. Only study for as long as you can focus. Make sure that
you give yourself enough time to preview what you will practice, do questions, and review
the questions you did every time you study.
Where to Study?
Study someplace where you can focus.
Coffeehouses aren't very good places to study. No one ever gets much done in a
coffeehouse. There are too many other people, some or all of whom you will be constantly
aware of. There are too many noises. That said, they are great places to look and feel like
you are studying while really just amusing yourself. Of course, you may not be able to
avoid studying in coffeehouses, especially if your actual house is even less conducive. It's
not impossible to learn at a coffeehouse, so if you have no other choice, don't give up on
the LSAT.
If you have a roommate, spouse, or children, your living room is also a terrible place to
study. If you have no one at home, but own a TV, stereo, or computer, the living room is
still bad. Never, ever, ever study while watching TV.
The best place to study is someplace quiet outside your home (but not so far as to
discourage you from going there). Having to travel is useful psychologically, to focus you
and keep you from stopping too soon since you had to drive or walk to get there, it makes

1
Roughly speaking; it's very difficult to quantify learning, and people vary, etc. etc. The
point is, studying less, but consistently, is much more effective than studying more, but
with large gaps between study sessions.
Studying Guide 19
that travel time seem wasted if you go home early. I prefer studying in libraries (as long as
I stay away from the fiction section).
Summary
The goal of studying is to improve your LSAT score. If, when studying, you do anything
that doesn't, directly or indirectly, help you improve your score, you are wasting your time.
You should study at least three times per week, and at most five. Learning new skills takes
consistent practice. If you wait too long between study sessions you lose whatever skills
you built up. If you study too much, though, you might burn yourself out. Study
someplace you can concentrate, where distractions will be minimal.
Before you do practice questions, you should put together a long list and a short list. A long
list contains every skill and concept you'll ever want to learn for the LSAT. You create your
long list as you work through this book, and as you pick up new pieces of knowledge while
practicing. A short list contains the skills and information you want to practice right now.
You can't learn everything on your long list at one time, so you prioritize and put your high
priorities on your short list. This list will change as you learn the skills on it; you'll take off
the skills you have mastered, and put new ones on.
Immediately before doing practice questions, visualize the processes you want to practice.
When visualizing you should imagine that you are doing a game or games, and walk
yourself through the process you want to follow. You should visualize before doing any set
of practice games. Visualizing helps focus you on what you are trying to practice, so that
when you actually do games you do them properly.
The point of practicing is to learn habits, and you can only learn habits by repeating a
process over and over. When you do the actual LSAT, you will tend to do questions in
whatever way is habitual for you. If your habits are good, you'll do well. Build habits by
always doing the questions the right way following the proper steps, leaving nothing out
no matter how easy or hard the question is. Look at your short list constantly as you do
practice questions, at least before every new question. Start out by doing practice questions
in small sets one game at a time.
After you finish a set of practice questions, review what you did and think about what you
did well and what you should do differently in the future. Don't worry about getting the
right answer or not; worry about following the proper procedure. Remember to be aware of
what you are doing well, and give yourself positive reinforcement. Use this review to
change your short list and guide your future practice.
You should take breaks whenever you need to, and at least every 10 minutes. This keeps
you alert and focused when studying, and also simulates what you'll do during the real
LSAT, thus building good test-taking habits.
To summarize the summary before you do questions, figure out what you want to learn
and make sure you understand it. As you practice, constantly focus on practicing what you
are trying to learn. After you practice, figure out what you have learned and what you need
to learn in the future. Use this information and start the process over again. This will help
to ensure that your studying time is always concentrated on changing your logic games
habits for the better.
20 Studying Guide
What Do I Do Next?
Do you have the energy to keep learning? If not, take a break or stop for the day.
Otherwise, continue.
Now that you've learned how to study, read the next chapter How to Take the LSAT
Logic Games Section taking notes for your long list as you go. Don't forget to take
breaks as needed. Pretty soon you'll be doing practice games, putting everything from this
chapter into effect. I hope you're as excited as I am.
How to Take the LSAT Logic Games Section 21
How to Take the LSAT Logic Games Section
Lets be honest: the LSAT logic games are hard. Lets take a second to be upset about
this Darn it! OK, feel better? That done, lets see what we have to do to get a good
score.
The LSAT logic games are hard for two reasons. First, the test just is hard there is a
certain amount of hardness built right into the test that is basically unavoidable. Second,
the way most people take the test makes it even harder. We cant avoid the built-in
hardness unless we skip the test, so we have to prepare for it and know how to deal with it.
At the same time, we clearly dont want to make the test any more difficult than it has to
be, so we need to know how not to do the stuff that makes it harder.
Built-in Difficulty
There are several aspects to logic games that make them inherently difficult:
1. You have to know logic
2. The games, questions, and answers are worded confusingly
3. There is a lot to think about at once
4. There are a lot of questions
5. There isn't a lot of time
Lets go through these issues one by one and discuss why they make the test tough and
what we are going to do to overcome them. I'll develop all of these ideas in more detail
later in this chapter, but for now I want to get us thinking about how we are going to make
the test easier.
1. You have to know logic
What are we going to do about this? Well, we can't avoid it we'll have to learn the logic
we need to know. That's part of what this book is for, to teach you all the logical concepts
and skills you'll need to succeed on the LSAT logic games. It's not so hard what makes
the test difficult is not the logic, but the way in which it is presented to you.
2. The games, questions, and answers are worded confusingly
You may actually know the logic involved in a question, but if you can't figure out what the
question is asking you to do there is no way you'll do it. The easiest way to understand
questions is to first know what types of questions will be on the test. There is a limited
number of types of questions they can ask, and every question of a given type is answered
in the same way. It's easier to identify an oddly worded question as being a certain type
than it is to figure out what it means from scratch. On the test, once we figure out which of
the question types we are presented with, we know exactly how to go about answering it.
This eliminates a certain amount of difficulty; we no longer have to think about how to
answer a question, we just have to answer it.
In addition, we are going to need to read slowly and carefully. It's easy to miss a "not" or
"exactly" or "at least," and thereby to miss the question. We need to take in and
understand what we read, and this requires reading carefully.
3. There is a lot to think about at once
Games involve a lot of information not just the information the games give you in the
form of rules, but also the steps you need to remember to go through for the questions. If
22 How to Take the LSAT Logic Games Section
you try to manage all this information in your head at one time, you'll get confused and
make mistakes. You'll also get tired, which will slow you down and make you more prone to
make mistakes. We deal with the information the game gives us by writing it down so that
we don't have to remember it. The more we rely on writing things down, the less we need
to think. The less we think, the less open we leave ourselves to making mistakes and the
less tired we get. In addition, we need not just to memorize all the steps for doing
questions and so forth, we need to make these steps into habits. Habits are things we do
without thinking. We can form habits only by studying properly.
4. There are a lot of questions
What is so difficult about this? Well, we will eventually get tired. When we're tired the
questions will be a lot more difficult. Also, even if we don't get tired, we may not have
enough time to answer all the questions.
We don't want to use any more mental energy on a question that we have to. One way to
avoid using too much energy is to write everything down, as I said above. We also need an
approach to the test that is simple and doesn't require too much thought. That way, when
we get tired (and we will), we can still stick to our system and do well. As we go through
my approach to games you will see that I have tried to make it as methodical and
memorizable as possible; I've done my best to eliminate unnecessary details and useless
information from the approach. A simple, memorizable system is one that can be applied
even when you are tired and not thinking as clearly as you normally do. It is also crucial to
have techniques to help you maintain focus and regain it when it has been lost. The best
technique for this is taking breaks during the test. We'll talk more about that later.
Because we may not have time to do all the questions, we need to have a system that
maximizes the number of questions we do do. In part we do this by speeding up on each
question having a simple and efficient approach. But we also need to do the questions in
the right order. We want to do the easiest and fastest questions first, because we can do
more of these in less time. I'll teach you a quick and easily memorized way to figure out
what order to do the questions in on every game.
5. There isn't a lot of time
Given the time limits of the test, it is crucial that you maintain the proper pace and don't
waste any time. On the logic games section you have 35 minutes to do four games. You
are capable of doing every game within these limits. You may not answer every single
question on every game, but we are going to get pretty close. However, you don't have a
lot of extra time if you spend time doing stuff that doesn't help you answer questions, that
is going to cost you points.
It is amazing how much time students waste on the test. Yet students generally don't think
they are wasting time they think that the steps they spend the most time on during the
test are helping them, when an objective observer would realize that these steps don't get
them points in proportion to the time they take. For example, students often want to figure
out as much as possible about a game before they look at the questions. This can be
helpful; it can save you a little bit of time on the questions. But I often see students
spending several minutes doing this. There is no way that this is worth so much time. You
only have about 8 minutes per game (35 divided by four). You probably spend at least a
minute, and more like two, understanding the structure of the game and the rules that are
given to you. Spending an additional 3 minutes figuring out how the rules can be
combined, and what may or may not happen in the game, gives you only 3 or 4 minutes to
answer the questions. That's not enough.
How to Take the LSAT Logic Games Section 23
Given that you don't have enough time, you can't afford to waste it. The key to speed on
the LSAT is not rushing through the questions. There is certain work that has to be done
slowly and carefully. The key to speed is avoiding doing anything that doesn't have to be
done. We're going to figure out just what you have to do to get a great score, and we won't
do anything else.
What Makes the Test Harder Than it Has to Be?
Here is a list of the most common ways test-takers hurt themselves on the LSAT:
1. Multitasking
2. Not knowing what to do
3. Not doing what they need to do
4. Working through fatigue
5. Worrying and/or changing your strategy
1. Multitasking
Multitasking is doing more than one thing at the same time. It is inevitable and not always
bad in the real world. For example, I am pretty adept at walking and chewing gum, or even
walking and talking at the same time. Neither of the tasks in these pairs uses the same
skills or the same part of the brain; only one of the two requires much (or any)
concentration. Now imagine you had to do two tasks simultaneously, both of which
required the same mental resources. Could you recite poetry while reading the newspaper?
Maybe, but only if the poetry were gobbledygook, or you didn't really pay attention to what
you were reading. The same problem arises when you do two tasks that both require
concentration. You can walk and talk, but could you walk on slippery ice while closing an
important business deal?
On the LSAT logic games section, every task uses the same part of the brain they are all
logic games and every task requires a decent amount of concentration. Doing more than
one task at a time on logic games means that you'll do each task slower and less accurately
than you would normally. Multitasking doesn't save you time, it slows you down.
Multitasking is a great LSAT sin, and it comes up in a variety of ways, such as thinking
about what you are going to do for the next step in a question before you've finished the
step you are on, or in trying to combine a rule with other rules while you are still writing it
down.
2. Not knowing what to do
It's perfectly OK to look at a question and not know what the answer is. Finding the answer
takes time and working through a process you don't get answers just by looking.
However, given that there are a relatively small number of question types for the logic
games section, and each type is always answered in the same way, there is no excuse for
not knowing the next step to take at any given stage of the section.
This doesn't mean that you have to be able to look at a game and know exactly how
everything fits together. It doesn't mean that you have to look at a question and know
what the answer is. You are going to have to work through a system; you are going to
have to figure things out slowly and carefully. But, you should know exactly how to do that,
and you will, because I'm going to teach you.
As an LSAT tutor, students often ask me for help with games that they can't figure out.
Typically, these are really challenging games (otherwise they wouldn't need help with
them). I can't count the times a student said, "What's the answer to this question?" or
"What's going on in this game?" and I said (after looking at it for a second or two), "I have
no idea, it looks really tough."
24 How to Take the LSAT Logic Games Section
"What's going on in this game?" and I said (after looking at it for a second or two), "I have
no idea, it looks really tough."
So what do I do in those circumstances? Tell the student sorry and refund their money? Of
course not. Instead, we go through the game or question step by step; we read the first
rule or the first answer and figure out what we have to do with that. Then we go to the
next one, and figure out what we have to do with it. By the time we get to last step we
have answered the question or we understand the game well enough to start working on
the questions, and it hasn't taken us that much time, either. This happens all the time;
when I first started tutoring, it scared me silly, because I didn't have enough experience to
trust my methodology. After a while, though, I started to take these almost miraculous
solutions to complex questions, which just emerge from doing the work, as normal.
What's the lesson here? Even the best test-taker (me, by the way) can't look at a hard
question or game and instantly figure it out. But, any good LSAT-taker knows all the steps
to go through, and they can tell them to you at a moment's notice.
3. Not doing what you need to do
There are certain steps that have to be taken in order to answer a question. For example,
you have to read the question. Most of the time you have to write out a diagram or two.
This involves going through the rules one by one. These steps are vital without them you
cannot get the right answer. Each of these steps takes a certain minimum amount of time;
you cannot do them properly in less time than this. Yet most people feel like the only way
to be good at the test is to rush through these most important steps.
What happens when you rush through the most important stuff? You end up making
mistakes, getting confused, and/or getting tired. You don't want that. What happens if you
slow down? You get the questions right, and everything feels much easier. In the long run,
you'll be faster overall because you'll be less tired and wont have to double-check your
answers. Why don't people slow down? Because they feel like they can't afford it if they
don't go faster, they won't finish the test.
This is a very common and very dangerous mistake to make. Let's analyze it: the goal is
to do well. If you don't rush (you assume), you won't do well because you won't finish in
time. But, if you do rush, you won't do well, because you can't be accurate if you are going
too fast. Damned if you do, damned if you don't.
Except you don't have to rush. The people who wrote the LSAT wrote the test to be
possible; if a high score were un-gettable (by regular people, not just by professional test
prep teachers), the test would be a waste. The people who wrote the LSAT know their
business; they know how long it takes to do the steps necessary for a question, and they
know what these steps are. Each game is written to be doable in the given time, doing all
the required steps. If you do what you need to do on a game and you do it in the amount
of time it takes (not rushing through), you will have enough time to get most of the
questions right. I promise.
Of course, this assumes that you aren't wasting a lot of your time wondering what to do, or
worrying about the test Wouldn't you rather give those things up, and be able to do the
logic games more comfortably?
How to Take the LSAT Logic Games Section 25
4. Working through fatigue
It's midnight. You're exhausted from a hard day at work or school. How do you become
un-exhausted? Go to the gym? Read a bunch of philosophy? Plan a new business venture?
No, of course not you go to sleep. The only way to relieve fatigue is through rest.
If you are fatigued on the LSAT, what should you do? Another question? Same idea that
question is not going to make you less tired; in fact, you are more likely to miss it because
you are tired. What should you do? REST.
While you are resting, you won't be doing any questions, and that's scary, isn't it? But, if
you don't rest, you'll keep slowing down (and missing questions) as you get more and more
tired. If you do take a short rest (20 minute naps aren't necessary) you'll come back faster
and more accurate, more than compensating for the time "lost." Resting time is never
wasted time.
5. Worrying and/or changing your strategy
The LSAT is going to be psychologically difficult; it's a tough test and at times you will feel
like you are getting questions wrong. Worrying about it doesn't do you any good (exactly
what my mom used to tell me in fifth grade) and it actually hurts you it wastes time and
distracts you (its a form of multitasking). It's tough, but work on putting how you are
doing out of your mind when you are practicing; eventually you will get used to focusing on
what you are doing, rather than on how well you did earlier.
During the LSAT, people often feel like they're doing badly when they are in fact doing quite
well. This causes them to change their approach to the test they deviate from the
strategy they've been using (the same strategy that has caused them to do well thus far).
This, in turn, causes them to do worse from that point forward. Don't do this, either when
practicing or during the test. Trust yourself and stick to what you know works; you aren't
going to be able to figure out a better strategy in the middle of the LSAT.
Putting It All Together
There are several key skills that we have discussed above. These are the things that you are
going to do to make the test easier. They are so important that I'm going to say them
again.
1. Learn the logic
2. Have a system for everything that might be on the test
3. Have an energy-efficient approach to work
4. Read carefully
5. Do one thing at a time
6. Take breaks when needed
I want you to notice something each of these points is phrased as a positive, something
for you to do, not something to not do. People tend to focus on negatives, but it is more
important to know the right thing to do. The only way to avoid mistakes and problems is
to do the right thing. It's important to avoid bad things, but you only get points by doing
the right thing. When you notice yourself making mistakes, don't just ask "What did I do
wrong?" Also ask, "What should I have done? What will I do next time in this situation?"
26 How to Take the LSAT Logic Games Section
1. Learn the logic
I'm going to devote sections of this book to teaching you the logic you need to know for this
test. It's not as complex as you may think, and it is all based on common-sense ideas you
probably already understand.
2. Have a system for everything that might be on the test
This is most of why I wrote this book. The logic you are tested on in the LSAT logic games
is, like I said, fairly straightforward. What makes the test so difficult is the way in which
you are tested the number of questions, the phrasing of the sentences, and the time
pressure. Knowing exactly what you are going to do, and in what order, at any given
moment of the test will relieve pressure, and it will eliminate time wasted wondering what
to do. And, if it is a good system (and my system is the best there is) it'll help you answer
the questions as accurately and efficiently as possible.
That said, I want to give a little warning. Once you learn a way of doing things from this
book, stick to it. The more you do it, the better you'll be at it faster and more accurate
and the less you'll have to think about it, which will make the questions easier. One of the
big time-wasters I see is that students look for a faster or better way of doing a question or
diagramming a game, even though they already know a viable method for it. Be consistent
in your approach to games. The best way to be good at the test is to have good habits, and
habits are formed by repetition.
3. Have an energy-efficient approach to work
Being efficient means minimizing waste. People waste energy on logic games by thinking
too much. Typically this involves trying to figure things out in their head, as opposed to
writing them down. Writing everything down and doing all your work on the page, rather
than in your head, seems slower because it involves the extra step of writing. But, in fact,
it is more efficient than just thinking questions through. There are several reasons for this.
First, if you write down everything you do, you can "forget" each step after you do it, since
your notes will remember all the relevant information for you. You can focus all your
attention on each step you do, rather than splitting your attention between what you are
doing now and what you just did. In addition, writing things down allows you to involve
more parts of your brain the visual and tactile in the test process. You are more likely
to catch mistakes you make when they are written down, and you are less likely to make
those mistakes in the first place. Finally, you can use your written work to help you out on
later questions. We'll learn an efficient method as we go through the book.
Often, as students get better, they feel that they can be a little sloppier on questions
doing things in their head and/or doing several steps of a question at once. This inevitably
leads to them missing more questions. The reason you can get answers quickly and
accurately is that you are good at doing what needs to be done on a question. Trying not to
do this will make you miss a question. Trying to do it in your head will, in the long run, tire
you out. Don't try to change the way you approach a question once you get good at it
this totally negates the advantage of having learned to do it.
4. Read carefully
Every single word in the Logic Games section is crucial. You don't want to miss a single one
of them. Read everything the rules, the questions, the answers carefully, making sure
you take everything in. You aren't reading slowly for the sake of reading slowly, you are
reading carefully understanding every single thing said.
How to Take the LSAT Logic Games Section 27
5. Do one thing at a time
Focus on the step you are doing while you are doing it, and ignore everything else. Don't
worry about what equation you'll solve next, or what you are going to do with the
information you get out of this diagram; once you've finished doing what you are doing,
then you can think about the next step.
6. Take breaks when needed
There is a science to taking breaks. This takes a bit of explaining, so get ready and pay
attention to this.
The first rule is: when you feel the onset of stress or fatigue, take a break. If you feel your
concentration slipping, stop working for a bit. Stare at the ceiling. Rotate your shoulders.
Breathe deeply. In other words, relax. Do this for as long as a minute or as little as a few
seconds, whatever you need. It is perfectly OK to spend 60 seconds doing nothing at all
during a test, as long as you are doing it on purpose.
Take these breaks every time you start to lose concentration. This may mean taking a
short break every four or five questions towards the end of the test. It is better to do this
than to lose focus.
When I took the Bar exam I went to the bathroom three or four times per section (a section
is three hours long). Was I nervous? Had I drunk too much coffee? No; I didn't even use
the facilities most of those times, just looked myself in the mirror, smiled, maybe splashed
some water on my face, and walked back to my seat.
Just walking to the bathroom took at least a minute, so these breaks probably lasted at
least 3 minutes each. This time was totally refreshing, and really useful in keeping me on
track. I ended up finishing each section before the majority of people taking the test,
despite having "wasted" 10 minutes a section going to the bathroom, in good part because I
was always focused.
Plus, since I was wearing boots, I made a lot of noise, and everyone had to look at me I
was wearing a three-piece suit, because I believe in dressing for success, so that made me
think of how good I looked, and how everyone was looking at handsome me, which boosted
my confidence and got my mind off the test. Ridiculous, I know, but these little confidence
builders can be very useful; I'll talk more about them in the chapter called During the
Test.
This will require that you learn to tell when you need a break. There are several ways to do
this. One is to be aware that you will inevitably lose focus when you are studying; if you
are ready for it you'll notice it when it happens, and take a break. Let me repeat that:
when you study, take a break every time your concentration starts to slip. This is practicing
what you should do on the actual LSAT; by the time you get to the real test, you'll be good
at this. You'll also get better at noticing when you need a break.
Another method of learning when to take breaks is to look at your performance on a series
of questions after the fact. If you tend to miss questions in blocks missing several in a
row, rather than spread out that is a sign of fatigue. Questions aren't grouped by
difficulty or skill; if you are missing a group of questions, that's a sign that the problem is
you and not the questions. Notice how often these blocks of missed questions occur. If
they occur with some regularity, that shows you when you are getting fatigued. For
example, I had a student who, like clockwork, would get five questions right, three wrong,
five right, right wrong. This told me that she got tired every five questions. Patterns don't
have to be so clear-cut; they are more likely to be a tendency, such as missing a few
questions in a row every ten or so questions. Once you notice a pattern of when you get
fatigued, start taking breaks at regular intervals right before when you would normally get
tired so as to break the pattern. The clockwork five-right three-wrong student needed to
stop after every fifth question and take a break.
28 How to Take the LSAT Logic Games Section
have to be so clear-cut; they are more likely to be a tendency, such as missing a few
questions in a row every ten or so questions. Once you notice a pattern of when you get
fatigued, start taking breaks at regular intervals right before when you would normally get
tired so as to break the pattern. The clockwork five-right three-wrong student needed to
stop after every fifth question and take a break.
This isn't so relevant to logic games, because people also often miss questions in groups
because they didn't understand the game, rather than because they got tired. But this is a
very useful technique for analyzing your performance on Logical Reasoning, so I thought I'd
throw it in for free.
You should also have a rule that you follow when studying, and when taking the test. My
rule is to take a break after every logic game, no matter what. This means that, even if I
feel great, I take a short break, generally just looking up from what I'm doing and moving
my shoulders and neck around. This gives me a chance to get that game out of my mind,
and to get sharp for the next game. Oftentimes you'll be too worried about the test, or too
keyed up, to notice if you need a break. This rule guarantees that you'll maintain your
focus and stay fresh. Follow this rule every time you study; by the time you take the LSAT,
it'll be a habit, and require no thought.
What Do I Do Next?
Feel tired? Take a break. When you feel mentally ready, begin reading the next chapter
The Fundamentals. Make sure you took notes on this chapter and that you use what you
have learned here. I'll expand on much of it in later chapters, especially in Game One
through Game Three, but I'll expect you to remember to take breaks, read carefully, and
not multitask.
The Fundamentals 29
The Fundamentals
Let's get our terminology straight before we start discussing things in depth. Every logic
game will start with a short paragraph explaining the situation you are dealing with. I call
this paragraph the setup. After the setup, there will be a series of sentences describing the
restrictions on the game. I call these the rules. After the rules, you will be asked
questions.
The setup and the rules give you a lot of information; the questions will ask you to put this
information together and make determinations about what can and cannot happen in the
game. There is going to be too much information for you to simply remember; you are
going to have to write most of it down. Typically, it will be easiest for you to organize your
information around some kind of chart or list. We'll call this the diagram: the diagram is
the way in which you visually organize the data you are given in the setup and rules.
There are three steps that you have to go through in every logic game. They are done in a
certain order: it would generally be odd to read the rules before the setup, since the setup
will give you the context for understanding the rules; likewise, it doesn't make any sense to
answer questions first and then read the setup or rules. Here is the order in which the
steps would normally be done:
Steps in Chronological Order
1. Read the setup and make a diagram
2. Read the rules and write them down
3. Answer questions
Before we discuss these steps in detail (and we are going to do so in great, great detail),
tell me which of these steps is the most important? Take a second to think about that
before looking at the next paragraph.
Answering the questions is the most important step. Your goal on the LSAT is to get a high
score. It is impossible to get a high score without answering questions. You can answer
questions correctly without doing any other step, although you aren't likely to do that well
(this is called guessing). But, even if you do all the other steps perfectly, you can't get a
high score without answering the questions.
What's the next most important step? Well, what step is most necessary to answer the
questions? Each question will be based on understanding the rules, so step 2 is the next
most important. It is very difficult to answer the questions correctly without a solid
understanding of the rules, but not impossible (you have a one in five chance of getting a
question correct if you just pick an answer randomly).
What's next? Well, the questions are a lot easier if you have a good system for organizing
your information. Thus, having a good diagram is important to answering the questions,
but it is not necessary. Any diagram, as long as it doesn't contradict what you have been
told, can be used to organize the information and answer questions. There is no "correct"
diagram for a game, only diagrams that are more helpful than others.
You need to know what is most important because that should dictate how you divide your
time on the LSAT and in studying you should spend more time answering questions on the
LSAT than you do on any other step, and you should spend more of your study time
30 The Fundamentals
learning how to answer questions. Don't obsess over trying to find the "perfect diagram,"
or in writing down rules in the most concise possible way (I'll talk about both of these topics
in more detail later), just make sure you understand what is going on, get it down on paper,
and get to the questions.
Deductions
Often, several of the rules have some common element. These rules can be combined to
give you a new rule. For example, if one rule says "R is after P," and another rule tells you
that "P is second to last," we know that R has to go last because that is the only position
after P. Combining the rules to find out new information that always has to be true is
known as making deductions. In some cases, as you answer the questions you will realize
that rules can be combined in this way (more often, though, the answers just ask you to
figure out what would happen in specific cases, not what has to be true generally). People
frequently try to make deductions before they look at the questions in order to know as
much as possible about the game before they answer questions.
The importance of making deductions before you look at the questions of trying to put
rules together as you look through them (what I list as step two), or after you have looked
at them but before looking at the questions (step three) is misunderstood by many, if not
most, students preparing for the LSAT. Most students feel that they must make deductions
before they look at the questions. They think that if they don't make deductions early on,
they won't be able to do the questions fast enough.
I strongly disagree, however. I know, both from first-hand experience and from working
with hundreds of students, that making deductions before looking at the questions is not
necessary either for speed or for accuracy on LSAT logic games. In fact, trying to make
deductions before looking at the questions is, for many students, the number one reason
why they can't finish the logic games section in time. If these students just read the rules
and began answering the questions, not trying to make deductions early on, they would be
able to do all four games, whereas with looking for deductions they can only do two or
three. For the majority of students I work with, including students who can and do get
scores in the 90
th
percentile on the LSAT, trying to make deductions before answering
questions hurts their score more than it helps it. Here's why looking for deductions before
answering questions deductions is unnecessary and often harmful on the LSAT:
First, you can answer any question on the test without having made any deductions prior to
answering that question. If the question requires you to make a deduction, it will push you
in that direction. The question will either give you a hypothetical situation to deal with
("Suppose X was 4
th
") that will lead you in the right direction, or the answers to the
question will give you concrete possibilities that you can then test out. Having something
concrete to work with makes combining the rules much easier. In either case, you don't
need to have thought out the deductions beforehand.
Second, many games have no deductions, or very few; in most other games the deductions
aren't terribly helpful. Looking for deductions in these cases wastes valuable time that you
need to spend answering the questions.
Finally, making the deductions is difficult to learn to do consistently. It can be learned, but
the payoff for learning it is often not worth the time and effort it costs. Most of you
probably have a limited amount of time to devote to studying the LSAT. Using that time
most efficiently means learning the most useful skills before any others.
The Fundamentals 31
This isn't to say that making deductions prior to answering the questions is useless; on the
contrary, deductions often make the games easier and faster. But, it is generally better to
wait to make deductions until you start working on the questions, and it is never necessary
to make them sooner. As I wrote this book, I did every logic games section in every LSAT
released since June of 1996. I purposely avoided making any deductions until I started
working through the questions. I used only techniques contained in this book. I finished
each section in the time given (35 minutes) with my normal (high) level of accuracy.
For this reason, I won't teach you to make deductions until much later in this book. For
now, we'll understand the rules individually, and only put them together if the questions
make us. We'll focus on learning the most important skills, the ones that will make the
biggest difference to our score. All those skills are contained in this chapter. Later on, once
you have mastered them all, you can learn to make deductions, which may help you
improve your score a little more. Those of you who haven't done any LSAT preparation
before now may not be surprised by this. Those of you who have previously taken an LSAT
preparatory course or read a book on the LSAT may be taken aback. You should read the
accompanying boxed text before going on.
If making deductions is so unimportant and potentially harmful, why do test preparation
courses stress it so much? Well, making deductions is difficult it's the hardest part of
logic games. Naturally, students want to learn how to do it. Students almost always spend
most of their time studying what is most difficult for them, rather than what will help them
get the most points. In addition, teachers enjoy teaching deductions more than anything
else. It allows them to show off their intelligence and LSAT skills, and it is an interesting
challenge to teach well. So there is great commercial and personal pressure to spend a lot
of time teaching deductions.
How to waste time on logic games
The biggest logic games time-waster is looking for deductions that you can't find. These
include deductions that don't exist and deductions that you are not capable of figuring out.
If you are going to make deductions, don't spend so much time making deductions that you
have insufficient time to answer all the questions. Another big time waster is looking for the
"perfect diagram." It is generally worthwhile to put together a diagram that you can
understand and use well, but trying to find some mythical Platonic ideal of a diagram is
foolish. The difference in efficiency between a "great" and a "decent" diagram, or even an
"excellent" and a "barely adequate" diagram is not worth a lot of time and effort. I'll teach
you how to put together good diagrams quickly. However, you will get into some situations
where the diagram you come up with just doesn't feel perfect. Forget about it. If you come
up with a diagram that accurately represents the game, but doesn't feel like the best
possible diagram, don't worry about it get on with the game.
I get calls from students all the time who say, "Brian, I can't figure out the diagram for this
game. I spent 10/20/30 minutes trying to do it, but I can't get a good diagram." I ask
them to describe the diagram they have, and then I say, "Let's use that and just go through
the questions." Almost every time, they get all or most of the questions right; they could
have done it themselves, but they were so freaked out about their diagram that they didn't
bother to actually do the game. Don't let this happen to you, especially on the LSAT.
Worst-case scenario you miss one or two questions because you have an inadequate
diagram. That's a lot better than running out of time and missing half the game because
you spent so much time on your diagram.
32 The Fundamentals
A note on pedagogy
I'm about to talk in detail about the theory and practice of each of the three steps. On the
one hand, it's going to be difficult for you to understand and digest all this information right
now, without any examples to work through. I really think people learn best by doing. On
the other hand, I think you need all the information to be located in one easy-to-find place
so that later on you can read through it and see how it fits together. So I'm going to say
almost everything I want to say about the steps in general right now. The next few
chapters will be us going through some logic games from past LSATs and seeing how to
apply everything I've just talked about. As we do that, I'll repeat a lot of this stuff and
show you how it works in specific cases. After you've read through that, I really encourage
you to reread this section; it'll make a lot more sense and you'll see how everything fits
together. But you should still read this section now, before going on to Game One,
because what we do in the rest of the book won't make any sense unless you understand
the foundations I'm about to give you.
Step 1: The Setup and Diagram
As you might be able to tell from my above rant against "perfect" diagrams, this is a much
misunderstood step.
In this step, you are reading through the setup the initial paragraph of the game trying
to figure out what is going on. You may also spot some rules as you read the setup. I'll
talk more about those in the next section. After you read through the setup, you will want
to figure out how to organize the game in a diagram.
There is only one kind of logic game, so you only need to learn one way of diagramming,
which will work for every game. Let me say that again, for those of you who don't believe
me: there is only one type of logic game. This is something that might shock those of you
who've done LSAT preparation before, but it's true.
Every LSAT game involves at least two types of things. There might be several things of
each type. The game will ask you to match up things of one type with things of the other
type. Take the following three examples I just made up:
A tutor is going to schedule the appointments of seven students P, Q, R, S, T, U,
and V one per day on Monday through Sunday of the same week.
Six boys A, B, C, D, E, and F are being grouped by height; three boys are of
average height, two are short, and one is tall.
Five flowers will be selected for a bouquet from a stall containing nine flowers:
begonias, daisies, irises, jasmine, lilies, orchids, poppies, roses, and snapdragons.
Each of these examples has two types of things. The first has students and days, the
second has boys and heights, and the third has the bouquet and the flowers. In each case,
you want to match up one type of things with the other. You want to match students with
days of the week, or boys with heights, or flowers with the bouquet.
It is possible that the game may contain three types of things. For example, the second
game could go on to say: "Two boys are blonde, one is a redhead, two are brunettes, and
one is bald." Then you'd be trying to match boys to height and to hair color. That doesn't
really change anything, as we'll see.
The Fundamentals 33
In every game, one of the two types of things is going to be more definite. That is to say,
it will change less, and/or you know more about it. In the first game you know more about
the days of the week. Why? Because Monday will always be next to Tuesday, which will
always be next to Wednesday, etc. This will never change. You know nothing about the
students, though you have no idea where P or Q or R will be. In the second game you
know more about the heights you know that there are three average kids, two short kids,
and one tall kid, which will never change but you know nothing about the boys, since
which boy is of which height might change. In the third game, you know more about the
bouquet it must always contain five flowers than about the flowers. We are going to
build our diagram around the definite group. The definite group is the one we know the
most about. Every other type of thing in the game is variable. I'm not going to use these
words ("definite" and "variable") much in the rest of the book, so you don't have to
memorize what they mean; you just need to remember that there are types of things that
you know more about and types of things you know less about, and your diagram should be
built around the ones you know more about.
Making the diagram is simple enough just write the names of the definite group members
(abbreviate if you want) in a row, horizontally or vertically. I generally prefer horizontally
for some reason, but it really makes no difference. There are occasionally games where it
makes more sense to do it one way or another for example if you are listing the floors of a
building you might want to do it vertically, since buildings are built up and down but it is
never necessary to go one way or another.
Here are our diagrams thus far:
Mon Tues Wed Thurs Fri Sat Sun
Short Average Tall
Bouquet:
Now, each definite thing will have a certain number of the other (variable) things associated
with it. Draw that number of blanks under (or next to) each definite thing (use lines to
show where the blanks are). Each day of the week will have one student, so each gets one
blank. In the next game, there are two short students, so Short gets two blanks, where
Average gets three and Tall gets one. There are five flowers in the bouquet, so Bouquet
gets five blanks. If the game doesn't tell you how many variable things go with each
definite thing, don't worry, just don't put any blanks.
Mon Tues Wed Thurs Fri Sat Sun
___ ___ ___ ___ ___ ___ ___
Short Average Tall
____ ____ ____
____ ____
____
Bouquet: ____ ____ ____ ____ ____
34 The Fundamentals
Write down the names of the variable things someplace so you can reference them (again,
feel free to abbreviate). Label them (heights, flowers, etc.). And that's it. That's it. We
have just generated what I call the original diagram; we'll recopy this diagram whenever
we want to test something out (see Making Test Diagrams later in this chapter).
What if there are three (or more) types of things? If there are more than two types of
things, go through the same process, and add blanks for the third type of thing; it can be
helpful to label the two types of blanks you have. For example, if each of the boys gets a
height and a hair color, then I'd just put a place for hair color next to every blank for a boy
(since every boy gets a hair color); I label the "boy" blanks b, and the "hair" blanks h (or if I
trust my memory, I just remember which blank is which).
Short Average Tall
b h b h b h
__ __ __ __ __ __
__ __ __ __
__ __
The short boys' names go under the "b" in the "short" group, and their hair color goes next
to their name (under the "h").
Wait! What if you can't figure out which type thing is more definite? If you can't figure out
which type of thing is more definite, just pick one of the types of things and base your
diagram on it in this way. Don't worry about whether or not you are "right." Like I said
above, the difference between a "perfect" and a "decent" diagram is not that big.
There is one type of game that doesn't diagram quite like this. This is where the definite
things have a certain spatial orientation. For example, they could be seats around a circular
table (see Game Seven). In that case your diagram should mimic the orientation. In a
circular table, the "last" person is next to the "first" person, so your diagram should include
this, probably by being a circle itself. This is rare, and circular tables are about the only
situation where it comes up.
You now know everything there is to know about diagramming logic games. After this it is
all just practice. Let's summarize: every game has two or more types of things in it, and
the game just consists of matching a thing from one category with one or more things from
another category. Figure out which type of thing you know more about, and write those
things down in a line. Figure out how many of the variable types of things (the ones you
know less about) go with each thing you have just written down (the ones you know more
about), and make that many blanks under each definite thing. Don't worry about the
vocabulary "definite" and "variable"; that's just for me to avoid clunky sentences, and isn't
that important for you to master.
I recently taught a student who had gone through a course with a major test prep company
whose name I won't mention. He told me that diagramming the games always confused
him. I taught him exactly what I just taught you and he stared at me in disbelief. He
couldn't accept that the diagrams were so straightforward. In fact, I thought he was on the
verge of getting mad at me. He kept asking why he had been taught that it was so
complicated and difficult. I couldn't tell him, and I can't tell you. Maybe test prep
companies want you to think the LSAT is harder than it is so you feel like you're getting
your money's worth (but I doubt it). My theory is that this just never occurred to them. It
took me years of constant one-on-one interaction with LSAT students (and the help of some
very smart people) to come up with this. I've never found a game that couldn't be easily
diagrammed like this. In fact, I've found that diagramming this way often gives me a better
diagram than the more complex approaches I've seen, and it never gives me a worse one.
And the student I told you about? He went from getting about 12 questions right per
section to getting about 20, in just a few days.
The Fundamentals 35
diagrammed like this. In fact, I've found that diagramming this way often gives me a better
diagram than the more complex approaches I've seen, and it never gives me a worse one.
And the student I told you about? He went from getting about 12 questions right per
section to getting about 20, in just a few days.
Step 2: Read the Rules
When you read the rules you are trying to understand them. If you don't understand a rule,
you can't apply it, and if you can't apply it, you are going to miss questions. This means
that you need to read the rules slowly and carefully, one at a time. As you read each rule,
think about what it means and what it doesn't mean. For example, if you are told that "A is
next to B," you aren't being told what order they come in (B might be before A or after BA
or AB). It is much worse to think a rule means more than it really does than to think it
means less.
Most of the time, rules will be easy to interpret. They will look like rules you have seen and
understood before. In the few cases where the rules aren't easy to understand, try making
them specific in order to make them clearer. If a rule says something like "No professor
teaches a class taught by a professor with the same specialty," try out some examples: say
to yourself, "What if professor X taught class A? What would happen?" Trying out specific
examples will make understanding the rule much simpler because people think about the
concrete better than the abstract.
Some seemingly difficult types of rules come up again and again. One of the most common
types of rules, called "If then" rules, typically takes the form "If such and such
happens, this other thing has to happen as well." These rules are so important that I will
provide an in-depth explanation of how they work in their own separate chapter ("If
then" Rules and Negations), which comes after Game One. I wait until after Game
One because that game doesn't have any "If then" rules, and it gives you an opportunity
to get used to the general format of games, as well as all these fundamental concepts,
before adding new ideas.
Once you understand a rule, you need to write it down. You will need to look at the rules
over and over, often several times for each question. The LSAT doesn't write the rules in
the clearest or most useful language; reading what they have written again and again will
just waste time. Instead, we write the rules in a way that makes sense for us.
When writing down a rule, make sure what you write is clear, complete, and accurate. In
other words, you need to be able to understand what you have written down, it needs to
include all the information in the rule, and it needs to be right (hopefully that last part is
obvious). One common mistake that people make is to try to write their rules in the most
compact form possible. They learn and try to use all kinds of strange symbols. This is fine
if you are completely comfortable with the symbols. If you aren't, don't use them. Don't be
afraid to use words when writing down rules. You don't need to write in complete
sentences, but there's nothing wrong with doing so if you wish. You wrote the rule down
well as long as you can understand what you have written.
My rule of thumb for writing down rules is this: I assume that I will turn dumb 10 seconds
after writing down the rule. I write every rule in a way that the dumb version of me can
understand. I figure if I have an opportunity to make a mistake I will make it, so I write the
rules in a way that I can't possibly misinterpret.
36 The Fundamentals
The final rule is this: be consistent. If you use a symbol or phrase to mean something,
always use it this way. Don't use a symbol one way in one game and another way in
another game. Otherwise you will get confused.
Symbols Used in This Book
__ This is a space that must be filled by someone
XY X is right next to Y in this order (or in the same group as Y)
X _ Y (X _ _ Y, etc) There is one space (or two, etc.) between X and Y in this order
Those are all of them. The rest of the time I use English words and phrases.
A final point: go through the rules one by one. When you are thinking about rule 2, don't
worry about rule 1 or rule 3. Don't try to put the rules together at this stage; do one thing
at a time. Otherwise its easy to get sidetracked or confused.
Let's summarize. Read the rules one by one, slowly and carefully. Make sure you
understand each rule. Once you understand a rule, write it down. Write it down in a way
that you will easily understand later on. Don't leave out any information. If you use
symbols, use ones you understand and always use them in the same way.
Another note on deductions
This note is especially for those of you who've done some LSAT practice before this. I
mentioned before that making deductions before you do the questions is not necessary and
can be bad. It can be helpful, too, and I'm here to help. I will teach you how to make
deductions eventually, but I'm not going to tell you how to make them now. As we practice
games we'll look at the rules individually, rather than trying to combine them. As you go
through the book you may see ways in which the rules can be combined. Please don't try to
look for these and don't use them if you see them; I don't want you practicing the wrong
skills. Instead, do the games as I do. You need to learn and get close to mastering these
fundamentals before you try to learn to make deductions. Even if you have studied for the
LSAT before and consider yourself pretty good at logic games, go through this book in
order; learning the most important skills will help your score more than jumping straight
into the more advanced ones (and you really won't be able to understand and apply the
advanced techniques without understanding my view of the fundamentals).
We are going to do the first eight games in this book without making a single deduction.
Why? For a couple of reasons. First of all, because you don't need to. In fact, trying to
make deductions is often a huge waste of time. By doing games without making deductions
I demonstrate that it is not just possible, it's often faster and easier. Second, by doing
games without deductions I will give you a solid grasp of the most important aspects of
games. Deductions are often a distraction at the beginning; once you have mastered the
most important skills you can turn your attention to the little things, like deductions. Third,
by not learning to make deductions you save studying time. Given that most LSAT students
have a limited amount of time to prepare for the test, and a lot of skills to master, limiting
the number of skills you try to learn is good for you. You should focus on the skills that
make the most difference to your score. Finally, many of the skills you will learn as we go
through the games in this book will be helpful in making deductions. When you finally turn
to the Advanced Techniques, deductions will make a lot more sense and be a lot easier.
The Fundamentals 37
Step 3: Answering the Questions
All right, you excited? We're about to go over the most important step. Fasten your seat
belts.
Remember, I'm about to inundate you with information. You won't be able to take it all in
right now. In fact, a lot of it won't make sense until we start doing games. Still, read
through this, skimming if you have to, before going on to the next chapters. After you've
read through the first couple of games I explain, you'll know what in all of this you need to
reread and make more of an effort to understand.
The order of the questions
Before we talk about answering a single question, we have to talk about the order in which
we will answer the questions. If you've ever done an LSAT logic game before, you've
probably noticed that the questions for each game are not given in order of difficulty. To a
certain extent, the first question is often the easiest and the last often the hardest, but this
is frequently not true, and even when it is true the middle questions don't follow any pattern
of difficulty. This is important, because you have only a limited time to answer questions on
the test. There is a chance that you will run out of time and not be able to answer every
question in the games section. Since that is the case, you'd better do the easiest questions
first. Why? Two reasons: they take less time, and you are more likely to get them right.
If you run out of time and have been doing the easy questions first, rather than just going
through the questions in order, you will have answered more questions and gotten more
right than you would have otherwise.
In addition, working through certain questions first will make other questions easier. Some
questions will help you understand the game better, and will generate scratch work that you
can use to eliminate answers on other questions. It's better to do these questions first,
then, so as to make the later questions easier. Fortunately for us, the easiest questions are
also the ones that give us the most helpful scratch work.
There is a science to the order in which you will do the questions. That is to say, you can
memorize the procedure ahead of time and simply go through a series of mechanical steps
to determine what questions you want to do when. Sound easy? Well, it is!
Logic Games Questions in Order of Difficulty (Easiest to Hardest)
1. What can be true? and If-questions
2. What can't be true?
3. What must be true?
4. What can be false?
5. Which is a complete and accurate list?
6. What is the maximum/minimum number of ?
7. How many diagrams are possible?
These are all the question types you'll ever see on the games section. (Notice that I've
italicized the "is" in "Which is a complete and accurate list?" That's because sometimes you
will see "Which could be a complete and accurate list?" which is actually the same as a
"What can be true?" question, a much easier question type) Now, questions will often be
asked in different words. For example, "Which of the following is possible?" means the
same as "What can be true?" I'll list common equivalent questions in the accompanying
boxed text.
38 The Fundamentals
Also, questions will generally involve more words than this. For example, a question could
ask, "Who can be in position 3?" This is a "What can be true?" question, because it asks for
what can happen. If the question asks, "Who must be in the same group as Todd?" this is a
"What must be true?" question, because it asks for something that must happen.
Common Equivalents for Each Question Type
Here are some different ways questions can be phrased.
1. What can be true?
! Which could be true?
! Which of the following is possible?
! Which of the following is acceptable?
! Which one of the following could be a complete and accurate list of ?
! Each of the following cannot be true EXCEPT (see #2 for equivalents of "cannot
be true"; any of them can be substituted for "cannot be true")
2. What can't be true?
! What must be false?
! What is impossible?
! Which of the following cannot be a complete and accurate list of ?
! All of the following can be true EXCEPT (see #1 for equivalents of "can be true")
3. What must be true?
! All of the following can be false EXCEPT (or "could be false")
4. What can be false?
! All of the following must be true EXCEPT
Question types 5 doesn't have any common equivalents.
6. What is the maximum/minimum number of ?
! What is the greatest/smallest number of ?
! What is the earliest/latest?
7. How many diagrams are possible?
! For how many of the can the position be determined?
! In how many distinct orders can ?
! Which of the following, if true, would allow to be determined?
! There is exactly one way in which if which of the following is true?
If-questions (these words mean "If")
! Suppose
! Assume
There is one additional complicating factor. Some questions start out, "If such and such
happens" and then go on to ask a question (such as, "If such and such happens, what
must be true?"). We call these If-questions (as opposed to "If then" rules). These
questions ask you to imagine a hypothetical situation, and then they ask a question about
that situation. The question can be any one of the seven types I've listed above. For
example, they might ask, "If M is in the first position, what must be true?" If-questions give
you new information about the game, but this new information pertains only to this
question. All the normal rules apply. These questions are easier than other questions
because they give you more information, and more information makes things easier. We
The Fundamentals 39
want to do them early on for that reason. In addition (as I'll explain in more detail later),
these questions give you helpful information for later questions, but you can't use the notes
from your previous questions to help you do them. Thus, if you do them sooner they will
help you with later questions, but there is no advantage to doing them later. Thus, we do
If-questions (those starting with "If such and such happens" before most other questions.
So, here is our procedure. We do all the If- and "What can be true?" questions first. Then
we do "What can't be true?" questions. Then "What must be true?", and so forth. It
doesn't take any significant amount of time to go through the questions and pick out the
one you want to do next, but it does save a lot of time to do the easier questions first.
Why this order?
I suppose you want to know how I arrived at the order I use for the questions. Like I said,
there's a science to it.
First of all, you have to notice that there are more wrong answers than right answers. Four
out of five answers are wrong. To answer a question you have to look at the answers. This
means that you spend most of your time looking at wrong answers, not right answers. For
any given question, the easier it is to figure out if an answer is wrong, the easier the
question is.
Second, answer a question. What is easier to determine: that an answer follows all the
rules of the game, or that an answer breaks a rule? It is easier to determine that an answer
breaks a rule. To determine that an answer follows all the rules, we have to look at all the
rules, and make sure every person in every slot of the game follows all these rules. To see
if an answer breaks a rule, we typically only have to look at some of the rules and some of
the parts of the game. It is almost always less work to spot answers that break the rules of
the game than answers that don't.
Put the two points together. We evaluate the easiness of a question based on how easy the
wrong answers are to spot. The easiest answers to spot are those that break rules.
Therefore, the easiest question should be the question where the wrong answers break
rules. Et voila! Look at question type 1. If the question says "What can be true?" you
know that the right answer can be true. Which tells you what about the wrong answers?
Yes! It tells you that the wrong answers can't be true, which means that they break the
rules! This is as easy as it gets.
Now, question type 2 asks "What can't be true?", which means that the correct answer
breaks a rule, and the wrong answers follow the rules. Not so easy. But it is much easier
than question type 3, which asks "What must be true?" On one of these questions we know
that the right answer must be true, but the wrong answers can be true, right? That is, they
might be true sometimes, but not all the time. So on one of these questions, potentially no
answer breaks a rule. That makes the answers more difficult to evaluate (there are other
difficulties, as we'll see later on). The other question types take even more work to
evaluate, but for now you'll have to trust me on that.
There is an additional consideration, which is how much we can use our previous work to
help us on a question. As we do the questions, we will generate diagrams which we use to
test out possible answers. Each of these will show us a possible way the people in the game
could be placed. Many questions require us to think about what is possible. For example,
in a "What can't be true?" question, we know that the wrong answers all describe possible
40 The Fundamentals
situations. That means that if we see the situation described in the answer in a diagram we
already have written down, we know that it is possible without doing any more work.
In question type 1, "What can be true?", only one answer (the right one) describes a
possible situation. That means that the wrong answers break the rules, so the situations
they describe won't show up in the diagrams we've previously generated. So there is little
point in waiting to do these questions not only are they pretty easy, they also aren't going
to get any easier if we draw more diagrams. But the other questions, types 27, will be
made a lot easier by having other diagrams to look at.
This is why we do If-questions the first time around. If-questions are pretty easy, because
they give us more information. Also, we can't use diagrams from other questions to help us
on If-questions (I'll explain this later). But we can use the diagram we make on an If-
question to help us on other questions. Which means that If-questions won't get any easier
if we wait, but doing them sooner will make other questions easier.
Science.
Making Test Diagrams
You will often have to test answers. This means that you will have to see if the situation
described in the answer is possible. You do this by making a new diagram and trying out
the situation described in the answer. Why do we make a new diagram? Well, when we
test something out we have to try it out, which means we have to write it down in a
diagram (we never do things in our heads, because that is too hard). We are going to have
to test out many answers. If we use the diagram we made originally (in step 1 of the
game), we'll have to erase what we put in it when we want to test another answer. This is
a waste of time, and, in addition, it means that we can't reuse the diagram we just made
(and erased) later on. So, whenever we want to test something out, we draw a new
diagram. Our new diagram should contain everything we put in our original diagram (see
Step 1: The Setup and Diagram earlier in this chapter), but we don't have to recopy our
rules (since we aren't going to add anything to them).
Once we have a new diagram, we add to it whatever we are trying to test out. If the
answer says "M is in group Z," we put M into a space under group Z.
After we've added anything to our diagram, we go through our rules and see if anything
else has to happen. For example, maybe N always goes with M. If M is in group Z, then N
has to be there too. If anything has to happen, we put that in the diagram. Then we go
back through the rules again, because, as I just said, after we've added anything to the
diagram we go through our rules.
If we go through the rules and find that nothing else has to happen, we then try to fill out
any empty spaces that are left. Put whoever remains in any empty space, as long as it
doesn't break a rule. Keep trying each space until you find one for each person. If you can
fill all the empty spaces, you have a possible diagram. If you can't fill the empty spaces
without breaking a rule, you don't have a possible diagram. Knowing this one way or
another is the key to answering every question.
If you have a diagram that doesn't work (meaning it breaks a rule), cross it out (erasing
takes too long). We don't want to look at the diagram ever again, because it will just
confuse us.
The Fundamentals 41
Don't erase or cross out diagrams that work. We'll call these previous diagrams and we
can use them to help us figure out later questions. Each previous diagram gives us a
possible situation, and that can be helpful, as we'll soon see.
Marking with a dot or star
Often the answers don't seem to give you enough guidance about where to put things. But
to complete your test diagram, you have to fill every spot; this means that you have to put
each thing someplace. Here is the worry students often have: "If I put this thing in one
place, maybe it won't work there, but it would have worked someplace else. I'll mark the
answer wrong, when it really is right. How do I know where things are going to work?"
Well, sometimes you just don't. But you have to draw a test diagram, which means you
have to put the person (or thing) someplace. Have no fear. You don't need to be able to
figure out the best possible place in your head. Instead, when you are uncertain where to
put someone, put them in the first available place. Then mark that person with a little dot
or star. This tells you that you made a decision where to put that person, but they might be
able to go someplace else. Then, if the diagram ends up not working out, try moving the
person you have dotted/starred to the next available place. This usually means moving
them from left to right. If you have multiple people marked, move the last one you put in.
Keeping doing this until you get a diagram that works, or you run out of places (which tells
you that it will never work).
I use an asterisk (*) in this book when marking things in this manner. In real life, I use
little dots, because they are quick and easy to put in; but they are hard to see in a book, so
I'll stick with the big, obvious asterisk.
Once you have dotted one person, you will often be forced to put other people in definite
spaces. Don't mark these people, just put them in. Again, only dot when you make an
arbitrary decision, so that you know where to go back and make a different decision. And,
finally, don't do this when you know already where the person can't go you are wasting
your time trying them in places that you see are impossible.
We are going to spend a lot of time making test diagrams as we work through the games in
this book; soon this will all become habit for you.
The Question Types
Let's take a detailed look at each type of question. In order to answer a question, you need
to know how to figure out what the right answer is and what the wrong answers are. The
question always describes the right answer to you. If it says "What must be true?", you
know that the right answer is a situation that must be true. Since each question describes
the right answer, it also describes the wrong answer: the wrong answers are those that
don't have the quality the question describes.
For every type of question, you need to know what you are going to have to do to test out
the answers. In addition, you need to know whether or not you can use previous diagrams
to help answer the questions. We'll examine all these issues for each question type, and for
If-questions.
These next sections contain a lot of information. You will eventually have to memorize it. I
am totally serious you can't do well on logic games without knowing how to answer the
questions. As you read through the section on each question type, write out the important
42 The Fundamentals
points what you know about the right answer, what you know about the wrong answers,
how to test the answers, and how to use previous diagrams. Don't try to memorize this yet
read through one or two games in this book to get a feel for how these things work in
practice. Then memorize. The sooner you understand each type of question, the better
you'll do on the test. I have a summary of this information in the Glossary.
I've had students who were really smart, and really good at understanding the games,
making diagrams, and making deductions. But they kept forgetting how to do the
questions. Once I reminded, them they'd do great, but they didn't bother to memorize the
processes on their own. Their scores suffered as a result. Eventually I had to yell at them a
little to get them back on track. But this is the most basic kind of knowledge, and the most
important. Remember, doing the questions is the only part of the test that impacts your
score, and you aren't going to get many right if you don't remember quickly and easily how
to do the questions. In fact, think how much less stressful the test will be if you never have
to struggle to think about how to do a question you can spend all your energy just doing
them, rather than trying to remember how.
1. What can be true? questions
What do we know about the right answer to these questions? We know that it can be true
that is to say, it is possible. This means that it follows all the rules of the game. What do
we know about the wrong answers? They can't be true they must break at least one rule.
How do we test out the answers? We test out the situation the answer describes (see
Making Test Diagrams earlier in this chapter). We draw a new diagram and put in it the
situation the answer describes. We see what has to be true from the rules. Then we fill in
all the empty spaces. If this works if we can fill the empty spaces without breaking a rule
the answer can be true and it is the right answer. If you see that you are going to break
a rule, stop, cross out the diagram, and cross off the answer. It is wrong.
Notice that you aren't going to get a lot of mileage out of previous diagrams. Most of the
answers are going to break the rules, which means that they won't show up in previous
diagrams (since your previous diagrams are all of possible situations).
2
Once you've found the right answer, go on to the next question. There is only one right
answer; have confidence in yourself and don't keep checking answers. Look at it this way:
if you don't know what you are doing, you probably aren't going to get the right answer no
matter what, so you might as well get it over with quickly. If you do know what you are
doing, then you've already got the right answer, so looking at the other answers is a waste
of time.
One more thing. Very often the first question of the game is a special kind of "What can be
true?" question. The answers to this question will actually be a complete spelling out of a
potential scenario. For example, the answer could list everyone in each position from 1 to
7, in order (see Game One question 1). These are the easiest type of questions. You don't

2
Some of you may be thinking, "Ah, so we should keep all of our previous diagrams even
the ones that break the rules, so we can use them on these questions." But then you have
to distinguish between diagrams that break the rules and those that don't. This is confusing
and you will end up making mistakes. Also, you won't know what caused the diagram to
break a rule was it the situation your answer describes, or something else? Finally, it is
so much easier to spot answers that break rules that looking at previous diagrams is more
trouble than it's worth.
The Fundamentals 43
have to draw a diagram for these at all, since everything is already spelled out for you. You
just go through each answer and compare them against the rules. Once you have found the
right answer, though, I recommend rewriting it yourself, so you can easily see it later as a
previous diagram. Often people overlook these diagrams and they can be very helpful (as
we'll see in Game One).
2. What can't be true? questions
The right answer to these can't be true, which means it breaks a rule. What does this tell
you about the wrong answers? They can be true, which means that they follow all the
rules.
Previous diagrams will be very useful on these questions. As you look at each answer, first
see if you have a previous diagram which contains the situation described in that answer.
For example, if the answer says "Q is with P" and the diagram from another question has Q
with P, you know that can be true. Cross the answer off (the answer, not the diagram!),
since it can be true. If the answer isn't in your previous diagrams, skip it. Look at the other
answers first. Hopefully you will be able to eliminate all of them using your previous
diagrams, so you won't have to do any work for the question.
If you end up having to test out one or more answers to the question, test out the situation
described in the answer as I talked about in Making Test Diagrams earlier in this chapter.
If it works that is, if you can fill out all the spaces in the diagram without breaking a rule
you know that the answer is wrong. If it doesn't work if you are forced to break a rule
then the answer is right. You are done.
If you eliminate all but one answer, pick it but don't test it out. You know it has to be right,
because there has to be a right answer. Trying it out is just a waste of time.
Here's a pet peeve of mine (and all of my pet peeves relate to things that make students
miss questions, so you should be interested): I really don't like it when students say a
diagram, or an answer, that breaks the rules is wrong. It doesn't work, it breaks the rules,
it can't be done, maybe you can even say it's false, sure, sure. But wrong? Not always.
On a "What can't be true?" question, the answer that breaks the rules is right. Breaking the
rules is a separate issue from being right or wrong. The reason I don't like this is that when
students start calling diagrams that break the rules "wrong," on some level they are
thinking that this answer can't be right. But that's not true; best case scenario, this sort of
confusion slows you down. Worst case scenario, it confuses you to the point that you miss
the question (I see it happen a lot). Lesson: don't confuse breaking rules with being
wrong, or following rules with being right. Don't call the two different things by the same
name.
3. What must be true? questions
These are probably the most misunderstood type of question in the games section; they are
also one of the most common (there being five or six of them on the average LSAT). They
are going to confuse you until you memorize the process for them and understand how and
why it works. Once you've done that, though, they aren't that much harder than the first
two types of questions.
What does it mean for something to have to be true ("have to be true" is the same as "must
be true," but it conjugates better)? It means that the thing must be true no matter what.
If something must be true, it will be true in any possible diagram for the game. Notice that
44 The Fundamentals
this is an "If then" sentence, so give me the contrapositive. That's right: if something is
not true in every possible diagram for a game, then it is not something that must be true.
People generally test the answers to these questions by trying out the situation the answer
describes. If the answer says "A must be first," then they put A first and see what happens.
Now, what if A works in the first position? Does this mean that A must be first? No. It just
means that A can be first. Typically, most answers to these questions can be true, so you
haven't really gotten any closer to the answer.
To test an answer, you need to see if you can make the game work in a different way. If
the answer says "A must be first" and you see that A could be second, then you know that
the answer is wrong. So here's what you do. First, look at your previous diagrams. You
are looking for a diagram which contains a situation other than what is described in the
answer. If you see one, you know that the answer doesn't have to be true it could be the
way the previous diagram shows. So cross it off. If you don't find what you are looking for
in previous diagrams, skip the answer. Hopefully you'll eliminate all the others and you
won't have to test anything out.
If you have to test out an answer, test out a situation other than what the answer
describes. If the answer says "A must be first," try A second, or third, or anyplace except
first. If you can make it work that is, if you can fill all the spaces in the diagram without
breaking the rules, doing something different from what the answer says the answer is
wrong. If you can't make it work, then the answer is right. I talk about this in more detail,
and give examples of how it works, in the chapter entitled "If then" Rules and
Negations.
Now this is backwards thinking, and it is confusing at first. If you generate a working
diagram, the answer is wrong, and if you are forced to break rules, the answer is right. But
it should make sense. If you can make a diagram work that doesn't do what the answer
says, than the answer doesn't have to be true you've got a diagram where it's false. If
you can't make it work any other way then it has to be true.
4. What can be false? questions
The right answer to this questions can be false. What does this mean about the wrong
answers? They can't be false. We have just talked about answers that can't be false this
means the same as "must be true." On a "What can be false?" question, the wrong answers
must be true, and the right answer is possibly false (if it has to be false that's fine as well
something that has to be false is also possibly false). These questions are the mirror image
of "What must be true?" questions, just like "What can be true?" and "What can't be true?"
questions are inverses of each other.
How do we do "What can be false?" questions? We test each answer as we would the
answers in a "What must be true?" question. Try and make a diagram which contains a
situation other than what the answer describes. If you can't make something other than
what the answer describes work, the answer must be true, and is the wrong answer. If you
can, the situation described in the answer doesn't have to occur, and the answer is correct.
You can use previous diagrams on these questions, but they aren't that useful. Look for
previous diagrams which contain something other than what the answer says (as we do in
"What must be true?" questions). If you see one, then you know the answer doesn't have
to be true, and it is the right answer. If you don't have any previous diagrams that show
situations other than what the answer describes, then the answer might have to be true,
but you won't know until you test it out. Don't eliminate the answer in this case.
The Fundamentals 45
These questions are a bit time consuming, but they are very rare, occurring less than once
per LSAT.
5. Which is a complete and accurate list? questions
Notice that this question says "Which is a complete and accurate list?" not "Which could be
a complete and accurate list?" The second version is actually a "What can be true?"
question (see above).
These questions ask for a list of things that fit a certain description. The list must be
complete, so it has to include everything that fits that description, and it has to be accurate,
so it shouldn't include anything that doesn't fit that description. If the question says,
"Which of the following is a complete and accurate list of boys who can have red hair?", the
answer must have all the boys who could be redheads, and no boys who can't. Complete
and accurate lists include all the things that fit this description in any possible diagram. If
only two boys can be redheads at one time, the list of possible redheads might still be four
or five boys long. Any of those four could have red hair in some possible diagram, but only
two at once.
The wrong answers to these questions are going to leave someone off the list, or will have
someone in the list that shouldn't be there. Previous diagrams are going to be very useful
here. Any person or thing that fits the description in any previous diagram should be in
your list. You can cross off any answer that leaves one of these people/things off. Thus, if
you see that A can have red hair in one diagram, and B in another, then any list without A
or B is wrong.
If you don't eliminate all the wrong answers using previous diagrams, you are going to have
to test out at least one answer. Look at your remaining answers and pick a person/thing
that at least one of them lacks (if they all have that person/thing, you know it has to be in
the list, so don't bother testing it). Draw a diagram where that person/thing has that
quality, and test it out. If you can make the diagram work, then any answer without that
person/thing in it is wrong. If you can't make it work, then any answer with that person or
thing is wrong. Do this until you have one answer left.
6. What is the maximum/minimum number of ? questions
These questions ask you to determine the largest or the smallest number of things that can
have a certain property. For example, they might ask you, "What is the maximum number
of students who can be scheduled before A?" The right answer will be the largest or
smallest number that can have that property, and the wrong answers will be less or greater
than this.
First look at your previous diagrams. Find the largest/smallest number of things fitting the
description. If you have two diagrams, one where A is 4
th
and one where A is 5
th
, the
largest number of students before A is at least 4 (there are 4 people before the 5
th
position). Thus, we know that any answer less than this is wrong. Often you will be
pressed for time by the time you get around to these questions this is one of the last
question types you will do. You may need to guess; you can pick any one of the answers
you didn't eliminate, but I think you are safest picking the number you know can happen.
That is, pick the one that you see in a previous diagram.
If you have time, and haven't eliminated all but one answer using your previous diagrams,
here's how to answer the question. Look at the largest (or smallest) answer (whichever the
question asks for). Try to make that work. Imagine the question asks for the maximum
number of students who can be scheduled before A. There are seven slots in the game.
46 The Fundamentals
The largest answer is six. If this were right, then A would have to be seventh. Put A
seventh and test out the diagram. If you can make it work you have the answer. If you
can't, go to the next largest answer, and so forth. If the question asks for the minimum,
start with the smallest answer and work your way up.
7. How many diagrams are possible? questions
These questions all rely on your ability to figure out who in a diagram can be moved without
breaking a rule and who cannot. They are not terribly common in the last ten years, they
have appeared slightly less than once per LSAT.
In their most common form, these questions ask how many different ways the game could
be worked out, usually given some condition specified in an "If." For example, "If X was
fourth, how many different ways could the people be ordered?"
Since these are usually If-questions, previous diagrams aren't terribly helpful. We generally
don't use previous diagrams in If-questions (see If-questions later in this chapter).
To do these questions, we draw a test diagram, putting in any conditions the question
stipulates (if it is an If-question). We then go through the rules and see if these conditions
cause anything to have to be true. After we've gone through the rules, we fill the remaining
spaces in the diagram. Be sure to mark anything that might have gone elsewhere with a
dot or a star (see Marking with a dot or star under Making Test Diagrams earlier in
this chapter). Once you have created a diagram that works, move the first thing in your
diagram that is marked with a dot or a star. See if this allows you to make a new diagram.
Keep moving things that you can move until you run out of ways to change the diagram.
Count up the number of new diagrams you've made; this is the answer to the question.
There is a simpler variant of this question, which asks "For how many of the things can the
position be determined?" These are usually If-questions as well, so previous diagrams
aren't helpful. You need to count up the number of things that can only go one place.
Generally, you will have a good idea of who can and can't be moved the conditions in the
question, together with the rules, will cause certain people to have to go in certain places.
If you think someone's position can be changed (for example, if they are marked with a dot
or star), try to put them someplace else. If there is no place else to put them, their position
is determined.
A final variant of this question type asks for information that would allow you figure
something out for sure. For example, the question could ask, "Which of the following would
allow you to determine who is in the seventh position?" or "Which of the following would
allow you to determine what group A goes in?" This is really asking, "Under what condition
is only one diagram possible?" That is, which answer makes it so that things can only be
put in the diagram one way? The wrong answers will be conditions that allow more than
one diagram.
Previous diagrams aren't terribly useful on this variant of the question either. You want to
see if knowing the information in the question forces something to happen. You would need
two previous diagrams, both of which match the scenario in the answer. If they differ as to
what the question asks if, for example, a different person were in the seventh position in
each diagram then you know that scenario doesn't force anything to happen. You can see
that this is hard to do. I recommend skipping these questions unless you have plenty of
time; if you have the time, just work through the answers.
In these questions, you check an answer by drawing a diagram that matches the scenario in
the answer. You then try to see if there is only one way the position or person in the
The Fundamentals 47
question can go. For example, if the question asks about the seventh position and the
answer says "B is fourth", you would put B fourth and see if there is a definite person who
must go seventh. If there doesn't seem to be, try putting each person seventh, one by one.
If more than one person can go seventh without breaking any rules, the answer is wrong.
For an example, see Game Eleven, question 16.
If-Questions
As I said earlier, If-questions give you a hypothetical situation. This situation is almost
always compatible with the rule you are given in the beginning. If it isn't, the question will
explicitly say so (see below). If the question doesn't explicitly say it is changing a rule (and
they almost never do), then all the rules are in effect. So If-questions give you more
information. This makes them easier than normal questions. Because they are easier, we
want to do them early on. If-questions can ask any question you can have an If with a
"What can be true?", a "What can't be true?", a "What must be true?", and so forth. But,
generally, If-questions, no matter what they ask, are as easy as "What can be true?"
questions and we should do them first (along with "What can be true?" questions).
If-questions give you new information. Because of that, you can't use previous diagrams
from other questions these diagrams don't always match the scenario given in the If-
question, and so are irrelevant.
3
However, and this is important, the scenarios in If-questions don't break any rules.
Because of that, any possible diagram you generate for an If-question will be possible for
any question (except other If-questions). So you can use previous diagrams from If-
questions when doing other questions (except other If-questions).
Let's review: If-questions should be done first, along with "What can be true?" questions.
You can't use previous diagrams to help you with them. You can use the diagrams you
make on If-questions to help you on other questions.
How to do If-questions
This is hugely important, because If-questions make up about 50% of logic games
questions. We'll go through a lot of If-questions as we do the games in this book; here is
the process we will use:
Steps for Doing If-Questions
1. Draw a new diagram.
2. Put in your diagram whatever the "if" part of the question describes.
3. Go through the rules and see if anything has to be true based on that; if
so, write it down. Also write down things that can't be true (e.g. "X can't
go in space 3").
4. Answer the question based on the question type ("What can be true?",
"What can't be true?", etc.).

3
There is an exception to this, but it is complex to learn and not necessary for doing well on
logic games. I will talk more about it in Advanced Techniques, but you will do just fine
not thinking about it. Don't use previous diagrams with If-questions and you will do great.
48 The Fundamentals
Step 3 is a crucial step, and involves a repeated process: Every time you figure out
something that has to be true (or can't be true) from the rules, go back through the rules
again and see if that makes something else true. Do this until you go through the rules
without learning anything new. Don't put anything into your diagram that might or might
not be true if you aren't sure who goes in a certain blank, leave it blank. Then go through
the answers in the way relevant to the type of question it is "must be true," "can be true,"
etc. using the diagram you just made. You may have to recopy it a couple of times since
you may have to test out several answers.
If-questions that break rules
Once in a while an If-question will tell you that it is changing the rules either taking a rule
away or modifying one (an If-question that adds a rule is just a regular If-question, giving
you new information in addition to what you already have). These If-questions will be very
clear about what they are doing. They will tell you explicitly that they are changing or
removing a rule. It might say, "If the rule about blah blah were changed to say such and
such, but all other conditions remained the same" Save If-questions that change or
remove rules for last. We can't use the diagrams from these questions on other questions,
because they don't follow all the rules. Other than that, do these questions as you would
any other If-question.
These questions are rare. About 2% of the logic games questions since 2000 have changed
or removed rules. This works out to less than one of these questions per LSAT, on average.
What Do I Do Next?
Well, you've just worked through a huge mass of facts and concepts. You might not have
understood them all many of them won't make sense until you've used them and you
definitely won't have memorized them all. At this point don't worry about that too much.
Go and work through Game One. Don't try to do the game on your own; instead, work
through the game along with my explanation in the chapter (later I'll tell you when you
should do a game on your own before reading the chapter). You'll start to see how all this
fits together and what it all means. Then you can start thinking about memorizing things.
Much of this chapter for example, how to diagram games you will memorize without
trying to, as you work through the games in this book. If you find that you don't remember
general concepts like how to diagram games or how to write down certain types of rules
even after going through a couple of games, then make a conscious effort to memorize
them. Flash cards are helpful (see below). Other things are more detail-oriented, and
you'll certainly have to make an effort to memorize them. Things you should make a
conscious effort to memorize are: the order in which you do the questions, the common
ways of phrasing each question (Common Equivalents for Each Question Type earlier in
this chapter), and the way to answer each type of question (including how to use previous
diagrams on each). I recommend making flash cards for each question type.
A Note on Flash Cards
Flash cards should help you prepare for the test. The test will use certain ways of talking
which are different from my way of talking, and probably from your way as well. You don't
want to memorize vocabulary words like "deduction" or "definite entity" because these won't
appear on the LSAT. I try to use vocabulary like these as little as possible in this book so as
to save you the work of learning them. The LSAT will use certain words and phrases over
and over again though, like "What must be true?" or "exactly." You want to memorize what
The Fundamentals 49
to do when you see these words and phrases. On the front of your flash card, you'll have
the word or phrase that will appear on the LSAT. On the back, you should have exactly
what to do when you see it.
You should have a flash card for each question type, and a separate flash card for every
alternate way of asking the same sort of question. So you should have different flash cards
for "Which of the following is possible," "Which of the following could be true," "Which of the
following is acceptable," "None of the following can be true EXCEPT," and so forth (all of
those are different ways of asking the same type of question, which is "What can be true?").
The back of the flash card for a question type will say how to test the answers, how to use
previous diagrams, and when you should do that type of question. For different cards that
mean the same thing, as I've just listed, the back of each will have the same stuff on it.
Why should you have several different cards for each question type? Why not just put all
the ways of asking the same question on the same card? Well, putting them on the same
card gives you a hint about what they mean you might not remember what "acceptable"
means, but you would know that it means "could be true" because they are on the same
card. This doesn't help you prepare for the test. The LSAT won't give you any hints, so if
you don't know what the question means just by looking at it, you aren't going to do too
well.
You should also have flash cards for important LSAT words like "exactly" or "at least." I'll
point out words like these as we go through the rest of this book.
Run through these flash cards regularly (mixing them up periodically). You should spend
more time on the cards you have a hard time remembering put them in a separate pile
and go through them more often but don't neglect the cards you remember easily. Every
so often, shuffle all the cards together, even the ones you know well, and go through the
pile. Your goal is to know every card without pausing to think the instant you look at the
card, you should know exactly what you are supposed to know. Having to pause to
remember slows you down on the test and forces you to think, and thinking makes you
tired.
If you take a look at the Big Fat Genius website (www.bigfatgenius.com), there should be a
set of flashcards available for you to download for free (I cant give you an exact address in
this book because nothing on the internet ever stays the same for long; look around, they
should be easy to find). These should give you a useful starting place for making your own
cards.
50 The Fundamentals
Statistics on Question Frequency
Here are some statistics on how often each type of question has appeared in LSAT 19
through 48 (the most recent LSAT as of the writing of this book). I consider the LSATs
before 19 not representative of modern logic games. I've broken the statistics down into
two groups: before 2000 and 2000 and after; the frequencies have changed slightly in
recent years. Notice, for example, that these days you are somewhat more likely to see the
more difficult questions types, like "What is the maximum/minimum?" or "How many
diagrams are possible?", although you still are not likely to see more than one of these on a
given LSAT.
I've given the following statistics: what percentage of the total questions are of each type,
what the largest number of that type on a single LSAT has been, what the smallest number
of questions of that type on a single LSAT has been, and the average number of that type of
question per LSAT. Percentages may not add up to 100 due to rounding. If the average is
less than one, this means that there is a significant chance that an LSAT will not have a
single question of this type on it.
One thing to point out is that each If-question is also another type of question the
question will say, for example, "If such and such, which of the following can be true?" Don't
try to add the percentages for If-questions with the other types of questions. If-questions
that change or remove rules are counted separately from regular If-questions.
Before 2000
"What can be true?" questions
Percentage: 45%
Largest number: 13
Smallest number: 8
Average: 10.75
"What can't be true?" questions
Percentage: 15%
Largest number: 5
Smallest number: 1
Average: 3.5
"What must be true?" questions
Percentage: 32%
Largest number: 10
Smallest number: 6
Average: 7.5
"What can be false?" questions
Percentage: 1.5%
Largest number: 1
Smallest number: 0
Average: .3
Since 2000 (2000 and later)
"What can be true?" questions
Percentage: 41%
Largest number: 14
Smallest number: 5
Average: 9
"What can't be true?" questions
Percentage: 21%
Largest number: 9
Smallest number: 3
Average: 5
"What must be true?" questions
Percentage: 24%
Largest number: 10
Smallest number: 3
Average: 5.5
"What can be false?" questions
Percentage: 2%
Largest number: 3
Smallest number: 0
Average: .6
The Fundamentals 51
Before 2000 (continued)
"What is a complete and accurate list?"
questions
Percentage: 4%
Largest number: 2
Smallest number: 0
Average: 1
"What is the maximum/minimum?"
questions
Percentage: 1.7%
Largest number: 2
Smallest number: 0
Average: .4
"How many diagrams are possible?"
questions
Percentage: 1%
Largest number: 1
Smallest number: 0
Average: .25
If-questions
Percentage: 52%
Largest number: 17
Smallest number: 9
Average: 12.4
If-questions that change or remove rules
Percentage: 2%
Largest number: 1
Smallest number: 0
Average: .5
Since 2000 (continued)
"What is a complete and accurate list?"
questions
Percentage: 4%
Largest number: 3
Smallest number: 0
Average: .9
"What is the maximum/minimum?"
questions
Percentage: 5%
Largest number: 4
Smallest number: 0
Average: 1
"How many diagrams are possible?"
questions
Percentage: 1%
Largest number: 3
Smallest number: 0
Average: .3
If-questions
Percentage: 50%
Largest number: 16
Smallest number: 8
Average: 11
If-questions that change or remove rules
Percentage: 2%
Largest number: 2
Smallest number: 0
Average: .4
52 Game One
Game omitted from electronic version of book.
Game One 53
Game One
(From LSAT 28)
Step 1: Read the Setup/Draw a Diagram
Every game involves two types of entities that you match together. What are the two types
of things in this game? They are the racehorses and the positions 1 through 6. The next
question we ask is, which is more definite? That is, which do we know more about? We
know more about the positions, because 1 is always next to 2, which is always next to 3,
and so forth. Thus, we are going to base our diagram around the positions. When you see
an LSAT game involving numbered positions, your diagram will almost always be based
around the numbers, as numbers are pretty definite (numbers don't move or change). We
write the numbers 1 to 6 out in a row, which is what we always do with the definite things.
Now we ask how many of the variable things will get matched up with each definite thing.
How many horses per position? The game says "one horse per position," so there's your
answer. Each position will get one blank associated with it. This is a very simple diagram:
1 2 3 4 5 6
__ __ __ __ __ __
Write down the names of the horses (K, L, M, N, O, P) somewhere off to the side so we can
remember them. We end up with something that looks like this:
1 2 3 4 5 6 K L M N O P
__ __ __ __ __ __
Now, here's a question you might be asking: where do you draw the diagram? You don't
get any scratch paper on the LSAT, so you've got to fit all your work in the blank space
provided for you. Put the diagram any place you want; don't write it so big that you don't
have any space for test diagrams, but don't write it so small that can't read it. Many people
prefer to put it at the very top of the page; I have no strong feelings either way.
Step 2: Read the Rules
Let's go through the rules one by one, making sure we understand them and writing them
down in a clear and complete manner. Ready? Let's rock!
First rule: "K and L must be assigned to positions that are separated from each other by
exactly one position."
What does it mean for K and L to be separated by exactly one position? It means that there
will always be one (and only one) space between them. The word exactly is an important
LSAT word; it means "this much and no more or less." Now here's the second question:
what does this rule not mean? Specifically, does it tell us who comes first, K or L? I ask
about who is first because order is very important in this game it is a game about putting
horses in order.
The rule doesn't explicitly say anything about the order of K and L, so either of them can
come first. For the rule to tell us explicitly, it would have to mention order. It could say "in
that order" or "L is before K" or something like that. So we know we have either K and a
space and then L, or L and a space and then K. These are the only two options. The
54 Game One
easiest way to write this rule is to write the options down. When a rule only gives you two
options, write both of them down. So here is our rule:
K __ L or L __ K
This is the best way to write down the rule, but it isn't the only way. You could have
written:
Exactly one space between K and L (or L and K)
This is much longer, but it communicates everything in a clear fashion. Memorize and try to
master my guidelines for writing the rules (so you can use them without having to think
about them much), but if you forget them on the test, or you can't figure out how they
apply to a certain rule, never forget this: it's better to write down a rule any old way than
to waste time trying to figure out the best way to write it.
Next rule: "K and N cannot be assigned to positions that are next to each other."
What does this mean? It means that K and N will never be next to each other; that is,
you'll never have them in consecutive positions. If, for example, K is in 1, N won't be in 2.
What does it not mean? Do we know what order they will come in? Well, the rule doesn't
say explicitly, so we don't know. They might be in either order.
There are many ways K and N could be situated. There might be one space between them,
or two, or three, and so on. But there are only two ways they can't be we can't have KN
and we can't have NK (when I write the horses next to each other I mean that they are in
consecutive spaces). So our rule is
No KN no NK
Of course, we didn't have to write this down so nicely.
K and N aren't next to each other
That works pretty well, too.
I don't mean to say that it's not good to write the rules down nicely. It is it's easy to read
(usually) and it means that you understand the rules. But it's not absolutely necessary. It's
supposed to save you time overall, so if it takes you a long time to figure out the best way
to write the rules that defeats the purpose of writing them. Do your best to learn how to
write rules well, but don't spend more time studying it than you do something really
important, such as memorizing how to do each type of question.
Third rule: "N must be assigned to a higher-numbered position than M."
This is an easy rule to misinterpret and to write down badly. First ask yourself what it
means. I'll give you a second to think. OK, here the answer: it means that the space N is
in has a higher number than the one M is in (but we don't know how much higher, because
it doesn't say). That means that N is further to the right and M further to the left, because
we write low to high from left to right. When you write this rule down, make sure that this
relationship is in what you write. Many people write something like N>M. This is very
confusing it has N on the left of M, not on the right. If you write something like this, you
are almost guaranteed to make a mistake (oh, how many times have I seen this!).
Game One 55
If a rule talks about a spatial relationship, make sure that the way you write the rule mimics
that relationship. That is, if a rule tells you that someone is to the left of someone else, you
should show that in the way you write it. If you are told that someone is above someone
else, make darn sure that they are above them on the page when you write it down.
Here is our rule as we write it down:
M before N
Last rule: "P must be assigned to position 3."
Not a hard rule to interpret. Put P directly in position 3. If a rule can be written directly
into your diagram, do so. But also write this with our other rules so that we don't forget it.
Cross P off the list so we know we don't have to worry about him. As things get assigned to
spaces, cross them off the list.
So now here is what you should have written down on your page:
1 2 3 4 5 6 K L M N O P
__ __ P __ __ __
1. K __ L or L __ K
2. No KN no NK
3. M before N
4. P is 3
You don't have to number your rules (really, there is absolutely no reason why you should).
I'm doing it so that I can refer you back to them quickly and easily as I work through the
questions.
Some of these rules can be combined to give you new information (deductions), but we
aren't looking for that sort of stuff at this stage in our LSAT life. Let's just focus on learning
the fundamental, and most important, skills.
Step 3: Answer the Questions
Here is where the magic happens. We are going to walk through the questions one by one.
If a question is one that we should do, we'll do it; otherwise we'll skip it and come back to it
later. It will all make sense as we go through.
Question 1: A "What can be true?" question do it now
The word "acceptable" means "can be true," and "What can be true?" questions are the first
ones we do. We are going to do this question now.
Notice that each answer fills out every space in the diagram, in order. This is going to make
the question pretty easy, because we don't need to make our own diagrams. We know that
the right answer can be true, so it follows the rules. This means that the wrong answers
break the rules.
When going through the rules to check whether something follows them or not, always go
through the rules in the order in which you have them written down. We do this because it
is easy and it ensures that we don't skip a rule. This means that we actually look at our
56 Game One
diagram first, because the diagram is the first thing we have written down. So we are going
to look at rule 4 (P is 3) before any other rule because it is written directly into our diagram.
Answer A
OK, P is third, so that's good. Next rule says K and L are one space apart, but here K is
next to L. That breaks a rule, so this is wrong. Cross it out and go on.
Answer B
P is third; good. K and L have P between them, so that's good. The next rule says that N is
not next to K, and it isn't. Finally, M must be before N, and it is. This follows all the rules,
which means what? It is correct.
Since this follows all the rules, it gives us a possible diagram. We'll want to use this later.
People often forget to look at the answers to this type of question when looking at previous
diagrams to see what they know can be possible. To avoid this, we are going to recopy
this answer ourselves, putting it into a diagram, so we'll recognize it as such.
1 2 3 4 5 6
M K P L N O
Question 2: A "Which is a complete and accurate list?" question skip for now
Question 3: A "What can't be true?" question skip for now
Question 4: A "What must be true?" question skip for now
Question 5: A "What can't be true?" question skip for now
Well, we just skipped all the questions. Did we do something wrong? No, the game just
gave us only one of the easiest kind of questions. Now we go back and do the "What can't
be true?" questions numbers three and five.
Question 3: A "What can't be true?" question do the second time through
We know that the wrong answers are things that are possible, since the right answer is
impossible (can't be true). We want to use our previous diagrams to show us quickly what
is possible. We only have one diagram, which we found in answer B on question 1.
Answer A
Look at our previous diagram. It shows us K in position 2, so we know that this is possible.
We cross off this answer.
Go through the rest of the answers in order. We can also eliminate answer C, since M is in
position 1 in the previous diagram. We can't eliminate any of the others, because the
situations they describe (e.g., L in position 2, or M in position 5) are not shown in our
previous diagram (e.g., the diagram has K in 2 and N in 5). We are going to have to test
out at least some of the rest of the answers.
Answer B
We have to test this and see if it is possible. So we draw a new diagram (copying our
original diagram, so putting P in 3) and put L in position 2. Also, cross L off the list because
we've placed it.
1 2 3 4 5 6 K L M N O P
__ L P __ __ __
Game One 57
Now we go through the rules one by one and see if anything has to be true. Rule 1 tells us
that we must either have K __ L or L __ K. We can't have K __ L because that would put K
off the chart (there is nothing to the left of position 1), so we have to have L __ K. Thus, K
goes 4
th
(cross K off).
1 2 3 4 5 6 K L M N O P
__ L P K __ __
The next rule says that N and K aren't next to each other. So N can't go in position 5.
Write that down next to position 5 (so it is easy to see). But we don't yet know where N
has to go (as far as we can see so far it can go in position 1 or 6) so we don't put N in
anyplace.
1 2 3 4 5 6 K L M N O P
__ L P K __ __
no N
The last rule says that M has to go before N. This is going to limit where M and N can go.
To see how, we look at each empty space in the diagram, one at a time. M can go 1
st
or 5
th
,
but not 6
th
if it were 6
th
, N would have to be 7
th
, and there is no 7
th
. N can't go 1
st
,
because then M would go off the chart. It can't go 5
th
, because we wrote down "no N"
under 5, but it could go 6
th
. N has to go 6
th
that's the only place it is able to go, and it
has to go somewhere. If you aren't sure what a rule tells you, look at each person in the
rule and each empty spot in the diagram. Earlier, rule 2 told us that N can't be next to K.
Since we knew were K was, it was clear that this told us only that N couldn't be 5. The rule
we just looked at, though (M before N), doesn't clearly tell us anything, since we don't yet
know where either M or N goes. But it seems like it should tell us something, so we try it
out, trying both M and N in each empty space. And this gives us new information, that N
has to be 6.
1 2 3 4 5 6 K L M N O P
__ L P K __ N
no N
All that are left are M and O. They can go in either 1 or 5 (we already tried M in both
spaces, and we have no rule about O). We can either write "M/O" in either space to show
that they can go either way, or just put them in someplace. Let's just put them in, in
alphabetical order to be systematic.
1 2 3 4 5 6 K L M N O P
M L P K O N
no N
This works. We've filled every space without breaking any rules. So it is possible for L to
be in position 2. This may seem like it took a long time, but after you practice this a bit, it
will become second nature and you'll soon be whipping right through it. And now we have
an additional diagram to help us out in later questions.
Use this new diagram to see if we can eliminate our remaining two answers (D and E).
Does the situation in D or E appear in this diagram? Unfortunately not; D talks about M
being 5, but here M is 1, and E talks about O in 2, but here O is 5.
We've already eliminated answer C based on the diagram for question 1, so go to answer D.
58 Game One
A quick point. In this book, as we test one answer, I redraw the diagram every time I add
something new. I'm not saying you should do that; I'm doing it to show you how the
diagram evolves. In these cases, where you are adding something to a diagram you
already have, just add the new information to the same diagram. Draw a new diagram to
test a new answer or a new question.
Answer D
Let's try this; recopy your diagram and put M 5
th
.
4
1 2 3 4 5 6 K L M N O P
__ __ P __ M __
The first rule says that we have to have K __ L or L __ K. Where can we put in L and K?
When you don't know where someone has to go, but you have to put them someplace
because you are drawing a test diagram, put them in the first available space. But you
want to remember that they didn't have to go there putting them there might make the
diagram break a rule later on, and you might want to move them someplace better. So put
a little dot or star next to them; this will tell you that their position was your choice. When
you choose where to put someone, mark them with a dot or star (see Making a Test
Diagram in the chapter The Fundamentals).
Here, we want to put K and L in the first available spaces. Position 1 is first, but if we put K
or L there, the other would have to be 3, so this won't work. You can put one of them in 2,
because spot 4 is empty. Put K there (since it is before L in the alphabet); however, it may
turn out that this breaks a rule, so mark it to remind us that this was our option. L,
however, will now have to go 4
th
because of the rules, so we don't mark L.
1 2 3 4 5 6 K L M N O P
__ K* P L M __
The next rule says that N can't be next to K. Write "no N" under position 1. There is only
one place left for N to go, space 6, so put N there. Because N had to go 6
th
, we don't mark
it.
1 2 3 4 5 6 K L M N O P
__ K* P L M N
no N
The next rule tells us that M is before N. This agrees with what we have here, which is
good. This leaves O 1
st
. There is no rule against this, so put it in.
1 2 3 4 5 6 K L M N O P
O_ K* P L M N
no N
We've filled every space without breaking a rule, so this is possible. We can cross off
answer D, so the answer has to be E. Don't bother checking it, we know it has to be the
answer.

4
When we did B, we could have put M 5
th
. If you remembered this, then you would just
cross off this answer and be done with it. If not, you'd have to test it out.
Game One 59
This wasn't too hard. We only had to test out two answers, and they were pretty
straightforward. Notice how systematic we were we went through the rules in order, we
put people in the first available spaces, in alphabetical order. Don't be afraid to put people
wherever it seems they can go. Sometimes you'll have to move them later on, but this is
no big deal.
Question 5: A "What can't be true?" question do the second time through
Let's use our previous diagrams, because we can do that on "What can't be true?"
questions. If we see that what an answer describes occurs in a previous diagram, we know
it is possible, so it is wrong the right answer can't be true.
Answer A
Notice that this answer doesn't specify an order. If we have LN or NL we know that this is
possible. Check your previous diagrams first. We have the diagrams from answer 1B and
from answers 3B and 3D. The diagram for 1B has L next to N, so we know that this is
possible. Cross off the answer and move on.
Answer B
The diagram for 1B has M and K next to each other, so this is possible. Cross it off.
Answer C
We don't have M and O next to each other in any previous diagram, so skip this.
Remember, if you are using your previous diagrams, skip answers that you can't eliminate
with the diagrams. Hopefully you will be able to use your previous diagrams to eliminate all
the other answers and won't have to test the answer you skipped.
Answer D
What does this mean? It means that there is one space between L and N, but no order is
specified. We could have L __ N or N __ L. We see this in the diagram for 3D L is 4
th
and
N 6
th
. O is in the only position between them. So we know that this is possible and we can
cross off this answer.
You might be wondering, what would we do if our previous diagrams weren't so helpful?
Well, we'd have to do some more work we'd have to test these answers. That's not so
bad. In most games, our previous diagrams should do some work for us because it's very
difficult to write 20 or more answers that are wholly unique.
Answer E
This is similar to D, except we are looking for M __ P or P __ M. Look for it in your previous
diagrams and there it is! In the diagram for 1B (as well as in 3B and 3D). We've
eliminated every answer except C, so C is the answer.
Now do you see why we skip answers we can't use our previous diagrams on?
We've made our second pass through the game. Now we do the third-easiest type of
question the "What must be true?" questions.
60 Game One
Question 4: A "What must be true?" question do the 3
rd
time through
The answer to a "What must be true?" question must be true in every possible diagram. So
if an answer is false in a possible diagram, it is wrong. When we look at our previous
diagrams, we look to see if we have a diagram where something other than what the
answer describes happens.
Answer A
"Either" means K could be 2
nd
or L could be 2
nd
. We want something other than this, so we
look for a diagram where K and L are not in position 2. Someone else maybe M, maybe
O, maybe N will be there. But you don't see it. 1B has K 2
nd
, as does 3D. 3B has L there.
Skip this answer.
Answer B
Again, look for a diagram where someone other than K or L is in position 4. We don't see it.
Skip this answer too.
Answer C
Look for a diagram where someone other than M or N is 2
nd
. All of our previous diagrams
have someone other than M or N in position 2 (they all have K or L 2
nd
). So M or N doesn't
have to be 2
nd
. Cross this off.
Answer D
The diagram for 3B has O in position 5, so M or N doesn't have to be there. Cross this off.
Answer E
3B and 3D have N 6
th
, so M or O doesn't have to be there. Cross this off.
We still have A and B left. Let's test A out.
Answer A
To test the answers on a "What must be true?" question, we make a test diagram doing
something different from what the answer describes. We don't want to put K or L in
position 2. Write this in your diagram.
1 2 3 4 5 6 K L M N O P
__ __ P __ __ __
no K
no L
Now go through the rules and see what has to be true. We know from rule 1 that K and L
have to be one space apart. Let's put them in the first available space. Figure out what this
is on your own before looking further.
OK, done? Here's what you should have seen. Neither can go 1
st
, because the other would
have to go 3
rd
, and P is already there. Neither can go 2
nd
, because that would defeat the
purpose of what we are doing: we want to see if we can avoid putting them 2
nd
. We can
put one 4
th
, and the other would have to go 6
th
. Notice, by the way, that neither can go 5
th
,
because the other would have to go 3
rd
or 7
th
. So they have to go 4
th
and 6
th
that's the
only way they can fit. Put them in, in alphabetical order. Since the order is our choice
(although not the position), mark K with a dot or star (only K, because once K is in, L has to
be 6). If the order ends up mattering, you can always switch them later.
Game One 61
1 2 3 4 5 6 K L M N O P
__ __ P K* __ L
no K
no L
The next rule says that N can't be next to K. So N can't be 5
th
.
1 2 3 4 5 6 K L M N O P
__ __ P K* __ L
no K no N
no L
The next rule says that M has to be before N. See where M can go. It can go 1
st
, because
then N would go 2
nd
. Put it there and mark it, because we haven't figured out that it has to
go there. (By the way, M can't go 2
nd
, because then N would have to go 5
th
, but N can't be
5
th
. So M has to be 1
st
).
N has to be 2
nd
because it can't be 5
th
. This leaves O for 5
th
.
1 2 3 4 5 6 K L M N O P
M* N P K* O L
no K no N
no L
This works. That means that the answer is wrong remember that when you test
something out in a "What must be true?" question and you make the diagram work, the
answer is wrong. That's because you tested out something other than what the answer
describes. If the diagram works, it means that the scenario in the answer doesn't have to
be true; something else (that you just tested) could be true. Here, we've just learned that
we don't have to have K or L 2
nd
. Notice how important it is to memorize the procedure to
answer the question. We just did a lot of reverse thinking we tested out something
different from what we were told, and we crossed off the answer because the diagram
works. That's weird, and it can be confusing if you don't have it thoroughly memorized. I
beg you, memorize this. Please
Anyway, we've eliminated every answer except B, so B is the right answer.
I want to point something out. Notice that once we write something down like "no N" under
position 5, we don't question this later on. Imagine the following internal dialogue:
"Well, hmmm, can M go 2
nd
? Well, if it went 2
nd
, then N would be 5
th
. Can N be 5
th
?
I don't know. I wrote down that it can't, but why did I do that? Let's work it out."
What a massive waste of time and effort. Once you write something down you have to trust
that it is correct. Don't question what you write down later on.
There's only one question left.
62 Game One
Question 2: A "Which is a complete and accurate list question" do it our fourth
time through
The question asks about K. The right answer has to include every place K could possibly be.
We have diagrams with K 2
nd
and 4
th
. We can eliminate any answer without 2 or 4 in it.
That gets rid of A and B. Now, to eliminate D, we wonder if K can be 5
th
. Try this out.
1 2 3 4 5 6 K L M N O P
__ __ P __ K __
Our first rule tells us that we have either K __ L or L __K. If we have K __ L, L is 7
th
, which
won't work. If we have L __ K, L is 3
rd
, which also won't work. So we can't have K 5
th
. Any
answer with 5 is not an accurate list, so it's wrong. Cross off answer D, and cross off this
diagram. Always cross off diagrams that break rules so that you don't try to use them later.
I know that this is the last question of the game, and that there won't be any "later" for this
diagram, but always be consistent. That's the only way to build good habits.
Now let's try K 6
th
.
1 2 3 4 5 6 K L M N O P
__ __ P __ __ K
Our first rule tells us that L will have to be 4
th
(so no dot or star for L)
1 2 3 4 5 6 K L M N O P
__ __ P L __ K
Our next rule tells us that N can't be 5
th
.
1 2 3 4 5 6 K L M N O P
__ __ P L __ K
no N
Our next rule tells us that M is before N. Let's see where they can go. M can go 1
st
, which
would put N 2
nd
. (M can't go 2
nd
, because that would put N 5
th
) O has to be 5
th
.
1 2 3 4 5 6 K L M N O P
M N P L O K
no N
This works, so K can go 6
th
, which means that the answer has to have 6 in it. Only E has 6,
so E is the right answer. And we are done.
5
Notice that if we were running out of time we could make a really quick guess at this
answer. It's pretty easy to see that A and B are wrong, and it looks like L and K can go 4
th
and 6
th
, so E is pretty likely the answer.

5
Some of you may have noticed that we could just switch K and L in the diagram for 4A and
see that K can go 6
th
. This is true, but it doesn't always work. I'll talk about when you can
and can't do this sort of thing in the Advanced Techniques section. For now, if you don't
see something in a previous diagram, use a new diagram to test it out (unless you are really
pressed for time; then just make the best guess you can).
Game One 63
That's It Summary
Well, congratulations. You've either done your first game, or you've done your first game
the right way. Let's see what we've learned:
! Be methodical in looking at the rules: start with the first rule and go through the
rest one by one
! When testing answers, put in people where you can as rules about them come up
! When testing answers, fill in any empty spaces after looking at all the rules
! Doing the questions in the right order sure makes things easier
! Always try things out on paper it is easier, and it gives you diagrams to look at
later on
At this point there are a couple of things we haven't seen. We haven't seen any "If
then" rules, and we haven't seen any If-questions. We'll see both in the next game,
though.
What Do I Do Next?
What should you do before you go on? Well, at the very least you should go through the
notes you took as you worked through this game. What? You didn't take any notes? Well,
go back through this game and write down everything that seems useful or important.
Then go through those notes and think about what they mean and how it all fits together. I
want you to visualize working through a game or two; imagine yourself going through these
steps. This will allow you to practice these steps without having to do any questions.
This game should have helped you to understand a lot of what we covered in The
Fundamentals. At this point you should be a bit tired. Take a break. Think about what
you have learned and how it all relates. Then work through this game on your own and try
to apply what you have learned here (although feel free to use your general notes on the
steps and so forth just don't look at the answers until you are done).
Once you have done that, you are in a position to think about what you need to start
memorizing. Go back and read What Do I Do Next? at the end of the chapter The
Fundamentals. There I talk about what you'll need to work on memorizing, and how to do
so. Put together some flash cards. You don't need to memorize them right now, but go
through them for a few minutes every day as you practice logic games. Your goal is to have
memorized them by the time you get to Game Four through Game Eight.
Once you've made up a list of things you need to memorize (and hopefully made some flash
cards), proceed to the next chapter, "If then" Rules and Negations.
Answer Key
1. B
2. E
3. E
4. B
5. C
64 "If then" Rules and Negations
"If then" Rules and Negations
"If then" Rules
"If then..." rules generally, but not always, are sentences containing the words "if" and
"then". For example,
If a creature is a human being, then they have a heart and kidney(s).
Let's take a look at this sentence. First of all, it is true human beings have hearts and at
least one kidney (assuming that they are alive, but let's ignore the philosophical question of
whether or not a corpse is a human being). So any time I meet a human being, I know
(without surgery) that they have a heart and kidney(s). However, there are plenty of
animals who have hearts and kidneys but aren't humans. For example, my dog has a heart
and two good kidneys, and, while she thinks she's a human and tries to sit on the couch,
she really isn't.
What we've just seen with this "If then" sentence can be generalized to all "If then"
sentences. Here, then, is the first thing we've learned about "If then" rules: when the
"if" part is true about a thing or situation, the "then" part has to be true as well. If I say
that it is true that so and so is a person, then it has to be true that they have a heart and
kidneys. In other words, the truth of the "if" part guarantees the truth of the "then" part
(or, as philosophers say, the truth of the "if" is sufficient to show the truth of the "then").
However, the truth of the "then" part doesn't tell you anything for sure about a given
person or situation. If I know that it is true that Edward has a heart and kidneys, I don't
know that it is true that Edward is a human (in fact, he is my cat).
Let's look at another example:
If Fred eats ice cream, then he is happy.
Let's say you knew that Fred was eating ice cream. What else would you know? Right,
you'd know he was happy. What if you were only told that Fred was happy? Would you
know that was eating ice cream? No, because "is happy" is in the "then" part of this
sentence, and knowing that the "then" part is true doesn't tell you anything for sure.
The contrapositive
Let's see what else we know. Sticking to the first example I gave, "If a creature is a human
being, then they have a heart and kidneys," let's say I know that Mothra doesn't have a
heart or kidneys. Can Mothra be a human? Well, if Mothra was a human, she'd have to
have a heart and kidneys, right? But she doesn't have them, so she can't be a human. So
we can say,
If a creature doesn't have a heart or kidneys, then it isn't a human.
This is always true about "If then" sentences; if the "then" part isn't true about a person
or situation, the "if" part can't be true either. From this, we arrive at the idea of the
contrapositive. The contrapositive is a way of rewriting an "If then" sentence. Let's
take a look at the form this will take; here's a general example of an "If then" sentence:
"If then" Rules and Negations 65
If something is X, then it is Y.
This means that if something isn't Y, it can't be X, because if it were X, it would have to be
Y.
If something isn't Y, then it isn't X.
Remember that the "if" part being true guarantees that the "then" part is true. Whenever
something is X, we know immediately that it has to be Y. We can say that the "then" part
being true is a requirement for the "if" part to be true (another word for "requirement" is
"necessity"). This means that the "if" part can't be true without the "then" part being true;
for example, something can't be X without also being Y. But the "then" part might only be
one of many requirements, so knowing that it is true doesn't tell us that the "if" part is true.
We might have something that is Y, but not X, but we can't have something that isn't Y and
is X.
Let's look at another example and see how it works like these X and Y examples:
If you have read The Brothers Karamazov, then you think Dostoevsky is a genius.
If X, then Y.
If you don't think Dostoevsky is a genius, then you haven't read The Brothers
Karamazov.
If not Y, then not X.
These two sentences are the contrapositives of each other. Notice that the two sentences
are structurally the same as the examples with X and Y that is, the second sentence tells
you that if the "then" part (the Y) isn't true, the "if" part (the X) can't be true.
The contrapositive of an "If then" sentence is a way of expressing the fact that if the
"then" isn't true, the "if" can't be true. There is a simple rule for making the contrapositive
out of an "If then" sentence. To make the contrapositive, do two things: 1) make the
"if" part into a "then," and make the "then" into an "if"; and 2) make both sides negative.
If one side is already negative, make it positive (two negatives make a positive). Look at
the Dostoevsky example and see that this is what I did there.
Take the following example with negatives. The contrapositive of
If something is A, then it is not B.
is
If something is B, then it is not A.
We switched the "if" and the "then" and made both negative; but since the "then" is already
negative, it becomes positive. This works because we always know that when the "then"
part isn't true (when it is negative) then the "if" part can't be true (so it will be negative as
well).
The contrapositive of every "If then" sentence is true, and it is useful. So every time you
have an "If then" rule, always write down the contrapositive of the rule.
66 "If then" Rules and Negations
Some examples
OK, here is a true sentence (I'll put Xs and Ys below the clauses to show how this sentence
relates to the more abstract examples above):
If someone lives in Los Angeles, then they live in California.
If X, then Y.
Imagine that I have a friend who lives in California. Do you know that this friend lives in
Los Angeles? You don't know they may or may not. Now imagine that I have a friend
who doesn't live in California (let's say they live in Oklahoma). Do they live in Los Angeles?
Of course not; they haven't met the requirement for living in Los Angeles.
The contrapositive of the sentence is
If someone doesn't live in California, then they don't live in Los Angeles.
If not Y, then not X.
Second example:
If Mary goes to the dry cleaner, then Bob will eat a hamburger.
If X, then Y.
Here we can't rely on our intuitions. We have to apply the rules. Now, what happens every
time Mary goes to the dry cleaner? That's right, Bob eats a hamburger. What if we know
that Bob eats a hamburger? Do we also know that Mary went to the dry cleaner? No, we
don't since Bob eating a hamburger is on the "then" side, it tells us nothing about Mary.
What's the contrapositive of this sentence?
If Bob doesn't eat a hamburger, then Mary doesn't go to the dry cleaner.
If not Y, then not X.
Now imagine that we know Bob doesn't eat a hamburger. What does this tell us? It tells us
that Mary doesn't go to the dry cleaner. What if we know that Mary doesn't go to the dry
cleaner? This tell us nothing, because it is on the "then" side.
Now let's look at an example with negatives:
If I ate lunch, I will not order pizza for dinner.
If A, then no B.
Notice that the "then" part has a negative in it ("will not order pizza"); this negative will
become a positive when we form the contrapositive.
If I order pizza for dinner, then I didn't eat lunch.
If B, then not A.
What does this mean? If I ate lunch, what do you know? You know that I won't order
pizza. What if I order a pizza? Then I didn't eat lunch. What if I didn't eat lunch? Well, I
may or may not order a pizza; knowing that the "then" part ("didn't eat lunch") is true
doesn't tell you anything about the "if" part.
"If then" Rules and Negations 67
A sentence with negatives in the "then" part, or in both the "if" and "then" parts, follows the
same rules. For example, these two sentences are contrapositives of each other:
If you don't like to laugh, then you won't like Happy Gilmore.
If you like Happy Gilmore, then you like to laugh.
To summarize: whenever the "if" part is true, the "then" part is true too. But when the
"then" part is true, we don't know anything about the "if" part.
"If then" Equivalents
There are many ways the LSAT will state "If then" rules. For example:
All kittens are cute.
is really an "If then" sentence. It means
If something is a kitten, then it is cute.
When you see something that means "If then", rewrite it as an "If then" sentence; we
know how to deal with "If then" sentences, and we know how to form their
contrapositives, so we like the format.
Here are some common things you will see on the test and how to rewrite them as "If
then" sentences; memorize these sentences and how they are equivalent to "If then".
Each of these sentences means "If something is an X, then it is a Y."
All Xs are Ys.
Any X is a Y.
Every X is a Y.
When something is an X, it is Y.
Nothing is an X unless it is a Y.
No one can be an X without being a Y.
Only Ys are Xs.
Something can be an X only if it is a Y.
To illustrate how to use this list, let's look at the sentence "Only mammals are cows."
"Mammal" is the Y in the "only" version of the sentence, and "cow" is the X. Each of the
sentences on the list means "If X, then Y"; we replace X with "cow" and Y with "mammal"
and get "If something is a cow, then it is a mammal."
X and Y can be replaced with anything in these sentences; for example, "Something is a
monkey only if it is mean," means "If something is a monkey, then it is mean." Remember,
this tells you that all monkeys are mean, but something can be mean and not be a monkey
(because "mean" is the "then" part).
One way of understanding why all these are equivalent is to think about guarantees and
requirements. If I tell you that "All monkeys are mean," this tells you that being a monkey
guarantees that something is mean (because all monkeys are mean). But being mean is a
requirement you can't be a monkey without it. Since it is a requirement, it might not be
the only requirement; just because you are mean doesn't mean you have to be a monkey,
because you might not have met the other requirements to be a monkey (like having a tail).
68 "If then" Rules and Negations
So "monkey" is the "if" part and "is mean" is the "then". Likewise, if I say that "Only
Americans are Californians," this means that being an American is a requirement for being a
Californian. However, it isn't a guarantee you can be an American without being a
Californian.
People often have trouble remembering how to translate "unless" into "If then" Here are
some tips for dealing with these sentences:
"Unless"
An "unless" sentence has to have two parts. Take the following example: "You can't dance
well unless you have the right shoes." (It's true have you ever tried to dance in combat
boots? It's tough, real tough.) The two parts are "can't dance well" and "have the right
shoes." One of the two parts will become the "if" and the other the "then" of an "If then"
rule. Whenever you have a sentence with "unless" in it, replace "unless" with "if not," and
put a "then" in front of the other part.
Applying this to our example "You cant dance well unless you have the right shoes," we
get:
Then you can't dance well if not you have the right shoes.
Sounds weird, but we'll fix it. Change the order to put the "if" first (this is perfectly
acceptable, and something you commonly have to do with "unless" sentences):
If not you have the right shoes, then you can't dance well.
Now, what does "if not you have the right shoes" mean? It means "if you don't have the
right shoes" that's how we'd say this in English. Putting "not" in front of a word or phrase
negates it; later in this chapter well talk about how to negate some common LSAT words
and phrases. Now we have
If you don't have the right shoes, then you can't dance well.
and its contrapositive:
If you can dance well, then you have the right shoes.
Perfect. To summarize, "unless" means "If _______ is not the case" After youve made
this substitution, you may have to move the "if" part of the sentence to the beginning.
I recommend that you don't rely on your intuition in interpreting "If then" rules.
Memorize these equivalencies (but use the idea of guarantee and requirement, or necessity
and sufficiency, to help you understand and memorize them) because logic games will give
you situations that you have no intuition about, such as "Sally wears a hat only if Jane
doesn't."
"If then" Rules and Negations 69
Complex "If then" Rules
Sometimes "If then" rules will be more complex than this. The "if" part or the "then"
part will contain several pieces. For example,
If you drive too fast or run a red light, then you will get a ticket.
Complex "If then" rules should generally be split into two "If then" rules. For
example, in this case we know that both of these are true:
If you drive too fast, then you will get a ticket.
If you run a red light, then you will get a ticket.
Splitting these rules into two rules makes them easier to think about and much easier to do
the contrapositives for. Sometimes (but very rarely) these rules can't be split. For
example,
If you drive 35 mph and you are on the freeway, then you will get a ticket.
Driving 35 mph won't guarantee a ticket by itself, and being on the freeway won't
guarantee a ticket by itself you need to do both together. So we can't split this into two
rules. More often than not, you can split a complex rule into two rules. I am about to give
you a list of times when you can and can't split the rules. This should be low priority on
your list of things to memorize generally, just split the rules unless that clearly doesn't
make sense but if you find that you get everything else under control and still have time
and space left in your brain, go ahead and memorize these.
If X or Y split
If X and Y don't split
If X then Y and Z split (If X then Y, If X then Z)
If X then Y or Z don't split
Here are the contrapositives of the two you can't split:
If X and Y, then Z If no Z, then not both X and Y (which means one or the other of
X and Y won't be true)
If X, then Y or Z If no Y and no Z, then no X.
See why we split when we can? It makes the contrapositives sooooo much simpler. I'll
explain more about what these complex contrapositives mean below, in Negations.
To summarize, when you have an "If then" rule with more than one thing in the "if" or
"then" part, you generally can (and should) split it into two rules.
Negations
When we form the contrapositive, we have to make both sides of the contrapositive
negative. We also need to know how to make things negative for "What must be true?"
questions. On these questions, we test answers by seeing if a situation that is different
from what the answer describes can work. To be different from the answer, a situation has
to be the negation of the answer. There are certain terms and phrases on the LSAT that
70 "If then" Rules and Negations
people often have a difficult time making negative. It's important that you understand how
to do this, so let's spend some time going over it.
We can define the negative of a sentence as anything that would make the sentence false.
"Not A" refers to any situation other than A, because "A" can only be true if it is true, not if
something different from it is true. So if the sentence says "X is 1
st
," then X being 2
nd
, 3
rd
,
4
th
, etc. would be the negation of the sentence, because in these cases it is false that X is
1
st
. Sentences, or answers, usually have more than one negation a whole range of
situations will negate a single sentence. Let's go through the types of sentences you will
have to negate on the LSAT; you don't have to memorize all of these right now, but you
should at least understand the type of thinking that underlies them. Later on, when you
practice games, you will have to make negations like these. If you find that you have a
hard time doing it, refer back here and then start to memorize.
Negating negatives: (For example, "X is not large.") This is the easiest thing to do; the
negation of a negative becomes a positive. So the negation of the example sentence would
be "X is large."
Negating positions: (For example, "X is 1
st
," or "X is in group A.") Some sentences talk
about X being in one specific position. The negation of these sorts of sentences is having X
not in the position described. Any situation where X is in a position or group other than the
one described would be a negation of this type of sentence. Here's how it works in an
example "If then" sentence:
If X is 1
st
, then Y is 2
nd
.
If Y is not 2
nd
, then X is not 1
st
.
Here we have an "If then" sentence and its contrapositive. What does the contrapositive
mean? Well, if you see Y in 1
st
, 3
rd
, 4
th
, and so forth, then you know X isn't 1
st
; it has to be
2
nd
or 3
rd
, etc.
Now let's look at "What must be true?" questions. We test answers on these by looking for
diagrams where the negation of the answer is true. If the answer says "X is 1
st
," then you
want to look for a diagram where X is not 1
st
. This means that you are looking for a
diagram where X is 2
nd
, 3
rd
, or 4
th
any position but 1
st
. If you need to test the answer,
then put "not X" under position 1 in your test diagram. Then go through the rules and see if
you can fill all the spaces in the diagram without breaking a rule.
Negating "before," "after," and "at the same time": (For example, "X is before Y.")
Most games involving "before" and "after" don't allow things to be at the same time (like
Game One), but some do. If a game doesn't allow things to be at the same time, ignore
where I have "At the same time" in what follows.
The negations of "before" are "after" and "at the same time." If X is after or at the same
time as Y, you know X is not before Y. The negations of "after" are "before" and "at the
same time." And the negations of "at the same time" are "before" and "after." Let's look at
an "If then" sentence and its contrapositive:
If A is before B, then B is before C.
If B is not before C, then A is not before B.
"If then" Rules and Negations 71
What do these sentences tell you? Well, if you see B after (or at the same time as) C, then
B is not before C. This means that A can't be before B, so A has to be after (or at the same
time as) B.
Now let's see how this would work on a "What must be true?" question. Imagine the
following was an answer to a "What must be true?" question:
(B) G is before E
To test this out, we want G to be not before E. So look for previous diagrams where G is
after E (or at the same time as E); or, draw a diagram in which you put G after E.
Negating "more than," "fewer than," and "exactly": (For example, "Group X has
fewer members than does group Y.") This is very similar to before/after/at the same time,
except "exactly" is a very important LSAT term and gets used all the time, whereas "at the
same time" isn't and doesn't.
The negations of "more than" are "fewer than" and "exactly"; the negations of "fewer than"
are "more than" and "exactly", and the negations of "exactly" are "more than" and "fewer
than." This means that if something is not more than three, it is either less than three or
equal to three. If something is not exactly three, then it is more or less than three. Let's
see how this works with contrapositives:
If group A has more members than group B, then B has exactly as many members
as C.
If B doesn't have exactly as many members as C, then A doesn't have more than B.
These are real mouthfuls. Let's see what the second sentence means. If you see that B has
either more or fewer members than C for example, B has 2 and C has 3, or B has 4 and C
has 2 then you know that A has to have either the same number or fewer members than
B.
If you saw one of these terms in a "What must be true?" answer, you'd test it by looking for
a diagram which has the opposite. If the answer said
(D) A has more members than B.
you'd look for a previous diagram where A has fewer members than B, or the same number.
To test it out in a diagram, you'd give B fewer or the same number of members as A. For
an example of this, see Game Two, question 11.
Negating "only": (For example, "Only X is in group B," or "X is the only red car.")
Sentences like this mean two things. First, they mean that X has the property described
i.e., being in group B, or being red. Second, they mean that no one else has this property.
In the examples I just gave in parentheses, group B has one member only X and in the
second example every car other than X is a color other than red. Since "only" sentences
require two things to be true, this tells us that we can negate these sentences in two ways.
First, we could make X not have the property described X could be in group C, or it could
be blue. Second, we could give other people that same property. We could put more than
one person in group B, or make another car red.
72 "If then" Rules and Negations
Let's see this with contrapositives:
If only X is red, then only Y is green.
If not only Y is green, then not only X is red.
What would tell us that "not only Y is green," as we see in the contrapositive? Well, if Y
were not green (say, if it were red), or if someone else (say, Z) was green as well, then you
would know that not only Y was green. If this were the case, we'd know that not only X is
red. Unfortunately, that isn't terribly informative, because there are two ways "not only X is
red" is true: X might be red, but someone else is red as well, or X might not be red at all.
Here's a little bonus hint regarding the above. Let's say you knew that Z was green, but
you didn't know anything about Y. What would this tell you? Well, it would tell you that not
only Y was green. See, if Y ends up being green, then both Y and Z are green, so it's not
only Y. And if Y ends up not being green, then not only Y is green.
Now let's see how this works on a "What must be true?" question. Here's our sample
answer:
(A) Only X is red.
To test this, look at previous diagrams. If you see one where X is not red (where it is
green, or orange, or blue, etc.), then you know that not only X is red. If you see one where
someone else is red, say Y or Z or Q, then you know that not only X is red. In either case,
A doesn't have to be true. To test this out by making a diagram, you'd either make X not
red (so make him green or orange, etc.) or you'd make X red and someone else as well.
Negating "either/or": (For example, "Either X or Y is blah blah.") Whatever fills out the
blah blah is going to be one of the things described above; e.g., "Either X or Y is 1
st
," or
"Either X or Y is before Z" or "Either X or Y has fewer members than Z," or "Either X or Y is
the only member of A." To be able to negate an either/or sentence, then, you have to be
able to negate the type of sentence that fills out the blah blah you have to understand
"before," or "fewer than," or "only," and so forth. But, of course, that's not all.
For an "either/or" sentence to be true, one or both of the people referred to have to have
the property referred to. If it says, "Either X or Y is 1
st
," then if X is 1
st
, it's true, and if Y is
1
st
, it's true (and if they are both 1
st
? It's true!). The only way for this type of sentence to
be false is for neither person to have the property in question.
For example,
If either X or Y is first, then either Z or W is fourth.
If not either Z or W is fourth, then not either X or Y is first.
You should notice that these are the sorts of "If then" sentences I told you above to split
up into smaller sentences. I don't split them up here because I want to show how
"either/or" is supposed to work, but you should do so on the LSAT. Since the "either" part
is true if X is first or Y is first, you can split the sentences into two rules, one about X and
one about Y:
If X is first, then either Z or W is fourth.
If Y is first, then either Z or W is fourth.
"If then" Rules and Negations 73
Now the contrapositives are a little easier to make:
If not either Z or W is fourth, then X is not first.
If not either Z or W is fourth, then Y is not first.
Of course, the question still stands: "What does 'not either Z or W is fourth' mean?" I'll
answer this question just below.
How do we know that not either Z or W is fourth? Simple we know it if neither of them is
fourth. We know this if both of them are someplace else, for example, if Z is 5
th
and W 3
rd
.
And if that's the case, we know that neither X nor Y can be first they both have to be
someplace else.
Let's see how this works with "What must be true?" questions. Take a look at this sample
answer:
(C) Either A or B is before C.
If you see a previous diagram where neither A nor B is before C, you know that this doesn't
have to be true. So look for diagrams where both A and B come after C. To test this out
with your own diagram, you'd have to put both A and B after C. You'd try putting C first,
and A and B somewhere after it; if that didn't work, you'd move C to spot 2, and so forth.
(See Game One, question 4.)
By the way, the negation of "neither/nor" is "either/or." A "neither/nor" sentence is false if
one of the entities mentioned has the property mentioned. That sounds very abstract, so
here's an example answer to a "What must be true?" question:
(A) Neither X nor Y is happy.
X being happy is a negation of this answer. Y being happy is also a negation (and both of
them being happy is a negation as well). If you saw a previous diagram where X was
happy, you would know that this answer doesn't have to be true. To test this out with your
own diagram, you would make one of the two happy. If the one you picked didn't work,
you'd try the other one. (See Game Three, question 8, answers C and D.)
The LSAT can say "or" and mean "either or" For example, "A or B is before C" means
"Either A or B is before C."
Negating "both and "and": (For example, "Both E and F are red-haired." "X and Y and
Z are painters.") A statement with "both" is only true if both parts of it are true; "and" is
just like "both," but with potentially more parts. Here, E would have to be red-haired and
so would F. If either of them was not, then the statement would not be true; so if E was
blond, or if F was a brunette, the statement would not be true. The negation of "both" is
"not both," and it means that one or the other part isn't as described. Take the following
"If then" sentence and its contrapositive as an example:
If both E and F are red-haired, then both J and H are blondes.
If not both J and H are blondes, then not both E and F are red-haired.
You could break these rules down into simpler ones if you wanted. With the "if" part, you
know that both E and F have to be redheads for the rest of the rule to be relevant. This
means you can't break up this part of the rule having only E be a redhead doesn't tell you
anything, nor does having just F be red-haired you have to have both of them. But you
can break up the "then" part. If both E and F are red haired, J has to be blonde. So does
H. You can make this into two rules, one with the "then" about J and one with the "then"
about H:
74 "If then" Rules and Negations
anything, nor does having just F be red-haired you have to have both of them. But you
can break up the "then" part. If both E and F are red haired, J has to be blonde. So does
H. You can make this into two rules, one with the "then" about J and one with the "then"
about H:
If both E and F are red-haired, then J is blonde.
If both E and F are red-haired, then H is blonde.
Now the contrapositive is nice and simple.
If J is not blonde, then not both E and F are red-haired.
If H is not blonde, then not both E and F are red-haired.
Breaking up "both," "either," "and" and "or" rules makes these sorts of sentences, or their
contrapositives, much simpler to think about.
What does "not both J and H are blonde" mean? It means that one of them, or both, is not
blonde they have some other hair color. So if you see J is a redhead, or H is bald, you
know that not both of them are blonde. Then you know that "not both E and F are red-
haired." What does this tell you? It tells you that one of them (maybe both) has to have a
hair color other than red. You don't know which one it is, unless one of them is a redhead.
Then you know the other can't be.
Now let's take a look at "both" in a "What must be true?" question. Imagine this were an
answer:
(E) Both T and R teach English.
The negation of this is that one of them doesn't teach English. If you see a previous
diagram where one of them teaches a different class, you know that (E) doesn't have to be
true. To test this by making a new diagram, you'd have to give one of them a different
class to teach, but you wouldn't know which one to give it to. You might try T first, and
give them every possible other class (while giving R English). If that didn't work, you'd try
to give R every possible class other than English (and give T English). If no class other than
English worked for either of them, you'd know that they both had to teach English.
By the way, the LSAT can just say "and" and mean "both." For example, "X and Y teach
English" means "both X and Y teach English."
Negating If then sentences: (For example, If A is 1
st
, then B is 3
rd
.) This is very
rare, but once in a while a What must be true? question will have If then sentences
for answers (see, for example, LSAT 29, question 6 in the logic games section; each answer
is an If then sentence). In order to see which answer must be true, you need to test
the negation of each answer. Thus, you need to know how to negate If then
sentences. Heres how to do it:
To negate an If then sentence, make the if true and the then false. For example,
to test
(B) If A is 1
st
, then B is 3
rd
.
You would make a diagram in which A is 1
st
, but B is not 3
rd
. In other words, youd write
No B under space 3, and try to put B someplace else (2
nd
, or 4
th
, or 5
th
, etc.). If you can
"If then" Rules and Negations 75
put B someplace else (other than 3
rd
), you know that B doesnt have to be 3
rd
if A is 1
st
.
Thus, the answer doesnt have to be true. If you cant put B anyplace else, you know that if
A is 1
st
, B has to be 3
rd
, so the answer must be true.
Like I said, this is very rare, so I wouldnt put it high on your list of things to memorize.
The moral of the story on negations
Negations often confuse people because they involve negatives, but you generally can't see
negatives you see positives. That is, you don't see what X isn't, you see what X is. What
X is tells you what it isn't it isn't anything else other than what it is. What you have to do
is learn how to recognize when one situation is the negation of another. If, to answer a
question, you have to find a situation where X is "not such and such," you should be able to
know, in advance, what types of actual situations you could see (X being something) that
would mean "not such and such." Learn what actual situations are the negations of
sentences, and you'll be able to spot right and wrong answers much more quickly and be
able to apply contrapositives much more comfortably.
What Do I Do Next?
You need to understand what "If then" rules mean and how to make contrapositives. If
you don't, read the first part of this chapter again. Make up your own "If then"
sentences and make contrapositives out of them. When you feel somewhat comfortable
with that, go on to Game Two.
Once you have "If then" sentences and contrapositives down, you need to think about
equivalents of "If then", like "all," "any," "only," and so forth. You don't need to
memorize these before you go on to the next chapters, but you should be memorizing them
while you study the next chapters. You want to have the "If then" equivalents nailed
down by the time you get to Intermediate Techniques. Try rewriting some of the "If
then" sentences I told you to make a few sentences ago using the equivalent terms. You
may also want to make flash cards out of some of these; most people find sentences with
"unless" and "only" difficult, so I'd be especially careful to understand them.
You don't have to have to be comfortable with negations at this point. Just be aware that
they exist, and you'll be forced to think about them as you go through games. If you see
that you have problems with them in general, or with specific types, refer back to this
chapter and maybe make some flash cards.
76 Game Two
Game omitted from electronic version of book.
Game Two 77
Game Two
(From LSAT 28)
Step 1: Read the Setup/Draw a Diagram
Every game involves two types of entities that you match together. What are the two types
of things in this game? Read the setup and figure it out. We've got some researchers and
some languages, and we are going to match one to the other. Which is more definite?
Well, you don't know too much about either group. You are told that each researcher can
only learn three languages at the most (which you should write down), but you don't know
how many languages each will actually learn one? two? or three? Since we don't really
know which of the researchers or the languages is more definite, we don't know how to
diagram this. Now, here is an important tip:
When you aren't sure how to diagram a game, glance quickly at the rules and see if they
help you.
Take a quick peek at the rules. Go ahead Now, notice that the first three rules tell you
definitely how many times each language is learned. That makes the languages more
definite. So our diagram should be based around them; write down their names
(abbreviated) horizontally (or vertically, it doesn't matter). Under each language we are
going to put a blank for each time it will be learned; those blanks will eventually be filled
with the names of the researchers. But that information will come from the rules, so let's
not get ahead of ourselves. Write down the names of the researchers off to the side. Also,
I've written down the rule about how many languages each can learn. This is a rule, so I'm
writing it where the rules go.
R S T Y researchers: G H L P
Each researcher can learn 1 to 3 languages.
Remember, this is the format of every diagram: the definite things are written down in a
row, and each gets a blank for every variable thing that will be assigned to it.
Step 2: Read the Rules
The first three rules tell us how many blanks to write under each researcher. Notice the
word "exactly" in each, which means that each has to be learned by that many researchers,
no more and no fewer. R gets one, because it will be matched with only one researcher, S
gets two, and so forth. We put these directly into our diagram:
R S T Y researchers: G H L P
__ __ __ __
__ __ __
__
Each researcher can learn 1 to 3 languages.
78 Game Two
Next rule: "Any language learned by the linguist or paleontologist is not learned by the
geologist."
First thing, you should have noticed the word "Any." The word "Any" tells you that this is
an "If then" rule. The "any" part is the "if" part, and the other half of the sentence is the
"then" (see "If then" Equivalents in the chapter on "If then" Rules and
Negations). Second, this is a complex "If then" rule, because it has two things in the
"if" part. When you see an "If then" rule with two things in the "if" part, ask yourself if
you can split it into two rules. This can be split into one rule about the linguist and the
other about the paleontologist. We know this because of the word "or." If either one of
them (the linguist or paleontologist) learns the language then the geologist can't learn it.
So we'll write this as two rules to make it easier to understand.
If L learns a language, then G can't (learn that language).
If P learns a language, then G can't (learn that language).
You don't have to write the part in parentheses, but you might want to if you find the rule
confusing. Now that we have "If then" rules, we want to write the contrapositives.
Always write the contrapositive when you see an "If then" rule. We make the
contrapositive by switching the "if" and "then" parts, and by adding negatives to both parts.
Since the "then" part is already negative (G can't learn that language), we make it positive.
If G learns a language, then L can't (learn that language).
If G learns a language, then P can't (learn that language).
What do these rules tell us? If you aren't sure you understand a rule, give yourself a
specific, concrete example to make things clearer. What if L learns S? The G can't learn S.
And if G learns S? Then L and P can't.
Next rule: "Any language learned by the geologist is learned by the historian."
How should we write this? I'll give you a second to think. That's right, this is an "If
then" rule (the word "any" tips us off), so we write it with "if" and "then."
If G learns a language, then H learns it (the same language).
Write the contrapositive as well.
If H doesn't learn a language, then G doesn't learn it (the same language).
Does this mean that G and H always learn the same languages? No. This is a common
mistake. The rule tells you that if G learns something, then H will, but what if H learns
something? Since H is in the "then" part, it doesn't guarantee that G will learn it. H learns
all of G's languages, but H might learn more than these. Get used to interpreting "If
then" rules in this way.
Game Two 79
So here is our original diagram and rules (with optional words in parentheses):
R S T Y researchers: G H L P
__ __ __ __
__ __ __
__
1. Each researcher can learn 1 to 3 languages.
2. If L learns a language then G can't (learn that language).
3. If P learns a language then G can't (learn that language).
4. If G learns a language then L can't (learn that language).
5. If G learns a language then P can't (learn that language).
6. If G learns a language then H learns it (the same language).
7. If H doesn't learn a language, then G doesn't learn it (the same language).
This is a lot of rules, but they are simple to understand. The numbering is just so I can
refer back to them, and it doesn't have much to do with the order in which they were told to
us by the LSAT.
There are several things we can figure out by combining these rules right now
deductions but we aren't going to look for them. It isn't necessary and I don't want to
distract you from the more important things you need to learn now. I point out that there
are deductions because after we finish the game I want you to notice how well we did
without making them.
Step 3: Answer the Questions
We are going to walk through the questions one by one. If a question is one that we should
do, we'll do it; otherwise we'll skip it and come back to it later.
Question 6: A "What can be true?" question do it now
It's a "What can be true?" question, so we'll do it ("could" means the same as "can"). We
have to test each answer out by drawing a new diagram and putting in the scenario the
answer describes.
Answer A
Redraw your diagram and put the linguist in under each of the languages listed in the
answer (R, S, and T).
R S T Y researchers: G H L P
L L L __
__ __ __
__
Now let's go through the rules.
80 Game Two
Rule 1 says that each researcher can only learn one to three languages. L has learned
three, so it can't learn any more. This means that L is finished. Cross it off our list. Rule 2
says that G can't learn any languages that L learns. So G can't learn R, S, or T. Write this
in the diagram.
R S T Y researchers: G H L P
L L L __
__ __ __
__
no G no G no G
Every time we learn something new about the game we have to go back through the rules
from the beginning. Here, we've learned that G can't learn R, S, or T, so we start again
with rule 1. It says that every researcher has to learn from one to three languages. That
means that G has to go someplace, and the only place it can go is Y. We can cross G off the
list because it can't go anyplace else.
R S T Y researchers: G H L P
L L L G
__ __ __
__
no G no G no G
Keep going through the rules. Rule 3 says "If P learns a language," but we don't know what
P learns. We could look at the "then" part of the rule, but we shouldn't. Remember, with
"If then" rules, the "then" part doesn't tell you anything. When looking at "If then"
rules, focus on the "if" part first. If it is true (if P learned some language here), then you
know that the "then" part is true. If you don't know if the "if" part is true, then the "then"
might be true or it might not.
Rules 4 and 5 tell you that L and P can't go where G goes, so they can't go under Y. Rule 6
tells you that H has to go under Y. Put all this in.
R S T Y researchers: G H L P
L L L G
__ __ H
__
no G no G no G no P
no L
Skim through the rules one more time because we've added some new stuff. Nothing new
happens. Now we are done with the rules. We have to fill in the rest of the spaces to see if
we can't make this diagram work. We can put P or H under S and T, since there is nothing
to say that we can't. Who do we put in the last spot for Y? We can't put P in, we can't put
L in, and we've already used G and H. What does this mean?
It means that there is no one that can go in that last space under Y. We can't fill all the
spaces of the diagram without breaking the rules. This diagram doesn't work, so this
answer is wrong. Cross off the diagram (always cross off diagrams that break the rules so
that you don't use them later on by accident) and cross off the answer.
Game Two 81
Answer B
Draw a new diagram again and put in the situation described in the answer.
R S T Y researchers: G H L P
__ L L L
__ __ __
__
Rule 1 tells us that we can cross off L because we have used it three times, and rule two
tells us that G can't go under S, T, or Y, because it can't go with L.
R S T Y researchers: G H L P
__ L L L
__ __ __
__
no G no G no G
Since we've learned something new that G can't learn S, T, or Y we have to start again
with rule 1. We know that G has to go under R, because it has to be used at least once.
We cross off G because it can't be used any more times there is nowhere else for it to go.
R S T Y researchers: G H L P
G L L L
__ __ __
__
no G no G no G
Continue with the rules. Rules 4 and 5 tell us that L and P can't go under R (which is no
problem because there isn't any more room under R). Rule 6 tells us that H has to go
under R, because H goes wherever G goes. But there isn't any room under R, so H can't go
there. This means that this diagram breaks the rules, so the answer is wrong. Cross off the
diagram and cross off the answer.
Answer C
Again, draw a new diagram and put H in where you are told.
R S T Y researchers: G H L P
H H H __
__ __ __
__
Go through the rules. Cross off H because of rule 1. Rules 2 through 6 are "if" rules about
the languages L, P, and G learn. But we don't know what languages they learn; when you
don't know if the "if" part of an "If then" rule is true, we skip the rule. That brings us to
rule 7. It says "If H doesn't learn a language." We know that H doesn't learn Y, because
there isn't an H under Y, and there can be no more Hs in the diagram. Since the "if" part is
true (H doesn't learn a language, Y), the "then" part has to be true as well. G can't learn Y.
R S T Y researchers: G H L P
H H H __
__ __ __
__
no G
82 Game Two
You might now notice that there are only two people that can go under Y L and P. But
there are three spaces under Y. This won't work, so this answer is wrong.
Maybe, though, you didn't notice that. You can still get the answer right. We're done
looking at the rules, and we have many empty spaces. We are just going to have to try
things out and see if we can make it work. Start putting people in. When you have to just
put people in a diagram, start with the first person and put them in the first available space
(that's the first space that isn't filled and that has no rule explicitly saying they can't go
there). Put G under S. Mark it with a dot or star, since we chose to put G there.
R S T Y researchers: G H L P
H H H __
G* __ __
__
no G
Once we've added something to the diagram, we go through the rules. The only rules about
G say that L and P can't go with G. There isn't any more space under S, so this isn't worth
writing down. Now we have to put in L and P. Put L in the next available spot under T.
Mark it, because this is our choice.
R S T Y researchers: G H L P
H H H __
G* L* __
__
no G
Now we have three spots left under Y. Put P under Y because P is next on our list. Now we
go back to the beginning of the list. G can't go under Y, and H is all used up. This leaves
only L, which means that we can't fill all the spaces under Y. This won't work.
Could we make this work by changing some people around? We can move G to T, but this
won't make a difference in filling Y the H's are all assigned by the answer, and we've
already got L and P under Y. We can't move G to Y, because we have "no G" under it.
Cross off this diagram and cross off answer C.
Answer D
Try this out on your own, then read my explanation.
We put H where the answer tells us.
R S T Y researchers: G H L P
__ H H H
__ __ __
__
Then we go through the rules. Rule 1 tells us that H can't be used any more. Rules 2
through 6 are about the languages learned by G, L, and P, and we don't know what those
languages are. Rule 7, though, tells us that G can't learn R. This is because H isn't under
R, and if H doesn't learn a language, G doesn't either. This is all the rules tell us.
Game Two 83
R S T Y researchers: G H L P
__ H H H
no G __ __ __
__
We just have to start putting people in. I'll put G in first. The first available space for G is
under S, since there is a "no G" under R. Mark G because this was our choice.
R S T Y researchers: G H L P
__ H H H
no G G* __ __
__
Look at the rules because we just put someone in. L and P can't go with G, but since S is
now full this is no big deal. Now let's put in the next person on our list (who isn't crossed
off): L. Put L in the first available space, which is under R, and mark this as our choice.
R S T Y researchers: G H L P
L* H H H
no G G* __ __
__
Now on to P. The first space open for P is under T. Put it there and mark it.
R S T Y researchers: G H L P
L* H H H
no G G* P* __
__
Looking through the rules, we see that this doesn't tell us anything new (well, it tells us that
G can't go under T, but T is filled anyway). Now we have two spaces left under Y. Start
back at the beginning of our list. Put G under Y. Look at the rules again. They tell us that
neither L nor P can go with G (you probably remember that without looking at this point), so
we couldn't put anyone in the last spot under Y. This won't work, so we think about the last
thing we put in. Let's not put G under Y. Instead, put L there. This is fine. Now put P in
the last spot.
R S T Y researchers: G H L P
L* H H H
no G G* P* L
P
Check the rules one more time. This works, so it is possible; this (answer D) is the answer.
84 Game Two
Question 7: An If-question do it now
This is our first If-question. Let's savor the moment as Sade says, it's never as good as
the first time.
If-questions give us a new piece of information that we assume is true (on this question
only). The first step on an If-question is to draw a new diagram so we can put this
information in. Then we are going to see if this new information, together with the rules,
makes anything have to be true.
R S T Y researchers: G H L P
__ __ __ __
__ __ __ 3 Ls
__
We can't put this situation right into the diagram we don't know where our three Ls have
to go. So I wrote it off to the side. When you first look at an If-question, before you look at
the answers, only put things in your diagram that have to be true.
You have an original diagram which is true for the entire game that's what you set up in
step 1 of the game. You copy this diagram every time you do a question. We don't put
stuff that might be true in this diagram, because it might be false, and we want this
diagram to apply to every question.
On an If-question, you have been given new information to put in your diagram; from this
information you can usually but not always figure out other things that have to be true.
These things will have to be true for every diagram based on the "If." That means that
every answer you test out on that question will have to incorporate these. The "If" gives us
a new diagram, which we use instead of the original diagram to test every answer on that
If-question. But, we don't put things that might be true into this diagram, because we want
it to use it on every answer (and stuff that might be true is not always true).
Now, when we look at an answer, we want to see if it could be true. So we plug people in
where they might go; we try things out that might be true, not just things that have to be
true. The difference between this and what we do when we first look at an If-question is
that the diagram we draw initially for the question has to be used on every answer, so it
should only include things that have to be true for every answer. The diagram we draw to
test a single answer only has to be possible, and we are only using it to test that one
answer, so it doesn't have to be true for any other answer.
Because we don't know exactly where the Ls will go, we probably aren't going to figure
much out. Go through the rules quickly. Rule 1 tells us that we can have three Ls at the
most, which just goes with what the question tells us. Rule 2 tells us that G can't be where
L is. Great. Nothing else mentions L, so that's all we know. We are going to have to use
the answers here. This might be an OK question to skip, since we didn't figure much out,
but I'm going to do it because I have confidence in us.
The question is a "What must be true?" question. How do we test the answers on this type
of question? You should have this memorized by now. We do something other than what
the answer describes and see if you can make it work. Remember, we have to have three
Ls, as the question says. Also, we can't use our previous diagram because this is an If-
question.
Game Two 85
Answer A
The answer says that L must be under T, so we are not going to put the L under T in order
to test this out. Since we have to have three Ls, that means L goes under everyone else.
Now that we've put our three Ls in, we won't put in any more Ls, so we cross L off.
R S T Y researchers: G H L P
L L __ L
__ __ __ 3 Ls
__
Let's go through the rules. Rule 2 tells us that G can't go where L goes. We can't have G
under R, S, or Y; write this into your diagram. Go back through the rules because we've
learned something new. We see that G has to be used at least once (rule 1), so it has to go
under T. G can be crossed off, because there are no other places it can go.
R S T Y researchers: G H L P
L L G L
__ __ __ 3 Ls
__
no G no G no G
Continuing through the rules, we see that H has to go with G (rule 6) (the other rules just
repeat what we already know, that G can't go with L, or talk about things we don't know
about, such as the language learned by P). This isn't the only place H can go, because H
can go someplace G can't.
R S T Y researchers: G H L P
L L G L
__ H __ 3 Ls
__
no G no G no G
Going back through the rules one more time, we learn nothing new. Rules 2, 4, 5, and 6
talk about things we know (that G can't go with L or that G cant go with P), and rule 3 talks
about the language P learns, and we don't know which one that is. Now we just fill in our
empty spaces. We have one under S and two under Y. We haven't used P yet, so put it
under S (and mark it as optional).
R S T Y researchers: G H L P
L L G L
P* H __ 3 Ls
__
no G no G no G
Looking through the rules, this tells us nothing new. We have to fill the spaces under Y. G
and L are crossed off, so we have to put P and H there.
R S T Y researchers: G H L P
L L G L
P* H P 3 Ls
H
no G no G no G
86 Game Two
This is possible, so the answer is wrong.
The next thing we should do is see if this diagram helps us with the other answers. We
can't use previous diagrams from other questions to help us on If-questions, but we can use
them from previous answers within the same question. We can do this because the
diagram will have to incorporate the situation described in the If-question, whereas
diagrams from other questions might not. We want to see if the diagram we just made is
different from the situation described in any other answer; if it is, we can eliminate that
answer, because we know that it doesn't have to happen. Answer B has L under R, which
agrees with our diagram. We can't eliminate B, then, because we don't know that that
doesn't have to happen. Answer C has the L learning S. Our diagram has that, so we can't
cross off C. Answer D has the paleontologist learning R, which is different from our
diagram. We can cross that off. Answer E has the paleontologist learning S, which is in our
diagram. We can't cross that off. We have to test out at least one more answer. Look at
B.
Answer B
To test this answer, don't put L under R. Since we have three Ls, we have to put it under S,
T and Y.
R S T Y researchers: G H L P
__ L L L
__ __ __ 3 Ls
__
Look through the rules. Rule 2 tells us that G can't go with L, so it has to go under R.
R S T Y researchers: G H L P
G L L L
__ __ __ 3 Ls
__
Go through the rules again. Rule 6 says that H has to go with G, but there is no room
under R. This won't work, so it (answer B) is the answer. Remember, when you can't make
a diagram on a "What must be true?" question work, the answer is right (because this
means that nothing other than what the answer describes can be true).
Notice how this seemingly difficult question we didn't figure out anything from what they
initially told us ended up being pretty easy. Why? Because we are comfortable going
through the rules and trying things out. We know how the question works that is, how to
determine if an answer is right or wrong. And we know how to re-use the diagrams we
create.
Question 8: A "What can't be true?" question do it the second time through
This question looks like a "What can be true?" question, except for the word "EXCEPT." The
word "EXCEPT" means that the test has just described the wrong answers to you, so the
right answers are the converse. Here we know that the wrong answers could be true, so
the right answers can't be true. We'll skip this and do it after we've done all the "What can
be true?" and If-questions.
Game Two 87
Question 9: A "What can't be true?" question do it the second time through
This looks like a "Which is a complete and accurate list?" question, except that it says "could
be a complete and accurate list", which makes it a "What can be true?" question. Oh, and
the word "EXCEPT," which makes it a "What can't be true?" question. Skip it for now.
Question 10: An If-question do it now
Draw a new diagram and write down what you are told.
R S T Y researchers: G H L P
__ __ __ __
__ __ __ 2 Gs
__
Like the previous If-question, you can't do much with this information. The rules tell you
that G can't go with L or P, and has to go with H. But we don't know exactly where G goes
(it is possible to figure this out, but we'll talk about how to do this later in the Advanced
Techniques chapter). Notice that it is also a "What can be true?" question. Let's go through
the answers. We'll try out what they describe, and if it works, the answer is right.
Answer A
Since it's a "What can be true?" question, we just try out what the answer says. Put P
under R.
R S T Y researchers: G H L P
P __ __ __
__ __ __ 2 Gs
__
Notice that the answer doesn't say "The paleontologist learns only Rundi," so we can have P
someplace else too. Don't read more into the rules than they say; if a rule says something
happens, we know that it happens, but it isn't the only thing that can happen. Other things
can happen too (unless it says "only").
Let's go through the rules. Rule 2 is about the language L learns, but we don't know what
that is, so we skip it. We know that G can't learn Rundi from rule 3, but since there is no
more room under R, it wouldn't go there anyway. The rest of the rules are about the
languages learned by G or not learned by H, but we don't know anything about G or H, so
these rules don't tell us anything.
We now have to fill our empty spaces. Start by putting G in someplace because it is first
alphabetically, and because we have lots of rules about it. We have to put G in twice. Put it
under S and T because they are our first two empty spaces. Put marks next to them
because we decided to put G here. Going through the rules, we see that H has to go with
G. Since it has to go with G, H doesn't get a mark.
R S T Y researchers: G H L P
P G* G* __
H H __ 2 Gs
__
We now have three spots left under Y. We can't put G there because we only get two Gs.
We have H, L, and P left, so put them there. Remember, we can have H without G (but not
vice versa).
88 Game Two
R S T Y researchers: G H L P
P G G L
H H P 2 Gs
H
This works, doesn't it (look at the rules and make sure). So this is possible, and it (answer
A) is our answer. Don't look at any other answers. Really, I mean it. Don't do it.
Looks easy, but what if we had done it differently for some reason? What if we had put G
under Y? It wouldn't have worked we wouldn't be able to fill the third space because
neither L nor P could go there. When you come across a diagram that doesn't work, change
the last thing you marked with a dot. We didn't have to put G under Y, so try someplace
else. Well, hey, you can put G under S and T; you'd try that out and it would work. Don't
be afraid to play around with the diagrams a bit if they don't work, if there are things that
you can change (i.e., if you have marked something). If not if, for example, you were
told that G had to go under S or T then don't change things. Note that it is important to
mark things only if you chose to put them where they are, not if the rules forced you.
Question 11 A "What must be true?" question do it the third time through
Question 12 An If-question do it now
Redraw your diagram and plug in what you are told.
R S T Y researchers: G H L P
__ __ __ __
__ __ __ 2 Hs
__
Again, we can't put what we are told directly into the diagram because we don't know where
the two Hs have to go. We could put the Hs into places where we think they could go, but
when we first set up a diagram for an If-question we only put things where they have to go.
Look at the rules before you look at the answers. All we know is that wherever we put a G,
we have to put an H, so at least one of the Hs will go with a G (since we have to have at
least one G). Also, we can't put a G anywhere we don't put an H (rule 7), so we have two
Gs maximum. On to the answers. Work through them yourself and then compare what you
did to what I say; remember, this is a "What must be true?" question, so we test the
answers out by doing the opposite of what they say.
Answer A
We test this by putting the paleontologist under Rundi (since the negation of "does not learn
R" is "does learn R") and seeing what happens.
R S T Y researchers: G H L P
P __ __ __
__ __ __ 2 Hs
__
Go through the rules. All you learn is that we can't put a G under Rundi (rule 3), but since
there is no more room under R anyway, this doesn't tell us much. Let's fill in the spaces
and see if we can make this work. We could start by putting in G, because it's first, but
since the rule for this question tells us we have to have two Hs, put them in first. The first
open spaces are under S and T. Mark the Hs.
Game Two 89
R S T Y researchers: G H L P
P H* H* __
__ __ __ 2 Hs
__
Put a G and an L under S and T, since G and L are our first two researchers. Going through
the rules, we learn that G can't be under Y, because H doesn't learn Y. Now we have to fill
the three spaces under Y. We can't put in any more Hs because we've used both of them.
G can't go there. That leaves L and P . This can't work.
When you have a diagram that doesn't work, change the last dotted thing you put in. You
can either draw a new diagram (cross off the one that broke the rules) or erase parts of the
one that didn't work. We put H under S and T, but we don't have to do that. Let's put H
under S and Y instead. I don't mark the second H because there is nowhere else it could
go.
R S T Y researchers: G H L P
P H* __ H
__ __ __ 2 Hs
__
Go through the rules. They mostly talk about P and L, and we don't know where those two
are, but rule 7 tells us that G can't learn T because H doesn't. Let's put G under S, the first
open spot.
R S T Y researchers: G H L P
P H* __ H
G* __ __ 2 Hs
no G __
Let's fill the spaces under T with L and P, the next two researchers on our list.
R S T Y researchers: G H L P
P H* L H
G* P __ 2 Hs
no G __
By this point we probably remember that G can't go with L or P, so we put L and P, but not
G, under Y. If you didn't remember that, you could try putting G under Y and marking it,
because you didn't have to put G there. It would end up not working, since you couldn't fill
the last spot, so you'd get rid of G (the last thing you put in) and put in L and P.
R S T Y researchers: G H L P
P H* L H
G* P L 2 Hs
no G P
This works, so this answer is wrong.
Let's see if this diagram helps us to eliminate any other answers. See if this diagram differs
from what other answers describe. We can eliminate B, since this diagram shows us that
the Geologist can learn S. We can't eliminate any of the other answers, however, since this
diagram agrees with what each of them say. On to C.
90 Game Two
Answer C
Let's try to put the linguist under Rundi.
R S T Y researchers: G H L P
L __ __ __
__ __ __ 2 Hs
__
Now we have to put our two Hs in. I saw that putting them under S and T was problematic
in answer A, so I'll put them under S and Y again.
R S T Y researchers: G H L P
L H* __ H
__ __ __ 2 Hs
__
This doesn't tell us anything other than that G can't go under T (rule 7); let's put the G in
the first available spot:
R S T Y researchers: G H L P
L H* __ H
G* __ __ 2 Hs
__
Now I just fill out the spaces with Ls and Ps like before.
R S T Y researchers: G H L P
L H* L H
G* P L 2 Hs
P
This works also, so cross off this answer. Unfortunately, this diagram doesn't help us
eliminate D or E, so we have to test one of them out (but only one, since it will be either
right or wrong; if it's right, we're done. If it's wrong, then we know the last one left is
right.)
Answer D
Put the historian under R.
R S T Y researchers: G H L P
H __ __ __
__ __ __ 2 Hs
__
Game Two 91
Now we have one more H left. Our previous work showed us that we have to put one of the
Hs under Y.
R S T Y researchers: G H L P
H __ __ H
__ __ __ 2 Hs
__
Go through our rules. We have to put in a G (rule 1), and the G has to go with one of the
Hs. So we put the G under Y.
R S T Y researchers: G H L P
H __ __ H
__ __ G 2 Hs
__
Going through our rules again we see that L and P can't go under Y. But that means that
we can't fill in all three spaces under Y. This can't work. Can we make any changes to fix
it? Nothing here is marked, so no. The first H is stuck under R, because that's what we
need to do to test out the answer; we can put our second H under S or T instead of Y, but
then we couldn't fill all the spaces under Y (we'd have to put G, L, and P there to do so, and
that doesn't work). There is no way this diagram will work, so D is the answer.
Question 8: A "What can't be true?" question we're doing it now, our second
time through
The question asks for something that can't be true about the researcher who learns R
(although it looks like a "What can be true?" question, the EXCEPT makes it a "What can't
be true?" question). So we are focusing on the person who is under R. Use our previous
diagrams wherever possible; if the answer shows up in a previous diagram, we can cross it
off because we know it can be true.
Answer A
When they say "the researcher" they mean "the researcher who learns Rundi," because
that's who the question is asking about. So, do we have a diagram where the same person
learns R and T, but not S? (Notice that the answer doesn't talk about Y, and doesn't say
"learns only Tigrinya," so we don't worry about Y at all.) 12A and 12C have this situation
(P learns R and T but not S in 12A, and L does so in 12C). You don't have to spot both of
these in fact, once you've seen one diagram that matches, stop looking but I'm putting
them here for your reference. Cross this answer off.
Answer B
In this case we want the guy who learns R to not learn T or S, and again we don't care
about Y. Look at our previous diagrams. 6D and 10A contain this situation. In 6D, L learns
R and Y but not S or T, and in 10A, P learns R and Y but not S or T. Cross this off.
Answer C
Look for a diagram where the same person learns R and T but not Y (ignore S, since the
question doesn't mention it, nor does it say "learns only Tigrinya"). None of our diagrams
have this in each case, the person who learns R also learns Y (and they may or not may
not learn T). What do we do when our previous diagrams don't help us? We skip the
answer and look at the other answers. If we can cross them off using previous diagrams,
great! If we can't, well, then we have to do a little work.
92 Game Two
Answer D
Look for a diagram where the same person learns R, T, and Y (they can't learn S in this case
because they can only learn three, but you don't need to worry about realizing that,
because the question doesn't even mention S). 12A and 12D all work. Cross this one off.
Answer E
Look for an answer where the same person learns R and Y, but doesn't learn T. 7A and 10A
work. On 7A, L learns R, S, and Y; S wasn't mentioned in the answer, so it doesn't matter if
L learns it or not. This answer is wrong as well. So the answer has to be C.
Question 9: A "What can't be true?" question we're doing it now, our second
time through
Make sure you understand the question. It says EXCEPT, so the right answer will give a
list of people who cant possibly all learn both S and Y at the same time (notice the answer
doesn't talk about learning other languages, nor say "only S and Y," so the people on the
list can learn more than these). That's the difference between questions that say "is a
complete and accurate list" and "could be a complete and accurate list" The first will
give you a list of people that could do whatever in any possible diagram, while the second
will be a list of people that could do whatever in the same diagram.
Answer A
Look at our previous diagrams first. Do we have one where the historian, and only the
historian, learns both S and Y? Where other people learn one or the other, but no one else
learns both? Sure; 6D, 10A, 12A, and 12D. In each of these, the historian learns S and Y,
and no one else does (because in each case G learns S, but doesn't learn Y). Sure, the
historian learns other languages as well, but that's not important, because the question
doesn't ask for people who only learn S and Y. So we know that A gives a possible list,
which means that it is wrong (the "EXCEPT" in the question tells us we are looking for list
that can't be complete and accurate).
In going through your previous diagrams you might also have noticed that in question 7A, L
and P learn S and Y. This allows us to eliminate answer E. We now have to look at answers
B, C, and D.
Answer B
Put just the paleontologist under S and Y.
R S T Y researchers: G H L P
__ P __ P
__ __ __
__
The answer only lists P, so this means that whoever else goes under S can't go under Y, and
vice versa (otherwise, if they went under both, the answer would have to include them,
since it is a complete list). Go through our rules. Rule 3 says that G can't go with P. Write
that in our diagram.
R S T Y researchers: G H L P
__ P __ P
__ __ __
__
no G no G
Game Two 93
We know that whoever we put under S can't also go under Y. So let's see who can learn S.
Not G, so try H; this would mean that H can't go under Y. Mark H with a dot or a star,
because we don't know that H has to go here.
R S T Y researchers: G H L P
__ P __ P
H* __ __
__
no G no G
no H
But who now can go under Y? Only L, and that doesn't fill enough spots. So H can't go
under S. Try L instead; we put L under S, but then it can't go under Y (otherwise it would
have to be in the answer, since it would be under both S and Y), so we put "no L" under Y
to remind ourselves of this.
R S T Y researchers: G H L P
__ P __ P
L __ __
__
no G no G
no L
Same problem; we can't put enough people under Y. So there is no way to make this work
we've tried everyone under S, and no matter who we put there, we can't find enough
people to put under Y. We can't make B work, so B can't be a complete list of the people
who learn both S and Y at the same time. B is the answer.
Question 11 A "What must be true?" question we're doing it now, our third
time through
Test the answers out by looking for a previous diagram showing that the negation of the
answer can be true.
Answer A
The opposite of this is that H learns as many as P or more. You may not remember that "as
many" (or "exactly") is the negation of "fewer," so I'm reminding you (see "If then"
Rules and Negations). Even if you don't remember, you can still get this question right
just looking for "more." Look for a diagram where H learns as many or more languages
than P.
In 6D, 7A, 10A, and 11A, H learns more than P. In 12C, H learns as many as P. Again, you
only need to see one of these diagrams to eliminate the answer, but I'm giving you all of
them just so you can see what you are looking for. Cross off this answer.
Answer B
This means that G learns fewer than H. So we want to look for a situation where G learns
as many or more than H. Notice how useful it is to understand the answer (putting it in our
own words) before figuring out what we are supposed to be looking for in the previous
diagrams. We don't see any diagrams that show us what we are looking for; H always
seems to learn more than G. Does this mean that this has to happen? No, just that it has
always happened so far. Skip this answer.
94 Game Two
Answer C
This means that G learns fewer than L. Look for a diagram where G learns as many or more
than L. In 10A, we have two Gs, but only one L. Cross off this answer.
Answer D
This means that P learns fewer than L. Look for a diagram where P learns as many or more
than L. In 10A and 12A, P learns more than L. Cross off this answer.
Answer E
This means that P learns fewer than H. Look for an answer where P learns as many or more
than H. In 7A and 12C, they learn the same number of languages. In 12A, P learns three
while H only learns two. Cross off this answer. The correct answer has to be B.
Summary
What has this game demonstrated to us? Well, hopefully it's reinforced how to construct
diagrams. You can use a different diagram for example, try putting the names of the
researchers along the top of the diagram, and leave blanks underneath them to put in the
names of the languages. It'll work this way too, but it's a bit easier to do it the way we did.
Also, I hope it helped you understand how to write the rules of a game; it was very easy for
us to use our rules because we wrote them out so clearly.
Here are some other key points this game illustrates about rules: You can use the rules to
help you figure out how to diagram a game. Split "If then" rules into two rules whenever
possible. On If-questions: Do them the first time through. Make a new diagram for the
question, put in what you are told, then go through the rules and figure out what has to be
true (put only these in the diagram). Use this diagram on all the answers; use diagrams
from previous answers on later answers of the same question. Other points: don't read
more into a rule, question, or answer than it says; if it says "So and so learns X," or asks
you who learned X, don't assume that person has to learn only X.
What Do I Do Next?
Take a break. Then read through your notes on this game and think about them until you
feel like you understand how the various steps work. Then go back and do this game on
your own. You can use your general notes to help you remember how to do the various
question types. When you are done, go on to Game Three.
Answer Key
6. D
7. B
8. C
9. B
10. A
11. B
12. D
Game Three 95
96 Game Three
Game omitted from electronic version of book.
Game Three 97
Game Three
(From LSAT 33)
Step 1: Read the Setup/Draw a Diagram
Every game consists of two or more types of things which you have to match together,
right? So what are the two types of things in this game? The setup mentions bird-
watchers, but they don't actually do anything; they just explore the forest. The setup also
mentions some birds. It may be unclear to you what you are matching the birds to
maybe bird-watchers? Where can we look for more insight into the game if the setup
doesn't make the game clear? If we are uncertain how to diagram a game, look at the
rules.
Each rule talks about whether or not a certain bird (or birds) are in the forest. So that
should make it clear that we have two types of things: birds and a forest, and we are
matching birds to the forest. We are trying to figure out which of the six birds will be in the
forest.
Now here's the next question. Which of these things is more definite? We don't know how
many birds are going to be in the forest (if we did, that would make the forest more
definite). In fact, we know next to nothing about the forest, except that the birds are going
to be in it. So we will base our diagram around the birds. Write down their names:
G H J M S W
Our other type of thing is the forest. Some of these birds will be in the forest, and some
will not. That means that the game will divide up into two groups: the in-forest group and
the not-in-forest group. We always label our groups, so label those:
G H J M S W in-forest:
not-in-forest:
So we've written down everything we've been told. We are done.
This is a common sort of thing to see in a game: picking things off a list to put in one or
more groups. If some of the things won't be in the group (or any group, if there is more
than one group), make a group for those excluded (here, the not-in-forest group). This
allows you to keep track of who isn't in a group; often, although not here, only a limited
number of people can not be in groups, so once you've filled the "not" group, everyone else
has to go in a group (see Game Five).
Step 2: Read the Rules
I'm going to warn you in advance that all of these rules are "If then" rules. So let's
briefly review what "If then" rules mean and how we write them down (see "If
then" Rules and Negations).
"If then" rules give you two situations; one is in the "if" part and one is in the "then"
part. Whenever you know that the "if" part is true, you know also that the "then" part has
to be true. However, knowing that the "then" part is true doesn't mean that the "if" part is
true. For example, imagine I say that "If X is true, then Y is true." Whenever I know that X
is true, I know that Y has to be true. But if I know Y is true, X may or may not be true.
98 Game Three
The fact that every time the "if" part is true the "then" part is true tells us something else.
It tells us that we can't have the "if" without the "then," because the "if" always makes the
"then" true. So, if we don't have the "then" part, we can't have the "if." In our "If X is true,
then Y is true," example, we know that "If Y is not true, then X can't be true." We know
this because X being true would make Y have to be true. This is called the contrapositive.
We always make the contrapositive when we have "If then" rules, and we make the
contrapositive by 1) switching the "if" and the "then" (the "if" part becomes a "then" and
vice versa); and 2) making both parts negative (something already negative becomes
positive).
Finally, some "If then" rules are more complex than they need to be. When we get
these rules we are going to make them simpler by breaking them into multiple rules.
First rule: "If harriers are in the forest, then grosbeaks are not."
Our first rule is an "If then" rule. It is easy to write down we just write it as it is, a
little abbreviated:
If H is in-forest, then G is not-in-forest.
This means that whenever the H is in the in-forest group, we have to have the G in the not-
in-forest group. My way of writing things is a suggestion; you can abbreviate more than I
have here (the "is" can be taken out in each clause without a problem) write whatever
makes sense for you, just make it easy to understand.
In reality, I abbreviate even more than this, but I don't in this book to make it easier for
you to read the rules. See the end of this section for the rules as I normally write them.
We should also form the contrapositive; this tells us that whenever the G is not in the not-
in-forest group, the H can't be in the in-forest group.
If G is not not-in-forest, then H is not-in-forest.
This can be simplified. We only have two groups; if something is not in one group, it has to
be in the other group; if something is not in the in-forest group, then it is in the not-in-
forest group, and vice versa. So, instead of writing "G is not in not-in-forest," we can write
that "G is in-forest." Our contrapositive is:
If G is in-forest, then H is not-in-forest.
Whenever something has to be in one of two groups, instead of saying it is not in one
group, say it is in the other group.
Next rule: "If jays, martins, or both are in the forest, then so are harriers."
This is a more complicated "If then" rule, because it has three things (jays, martins,
both) in the "if" part. We want to break it down into simpler rules. So ask yourself, what if
jays were in the forest? Well, harriers would have to be as well. What if martins were in
the forest? Harriers would have to be as well. So we can break this into two rules, one
about jays and one about martins, because the rule tells us something has to happen when
we have just jays or just martins.
Game Three 99
If J is in-forest, then H is in-forest.
If M is in-forest, then H is in-forest.
Make the contrapositive of each. Switch the "if" and "then," so H goes in the "if" part of
each and J (or M) goes into the "then" parts. Make both sides negative, which just means
put each person into the other group, so we get
If H is not-in-forest, then J is not-in-forest.
If H is not-in-forest, then M is not-in-forest.
We don't have to worry about the "or both" part if both J and M are in the forest, then J is
in the forest which means H will have to be in the forest.
Next rule: "If wrens are in the forest, then so are grosbeaks."
Another "If then" rule. Write it down and make the contrapositive.
If W is in-forest, then G is in-forest.
If G is not-in-forest, then W is not-in-forest.
Last rule: "If jays are not in the forest, then shrikes are."
If J is not-in-forest, then S is in-forest.
If S is not-in-forest, then J is in-forest.
One quick point. People are often very quick to try to combine "If then" rules and their
contrapositives in order to make deductions. They generally do this incorrectly. For
example, people might think that the last rule, together with its contrapositive, tells us that
J and S can't both be in the forest. After all, if J is out, S is in, and vice versa. This is a
mistake, though, because the rule only tells us about what happens if J or S are out of the
forest. Remember, the "then" part of the rule being true tells us nothing about the "if" part.
So if S is in the forest (the "then" part), J could also be in the forest. Take the rules as they
come, and take them literally; don't try to think too much about them.
So here is our original diagram and the complete list of rules. I'm spacing like I do to
make them easier for you to read.
G H J M S W in-forest:
not-in-forest:
1. If H is in-forest, then G is not-in-forest.
2. If G is in-forest, then H is not-in-forest.
3. If J is in-forest, then H is in-forest.
4. If M is in-forest, then H is in-forest.
5. If H is not-in-forest, then J is not-in-forest.
6. If H is not-in-forest, then M is not-in-forest.
7. If W is in-forest, then G is in-forest.
8. If G is not-in-forest, then W is not-in-forest.
9. If J is not-in-forest, then S is in-forest.
10. If S is not-in-forest, then J is in-forest.
100 Game Three
Here is how I write these rules for myself, more abbreviated (I use arrows to mean "then;"
it's just something I learned doing philosophy):
H -> no G
G -> no H
J -> H
M -> H
no H -> no J
no H -> no M
W -> G
no G -> no W
no J -> S
no S -> J
I'm showing you this just to demonstrate that you don't have to write as much as I do when
I write the rules, although there is nothing wrong with it. You only write the rules once, so
spending a little extra time to write them isn't so bad. If you want to abbreviate, go ahead;
just make sure that you use an abbreviating scheme that you like and use easily. Don't
bother making one up; it's not worth the effort. Mine works well for me because I had it
drilled into my head from years of doing formal logic for philosophy classes.
Many of the rules we have can be combined to make deductions. However, even if you
knew how to do this, there are so many rules and so many ways to combine them that
trying to put them together would take you forever. In a game with this many rules,
deductions definitely aren't worth it.
Step 3: Answer the Questions
We are going to walk through the questions one by one. If a question is one that we should
do, we'll do it; otherwise we'll skip it and come back to it later.
Question 6: A "What can be true?" question do it now
This looks like a "Which is a complete and accurate list?" question, but the fact that it says
"Which one could be a complete and accurate list" tells otherwise. The question is
asking for a list of all of the birds who could be not-in-forest at one time. The right answer
will be a list that could be true, that follows all the rules, so this is a "What can be true?"
question. Let's do it now.
Each answer will tell us a complete list of birds in the not-in-forest group. That means that
no other birds (no birds not on the list) are in the not-in-forest group. What does that tell
you about the other birds? It means that they have to be in the in-forest group.
Remember, when we have only two groups, if you are not in one group you are in the other.
It is crucial to understand what the answers are supposed to mean before you evaluate
them.
Answer A
Put J and S in the not-in-forest group, and every other bird in the in-forest group. Notice
that I cross birds off the list as I put them in a group.
G H J M S W in-forest: G H M W
not-in-forest: J S
Game Three 101
Now we go through the rules. When looking at "If then" rules, we only apply a rule
when the "if" part is the case. Rule 1, for example, says "If H is in-forest" H is in the
forest, so we look at this rule. It goes on to say "then G is not-in-forest." This contradicts
what the answer tells us (it tells us that only J and S are not-in-forest), so this answer can't
be right. Cross off this answer, because this can't be a complete list of birds not in the
forest. Cross off the diagram, because it breaks a rule, and we don't want to use it
accidentally later on.
Before we go on, look at rule 3. We wouldn't do this on the LSAT, but I want to do it now to
illustrate what I mean by "When looking at 'If then' rules, we only look at the rule if the
'if' part is the case." Rule 3 says "If J is in-forest" In this answer J is not in the forest, so
we ignore this rule. We don't assume the opposite of the rule (that H can't be in the
forest), we just ignore the rule entirely. This is because "if" rules only tell us what has to be
the case if the "if" part is true. So if the "if" part is not true, the rule has nothing to tell us.
Answer B
Again, put everyone in the answer H and G into the not-in-forest group, and everyone
else into the in-forest group.
G H J M S W in-forest: J M S W
not-in-forest: H G
Now let's go through the rules.
Rule 1 says "If H is in-forest", but H is not in the forest, so ignore this rule. Rule 2 says
"If G is in-forest", but G isn't in the forest, so ignore this rule. Rule 3 says "If J is in-
forest" and J is in the forest, so we look at this one. H has to be in the forest as well, but
it isn't. Thus, this answer breaks a rule, and we cross it off because it can't be an accurate
list of birds not in the forest. Cross off the test diagram as well, because it is a rule-
breaking diagram.
Answer C
Make your test diagram, putting G, J, and M into the not-in-forest group.
G H J M S W in-forest: H S W
not-in-forest: G J M
Go through the rules. Rule 1 says "If H is in-forest" and H is, so we look at this one. It
tells us that G can't be in the forest. Well, G isn't in the forest, so that rule is being
followed. On we go. Rule 2 says "If G is in-forest", but G isn't, so we ignore this rule.
Rule 3 says "If J is in-forest", but J is not in the forest, so we ignore this. Rule 4 says "If
M is in-forest", but M isn't, so ignore this rule. Rule 5 says "If H is not-in-forest", but H
is in the forest, so we ignore this. Rule 6 says the same, so we ignore that one too. Rule 7
says "If W is in-forest" W is in the forest, so look at this one. If W is in the forest, G has
to be. But G isn't in the forest, so this rule is broken. Cross off this answer and cross off
this diagram.
Answer D
Put G, M, S, and W in the not-in-forest group.
G H J M S W in-forest: H J
not-in-forest: G M S W
102 Game Three
Rule 1 says "If H is in-forest" H is in the forest, so we look at the "then" part. It says that
G can't be in the forest, and, indeed, G isn't. So far so good. Rule 2 says "If G is in-
forest", which isn't the case, so we ignore it. Rule 3 says "If J is in-forest" and J is in-
forest, so we look at the "then" part. It says that H has to be in the forest, and H is.
Another rule followed. Rule 4 says "If M is in-forest", but M isn't, so ignore it. Rules 5 and
6 talk about H not being in-forest, so we ignore them. Rule 7 says "If W is in-forest", but
W isn't. Rule 8 says "If G is not-in-forest", which is true, so look at the "then" part. W
can't be in the forest, and W isn't, so this rule is followed. Rule 9 says "If J is not-in-
forest", but J is, so we ignore this. Finally, rule 10 says "If S is not-in-forest" S is not in
the forest, so we have to have J in the forest, and it is. This diagram follows every rule.
This (answer D) is the right answer.
A couple of points about this question. First, you see how necessary it is to realize that
everyone not listed in the answer is in the other group. If you hadn't listed the "in-forest"
birds, you couldn't have gotten this right. Second, you should see that you need to develop
the skill of skimming down lists of "If then" rules, looking only at the ones that are
relevant to the situation you are working on. If you can't do this, you will be very slow.
Third, you should see that it is important to realize that this is not a "Which is a complete
and accurate list" question, but really a "What can be true?" question. All of these are
skills you can practice or ideas you can memorize, so start doing so.
Question 7: An If-question do it now
We do If-questions first, along with "What can be true?" questions.
The first step in every If-question is to redraw our original diagram.
G H J M S W in-forest:
not-in-forest:
The second step is to write down the situation described in the question. In this case, we
put M and H in-forest and cross them off our list.
G H J M S W in-forest: M H
not-in-forest:
The third step is to go through our list of rules and see what has to be true. Remember,
every time you figure out something new that has to be true, go back through the list of
rules one more time. Also, when looking at "If then" rules, only pay attention if the "if"
part matches what you have written down in your diagram.
Rule 1 says "If H is in the forest" Well, H is in the forest, so let's look at the "then" part.
G can't be in the forest, so we put G in the not-in-forest group and cross G off our list.
G H J M S W in-forest: M H
not-in-forest: G
Since we've learned something new, we have to go back through the rules. Not too hard
when we've only looked at rule 1. Rule 2 says "If G is in-forest", but we just learned that
G isn't in-forest, so ignore this rule. Rule 3 says "If J is in-forest", but we don't know if J
is the forest or not. Ignore this rule. Ignore a rule if you don't know whether or not the "if"
part is true. Rule 4 says "If M is in-forest" and we do know that M is in forest. So we also
know that the "then" part has to be true. This tells us that H is in-forest, which we already
Game Three 103
knew. Since we haven't learned anything new, we don't change our diagram, and we don't
start going back through the rules.
OK, on we go. Rules 5 and 6 say "If H is not-in-forest", so we skip these. Rule 7 says "If
W is in-forest", but we don't know one way or the other, so we skip this. Rule 8 says "If G
is not-in-forest" Bingo, we know that G isn't in forest, so we look at the "then" here. W
can't be in the forest. Add this to your diagram.
G H J M S W in-forest: M H
not-in-forest: G W
Go back through the rules; nothing changes up to rule 8 (reread the previous two
paragraphs to simulate going back through the rules if you'd like). So let's continue from
there. Rule 9 says "If J is not in-forest", but we don't know about J one way or the other.
And rule 10 says "If S is not in-forest", but we don't know about S either, so we skip this
one too. We know the status of G, H, M, and W; J and S may or may not be in the forest.
Let's look at the answers. This is a "What must be true?" question, so we can test the
answers out by doing something different from what they say.
Do you see how putting these rules together works? Not so hard, as long as you are
methodical, and are comfortable with it. You need to practice the skill until you just
automatically skim through lists of "if"s, ignoring those that aren't relevant to your
situation. Once you can do this, going back through the list every time you learn something
new doesn't take much time. Get used to this and you'll be able to figure out all sorts of
exciting stuff on games like this one (and they are pretty common about one in five
games has three or more "If then" rules).
Answer A
Before we test this answer out, we have to understand what it means. What do they mean
by "only other birds"? They mean "other than what we've just told you about." Specifically,
other than M and H. This would be false if another bird was in the forest, or if S wasn't (see
"If then" Rules and Negations). We test this answer out by doing something other
than what it describes. In this case, let's not put shrikes in the forest, and put someone
else in. Alternately, we could have both S and someone else in, or neither S nor anyone
else. Who is another bird that we can put in? J's the only other one unassigned, so let's
put J in the forest (and not S).
G H J M S W in-forest: M H J
not-in-forest: G W S
Go through your rules and see if this works. The only rule that applies here that didn't
apply before is rule 3. It says "If J is in-forest, then H is in-forest." That is followed here,
so this is possible (we know that everything we already had follows the rules). We know
that this answer doesn't have to be true, because something different than it can be true.
Cross the answer off.
Let's use this diagram and try to eliminate some other answers. We can eliminate an
answer if the situation it describes is different from what this diagram shows. Answer B
wants just J, M, and H to be in the forest, which is what we have here, so we can't eliminate
it. It might have to be true. Answer C says that J and S should not be in the forest. Well,
this diagram has J in the forest, so C doesn't have to be true. Cross it off. D says that
there have to be two or more birds in-forest besides M and H. We know that doesn't have
to be true because this diagram only has one other bird in-forest. We can cross off D. E
104 Game Three
says that there are no more than two birds besides M and H in-forest. That agrees with this
diagram only one other bird is in-forest, which is not more than two. We can't cross off E,
because it might have to be true. So we have to test out either B or E.
Answer B
Again, "other" here means "other than what we've told you" (M and H, specifically). The
answer says that J is the only bird other than M and H in the forest, so everyone else is not-
in-forest. We can't use the diagram for A because it has J in-forest, which is just what the
answer says. So let's draw a new diagram, and try not putting J in the forest, while putting
someone else in (who'd have to be S, since no one else is left). Again, we also could have
tried putting both S and J in, or leaving both out.
G H J M S W in-forest: M H S
not-in-forest: G W J
Look through our rules. The only one that applies here that didn't apply initially is 9, which
says "If J is not-in-forest", so I'm not going to rewrite the process of checking each of
these (but you should go through it). Rule 9 is followed: S is in-forest. So this diagram
follows all the rules. Cross off this answer.
This leaves only E, so E is the answer.
Question 8: An If-question do it now
Redraw your diagram and plug in what you are told.
G H J M S W in-forest:
not-in-forest: J
Now we go through the rules. I'll remind you again, when you look at "If then" rules,
only pay attention if the "if" part matches what you know to be true. Rules 1 and 2 talk
about H and G, who we know nothing about so far, so we ignore them. Rule 3 says "If J is
in-forest", but that isn't the case, so we ignore it. Rules 4 through 8 are about birds other
than J, so we ignore them too (yes, I know that rule 5 has J in the "then" part, but we
aren't looking at the "then" parts, just at the "if"s). Rule 9 says "If J is not in the forest",
which is the case, so we know that the "then" part of this rule has to be true. S has to be in
the forest.
G H J M S W in-forest: S
not-in-forest: J
Since we've learned something new, go back through the rules. Go ahead, I'll wait. Did
you see any rules saying "If S is in-forest"? No? Then no rules are relevant here, so we
end up ignoring them all. But you have to look at them, otherwise you might miss
something.
This is a "must be false" question, so the answer is something that can't be true. Any
answer that is possible can be crossed off.
Answer A
Test this out by putting M in-forest.
G H J M S W in-forest: S M
not-in-forest: J
Game Three 105
Go through the rules. Rule 4 is relevant. It tells us that H has to be in the forest.
G H J M S W in-forest: S M H
not-in-forest: J
Since we just added something new, go back through the rules. Rule 1 is relevant; we can't
have G in the forest.
G H J M S W in-forest: S M H
not-in-forest: J G
Keep going. Skip rules 2 and 3, because their "if" parts aren't the case here; we've already
looked at 4; rules 5 and 6 aren't the case (because we have H in-forest); we don't know
about W, so we skip 7; ahhh, rule 8. G is not-in-forest, so we can't have W in-forest.
G H J M S W in-forest: S M H
not-in-forest: J G W
One more time through the rules. We've looked at everything relevant already except 9,
and this diagram agrees with 9. This is a possible diagram. Cross off answer A. Now we
have a diagram we can use on the other answers.
The diagram tells us that H can be in-forest, so cross B off. Skip C because we can't use
our previous diagram it has both M and H in-forest, so it doesn't show us that M and H
can be not-in-forest. When we get to D we are done; it can't be true. We know it can't be
true because before we even looked at answer A, we learned that S has to be in the forest.
So it isn't possible for neither M nor S to be in the forest. "Neither" means that both M and
S are not-in-forest, but that can't happen if S has to be in-forest. So D is the answer.
Question 9: A "What is the maximum number of ?" question do much later
These questions are hard and time-consuming; we'll do it later, if at all (I'll explain a quick
way to make a good guess at these questions later on).
Question 10: A "What can't be true?" question do it the second time through
Question 11: An If-question do it now
Again, redraw your diagram and plug in what you are told.
G H J M S W in-forest: G
not-in-forest:
Let's go through our rules and see if anything has to be true before we look at the answers.
I'll say it for the millionth time (because it's so important): skip over "If then" rules
when you don't know that the "if" part is true (i.e., when it contradicts what you have in
your diagram, or talks about something you don't know about). Ignore rule 1 because we
know nothing about H. Look at rule 2, because G is in-forest. We know that H can't be in
the forest, so add this to our diagram and start over with the rules.
G H J M S W in-forest: G
not-in-forest: H
Ignore 1 because H is not in the forest; ignore 2 because we just looked at it. Ignore 3 and
4 because we know nothing about J and M yet. Look at rules 5 and 6 because both start
with "If H is not-in-forest" We learn that J and M are not-in-forest.
106 Game Three
G H J M S W in-forest: G
not-in-forest: H J M
Start over with the rules. Rules 1 through 6 don't give us anything useful; they are either
things we ignore, or thing we've already looked at. We ignore rule 7 because we don't
know about W; we ignore rule 8 because G is not not-in-forest. Look at rule 9, because J is
not-in-forest, so we know that S has to be in-forest.
G H J M S W in-forest: G S
not-in-forest: H J M
Back through the rules one more time. Nothing new here, so we are done. The only bird
we don't know about is W.
I hear you saying to me, "Brian, why do you always make us go back through the rules?
We've just looked at them, so you are wasting our time." Not so! Take a look at 8A. We
go back to rule 1 after looking at rule 4, and doing so allows us to figure out many other
things. Now, I'll admit, this doesn't happen every single time. But it happens often enough
to be important. It doesn't take us very long to look back through the rules, does it? OK,
well, it may seem like it takes a long time to you now, but that's just because you need to
practice. Get good at it (and it's not that hard) and you'll find that it doesn't take too long.
But if you don't do it, you'll miss things like question 8.
This is a "What must be true?" question, so we are going to test the answers by doing
something different than what they describe.
Answer A
Well, we don't even have to test this out. The work we've done so far tells us that S must
be in-forest, because it's in our diagram, so there is no way this could not be true. We're
done, this is the answer.
Question 12: An If-question do it now
This doesn't look like an If-question at first glance it has the word "suppose" at the very
beginning. But the first sentence is just telling you "Imagine we had a new rule," which is
just what If-questions do. This new rule just happens to be an "If then" rule. "Suppose"
in a question makes it an If-question; see Common Equivalents for Each Question Type
in the chapter The Fundamentals.
Draw your diagram, and then write down our new rule (and its contrapositive, since it is an
"If then" rule).
G H J M S W in-forest:
not-in-forest:
If S in-forest, then H not-in-forest.
If H in-forest, then S not-in-forest.
This is a "What can be true?" question, so we'll just work through the answers until we find
one that can be true.
Game Three 107
Answer A
Let's test this out by putting J and S in-forest.
G H J M S W in-forest: J S
not-in-forest:
Go through the rules. Look at the new rule (the one given by the question when it says
"Suppose") first. S is in the forest, so H can't be.
G H J M S W in-forest: J S
not-in-forest: H
Now go through the regular rules. Ignore 1, because its "if" part isn't the case: H isn't in
the forest. Ignore 2 because we don't know about G. Rule 3 says "If J is in-forest" and J is
in-forest, so we know that H has to be in-forest. But wait, H can't be in the forest (because
we have it not-in-forest and we don't question our previous work). So this breaks a rule.
Cross out the answer and cross out the diagram.
Answer B
Put W and S in-forest.
G H J M S W in-forest: S W
not-in-forest:
Go through the new rules, the ones from this question, first. If S is in the forest, H is not.
G H J M S W in-forest: S W
not-in-forest: H
Now go through the regular rules. Ignore 1, because H isn't in the forest. Ignore 2, 3 and
4 because we don't know about G or J or M. Look at 5 and 6, since we know that H can't be
in the forest. This means that J and M can't be in the forest.
G H J M S W in-forest: S W
not-in-forest: H J M
Go back through the rules. Nothing new until rule 7; if W is in-forest, G has to be.
G H J M S W in-forest: S W G
not-in-forest: H J M
Back through the rules again. Rule 2 tells us that if G is in-forest, H is not-in-forest. This
agrees with what we have. No other rules are relevant here. So this diagram works; it
(answer B) is the correct answer.
Question 9: A "What is the maximum number of?" question do much later
Still skipping this one; we don't do "What is the maximum number of?" questions until our
fifth time through.
108 Game Three
Question 10: A "What can't be true?" question we're doing it now, our second
time through
The right answer will be a pair of birds who can't both be in the forest; the wrong answers
will be pairs who can both be in the forest. If they say "What pair (or what group, or what
list) can't have such and such a quality," they mean "at the same time." The same is true
if they say "What pair/group/list can have such and such a quality;" they are looking for a
pair/group/list that can all have the quality at the same time.
We can (and should) use our previous diagrams on this question. We are looking for birds
who can't both be in-forest, so eliminate any answer containing a pair of birds who show up
together in-forest in a previous diagram.
Answer A
We don't have J and W both in-forest in any previous diagram, so skip it.
Answer B
J and S aren't both in-forest in any previous diagram; but, the diagram for 7 looks pretty
close we have both J and S as possibilities. Still, let's skip it for now.
Answer C
12B has S and W in-forest.
Answer D
The diagram for 7A has J and M in-forest.
Answer E
Both 7B and 8A have S and M in-forest.
At this point we have A and B left. Now, if we were pressed for time I'd just eliminate B;
the diagram we made for 7 (before we tried out the answers) makes it look like we can
have both J and S. But, since we have time, let's test out an answer; we'll test out A
because I like to go in order.
Put J and W in-forest and see what happens.
G H J M S W in-forest: J W
not-in-forest:
Go through the rules. Rule 3 tells you that you have to have H in-forest (because we have J
in-forest).
G H J M S W in-forest: J W H
not-in-forest:
Go back through the rules again. Rule 1 tells us that we can't have G in-forest, because H
is in-forest.
G H J M S W in-forest: J W H
not-in-forest: G
Keep going through the rules. Rule 7 tells us that if we have W in-forest, we have to have
G in-forest. But G is not-in-forest; this diagram breaks a rule, so A is the answer.
Game Three 109
Question 9: A "What is the maximum number of?" question our last question,
so do it now
A great question to skip if we are running out of time. There's a decent way, if you are
running out of time, to make an educated guess on a "What is the maximum number of?"
question: look at your previous diagrams and find the maximum number of whatever the
question talks about. Pick that as your answer (since you know that it is possible). Do the
opposite for "What is the minimum number of?" questions find the diagram with the
minimum number of whatever, and pick that.
Here, the most we have in-forest at once are three birds (diagram 7A, for example). So
we'd pick B. We'd be wrong, but that's not what guessing is for. Guessing is not supposed
to get you the right answer reliably; otherwise we'd guess all the time. Guessing is
supposed to get any answer fast, so that you can move on to questions you'd rather work
on. If it happens to get you the right answer, well, that's just a bonus.
But how do we do this question if we are not going to guess? We look at the biggest
answer (the smallest one if this asked for the minimum) and try it out. Here we try to have
six birds in-forest. Since there are only six birds, that means picking all of them.
Answer E
G H J M S W in-forest: G H J M S W
not-in-forest:
Now go through the rules. You immediately see that we can't have this we can't have
both H and G. Cross off the diagram, and cross off E. Go to the next biggest answer.
Answer D
We are going to have to get rid of one of the birds. Well, we know that we have to get rid
of G or H, because we can't have both of them. We want to get rid of a bird who causes
problems one where having it in-forest forces many birds to go not-in-forest and one
where putting it not-in-forest doesn't make any other birds go not-in-forest. Go through
the rules. If we put G not-in-forest, W has to go not-in-forest. But, if we put H not-in-
forest, J and M have to go not-in-forest. In either case, we are going to have to put more
than one bird not-in-forest, and we have to move either G or H. So we can never have five
birds in-forest.
Answer C
Let's move G, since putting G not-in-forest only makes us put W not-in-forest, while putting
H not-in-forest forces J and M to be not-in-forest. Let's see if this works.
G H J M S W in-forest: H J M S
not-in-forest: G W
Go through the rules. Rule 1 is followed; rule 2 is irrelevant, since G is not-in-forest. 3 is
followed, 4 is followed, and we ignore 5 and 6. Ignore 7, and 8 is followed. Ignore 9 and
10. So all the rules are followed; we can have four birds in-forest. Thus, C is the answer.
Whew, that's a lot of work. See why we skip these questions? Here's the way we do them,
in summary: Try the biggest answer. If that doesn't work, move one down and see if you
can make that work somehow. If not, move one more down, and so on. Try to make
changes that cause the least trouble; that is, ones that force you to make the fewest
changes. You want to change things that don't impact the rules much.
110 Game Three
Summary
We have learned to make "not" groups when the game might leave some people out of the
main group. We have learned the importance of understanding "If then" rules and their
contrapositives. We would not have been able to do as well on these questions as we did
without having the contrapositives of each rule written down. We have learned how to scan
through lists of "If then" rules by ignoring those that aren't currently relevant. Make
sure you understand these skills; from the year 2000 to the present, almost one in five
games has had three or more "If then" rules (with one game having eight of them!).
This means you are very likely to get a game with a list of these rules on your LSAT.
What Do I Do Next?
Take some time before you go to look back at the questions on this game, and especially to
practice skimming through lists of "If then" rules.
At this point you have been exposed to all the fundamental concepts you need to do well on
logic games. Now you need to practice some logic games. I provide a list of recommended
games below. Work through these games, applying the studying skills I talk about in the
Studying Guide. Once you feel comfortable doing these games, continue to
Intermediate Techniques.
I've also put a brief summary of the most important logic games skills in the next chapter,
The Five Most Important Skills. Take a look at this before you start practicing games.
Timewise, you want to move on to the intermediate techniques at least three weeks (and
hopefully more) before the LSAT. This gives you a week to work through Intermediate
Techniques and the ensuing five games, and to practice the techniques, and then gives
you two weeks to work on timing (I'll talk more about this later).
Answer Key
6. D
7. E
8. D
9. C
10. A
11. A
12. B
Recommended Games
I have grouped the recommended games together by similarity to each other; while I
believe, as I teach, that there is only one type of logic game, and that all games can be
diagrammed by applying the same technique, some games are more alike than others. Try
to divide your work somewhat evenly between the different groups listed below; this will
ensure that you get practice with a variety of different types of rules and setups. "Game #"
refers to the order in which the games are given in the specific LSAT; for example, game 3
is the third game in that LSAT. Again, you don't need to do all of these games before
continuing on in this book. Do enough so that you become comfortable with the
fundamental techniques get to the point where you remember what to do when, and
understand how to apply the techniques in most situations. I have excluded from this list
games that you are better off doing after covering the next few chapters.
Game Three 111
If you have access to LSATs which are dated but not numbered, there is a chart (LSAT
Number to Date Conversion Chart) which gives the date each number corresponds to at
the end of the Introduction chapter.
Group One Group Two Group Three
LSAT
19
23
24
26
27
29
30
32
32
33
34
34
40
41
42
42
43
43
45
46
47
48
48
Game #
1
1
2
2
1
3
4
1
3
1
1
3
1
1
2
3
1
2
1
1
1
2
4
LSAT
19
19
24
26
27
29
29
34
35
37
42
43
45
46
47
47
48
Game #
3
4
1
3
3
1
2
4
2
3
4
4
4
2
2
3
3
LSAT
20
20
23
25
32
33
35
36
40
42
45
48
Game #
2
3
2
1
2
3
1
1
4
1
3
1
112 The Five Most Important Skills
The Five Most Important Skills
(In order of importance)
You've read The Fundamentals. You've worked through three games with me. You are
probably starting to get a sense of how to do games the right way, and you are about to
embark on the great adventure of practicing games on your own. At this point I feel it is
crucial to make sure that you know what the most important skills for you to practice are,
and also to make sure you understand them. These skills are good candidates for putting
on your short list when you study. If you don't know what a short list is, you need to
reread Studying for the LSAT Logic Games Section.
These are only the top five skills, and, of course, there are many more. One useful resource
for you is the Glossary at the end of this book; there I define all the terminology you need
to know, and give brief summaries of what to do when you see each term on the test. It
may be helpful for you to read through that and make sure you understand each definition
given.
Here I will only touch on each skill, since I've gone through them in much more detail in
previous chapters.
1. Know how to read LSAT sentences.
If you can't understand what the LSAT is telling you, you won't do well on it. Reading on
logic games is always an active process. You never just look at the words in front of you;
instead, you are always looking for something, reading with a goal in mind. When reading
the setup, you are trying to figure out what things you are matching together. When
reading rules, you are trying to figure out what they mean. When reading a question, you
are trying to figure out what type of question it is and how to test the answers. Make sure
you read every word on the page, which means you need to read slowly and carefully; the
easiest way to miss questions is to miss one crucial word in a rule or question. Always ask
yourself what each sentence means. If you don't understand something, try to make it
clearer using an example. For example, if a rule doesn't make sense, try applying it to a
specific situation and see what happens; people think better about what is concrete, rather
than abstract.
You need to understand common LSAT vocabulary, such as "always," "only," "exactly,"
"neither," "except." If you don't know what these words mean yet (or how to negate
them), make some flash cards and go through them every day. A more comprehensive list
of these words is in the Glossary.
2. Know how to recognize the question types.
This is a pretty straightforward skill, but it's absolutely crucial. If you aren't 100% on this,
there is a list of common ways each question type is worded in The Fundamentals. If you
don't have these memorized yet, start using flash cards (I explain a good way to make flash
cards at the very end of The Fundamentals).
The Five Most Important Skills 113
3. Know how to determine if an answer to a question type is right or
wrong.
This includes knowing when and how to use previous diagrams, and how to draw test
diagrams for each type of question. The Glossary has a brief summary for each question
type, and The Fundamentals goes over this in more detail. If you find that you have to
struggle to remember this when you do practice games, you need to make flash cards with
this information and go through them every day.
4. Know how to make test diagrams.
The process for making test diagrams is, briefly, to redraw your diagram, put in whatever
you want to test, and then go through the rules and add to your diagram whatever the rules
tell you. When you are finished going through the rules, fill any empty spaces with whoever
is left, moving them around if the rules so require. If you can fill all the spaces without
breaking a rule, the diagram works; if you can't, it doesn't.
Here are all the steps, in order, in a little more detail:
a) Recopy your original diagram (or, if this is an If-question, the diagram you
drew when first setting up the question, which includes the rule the "If" gives
you, and anything that causes to be true).
b) Put in what the answer wants you to test (unless this is a "What must be true?"
question, in which case you put in something other than what the answer says).
c) Go through the rules does this new information cause something else to
happen? If so, write it down.
d) If you wrote something new down, return to step (c) and go through the rules
again (every time you learn something new, go through the rules and see if the
new thing makes something else happen).
e) Once you have gone through the rules and not added anything new, see if there
are any empty spaces in the diagram. If so, fill them in from left to right in
alphabetical order (to the extent the rules allow). As you put things into the
diagram, mark them with a dot or a star. Every time you put someone in, go
back through the rules again [step (c)].
f) Have you broken a rule? Try moving someone who is marked with a dot or a star
(from left to right). If no one is marked with a dot or a star, or this doesn't help,
the diagram won't work.
g) Have you filled all the spaces? The diagram works.
5. Be comfortable going through the rules.
When you go through the rules, start with the first rule you have written down and go
through the rules in order. It is essential to be systematic, because this ensures accuracy
and minimizes effort (which is important for speed and to reduce fatigue). Look at the rules
you wrote, not back at the way the LSAT has them (these are harder to read). Recognize
that sometimes rules don't tell you anything helpful about your current situation. For
example, in general, rules that say two people or things are together, or next to each other,
or not together or not next to each other, aren't worth thinking about if you don't know
anything concrete about either person or thing. This isn't true if you have only a few spaces
left in the diagram, though; at that point, there may be only one or two places to put these
people, and every distinction counts.
114 The Five Most Important Skills
This is not a skill that can be learned in the abstract. It has to be practiced the natural
human tendency is to skip around, which leads to skipping rules and making mistakes. As
you go through games, both in reading the explanations in this book and in practicing by
yourself, pay attention to which rules are important when. Eventually, you will be able to
go through lists of rules quickly and efficiently, putting them together with a minimum of
fuss.
Intermediate Techniques 115
Intermediate Techniques
By now you should be starting to get comfortable putting together diagrams for games,
writing down the rules, putting the questions in the right order, testing situations for
answers, and using previous diagrams. There are a few other things you might try doing
that will make your life a little bit easier. These aren't essential to your LSAT success, but
they can help quite a bit. Some of these are things that you might be doing already, or are
thinking of doing, but aren't sure if you should.
Diagramming with Columns
You might have noticed that we redraw our diagrams often. This generally doesn't involve
writing much maybe just hastily scribbling down 1 2 3 4 5 6, or R S T Y, or the names of
six birds. It doesn't take much time, but it does take some, and time is precious. There is
a way to save yourself a couple of seconds here and there which may also make your
diagrams easier to read.
Instead of redrawing your diagrams from scratch every time, when you first draw your
diagram, give yourself room and make columns underneath each space in your diagram. If
you draw the diagrams vertically, make rows. For example, on Game One, it would look
like this:
1 2 3 4 5 6
Your original diagram goes at the top of the columns, and you do the test diagrams for each
answer underneath; this allows you to test diagrams quickly without doing any redrawing.
We do all of our work in this chart, and, of course, cross off anything you put in there that
breaks a rule. For example, after a few questions, our chart on Game One might look like
this:
1
B
3
B
3
D
1
M
M
O
2
K
L
K
3
P
P
P
P
4
L
K
L
5
N
O
M
6
O
N
N
116 Intermediate Techniques
On Game Two, our chart might look like this (notice the first diagram breaks a rule, and is
crossed out):
6C
6D
R
__
H
L*
S
__
__
H
G
G*
H
T
__
__
H
L
H
P*
Y
__
__
__
L
P
__
L
P
H
This technique doesn't work that well for Game Three, mostly because that game doesn't
really have anything we can put into columns.
This technique is totally optional. It can save you a little time (not that much, but some),
and being organized like this may be helpful as well. But it won't work on some games
because you just don't have the room to do it. Try it out and see if you like it.
Remembering Rules
After you've tested out anywhere from a couple of answers to a couple of questions, you
might notice that certain rules stick in your mind. You know to use these rules before
looking at the list of rules you have written down. For example, in Game One, the rule
about L and K having one spot between them might have stuck in your mind; you might
have wanted to fit L and K into the diagram before anyone else because you knew they had
to have that relationship. In Game Two, you might have found it easy to remember that G
couldn't be with L or P, so that anytime you put G in a spot you immediately knew without
looking at the rules that you couldn't put L or P there.
This is fine. Certain rules will stay in your head without you having to try to memorize
them. Use them, and use them before you look back at the other rules. I often find that on
games with relatively few rules, like Game One and Game Two, all the rules stick in my
head after I do a couple of questions.
Finally, don't try to memorize the rules it's unnecessary, it's tough, and it wastes time. If
you find that you do memorize some of them as you go, without trying, that's good. If you
find that you have to look at the list every single time, that's fine too, just make sure you
do it.
Making Deductions from Questions Part 1
Once you've answered a "What can't be true?" question, you've discovered something that
can't be true, in any diagram. That means you can use this knowledge to help you on other
questions. You can write what you just learned as a new rule; often you can put it directly
into your diagram.
Look at Game One, question 3. The correct answer is E: O cannot be assigned to position
2. We can write this right into our diagram put a little "no O" over position 2 or we can
just write down "O not in 2" as a new rule. That's sorta handy.
Intermediate Techniques 117
This is likewise true about "What must be true?" questions. Their answers must be true
always, so they give us new rules as well. However, you cannot get new rules from If-
questions, even if they ask What must be true? or What cant be true?; what must be
true or cant be true in an If-question must or cant be true because of the new condition
the question added, so wont always be the case in other questions.
You might be wondering: if this is true, why don't we do these questions earlier on in our
process? That way we could have the benefit of these new rules for the other questions.
That is a good question, and it has a good answer. There are two reasons. The first, and
most important, is that these questions are generally harder and more time consuming.
You've seen this. Saving them for later makes them easier, because it allows us to
accumulate previous diagrams that we can use to eliminate answers. You've seen how
handy that is it saves us a huge amount of work, more than would be saved by doing the
"What can't be true?" and "What must be true?" questions sooner.
This leads us to the second reason why we don't do these questions sooner. The insights
you get into the game by doing these "What can't be true?" and "What must be true?"
questions are often not terribly useful. Look at Game One. How often in that game is
knowing that O won't go 2
nd
useful? Never. We end up never even trying to put O 2
nd
,
even without knowing this rule, because there is always someone else we want to put 2
nd
.
For another example, look at question 4. Answer B, the correct answer, tells us that K or L
has to be in position 4, which is something like a new rule. How useful is that? Only a tiny
bit useful, because our application of the rules automatically forces us to put one of them
there anyway. Again, we never (or almost never) even tried to put someone else there, so
knowing that no one else could go there wouldn't have saved us a significant amount of
time.
Now look at Game Two, question 8. It is a "What can't be true?" question. Look at it and
ask yourself how useful knowing the answer will be on later questions. It might be a little
useful to know that no one can learn R and T, but not Y, but only a little. Compare that
usefulness to how much time we save using previous diagrams to answer this question. We
end up doing no work to solve this question because we use our previous diagrams.
Now, once in a while, you may find that an answer to one of these types of questions is
useful in answering other questions. That's why I'm telling you that you can use these
answers as new rules. The great majority of the time, however, doing this won't save you
more time than doing the questions in the order we've discussed. So we are going to play
the good odds and continue to do the questions in the right order.
Making Deductions from Questions Part 2
From time to time, you will come to realizations as you work through questions. For
example, as you work through Game Two, you may realize that G will never go under R
(because G has to go with H, and there is only one space under R). Or you might notice
that G can't go under Y (because if it does, L or P can't go under Y, and then you can't fill all
three of the spaces under Y this might also show you that H, P, and L would have to go
under Y, because you have to fill those three spaces with someone, and G can't be used).
In each of these cases, you could write down what you've figured out as if it were a rule, so
that you can use it to help out on future questions.
Notice though, that in each of these cases you realize something has to be true, or can't be
true, and you can give a reason for it. It's not enough to think something is true, or has to
118 Intermediate Techniques
be true, or to be pretty sure. You must be sure, and to be sure you must be able to give a
conclusive reason for it based on the rules. If you realize, while doing questions, that
something must be true, based on the rules, write it down. If you are not sure, don't write
it down.
Redrawing Diagrams What Not to Copy
Redrawing your diagrams is a wonderful, wonderful technique. It is easy, necessary, and
extremely useful. But not every part of every diagram is equally necessary. I often find
that writing down the names of all the variable things the list of F G H I, etc., that we
write down to the right of our main diagram is not that useful, and its a little tedious. On
most games where every thing is used once (such as Game One and Game Two), it's
easier for me not to copy the names down. But you need to be able to figure out who the
last person left is, so you can fill that last empty space; for this I just run through the main
diagram, checking against each person on the list, until I find an unused one. With a bit of
practice this is fast and easy.
On the other hand, I have students who always make mistakes unless they write down all
the names and cross them off as they go. If you find yourself leaving people out, using
people twice, or just forgetting what to do next, err on the side of caution and copy
everything down every time. It doesn't take that long. But I did want to let you know that,
if you are careful, you can get away with not doing it. You do have to copy down the rest of
your diagram, though. We're being efficient, not foolish.
Picking the Answer to Test
In general, you should test the answers in order, from A to E. Anything else requires too
much thought, and thought is the enemy of all good LSAT-takers. However, there is one
(maybe two) situation(s) in which you might not want to test the answers in order.
Here is the most important one. You have eliminated all but two answers, usually by using
previous diagrams (which means you'll have at least glanced at both of the remaining
answers). You'll only have to test one of the answers, since it will either be right, in which
case you are done, or wrong, in which case the other is right, and you are still done. So it
doesn't really matter which you test. If one of the answers seems either pretty obviously
right or pretty obviously wrong, or really easy to test, then test that one.
The other case where you might not want to test the answers in order is this: you are using
previous diagrams to eliminate answers, and you come across an answer which you are
pretty sure is right (but no previous diagram proves that it's right, otherwise you wouldn't
even be considering testing it out). Try that one, you might save yourself some time; just
be careful to remember which answer you are testing. Doing things out of order has a habit
of confusing people, which is why I am usually against it, so you should be very confident
that you are right; don't just do this on a whim, or it'll cost you at some point.
What Do I Do Next?
Go through Game Four through Game Eight. These games will demonstrate some of
these intermediate techniques. These games will also show you how to apply your
fundamental techniques to games that people consider odd or difficult.
Game Four 119
120 Game Four
Game omitted from electronic version of book.
Game Four 121
Game Four
(From LSAT 30)
By this point you have learned and practiced the most important skills you will use on LSAT
logic games. I've given you a series of techniques that you can memorize and apply to
every logic game you come across. But sometimes the LSAT will throw something at you
that seems extremely difficult, that seems not to fit into what you understand. In the next
five games, I will demonstrate that even these seemingly different games are not, in fact,
different at all. We don't need to learn anything new or approach the games in a way other
than we have been. As long as we stick to our fundamentals and do what we have learned,
these games will be as straightforward as anything else.
Now that we've gone through a few games, I'm going to go a little more quickly through the
explanations of the steps. We'll still do everything you should do, but I won't explain the
fundamentals in quite as much detail. If something doesn't click, refer back to the previous
chapters. Don't work through this game on your own yet; read along with me until you get
the hang of how we are doing things, and then, if you want, finish the rest of the questions
on your own before looking at the explanation.
Step 1: Read the Setup/Draw a Diagram
Read the setup and see if you notice something different about this game. I've told you
before that every game involves two or more types of things, and you are figuring out which
of one type goes with which of the other. Up to this point, though, we have only seen
games with two types of things horses and positions, or researchers and languages. This
game has three: cars, order (first, second, etc.), and types of washes. For each car we
want to figure out where in the order it is washed, and what type of wash it receives; for
each number in the order we want to figure out what car goes there and what type of wash
it gets.
This does not require any new way of thinking. We build our diagram in the exact same
way. Which of the three types of things order, cars, washes do we know the most
about? Which will change the least? The order; numbers never change the order they are
in. So we will build our diagram around the order. Since there are five cars washed one at
a time, there are five possible numbers (if more than one car could be washed at once,
there could be fewer numbers):
1 2 3 4 5
One car will go with each number, and one type of wash. So we put a space under each
number for the car, and a space for the wash; car names (O, V, etc.) go in the top spaces
and the wash types they get in the bottom ones.
1 2 3 4 5
__ __ __ __ __
__ __ __ __ __
Finally, we put in the names of the cars and washes.
1 2 3 4 5 Cars: F M O T V
__ __ __ __ __ Washes: r, s, p
__ __ __ __ __
122 Game Four
You will see games like this, which involve more than two types of things, regularly on the
LSAT. They are not harder than any of the games we have seen before, and they don't
require any different kind of thinking. We just read what we are given and work through
the rules exactly like before.
Step 2: Read the Rules
Many of these rules are actually two rules disguised as one, like the first rule. It tells you
not only that the first car washed doesn't get a super wash, it also tells you that there has
to be at least one super wash. At least means there has to be one s and maybe more.
You need to write both of these things down.
1 is not s
At least one s
The next rule contains the word exactly. This means that one, and only one, car gets a
premium wash.
Exactly one p
The next rule is also simple:
2 and 3 same wash
This next rule can be written as two rules, one about O and one about T; we break it down
to make it easier to understand. We also translate it into a way we easily understand. If O
isn't before V, then V must be before O. Usually we say that someone is before someone
else, not "not before." Try to translate rules into a format you've used before and are
comfortable with.
V before O
V before T
This next rule should also be written as two rules. I translate "after" into "before."
M before F
O before M
The last rule is more complicated; breaking it into two rules one about M and one about
the car before M makes it easier to apply.
M is r
r before M (and next to)
I put in the "and next to" to point out that the r comes immediately before M. You could
write is somewhat differently (e.g., "r right before M," or "rM"), as long as you understand
what you wrote.
Game Four 123
Here is our original diagram and list of rules:
1 2 3 4 5 Cars: F M O T V
__ __ __ __ __ Washes: r, s, p
__ __ __ __ __
1. 1 is not s
2. at least one s
3. Exactly one p
4. 2 and 3 same wash
5. V before O
6. V before T
7. M before F
8. O before M
9. M is r
10. r before M (and next to)
Step 3: Answer the Questions
Before we go through the questions, I'm going to list them in the order we do them in. By
this time you should understand that we do this by going through the questions in order,
skipping the ones that we know we shouldn't do yet; thus, the game may take us two or
three go-throughs to finish. This game actually takes several go-throughs: we do the
"What can be true?" and If-questions first, but that leaves a "What must be true?", a "What
is a complete and accurate list?", and an odd question. We end up skipping these three
questions the first time through.
Here is the order in which we should do the questions:
Question Number Question Type
11 What can be true?
12 If
13 If
14 What must be true?
15 Which is a complete and accurate list?
16 If (that changes the rules)
Question 11
Although the question says "accurate list," it asks for a list that "could be an accurate list,"
so this is a "What can be true?" question. For each answer, go through all the rules until
you find a rule that it breaks.
Answer A
The first wash isn't s, it's p, so this follows rule 1. There is one s, so this follows rule 2.
There is only one p, and the 2
nd
and 3
rd
washes are the same, so this follows rules 3 and 4.
But this violates rule 5: V should be before O, not after.
124 Game Four
Answer B
This follows all the rules, so this is the answer. Write this diagram down, so that you
recognize later on that it is a usable previous diagram.
1 2 3 4 5
V O T M F
p r r r s
Question 12
This is a tough question, as we'll see. It tests our ability to draw sample diagrams to the
limit. Are you ready? Are you LSAT-gamer enough? Let's do it!
First, we are told that V doesn't get a premium wash. OK, write that down.
V not p
Glance through the rules and see if that tells us anything about V. Well, we know that V
has to be before O and T, but we don't know much else about V. I don't see where it will go
(although you might), since it could be 1, 2, or 3 and still be before O and T, and I'm not
sure whether it will have an s or r wash. So let's work through the answers. This is a
"What must be true?" question, so we test the answers by doing something other than what
they describe.
Now, where V goes and what wash V has are both things that it turns out we could figure
out. But there's not really any way for us to know we could figure them out, so unless we
somehow just see it, we shouldn't try to do so. Looking for nonexistent deductions is one of
the biggest logic games time-wasters, so once you've looked through your rules, start
looking at the answers.
Answer A
Since the answer says O and V have the same wash, we want to try and give O and V
different washes. We don't know what wash V is going to get, so we don't know what wash
to give O, either. Here is where some people will panic we don't know anything definite
about O or V, so what do we do? How do we test this out?
OK, let's calm down. The best way to test things out is to make them definite draw a
diagram and test something. So let's draw our diagram and write down what we are trying
to test that O is not the same as V.
1 2 3 4 5 Cars: F M O T V
__ __ __ __ __ Washes: r, s, p
__ __ __ __ __ O not same as V
I feel better already. We want to test out O and V, so we want to put them in our diagram.
We don't know quite where they will go, but we'll just try places until one works, or we
realize that nothing will work. We know a whole lot about V, so we'll put her in first. Let's
just put her in the first available spot, spot 1. That's an easy way to start. Then we'll try
place 2, and so on. We put a little star next to V to remind ourselves that we could have
put V someplace else later on we might move V.
1 2 3 4 5 Cars: F M O T V
V* __ __ __ __ Washes: r, s, p
__ __ __ __ __ O not same as V
Game Four 125
Look through the rules before we go on. The question tells us that V can't have p; rule 1
tells us that position 1 can't have s. So V must have wash r, since that's the only wash left.
We don't star r, because this has to happen (given that V is first).
1 2 3 4 5 Cars: F M O T V
V* __ __ __ __ Washes: r, s, p
r __ __ __ __ O not same as V
OK, now we try to put O in. Let's try putting O in position 2, because that's the first
position we have available. If that doesn't work, we'll try 3, and so on. We'll put a little
star next to O to remind ourselves that this was a decision we made we can try moving O
if it doesn't work.
1 2 3 4 5 Cars: F M O T V
V* O* __ __ __ Washes: r, s, p
r __ __ __ __ O not same as V
Look through our rules before we go on. Remember, any time you write something definite
in the diagram, go through the rules. This works so far. Now we assign a wash to O. It is
supposed to be different from V, so we don't want to assign it r. That leaves p and s. Can
we give it p? Look through the rules. I'll wait.
No, space 2 can't be p, because 2 and 3 must be the same, and p only gets used once
(rules 3 and 4). We can actually write this as a rule, because we see that it must always be
true.
11. 2 not p
12. 3 not p
So O has to be s, which makes position 3 s (rule 4). Again, we don't star s because s has to
be where it is.
1 2 3 4 5 Cars: F M O T V
V* O* __ __ __ Washes: r, s, p
r s s __ __ O not same as V
Now we start putting in the others. Go through our rules. We've used at least one s, so
rule 2 is happy. We don't have a p, so one of our last two slots has to be p. V is before O;
that's fine. V is before T, so T will go in someplace after 2, but we don't know where. M is
before F, so M has to be either 3 or 4 if it were in place 5, there would be no room for F.
Then we see that M has to be r (rule 9), so M can't be 3; this means that it has to be 4
th
.
1 2 3 4 5 Cars: F M O T V
V* O* __ M __ Washes: r, s, p
r s s r __ O not same as V
Keep going through the rules. This breaks rule 10 the place before M is not r. So this
won't work. Go back to the last place we made a decision and change it (look for the last
place we marked). So let's move O (it's the last dot/star we put in, because M had to be 4,
since she couldn't have s or be last). Put it in position 3 (and cross our the diagram we just
made, since it breaks rules); remember, when you move people around, be consistent in
moving them in the same direction (generally left to right) so that you don't lose track of
where you have tried them.
126 Game Four
1 2 3 4 5 Cars: F M O T V
V* __ O* __ __ Washes: r, s, p
r __ __ __ __ O not same as V
What wash is O going to get? It can't get r (we are testing answer A, so we want O and V
to be different). We just figured out that 2 and 3 can't be p because there is only one p.
So they have to be s.
1 2 3 4 5 Cars: F M O T V
V* __ O* __ __ Washes: r, s, p
r s s __ __ O not same as V
M was a problem before; where can it go? Well, it has to go after O (rule 8), and it can't go
in position 5 (rule 7). But we just saw that if it goes in position 4, then 3 has to be r, and 3
isn't r. So there is no place M can go. We can't move O over any more, because that would
leave M even fewer places to go. So moving O isn't going to fix anything.
Let's look at our other starred person, V. Let's try moving V around. Put V in position 2.
1 2 3 4 5 Cars: F M O T V
__ V* __ __ __ Washes: r, s, p
__ __ __ __ __ O not same as V
Now look through the rules. V is before O and T, so they have to go somewhere in 3
through 5; we know that they can't be in position 1.
1 2 3 4 5 Cars: F M O T V
__ V* __ __ __ Washes: r, s, p
__ __ __ __ __ O not same as V
no O
no T
Look through the rules again (every time we figure out something new, we look back
through the rules). O is going to be before M, so M also can't go in position 1 (if O is
somewhere in 3 through 5, M has to be after it).
1 2 3 4 5 Cars: F M O T V
__ V* __ __ __ Washes: r, s, p
__ __ __ __ __ O not same as V
no O
no T
no M
Look through the rules again. Rule 7 tells us that M is before F, which means F can't go in
position 1 either. But this means no one can go in position 1. This means that V can't be in
position 2 ever. This is a deduction and we can write it down as a rule (see Intermediate
Techniques). We should also put V in position 1 in our original diagram, so that we always
put it 1
st
when we draw a new diagram.
13. V is 1
Game Four 127
Now, we've tried to move everything we can; we can't get a diagram that works. Since this
is a "What must be true?" question, and we can't get a working diagram, this (answer A)
must be the answer.
Notice that, while we had no definite place to start on this question, just calmly working
through it gave us the answer, and it gave us a lot of information about this game. Feel
better? Be systematic and stick to the rules, and you can answer any question. Worst-case
scenario, though, you realize that you just can't figure it out, so you skip the question and
do another one.
I also want to make another point. This seemed to take a long time. Perhaps you are
thinking it would have been faster if we had figured out that V had to be 1
st
ahead of time.
But how could we have figured out where V goes without doing most of what we just did?
We couldn't, so doing it sooner wouldn't take less time, it would just mean we spent that
time earlier. We still would have had to work out where O could or couldn't go. By waiting
to think about V until the answers force us to, we only do what we have to. If we had
thought about V sooner, it might have turned out not to be useful. But now, since the
question talks about V, we know thinking about V will be useful.
Question 13
We are told that position 4 and 5 have the same wash. Write this down.
4 and 5 same wash
Look through the rules and see if any tell you what wash this will be. It can't be p, because
we can only have one p. I don't see anything else jump out at me no rules tell us
anything about positions 4 and 5 so let's look at the answers. This is a "What can be
true?" question, so we just try out the scenario the answer describes.
Answer A
Let's test it out. Put O 3
rd
(and V 1
st
, which we figured out in question 12).
By the way, even if we hadn't figured out that V has to be first, we'd approach the next few
questions in a similar way. Since V has to be before a lot of people, when testing the
answers it is safe to put V 1
st
. We wouldn't know that it has to be there, but we know that
it works there.
1 2 3 4 5 Cars: F M O T V
V __ O __ __ Washes: r, s, p
__ __ __ __ __
Go through the rules. Rule 7 tells you that O has to be before M, so M will be 4
th
or 5
th
.
Last question, we saw that M can't be 5
th
. If you had forgotten that, when you looked at
the rules again, you would see rule 7 says that M must be before F, which means M can't be
5
th
, since F would have no place to go. Anyway, we put M 4
th
. Since M has to go there, we
don't mark it with a dot or star. Since we are on rule 7, we know that F has to go after M,
which puts F 5
th
. There is only one spot left (2), so T has to go there.
1 2 3 4 5 Cars: F M O T V
V T O M F Washes: r, s, p
__ __ __ __ __
128 Game Four
Now, rules 9 and 10 tell us that 3 and 4 must get wash r. The question tells us that 4 and 5
will have the same wash, so 5 is r as well. Write this down, and then go back through the
rules. Rule 4 tells us that 2 is the same as 3, so it is r also.
1 2 3 4 5 Cars: F M O T V
V T O M F Washes: r, s, p
__ r r r r
Go through the rules again. We have to have at least one s and at least one p. This is
impossible, so this diagram won't work. Cross off the diagram and cross off answer A.
Answer B
Put T 5
th
.
1 2 3 4 5 Cars: F M O T V
V __ __ __ T Washes: r, s, p
__ __ __ __ __
Let's go through the rules. V is before O, and O is before M, so let's put them in someplace;
put them in as early as possible, and we'll move them if we have to. Mark O because we
chose to put it 2
nd
(although it will end up that it had to go 2
nd
).
1 2 3 4 5 Cars: F M O T V
V O* M __ T Washes: r, s, p
__ __ __ __ __
There is only one spot for F 4 and rule 7 tells us that F has to go after M, so space 4 is
fine. Rules 9 and 10 tell us that 2 and 3 are going to be r.
1 2 3 4 5 Cars: F M O T V
V O* M F T Washes: r, s, p
__ r r __ __
Go through the rules one more time. 4 and 5 have to be the same according to the "If" part
of this question, and rule 3 tells us that we can only have one p. This means that 4 and 5
can't be p, so make them s (because we need at least one s). Make 1 p, since we need a p.
1 2 3 4 5 Cars: F M O T V
V O* M F T Washes: r, s, p
p r r s s
This works, so this (answer B) is the answer. See, not much harder than any other game.
Question 14
Since this is a "What must be true?" question, use your previous diagrams first (we have
diagrams from 11B and 13B). We look for diagrams which have situations different than
what the answer describes.
Answer A
Every previous diagram has V getting a p, so we don't know if it can be false or not. We
skip this one.
Game Four 129
Answer B
The negation of "exactly" is "fewer" or "greater"; this doesn't have to be true if we have a
previous diagram where there are fewer than two s, or one in which there are more. The
diagram for 11B shows that we can have just one s, so this doesn't have to happen (see
why I make you recopy the answers to these questions it is so easy to overlook that as a
previous diagram because it just looks like an answer choice). Eliminate this answer.
Answer C
Every previous diagram has 5 getting an s, so skip this.
Answer D
11B shows that 4 doesn't have to get an s, so this doesn't have to be true. Eliminate this
answer.
Answer E
2 gets an r in each previous diagram, so we can't eliminate this.
OK, so we have to go back and test some of A, B, and E. Let's test A first. Don't give V a
premium wash. Since she is 1
st
, she can't have a super wash (rule 1), so give her a regular.
We don't mark r, because we have to use an r to test this answer.
1 2 3 4 5 Cars: F M O T V
V __ __ __ __ Washes: r, s, p
r __ __ __ __
Now go through the rules. V is before O, and O has to be before M, so let's just put O and
M in 2 and 3, like we did before. Rules 9 and 10 tell us that both will get r washes. Mark O
and M because they don't have to go 2 and 3, but don't mark the r's because they do have
to go with O and M.
1 2 3 4 5 Cars: F M O T V
V O* M* __ __ Washes: r, s, p
r r r __ __
Now we have to put in T and F; it doesn't seem to matter where they go. We also have to
put in one p and one s; it doesn't seem to matter where they go, either, so use alphabetical
order.
1 2 3 4 5 Cars: F M O T V
V O* M T F Washes: r, s, p
r r r p s
I could have switched T and F, and the s and p, if I wanted. This works, so A is wrong.
This also shows us that C is wrong; we didn't have to put the s in spot 5. The p could have
gone there as well. E is the answer.
Question 15
Use your previous diagrams before you do anything else. The question asks for people who
must have r washes. Any person who doesn't have to have a regular wash should not be in
the answer. If we see a previous diagram where a person doesn't have an r, we know they
don't have to have r, so shouldn't be in an answer.
130 Game Four
Answer A
F doesn't have to have a regular wash the diagrams for 11, 13, and 14 give him a super
wash.
Answer B
Both M and O have r in every previous diagram. Skip this.
Answer C
T has an s in 13 and a p in 14, so eliminate this.
Answer D
We just saw that T doesn't have to have r, so eliminate this.
Answer E
V has a p in the diagrams for 11 and 13. Eliminate this.
B is the answer.
Question 16
This gives us an additional fact, so it seems like a regular If-question, but it is really
changing the rules by adding a car. We have to expand our diagram we only have room
for 5 cars now, not 6. The additional space may complicate things. We don't know how it
affects the deductions we have made for example, perhaps V doesn't have to be 1
st
(maybe it could go after J?). This makes this question difficult, and so we save it for last.
It also means that we couldn't use diagrams from this question on other questions, another
reason to do it at the end. We always save questions that change the rules for last not
those that add new information, like regular If-questions, but those that alter the basic rules
of the game. These questions always explicitly tell you that they are altering the rules, so
don't worry about not spotting them.
Let's draw a new diagram with six spaces, and put J in with the cars.
1 2 3 4 5 6 Cars: F M O T V J
__ __ __ __ __ __ Washes: r, s, p
__ __ __ __ __ __
Now let's test out the answers, just like normal. This is a "What can't be true?" question, so
we try each answer out and eliminate those that are possible. We can't use previous
diagrams because we've added a new rule.
Answer A
We can't give O a premium wash until we know where he goes. We want to put O in the
first available spot. We know that O can't be first, because he still must be after V. Try
putting O 2
nd
, and mark him.
1 2 3 4 5 6 Cars: F M O T V J
__ O* __ __ __ __ Washes: r, s, p
__ p __ __ __ __
Now go through the rules. They tell us that 2 and 3 must get the same type of wash.
However, there is only one p. This can't work, nor can O go 3
rd
, for the same reasons. Put
O 4
th
.
Game Four 131
1 2 3 4 5 6 Cars: F M O T V J
__ __ __ O* __ __ Washes: r, s, p
__ __ __ p __ __
Now let's put in everyone else. V has to be before O, so V will be in 1, 2, or 3 (most likely
1). Let's put V 1
st
and mark it.
1 2 3 4 5 6 Cars: F M O T V J
V* __ __ O* __ __ Washes: r, s, p
__ __ __ p __ __
M has to be after O, so M will have to be 5 or 6. We've already seen that M can't be last,
since F has to be after her, so M has to be 5
th
.
1 2 3 4 5 6 Cars: F M O T V J
V* __ __ O* M __ Washes: r, s, p
__ __ __ p __ __
But this won't work. Rule 10 tells us that the car before M has to get an r wash. Cross off
this diagram and move O one over.
1 2 3 4 5 6 Cars: F M O T V J
__ __ __ __ O* __ Washes: r, s, p
__ __ __ __ p __
Go through the rules again. M has to be after O, but that makes M last. This won't work,
since F won't have anyplace to go. If we move O any more, M will not fit at all. So this will
never work. This (answer A) is our answer.
Summary
This game didn't involve any new skills. We still had to draw diagrams in the same way and
test answers out by trying to put them in diagrams that work. Games like this, which
involve more than two types of things, can be difficult, because sometimes it's hard to know
definitely where to put people or things. But that is not a serious obstacle for us; we try
them someplace, and if that doesn't work, move them to the next available place. As long
as we are consistent in the way we do this, and always look at the rules, we can do as well
on these games as on any others.
What Do I Do Next?
Take a break. Then look over your notes and think about how we applied the fundamental
skills to this game. Then, when you feel ready, maybe today, maybe tomorrow, maybe in a
day or two, go back and do this game with no notes. By this stage you should have
memorized the fundamental skills. By that I mean you should know the techniques for
diagramming and how to do every kind of question, including when and how to use previous
diagrams and how to eliminate answers on every type of question. If you haven't, you need
to make up some flash cards and be working on them.
132 Game Four
Answer Key
11. B
12. A
13. B
14. E
15. B
16. A
Game Five 133
134 Game Five
Game omitted from electronic version of book.
Game Five 135
Game Five
(From LSAT 40)
Step 1: Read the Setup / Draw a Diagram
Let's go through this setup piece by piece, as we would on the LSAT. The first sentence
tells us that five of seven medications are being tested. Immediately we ask ourselves the
question we ask about every logic game: what things are being matched up? Here we
have medications; what are we doing with these medications? We are testing them, but not
all of them, just some of them. So we have the "test" group, and the "not-tested" group.
This game should remind you a bit of Game Three. Keep reading the first sentence. It
then tells you that the medications that are tested (those in the "test" group) are being
ranked from 1 to 5.
So we're doing two things here: figuring out which medications are being tested, and then
ranking those medications. We have three types of things floating around: the "tested" /
"not-tested" groups, the numbers 1 through 5 (which occur in the tested group), and the
medications. Which type of thing do we know the most about? We know the most about
the numbers, as always, so we'll base our diagram around these numbers. These numbers
are the tested group, so we'll label them as such:
test: 1 2 3 4 5
Each number gets one medication assigned to it, so we'll put one blank under each number.
test: 1 2 3 4 5
__ __ __ __ __
Now, we also have a not-tested group, which we should also write down. Whenever we
have two or more groups, you should give space to both. And whenever only some
things/people/medications are selected, some are not selected, which means that you have
both a "selected" and a "not-selected" group (or, in this case, a tested and not-tested
group). How many medications go in the not-tested group? Well, we have seven
medications to start with, and we are testing five of them, so we are not testing two (seven
minus five equals two).
test: 1 2 3 4 5 not: __ __ F G H I K L M
__ __ __ __ __
Step 2: Read the Rules
The first rule is relatively simple: it tells us that L is 2
nd
. If L is 2
nd
, it must be tested, since
the "not" group is not ranked. So we can put L in position 2 in the test group in our
diagram, and write down the rule as well.
L is 2
The next rule is also simple: F or G is in position 1. Now, if F is 1, where is G? I don't
know we aren't told. G might be 3
rd
or 4
th
, and so forth, or not picked. Be careful not to
assume that one is 1
st
and the other is not picked.
136 Game Five
F or G is 1
The next rule tells us that I is tested, so it will go in the "test" group, but we don't know in
which position.
I is tested
The next rule is an "If then" rule. Don't be confused by the placing of the "if" towards
the end of the sentence a rule with an "if" in it is still an "If then" rule, regardless of
where the "if" falls in the sentence. We should write it as an "If then" rule (moving the
"if" part to the beginning of the sentence so it looks more normal):
If H and G tested, then H before G.
Notice that we can't break this into two rules because it says "H and G," so we need both for
the "if" to be true, not just one. What's the contrapositive?
If H is not before G, then H and G aren't both tested.
It's worth taking a minute to see what this means. Let's look at the first part of the
contrapositive, the "if" clause. How would we know that H is not before G? Imagine a
situation in which you knew that H couldn't be before G. Go ahead, do it. What is it? Well,
one way for us to know this is if a rule told us this, but that's too easy. Imagine a situation
in which the question doesn't tell you explicitly about both H and G, but you know that H is
not before G. Think hard, think very hard. How might this happen?
OK, here's my answer. If you knew, for example, that G was 1
st
, then H couldn't be before
it. Or, if you knew that G was 3
rd
, let's say, and there was no room in front of G for H to go,
then H couldn't be before G. This is an important thing to understand when you look at
an "If then" rule, you need to be able to know when what the "if" part describes is the
case; otherwise, you won't know whether or not to use the rule in a situation.
The next rule is very similar:
If K and F are tested, then K is before F.
If K is not before F, then both K and F are not tested.
Here, if we knew that K couldn't go before F (for example, if F was 4
th
but there was no
room in 1 through 3 for K) then you would know that you couldn't have both the medicines.
The last rule is really two rules, since M being tested tells us something about F, and also
something about H. We'll break it up (see "If then" Rules and Negations).
If M tested, then F tested.
If M tested, then H tested.
If not F, then not M.
If not H, then not M.
Game Five 137
Here is how our original diagram and list of rules end up looking:
test: 1 2 3 4 5 not: __ __ F G H I K L M
__ L __ __ __
1. L is 2.
2. F or G is 1.
3. I is tested.
4. If both H and G tested, then H before G.
5. If H is not before G, then H and G aren't both tested.
6. If K and F are tested, then K is before F.
7. If K is not before F, then both K and F are not tested.
8. If M tested, then F tested.
9. If M tested, then H tested.
10. If not F, then not M.
11. If not H, then not M.
Step 3: Answer the Questions
Here is the order in which you should do the questions:
Question Number Question Type
6 What can be true?
7 What can be true?
10 If
8 What must be true?
9 Which is a complete and accurate list?
Question 6
Since this is a "What can be true?" question, let's go through the answers, seeing if they
break the rules or not. Each answer lists all the medicines in the "test" group, so the other
two are not tested.
Answer A
This breaks rule 1: L is not 2
nd
.
Answer B
This follows rule 1 and rule 2. It breaks rule 3, though, because I is not on the test list.
Answer C
This follows rules 1, 2, and 3. We don't worry about rule 4 because H and G aren't both
tested, so the situation the "if" part of the rule describes isn't the case here. Rule 5 is
followed H isn't before G, because H and G aren't both selected (H can't be before G if
there is no G in the tested group). We don't have both K and F, so we ignore rule 6, and
rule 7 is followed. Rule 8 is followed, but rule 9 is not: we have M but no H. This is wrong.
Answer D
This follows 1, 2, and 3. We don't have both H and G, so rule 4 is irrelevant, and rule 5 is
followed. We don't have both K and F, so we ignore rule 6 (and rule 7 is followed). Rules 8
and 9 are followed, because we have M, H, and F. Rules 10 and 11 are irrelevant, because
138 Game Five
their "if" parts say "not F" and "not H," but we test F and H. So this follows all the rules,
and it is the answer.
Since it is the answer, it is a possible diagram; copy it down and we'll use it for later
questions. I'm also going to take the time to fill out the "not" section; this may or may not
come in handy later on, and it isn't hard to do.
test: 1 2 3 4 5 not: G K
F L I H M
Question 7
This is a "What can be true?" question, so we'll go through each answer and see if we can
make it work.
Answer A
If G ranks better than M, both have to be tested (after all, you can't be ranked higher than
someone if one or both of you isn't ranked at all). Now, we don't know where G will go, so
we'll try G 1
st
; if this doesn't work, we'll try him someplace else. Let's put a * next to G, so
we know that we can move him if we have to. I'm not going to put M in a spot because I
haven't looked at the rules yet, so I don't know where M can or can't go. So as not to
forget M, though, I put it floating around near the "test" group.
test: 1 2 3 4 5 not: __ __ F G H I K L M
M G* L __ __ __
Now let's go through the rules. We've got 1 and 2 covered L is already in our original
diagram, and we've just put G 1
st
. Rule 3 tells us that we also have to test I, but I don't
know where that's going to go, so I'll just sort of put it near M.
test: 1 2 3 4 5 not: __ __ F G H I K L M
M I G* L __ __ __
Looking at rule 4, I don't know if both H and G are tested, so I ignore it. Now we look at
rule 5. It says "If H is not before G"; here, there is no room before G, so H isn't before G.
Thus, we can't test both H and G. What does this mean? It means that we can't test H,
since we've already got G tested. Put H in the "not" area.
test: 1 2 3 4 5 not: H __ F G H I K L M
M I G* L __ __ __
Let's keep going through the rules. We can't test both K and F because we only have three
spaces left and we have to have M and I, so we ignore rules 6 and 7. We know that M has
to be tested, so we have to test F and H (rules 7 and 8). But wait we can't test H (it's in
the "not" area), so this won't work. Cross out the diagram.
We don't know if this answer will work or not, because we haven't tried all the possibilities.
Since G is starred, it could go someplace else, so we have to move it. G can't go 2
nd
(L has
to go there), so put G 3
rd
. Again, I'm putting M floating around near the test group,
because the answer wants us to put G before M, but I'm not sure exactly where to put M.
test: 1 2 3 4 5 not: __ __ F G H I K L M
M __ L G* __ __
Game Five 139
Rule 2 tells us that F or G is 1
st
, so put F 1
st
, since G is 3
rd
.
test: 1 2 3 4 5 not: __ __ F G H I K L M
M F L G* __ __
Rule 3 tells us that I is picked, so put that in our diagram someplace; this means that the
two empty spaces will have I and M in them. Put H and K in the "not" area.
test: 1 2 3 4 5 not: H K F G H I K L M
M I F L G* __ __
Keep going through the rules; we can't have both G and H, so ignore rule 4. Rule 5
confirms that H can't be tested. We ignore 6 because K isn't tested. Rule 8 is violated,
because we have M tested but not H. Cross off the diagram and move G over one more.
test: 1 2 3 4 5 not: __ __ F G H I K L M
M __ L __ G* __
We probably remember that F or G goes 1
st
, so put F in 1. The answer says M is after G, so
put M 5
th
.
test: 1 2 3 4 5 not: __ __ F G H I K L M
M F L __ G* M
Rule 3 says I has to be tested, so I will go 3
rd
. Now, you're probably seeing that we're
going to have a similar problem here as before M is tested, so H has to be, but there is no
room for H. Moving G over one more won't help us, because then G couldn't be before M.
This won't ever work cross off the diagram and cross off the answer.
Answer B
This is going to be a similar drill to the previous answer. We want H to be before F, but we
don't know where either one goes. We could try H 1
st
, but having worked through the
previous answer, we probably remember that either F or G has to go 1
st
(rule 2) So let's
put H 3
rd
(since rule 1 says that L is 2
nd
), and put F someplace as a reminder not to forget
it. I'll put G 1
st
(since F sure can't go there the answer wants F after H). I don't put a
star next to G because G has to go 1
st
.
test: 1 2 3 4 5 not: __ __ F G H I K L M
F G L H* __ __
Skim through the rules until you see one that is broken. Which is it? That's right, it's rule
4. We're testing both H and G, but H is not before G. Is there any way to remedy this
situation? The only medicine we have starred is H, so that's the only one we can move
around. We already know that it can't be in 1 or 2, which means that it can only be moved
to the right (this is the virtue of consistently starting with the left-most position and moving
things to the right). But this would still mean that H is after G, which it shouldn't be, so
moving H further to the right won't help. This will never work. Cross off the diagram and
cross off the answer.
140 Game Five
Answer C
Same drill try I wherever you can until it works. We start by putting I 3
rd
, since we know
that F or G is 1
st
and L is 2
nd
.
test: 1 2 3 4 5 not: __ __ F G H I K L M
F __ L I* __ __
We want I to be before F, so F can't go 1
st
. This means that G goes 1
st
(rule 2).
test: 1 2 3 4 5 not: __ __ F G H I K L M
F G L I* __ __
Let's go through the rules. 1, 2, and 3 are fine. Since we don't know if H is tested, we
ignore rule 4. We do know that H can't be before G (since G is 1
st
), so we look at rule 5. It
tells us that we can't test both H and G; since we have G, we can't test H (or we'd be
testing both). Put H in the "not" area.
test: 1 2 3 4 5 not: H __ F G H I K L M
F G L I* __ __
We ignore rules 6 and 7 because we don't know if K is tested, and we don't know where F
goes (which means we don't know if K could go before F or not). We also don't know if M is
tested, so ignore rules 8 and 9. Ignore 10 because F is tested. Look at rule 11, because H
is not tested. This means that M can't be tested. Put M in our "not" area.
test: 1 2 3 4 5 not: H M F G H I K L M
F G L I* __ __
Our "not" area is full; all the remaining medicines must be tested. Only K remains, so it
must be tested. And I happen to remember rule 6 says something about K
We know that K must be before F, so the order must be:
test: 1 2 3 4 5 not: H M F G H I K L M
F G L I* K F
This (answer C) is the answer, since it works.
Notice that the question started a little slowly we had to do a lot of testing. However, as
we got the hang of the game, things sped up. We start to remember the rules and
understand how they work, which means we spent less time looking at our list and trying to
figure things out, and just whip through our diagrams.
One of the big time-wasters on the LSAT is anxiety. You start working on a question, or a
game, and it feels like it is going to take too long. It hasn't actually taken you much time
yet, but you feel like it is going to. So you worry, and you fret, and you start looking for a
different way to do things, some kind of magic inspiration that will make the question so
much easier. But usually you don't find it, and you end up not only taking too long, you
also don't even answer the question. If you had just stuck to your program, you would
have answered the question quickly enough. The moral of the story is to use what you
know, not to look for some unique way of solving this question.
Now, that is not to say that you shouldn't skip a question if you know it will take you too
long (for example, if you have already spent too much time on it). But there is a difference
between worrying that what you are doing is too slow and it actually being too slow. You
have to have the confidence that you know what you are doing and that it will work. How
do you build this confidence? Practice and honesty. You have to practice enough times to
know what you are doing and to see that it works. And you have to be honest with yourself
when it does work. Too often students want to deny that they know what they are doing, or
that they can get questions right, when the evidence that they can is staring them right in
the face.
Game Five 141
long (for example, if you have already spent too much time on it). But there is a difference
between worrying that what you are doing is too slow and it actually being too slow. You
have to have the confidence that you know what you are doing and that it will work. How
do you build this confidence? Practice and honesty. You have to practice enough times to
know what you are doing and to see that it works. And you have to be honest with yourself
when it does work. Too often students want to deny that they know what they are doing, or
that they can get questions right, when the evidence that they can is staring them right in
the face.
Question 10
Since this is an If-question, we have to redraw our diagram and put the information we are
given into it.
test: 1 2 3 4 5 not: __ __ F G H I K L M
__ L I __ __
Now we go through the rules. Go ahead Anything else have to happen here? None of the
rules (except rule 3) is about I, or about position 3, so we don't know what else is going to
happen here. Let's look at the answers. This is a "What can't be true?" question (the
EXCEPT means that the wrong answers can be true, so the right answer can't be), so we'll
test out each answer to see if we can make it work.
Answer A
Let's put M before H. I happen to remember that F or G is 1
st
, so M can't go there, which
means that M and H have to go 4
th
and 5
th
(no other spaces available).
test: 1 2 3 4 5 not: __ __ F G H I K L M
__ L I M H
Now we go through the rules. L is 2, so rule 1 is happy. Rule 2 tells us that F or G will be
1
st
, but we don't know which. We've obeyed rule 3 because I is tested. We ignore rule 4
because we don't know if we have both H and G, so we don't know if the "if" part of the rule
is true. Rule 5 is worth a look we know that H can't be before G. Why? Because H is
last, so there is no room after H for G to go. So we can't have both H and G, which means
that we can't have G, since we already have H. Put G in the "not" zone.
test: 1 2 3 4 5 not: G __ F G H I K L M
__ L I M H
Now, since the rule about F or G being 1
st
sticks in my head so much, I start thinking about
it. G isn't picked, so F has to go 1
st
. This fills up the entire "test" area, so everyone else
goes in the "not" area (only K is left).
test: 1 2 3 4 5 not: G K F G H I K L M
F L I M H
Go through the rules again, since we've added a lot of stuff. Every rule is obeyed since
we don't have both F and K or both G and H we ignore 4 through 7 so this answer works.
This makes it wrong (we're looking for something that can't be true).
Since this is a "What can't be true?" question, let's use this diagram to eliminate other
answers if we can. Does this diagram show that any of the other answers are possible?
142 Game Five
Unfortunately not. None of the situations described in the other answers occurs in this
diagram.
Answer B
Recopy our diagram for this question. Remember, on an If-question we recopy the diagram
we create for the specific question, where we put in what the question tells us (here, that I
is 3
rd
), not our original diagram.
test: 1 2 3 4 5 not: __ __ F G H I K L M
__ L I __ __
The answer wants us to put K before G. Again, only F or G can be 1
st
, so we have to put K
4
th
. That makes G 5
th
(since we are trying to put K before G) and it forces F to be 1
st
(rule
1), since G can't be.
test: 1 2 3 4 5 not: __ __ F G H I K L M
F L I K G
There is no room left in the "test" area, so everyone else (H and M) goes in the "not" area.
test: 1 2 3 4 5 not: H M F G H I K L M
F L I K G
Let's go through our rules. Rule 1? Check. Rule 2? Check. Rule 3? Check. Rule 4?
Ignore, because we don't have both H and G. Rule 5? Check (we don't have both H and
G). Rule 6? Broken we've got both K and F, but K is not before F. This diagram doesn't
work. Can we fix it? Well, nothing is marked with a star, so everything we have in the
diagram is mandatory. Since there is nothing we can change here, there is no way of fixing
the diagram (so cross the diagram out). This (answer B) is the right answer.
Question 8
Since this is a "What must be true?" question, we can use our previous diagrams to
eliminate answers that don't have to be true. We are looking for a medicine that must be
tested, so if a medicine can be in the "not" area in a previous diagram, it is not the answer.
We have previous diagrams from 6D, 7C, and 10A.
Answer A
F doesn't appear in the "not" area of any previous diagram. Skip this answer.
Answer B
G appears in the "not" area of 6D and 10A, so it doesn't have to be tested. Cross off this
answer.
Answer C
H appears in the "not" area of 7C, so it doesn't have to be tested. Cross off this answer.
Answer D
K appears in the "not" area of 6D and 10A, so it doesn't have to be tested. Cross off this
answer (you know, I really like saying that).
Answer E
M appears in the "not" area of 7C, so you know
Game Five 143
The answer is A.
Question 9
The question asks us for a complete and accurate list of the medicines that can be 5
th
. The
right answer will include all medicines that could be 5
th
, leaving none out; the wrong
answers will either leave one out or have one that can't be 5
th
. We can use our previous
diagrams to see which medicines we already know can be 5
th
, and eliminate any answer
that leaves one out.
The diagram for 6D has M 5
th
; the diagram for 7C has F 5
th
; and the diagram for 10A has H
5
th
. So we can eliminate any answer that doesn't include F, M, or H.
This eliminates B, C, and D. We are left with A and E. We only want to test medicines that
are not on both lists, and the only one that is not in both answers is I. We will see if I can
be 5
th
; if it can, E is the answer. If not, A is the answer. Let's go!
Actually, why don't you go ahead and try it out? I'll wait.
OK, here's how to do it we put I 5
th
.
test: 1 2 3 4 5 not: __ __ F G H I K L M
__ L __ __ I
Now we go through the rules. No rule tells us much about I; this is a good indication that I
can go 5
th
(if an item is not limited much by the rules, it can usually do lots of things). But
this isn't good enough for me I need to know that I can go 5
th
. So I want to try some
things out. Let's start putting things in. First I'll put F in position 1. Why? Well, someone
has to go there, and I have seen that F works pretty well in my previous diagrams. I'll star
F so I can try someone else if it doesn't work.
test: 1 2 3 4 5 not: __ __ F G H I K L M
F* L __ __ I
Now I go through the rules. Rule 7 tells me that K can't be picked (because it can't be
before F).
test: 1 2 3 4 5 not: K __ F G H I K L M
F* L __ __ I
That's all the rules tell me. But I have two spaces left. Whom shall I put in? Well, I've got
G, H, and M left. I remember that H goes before G (rule 4), so I'll pick them.
test: 1 2 3 4 5 not: K M F G H I K L M
F* L H G I
This follows all the rules, so this works. Thus, E is the answer.
By the way, you could have arrived here in many different ways. If you had put G in
position 1, you would have known that H couldn't be picked (rule 5), which in turn means
that M couldn't be picked (rule 11). This leaves K and F, which have to go in that order.
This would work.
Alternately, if you started by putting F 1
st
, but you didnt know who to put in 3 and 4, you
could have put H and M in as well.
144 Game Five
could have put H and M in as well.
What's the lesson to be learned? Try things out, don't hesitate and worry that you'll try the
wrong things. In the absence of information guiding you as to what to try, just try
something. If that doesn't work, try something else; usually, working out one thing will
give you an idea of what to try next, or you'll have some hunch based on your previous
work on the game, but there is nothing wrong with just going through the rules one by one
and trying what they suggest (e.g., putting F or G 1
st
because the rules say so, and going
from there).
Summary
Another odd game, where you had to deal with several different issues who gets tested
and who is not tested, and in what order. But it wasn't that hard, you just had to work
through the rules carefully. Notice how crucial it was for us to do the contrapositives of the
rules about G and H, and K and F. If we hadn't done those contrapositives, and hadn't
understood what they meant, the game would have been much harder. Go back and look
at the rules again, and think about how we knew what the contrapositives meant. You are
likely to see rules like this on the LSAT, and understanding them now means that you will
be more likely to understand them then.
What Do I Do Next?
Take a break. Then look over your notes and think about how we applied both the
fundamentals and the intermediate techniques to this game. Then, when you feel ready, go
back and do this game with no notes. By this stage you should have memorized the
fundamental skills. By that I mean you should know the techniques for diagramming and
how to do every kind of question, including when and how to use previous diagrams and
how to eliminate answers on every type of question. If you haven't, you need to make up
some flash cards and be working on them.
Once you've done this game on your own and feel comfortable with it, go on to Game Six.
Answer Key
6. D
7. C
8. A
9. E
10. B
Game Six 145
146 Game Six
Game omitted from electronic version of book.
Game Six 147
Game Six
(From LSAT 39)
Step 1: Read the Setup/Draw a Diagram
Let's go through the setup piece by piece. The first sentence tells us that three people
attend a three-day conference. So far we have two types of things people and days. The
next sentence gives us a third type of thing sessions on different topics. So now we have
three types of things to think about topics, days, and people.
We have to ask which of the three types of things we know the most about. Generally we
know more about days than we do other types of things specifically, days have to come in
a certain order. Here, though, I'm not yet sure if we are concerned with the order of the
days. If they had told us that the days were numbered 1, 2, and 3, or that they were
Monday, Tuesday, and Wednesday, I'd know that order mattered, since numbers and days
of the week have order built right in. So far, however, they have told us no such thing.
What do we do when the setup doesn't give us enough information? Where do we look for
more information? That's right, we look at the rules or the questions.
Glance quickly through the rules and see if the order of the days is important. Do any of
the rules tell us that we should think about the order the days come in?
Yes, one does. The third rule tells us that Tate doesn't attend on the third day. The word
"third" means that there has to be a first and a second day, and words like "first," "second,"
and "third" have the concept of order built in. In fact, words like these are called ordinal
numbers (as opposed to "one," "two," and "three," which are cardinal numbers), which
name even contains the word "order". You don't have to know the words "ordinal" or
"cardinal" for the LSAT, but you should understand the idea that words like these tell us that
order matters in the game.
Rules containing words like "before" or "after" or "earlier" or "later" would have told us that
the order of the days was important. For example, "Tate can't be assigned to a day earlier
than Maria."
If a rule had said, "Tate can't be assigned to a session earlier than Maria," we would have
known that the order of the sessions was an issue. This might just mean that the order of
the days was an issue (maybe the sessions on each day are simultaneous), so that a
session on the first day is before a session on the second, or it might mean that the
sessions on each day are in a certain order. We would need more information to tell us
which. Never assume anything on the LSAT just rely on the definite literal meaning of
words.
Now that we know that the order of the days is important, we know that we know
something about the days their order. One of the days is first, another is second, and
another is third. We can name the days 1, 2, and 3. Since we know something about the
days, and not much about the people or sessions, we'll build our diagram around the days.
We write them down in order:
1 2 3
148 Game Six
Now, the next question we ask is, how do the other things session and people get
matched up with the days? You tell me; it says right in the setup. "Each day, there are
three sessions" OK, that's pretty clear, but what are the sessions? The sentence goes on
to tell us that they are "one on hiring, one on investing, and one on regulations." So each
day gets one of each kind of session. Let's write that out. We can write the three sessions
under each day, or just sort of make a chart, with each session getting a row:
1 2 3
H
I
R
Now, how will the people get matched up with the sessions and days? Is it one person per
session per day? One person per day? Something else? I don't know the setup doesn't
tell us. We've got enough of a diagram so far (well, we should add in the names of the
people write them down off to the side), so we can proceed on to the rules.
Step 2: Read the Rules
The first rule answers the question we raised at the end of the previous step how we
match up the people to sessions and days. Each person will attend two sessions; no one
attends multiple sessions on the same topic, or on the same day. What does this mean? To
better understand it, give yourself a concrete example of something that could happen or
something that could not happen. M, for example, couldn't attend two sessions on day 1, or
attend a session on I twice. Now, we have three people, each of whom will be used twice;
so it seems that we'll fill up six of the nine sessions, leaving three empty ones. This is
useful to know when not every space in a diagram will be filled, try and figure out how
many empty spaces there are. Here, we have three empty spaces.
This last point, about the empty spaces, is actually false, but it is an easy mistake to make.
I'll talk more about why it is false, and how to avoid making the mistake, later.
I'm going to write this down in two different ways. Since we will use each person twice, I
am going to write each of their names twice. That way, each time I put a person into a
specific session, I can cross off one of the names; once I have crossed off both, I won't try
to put that person in the diagram any more. This is a useful technique: when you will use
one of the variable things (the ones that aren't built right into our diagram) more than
once, write its name as many times as you will use it. (See The Fundamentals for more
on variable things; here, the people are variable because we don't have them built right into
our diagram.) We are also going to write down Xs for each empty space. A rule might tell
us, for example, that no one attends I on day 3, and we'll put one of our Xs there. We have
three empty spaces, so we have three Xs.
1 2 3 M S T X
H M S T X
I X
R
I'm also going to write this rule down as a rule, so that I don't miss it by accident. I'm
going to break it into pieces to make it easier to understand. You might abbreviate these
rules a little more than I do (but don't abbreviate to the point of incomprehensibility); I'm
Game Six 149
writing them out more explicitly because I don't want to force you to learn my own
idiosyncratic form of abbreviation.
Each person goes to 2 sessions.
No one goes twice on same day.
No one goes to same kind of session twice.
If you are curious, here's how I abbreviated the rules for myself:
everyone 2 times
no 2 times same day
no one I H R twice
Everyone abbreviates slightly differently, based on how they think. I want you to
abbreviate in a way that you find comfortable, and not to have to learn the way that I think,
which may not ever be comfortable for you. You don't even have to abbreviate at all it
saves very little time, as long as you write the rules in a clear, easy-to-understand way.
You've probably noticed that there are certain areas of the logic games where I just give
you guidelines and let you do things your own way diagramming is one of these, to a
certain extent, as is writing the rules. Writing these things down is supposed to help you,
which means you have to do it in a way that you are comfortable with, and that depends a
lot on your personal history. There is little to be gained by learning a very specific way of
doing these tasks (although you can make things somewhat easier by diagramming better,
which is why I spend more time talking about it). On the other hand, there are other tasks
which will give huge benefits if you do them a certain way, such as answering the questions,
and I am very dogmatic about doing these the right way, no matter how weird the right way
makes you feel. In these areas, the benefits of doing these tasks right far outweigh the
costs of having to learn the methods.
I'm going to break the next rule into two rules, one about Maria and one about Suki.
M doesn't go to I.
S doesn't go to I.
I abbreviated these as:
M not I
S not I
The next rule is quite simple.
T not on 3
The last rule is somewhat surprising. It tells us that we can't have more than two
employees per session. The word "given" means the same as "specific," and it tells us that
we can't have more than two employees in a specific instance of the session (such as H on
day 2), as opposed to "type of session" or "topic," which would be about all the Hs or all the
Is. But I (honestly) never even thought that we could have more than one employee per
session, much less more than two. Let's write this rule down and then we'll think about
what it tells us.
Not more than 2 people per session
150 Game Six
If we can't have more than two people per session, that means that we could have just two
(or one or none). This means we can double up employees in a single session. Earlier we
thought we'd have to fill six sessions, since we have three people and are using them twice
each. This meant that we'd have three empty spaces, which we signified using Xs. Now we
can double up people, which means we could have a lot more empty spaces (if everyone
went to a session with someone else we'd only use three sessions, so we'd have six empty
spaces), although we don't have to, because we don't have to double up. Let's erase those
Xs, because we don't know how many we have to have, and I don't want to confuse myself
later on.
Now, what lesson do we learn here? Never assume anything that you weren't told, but we
already knew that. More specifically, never assume that a space has to be filled unless you
are told. Now, how could we be told that a space has to be filled? One way would be for
the game to tell us that we were going to put things in spaces, one per space (see Game
One). Another would be for the game to tell us that things were going to get matched a
certain number of times. For example, in Game Two, we are told that each language is
learned a certain number of times, so we create that number of spaces for each language.
A game could also tell us that we couldn't have ties, or couldn't have more than one thing
per area (unlike this game), which would tell us how many spaces were needed if there
are five people, and no ties, then five spaces are needed. A game could also tell us that we
are going to order a certain number of things, and that the number of ordered spaces was
equal to the number of objects. For example, the game before this one in LSAT 39 says
"Eight files will be ordered from first to eighth." You couldn't have an eighth file if you
didn't have seven files in order in front of it.
One last point. We use the blank "__" to signify a space that has to be filled. Here we
haven't put any blanks in our diagram, because we don't know which spaces will be filled
and which won't (also, we'd have to put in nine blanks, which would take too long, so we
wouldn't use blanks even if we were filling all the spaces we only use blanks to help us,
not because we have to).
Let's summarize: the rules here told us that we were going to match people to sessions on
specific days. The rules reminded us that we might put people together. So the rules really
help to determine how we diagram this game. When in doubt on how to diagram, look at
the rules.
So, here is our original diagram (in its final form) and our list of rules:
1 2 3 M S T
H M S T
I
R
1. Each person goes to 2 sessions.
2. No one goes twice on same day.
3. No one goes to same kind of session twice.
4. M doesn't go to I.
5. S doesn't go to I.
6. T not on 3
7. Not more than 2 people per session
Game Six 151
Step 3: Answer the Questions
Here is the order in which you should do the questions:
Question Number Question Type
8 If
10 If
11 If
7 What can't be true?
9 What can't be true?
6 What is the maximum number of?
Question 8
The question tells us that two (no more, no less, that's what exactly means) sessions on
day 3 are attended. Since we don't know which sessions they are, let's make a note of that,
something like
2 sessions on 3
Be careful that you don't write "2 people on 3," because that might be false. Two sessions
are attended, but they can each be attended by more than one person.
Now let's look at the rules, and see if this tells us anything. We're looking for rules about
day 3, since that's what we have information on.
Rule 6 tells us that T doesn't go on day 3, so the sessions that are attended have to be
attended by M and S. Now that we've learned something new, we go back and look through
the rules again (remember, every time you learn something new, go back through the rules
and see if that leads to more information). We want to see what we know about M and S
we're always looking to use our new information. Rules 3 and 4 tell us that M and S can't
attend a session on I. Since M and S, and not T, are attending on day 3, and M and S can't
attend I, I can't be attended on day 3. But this question tells us that two sessions have to
be attended on day 3; what will they be? They can't be I, so they have to be H and R (the
only two sessions left). So we can put this in our diagram by drawing blanks in those spots,
to tell us that we have to put someone there, and put an X in the spot where no one can go.
I'm also going to jot down M and S under day 3, to remind me that they will be the ones to
fill those spots. We don't put the M or S in either spot at the beginning of an If-question,
before we look at the answers, we only write what has to be true. This is because we will
use this diagram for every answer, so we don't want to prematurely eliminate situations
that might be possible.
1 2 3 M S T
H __ M S T
I X
R __
M, S
Go through the rules again; nothing much more jumps out at me.
Now we want to look at the answers. This is a "What must be true?" question, so we'll test
the answers out by trying to do something other than what the answer describes.
152 Game Six
Answer A
Now, what is something other than "exactly two"? (See "If then" Rules and
Negations.) More than two or fewer than two is different from "exactly two." Let's try to
have fewer than two sessions attended on the first day. If this doesn't work, we can try
more than two. To have fewer than two sessions attended on the first day, we want to put
as many people as we can in the second day, since they have to go somewhere and the
third day is filled. Let's start by putting in our second M (the one not under day 3). Look at
the rules and see where this M can go.
Rule 2 tells us that M can't go on day 3 twice, and rule 3 says that it can't go to the same
session twice. Well, we don't know where M went on day 3, so we don't know where it can't
go on the other day. Let's just put M in H and S under R to make things easier on us
conceptually (it's so hard to think about them without putting them someplace specific); I'll
put a star next to them because they don't have to go in these places.
1 2 3 M S T
H M* M S T
I X
R S*
OK, now we know that M can't go under H again. We also know that M can't go under I
(rule 4). So let's put M under R on the 2
nd
day because we are trying not to put people on
day 1 (with a star because I could have put M someplace else).
1 2 3 M S T
H M* M S T
I X
R M* S*
Now on to S. Where do we put the second S? Well, checking the rules, similar rules apply
to S as to M. So let's put S on the 2
nd
day, under H, since we can't put it under R again.
1 2 3 M S T
H S* M* M S T
I X
R M* S*
Now let's think about T. Looking at the rules, the only rule about T is that it can't go on the
3
rd
day. Let's put a T on the 2
nd
day, since we are trying not to put many people on the 1
st
day (so as to test this answer). We could put it anywhere on the 2
nd
day, but why not put it
under I, since the space is empty (with a star as well, since this was a pretty arbitrary
decision).
1 2 3 M S T
H S* M* M S T
I T* X
R M* S*
Game Six 153
Now for the last T. It can't go on the same day as the other T (rule 2), and day 3 is booked
up, so we put it on day 1. Check the rules to see where it could go; as long as it doesn't go
on I again, we can put it anywhere. H is the first open space, so I'll put it there.
1 2 3 M S T
H T S* M* M S T
I T* X
R M* S*
This all works, so what does this tell us about the answer? It's a "What must be true?"
question, so that's right, since this works, the answer is wrong (because something other
than what the answer says can be true, which means that the answer doesn't have to be
true). Cross off answer A.
Let's see if this diagram allows us to eliminate any other answers. Does it give a situation
different from that described by any other answer? We don't have exactly two sessions
attended on day 2, we have more, so B doesn't have to be true. Cross off B.
C looks like it might have to be true, since we don't have M and S together in this diagram.
D and E look fine as well, since M and T are not together, and T does indeed attend an I
session. We are going to have to test these out ourselves.
By the way, in the course of doing this diagram, you may have noticed some deductions.
First, T has to attend sessions on days 1 and 2 (since he can't go on day 3). Second, the I
session on day 3 will never be attended (the rules say only M or S go on day 3 (rule 6) and
M and S don't attend I (rule 4 and 5), and these are general rules, not specific to this
question). However, we don't know that M and S have to go on day 3 that information
was specific to this question. If you think that M and S have to go on day 3, I'll bet you that
you can draw a good diagram in which they don't try it out. Here's one that works (and it
isn't the only way to do it):
1 2 3 M S T
H SM T M S T
I T
R SM
Now, you shouldn't actually have tried that out on the LSAT it takes too long. The point is
that if you want to make deductions while you do the questions, you should be careful not
to use information that comes from the "if" parts of If-questions.
If you noticed these deductions, feel free to write them into your diagram, or in your list of
rules. If you didn't, don't worry too much.
Answer C
We test this out by making M and S attend a session together. We already know they both
go on day 3; can they be together on day 3? Well, according to the diagram we drew when
we started this question, we have to fill two sessions on day 3, and M and S both have to be
used. We can't double them up if we want to fill these two spaces (otherwise we'd have to
use one of them twice, and we can't use someone twice on the same day, according to rule
2). So we have to try and put them together on a different day.
It'll be helpful to know which sessions M and S attend on day 3, so, as we did before, we
just put them in sessions, and put stars next to them, since we don't know which exact
sessions they have to attend.
154 Game Six
1 2 3 M S T
H M* M S T
I X
R S*
Now, we want to put M and S together. We already know they can't go together on day 3,
so let's try to put them together on day 1. Let's put them both in H on day 1 (we'll put a
star next to them so that we can move them if we have to).
Now that we've put M and S into definite places, go through the rules. Rule 3 tells us that
M can't go in H again, and S can't go in R again. We want to put M and S together. Where
could they both go? They can't both be in H or R, so they'd both have to be in I. Put them
in I and check the rules. This breaks rules 4 and 5. There is no place to move the pair that
will make this work. Cross off this diagram and the last one, since they break the rules.
What have we just learned about this answer? It must be the right one, since we can't
make anything different from what the answer says work. Answer C is the answer.
Question 10
Another If-question. It tells us that all the sessions on the first day are attended. It doesn't
tell us by how many people, so we should ask if they can be attended by more than one
person. Well, can they? How many people are there? Three. And there are three sessions
on that day. By this point we probably remember rule 2 by ourselves, which tells us that
people can't attend more than one session per day. So each person only attends one
session, at the most, on day 1; since all three sessions are attended, everyone must attend
only one. Let's put that in our diagram; again, we use blanks to say "Someone goes here":
1 2 3 M S T
H __ M S T
I __
R __
MST
Now let's go through the rules and see what we can learn. We've already considered rules
1, 2, and 3. Rules 4 and 5 tell us that M and S don't attend I; this leaves only T to attend I,
which we should note in our diagram.
1 2 3 M S T
H __ M S T
I T
R __
MS
Rule 6 tells us that T doesn't go on day 3; this means that the second T has to go on day 2
(but we don't know where).
1 2 3 M S T
H __ M S T
I T
R __
MS T
Go through the rules one more time; there is at least one more small thing you can figure
out, but I'm not going to tell you what it is. It isn't crucial, and it isn't the sort of thing that
Game Six 155
you have to be able to catch, so let's practice how we do LSAT questions when we can't
quite figure everything out.
This question asks "What must be false?" which is the same as "What can't be true?" We'll
try out each answer and see if it will work; if so, we cross it off.
Answer A
Let's try to put them together on day 3 and see if we can make this work. We don't know
quite where they should go on day 3, so where do we put them? What do we do when we
want to put someone someplace, but we don't know where? Put them in the first available
space and mark them with a star.
1 2 3 M S T
H __ MS* M S T
I T
R __
MS T
Now we try to make the diagram work. We have to put in one more M and one more S.
Our diagram tells us that they have to go in day 1. Where can we put them? Look through
the rules.
According to rule 3, neither M nor S can go in H on day 1, since they are already in H on
day 3. This leaves no one to go in H on day 1, but we have to have someone there. How
do we know? We have a space there, which means "Put someone here," and we trust our
notes. This won't work.
Since M and S are starred, we have to see if we can move them someplace else. Rule 4 and
5 tells us they can't go in I; can we put them in R? If you aren't sure, try it out. You should
see that they can't go there, for the same reasons that they couldn't go together in H (we
wouldn't be able to put anyone in R on day 1, and we have to).
This will never work, so this (answer A) is the answer. Cross off the diagram.
By the way, you could have gotten this question right without figuring out that T goes in I
on day 1. We know only that all three sessions have to be filled on day 1. So we put M and
S in H on day 3. This means they can't go in H on day 1, so we put T there. But then no
one can go in I on day 1, so this doesn't work. The point? Don't despair if you start doing
an If-question and can't figure much out work through the answers and you still should be
able to get it right, just a tiny bit slower.
By the way, the other small thing you could have figured out before looking at the answers
is that I on day 2 and I on day 3 are going to be empty. Totally useless, but you could have
figured it out just by looking at the rules.
156 Game Six
Question 11
The question tells us that M and T are on day 1, and that they are the only people on day 1
(which means S isn't on day 1). Let's write that down and look at the rules to see if we can
figure out where on day 1 they go.
1 2 3 M S T
H M S T
I
R
MT
no S
Rules 1 through 3 don't tell us much, except that M and T will only attend one session each
on day 1. Rule 4 tells us that M won't attend I (T certainly could, but doesn't have to
there's no reason anyone attends I on day 1). Rules 5, 6, and 7 are likewise not very
helpful. So let's look at the answers. The question says "could be true EXCEPT," so we
know that the wrong answers can be true, which means that the right answer can't be true.
Answer A
M and S can't attend 2 sessions together. We don't even have to diagram this. M and S
only attend two sessions each. One of the sessions M attends is on day 1, and S doesn't
attend a session on day 1. So one of the Ms won't be with an S. That leaves only one other
M, which might go with an S. One is not exactly two, so this is not possible. This is the
answer.
Question 7
"Must be false" means the same as "What can't be true?" This means that the wrong
answers can be true. We'll test the answers out one by one, but we can use our previous
diagrams to eliminate answers. We only have one previous diagram that didn't break a
rule, which is the diagram for 8A.
This diagram allows us to eliminate answer B, since the diagram for 8A shows S on only the
last two days. If you made the deduction I talked about in question 8 (see the boxed text
in question 8, answer A), that session I on day 3 can't be attended, as you went through
the answers you would have noticed that E can't occur, so E is the answer. However, for
those of us that didn't make this deduction, let's work through the other answers.
Answer A
Let's try to put M on just the first two days. By this point you might remember that M can't
go in I; if not, skim the rules to see what we know about M, and you'll see that rule 4 says
M can't go in I. So we'll put M in H and R; it doesn't seem to matter which session goes on
which day, but I'll put a star next to M just to be cautious.
1 2 3 M S T
H M* M S T
I
R M*
Game Six 157
Now let's put in the Ss. We can put them anywhere we want, but it's easiest to put them in
the first available spaces. Here those are in H and R, with M (since two people can go
together, spaces with one person in them are still available).
1 2 3 M S T
H M*S* M S T
I
R M*S*
Now let's put in the Ts. The first available spaces are in I on day 1 and H on day 2.
1 2 3 M S T
H M*S* T* M S T
I T*
R M*S*
This works, so we can eliminate this answer. We can also use this diagram to try and
eliminate other answers. Fortunately, it helps us eliminate C (we have two people, M and S,
attending a session together on day 2). If it didn't, we could test out C, which would be
easy to eliminate. This doesn't eliminate D, because we have two sessions attended on day
2. We've eliminated B from previous diagrams, so let's look at D.
Alternately, we could have put S on days 1 and 2, in R and H.
1 2 3 M S T
H M* S* M S T
I
R S* M*
Now let's put in the Ts. Rule 6 tells us that T can't go on day 3, so I'm going to have to put
T on days 1 and 2. Let's just double up T with M on each day (T is going to have to double
up on at least one of the days, since it can't go in I both times).
1 2 3 M S T
H MT* S* M S T
I
R S* MT*
This would also have eliminated answer C for us. I'm just showing you this to remind you
not to get hung up looking for the "perfect" place to put people when drawing test
diagrams.
Answer D
Let's try to have only one session attended on the 2
nd
day. That means we'll try to put all
but one person on days 1 and 3, so we want to avoid day 2 if possible.
Let's start with M, because she is our first person in alphabetical order. Can we avoid
putting Ms on day 2? Sure; we'll put M on days 1 and 3. Look at the rules to see where we
can put M; rule 4 tells us that M can't go in I. So put M in H and R (since we can't put M in
H twice, or R twice), in whatever order you like (since there are no rules specifying).
158 Game Six
1 2 3 M S T
H M* M S T
I
R M*
Now let's put in the Ss. The rules for S are much the same as for M; we could double up
the Ms and Ss if we wanted, or we could put them separately, as long as we avoid day 2.
Let's put them with M, since these are the first available spaces.
1 2 3 M S T
H M*S* M S T
I
R M*S*
Now we just have the Ts left. Look at the rules and see what we know about T. We know
that T can't go on day 3 (rule 6), and we can't put the Ts on the same day (rule 2). This
means that the two Ts go on days 1 and 2. Put them wherever you want, other than H on
day 1 (which already has two people in it).
1 2 3 M S T
H M*S* T* M S T
I T*
R M*S*
This works, so we can eliminate this answer. The answer has to be E.
Question 9
The question is an EXCEPT question, so it is explicitly describing the wrong answer to us.
The wrong answers are possible, which means that the right answer is impossible. This is a
"What can't be true?" question. Let's use our previous diagrams to test it. We now have
diagrams from 7A, 7D, and 8A.
Answer A
What does this mean? It means that every session with people in it (since there will be a
few not attended at all) has exactly one person at it, not two. If it said "Every session is
attended by exactly one Capital employee," it couldn't be true, since we don't have enough
people to fill all the spots; to avoid being so easy, it specifies that it is only concerned with
the sessions that are attended at all.
Is it possible to have no spots with two people in them? Sure, we see this in 8A. Eliminate
this answer.
Answer B
What does this mean? It means that there are no sessions with just one person in them.
Well, do we see this in a previous diagram? No, so skip the answer.
Answer C
We can eliminate this using the diagram for 7A or 7D.
Answer D
We don't see this in a previous diagram, so skip the answer.
Game Six 159
Answer E
We don't see this in previous diagrams (unless you'd diagrammed 7A differently; see the
boxed text in 7A). We have to skip this answer.
Notice, by the way, that dumb luck factors in a little in eliminating answers. Some of our
choices in diagramming are a little arbitrary, and if we are lucky then these will give us
diagrams that eliminate a lot of answers. If we had diagrammed 7A differently, we might
have been able to eliminate E, or maybe we'd be able to eliminate D, or some other answer
on another question. This is nothing to worry about; there is no way to predict the kind of
diagrams that will be helpful later on, so don't even bother thinking about this when you
have to make a random decision in trying things out.
Now we have to go back and test out answer B. We want to make a diagram in which
everyone is paired with someone else. Let's try it out.
I have no idea who I should be pairing up, so I'll just start pairing people up in order. Let's
start by pairing up M and S; we'll put them in the first possible position. Since this is totally
arbitrary, I'll mark it with a star.
1 2 3 M S T
H MS* M S T
I
R
Skim the rules. We now know that M and S can't be picked on day 1 again. We can put
them together on day 2.
1 2 3 M S T
H MS* M S T
I
R MS*
Now there's no one to pair with T, and T can't pair up with himself (rules 2 and 3). Hmm
so we don't want to pair up S and M twice. Let's pair M with T instead. They have to be
paired on day 2, since M can't go on day 1 twice, and T can't go on day 3; and they have to
go under R, since M is already in H once, and can't go in I. So they have to go in R under 2,
and they don't get a star (poor them).
1 2 3 M S T
H MS* M S T
I
R MT
Cool, now we can pair S and T. But where would they go? S is already on day 1, and T is
on day 2; T can't go on day 3, so there is no place to put them. Hmm go back to the last
place we have starred. Maybe we shouldn't have paired M and S on day 1? I don't see that
it would matter where on day 1 they went, and I don't see how it would help to put them on
day 2, so let's put them on day 3.
1 2 3 M S T
H MS* M S T
I
R
160 Game Six
Alright, now we know we can't pair M and S again and make this work (because no one will
go with T, as we saw above), so we'll pair M and T. They have to go in R, since M can't go
in H again and also can't go in I. Let's put them in day 1 (day 1 or 2 won't make a
difference).
1 2 3 M S T
H MS* M S T
I
R MT
Now we can pair S and T. But where do they go? S can't go in H again, or in I, so it
would have to go in R. But M is already in R. This won't work either.
By this point, and maybe before it, you should have realized that this answer won't ever
work. There is still one change we can make, which is not pairing M and S at all, but that
just means we'll have to pair only S and T, which leaves no one for M. As long as we pair M
and S once, we see that we can't make the diagram work, no matter where we put M and S.
Putting them under R, rather than H, won't make a difference; the same rules govern both
H and R, so if it won't work in one place, there is no reason why it would work in the other.
Since this won't work, it (answer B) is the answer.
This may have seemed like a lot of work, and I agree that it was. Once in a while you'll
eliminate all but two answers. We really should have tested out D, which would have been
much easier (see Picking the Answer to Test under Intermediate Techniques), but I
thought it would be a good learning experience to test B.
Question 6
A maximum number question; look at your previous diagrams and see what is the largest
number of sessions attended in any of them. The diagram for 8A has 6 sessions attended.
We can eliminate A through C, since we know that these aren't the largest possible number
of sessions attended. The only question is if we can have seven sessions attended. Think
about it.
Well, we only have three people, who attend two sessions each, and two times three is six.
A session has to be attended by someone, so I don't see how we can have seven sessions
attended. E can't be right, so D is the answer. This is the sort of thing they throw in to a)
confuse people who don't understand the rules, or b) waste the time of people who don't
have the confidence in themselves to eliminate ridiculous answers, and have to test out
things they already know to be true or false.
Summary
This game seems odd on its face, but I think it just reinforces the importance of taking the
setup apart piece by piece and applying the skills we have already learned to construct a
diagram. Once you have a diagram, you have to understand the rules; that requires
making the rules concrete thinking about how they apply to the specific people in the
game but not adding anything of your own. Review the rules to this game and think
about why we shouldn't assume that each person attends a session by themselves (before
we see the last rule), and when on other games you should or shouldn't assume this.
Game Six 161
The questions on this game test your ability to make test diagrams; the questions don't give
you a lot of guidance, which means that you have to be confident enough in yourself to just
make arbitrary decisions, and that you have to know how to modify your diagrams when
these decisions don't work out. The questions also test your understanding of LSAT phrases
like "exactly," and "or more."
If you feel comfortable with this game, you are in pretty good shape for the LSAT the
game is challenging, and draws upon almost every skill we have learned so far. Since the
game and its questions are so open-ended, it really tests your ability to be methodical and
not jump to conclusions.
What Do I Do Next?
Same as at the end of the previous chapter: take a break. Then look over your notes and
think about how we applied both the fundamental and the intermediate techniques to this
game. Then, when you feel ready, go back and do this game with no notes. By this stage
you should have memorized the fundamental skills. By that I mean you should know the
techniques for diagramming and how to do every kind of question, including when and how
to use previous diagrams and how to eliminate answers on every type of question. If you
haven't memorized these things, you need to make up some flash cards and be working on
them.
Once you've done this game on your own and feel comfortable with it, go on to Game
Seven.
Answer Key
6. D
7. E
8. C
9. B
10. A
11. A
162 Game Seven
Game omitted from electronic version of book.
Game Seven 163
Game Seven
(From LSAT 40)
Now let's take a look at a game that doesn't seem at first to fit what I've told you, and see
how we apply our fundamental skills to it.
Step 1: Read the Setup/Draw a Diagram
Before you take a look at the setup paragraph for this game, I want you to think about how
we diagram games. What do we look for in every game when we want to diagram it? Once
you've remembered and thought about this, read the setup paragraph and think about how
you might diagram this game.
I've told you that every game involves two or more types of things or characteristics that
you match together. What are the two types of things in this game? Well, we have cities.
What else do we have to think about? We have connections: we are trying to assign
connections to pairs of cities (a connection can't be with just one city, right?). So here are
our two types of things: connections and cities. Which do we know more about? At this
point I don't think we know much about either, so I'm just going to write down that these
two things exist:
Connections Cities: H M P T V
Not a very exciting diagram, but what else is there to write down? A couple of things to
remember: connections are between two cities, and every city gets connected to some
other city. Notice also that the setup doesn't say that there is a difference between H being
connected with M and M being connected with H; in fact, it tells you that cities are
connected with each other if there is a flight between them, so if there is a flight between H
and M, in either direction, these two are connected.
There's another way to diagram this game (well, there are probably a few other ways to
diagram this game, but there's at least one other good way to do it). Many people find this
easier to use, so I'll put it out for you to look at. I'll also illustrate how to work with this
diagram alongside our other diagram as we go through the questions.
The game involves connecting cities, which you would normally do on a map. So we'll draw
a little map, and connect the cities on that. Since we don't really have any guidelines as to
how the cities are arranged, we'll just arrange them roughly in a circle.
As the cities get connected, we'll draw lines between them. If a city can't be connected
anymore, we will cross it out.
Now, should you go out of your way to memorize this type of diagram? No, because this
sort of game is pretty rare: there haven't been any other games like it on LSATs 31
through 42 (the LSATs released from 2000 to the time this book was written), although the
fourth game on LSAT 33 has a slightly map like diagram as well. But you should learn this
lesson: if you see a way to diagram a game that seems easier to you, use it. But don't
spend a lot of time looking for alternate diagrams, because it isn't worth it.
H M
P
T
V
164 Game Seven
lesson: if you see a way to diagram a game that seems easier to you, use it. But don't
spend a lot of time looking for alternate diagrams, because it isn't worth it.
Step 2: Read the Rules
Before we read the rules we have been given, I'm going to write down something we were
told in the setup, because it seems like a rule. I want to write down that every city needs
to be connected to at least one other city.
Every city connected at least once
Now let's go through the rules. You should know how to do this by now, so I'm only going
to go into detail in places where we see something new and/or difficult.
M connected only one other city
The word "exactly" in that rule tells me that M is connected once and only once.
H not connected with T
This next rule has the word "any" in it, which means that it is an "If then" rule; so we
have to write it and its contrapositive.
If a city connected with H, then that city is connected with T
If not connected with T, then not connected with H
The next rule is also an "If then" rule:
If P connected with T then P not connected with V
If P connected with V then P not connected with T
And here is our original diagram with the complete list of rules:
Connections Cities: H M P T V
1. Every city connected at least once
2. M connected only one other city
3. H not connected with T
4. If a city connected with H, then that city is connected with T
5. If not connected with T, then not connected with H
6. If P connected with T, then P not connected with V
7. If P connected with V, then P not connected with T
Game Seven 165
Step 3: Answer the Questions
Here are the questions, and their types, in the order we should do them in:
Question number Question Type
11 What can be true?
12 If
14 What can be true?
15 If
17 If
13 What can't be true?
16 What is the maximum number of?
As we go through the questions, you will have to look through the rules over and over
again. I am going to assume that you know how to do this by now; instead of walking you
through every rule every time, I will just point out rules that are relevant (and some that
are irrelevant in interesting ways). I do this here to make these explanations easier to read
and follow. Understand, though, that each time I do this, it is because I looked through
each of the rules one by one, and I'm just skipping the ones that obviously don't apply.
Question 11
Although the question contains the words "complete and accurate list," it says "Which
could be a complete and accurate list" which makes it a "What can be true?" question.
The words "complete and accurate list" tell us that each answer is supposed to list all the
connections; if one of the connections in the list won't work, or if another connection is
required, the list is bad and the answer is wrong.
We go through the answers one by one, trying to see if they follow all the rules.
Answer A
Rule 1 says that all the cities must be connected at least once; well, this list includes all the
cities, so that is fine. Rule 2 tells us that M can only be connected once, and it is. Rule 3
says that H can't be with T, and it isn't. Rule 4 says that if a city is connected with H, it
must be connected with T as well. H and V are connected, and V is also connected with T,
so that's good. No one else is connected with H. Rule 5 says if someone is not connected
with T, they can't be connected with H, and that is fine only P is not connected with T, and
it isn't connected with H either. Rule 6 is irrelevant, because P is not connected with T
(remember, ignore "If then" rules when the situation described in the "if" part doesn't
occur). Finally, we have rule 7. P and V are connected, and P is not connected with T. So
this is the answer.
We should copy this diagram down in our own writing so we will remember that it is a
potential situation, and we can use it on later questions. Here it is using both diagrams
(although you should only use one or the other):
Connections
H and V
M and T
P and V
T and V
H M
P
T
V
166 Game Seven
Question 12
Copy down our original diagram and plug in what we have been told. Here we are told that
P will be connected three times (exactly), so I'm going to put three Ps in the "connected"
section of my diagram and cross P out, so that I won't try to use it again. In the alternate
diagram, I'll just make a note next to P that it has to be connected three times.
Connections Cities: H M P T V
P
P
P
Look through the rules. As you skim through them, nothing much should catch your eye
until you get to rules 6 and 7 (because rules 1 through 5 are about cities other than P, but
we only know about P here). We are told that if P is connected with V, it can't be connected
with T; and, if P is connected with T, it can't be connected with V. So P can't be connected
to both of these. Why does that matter? P has to be connected to three cities. What does
this tell us?
I'll let you think about that for a bit, and go on to look at the answers. There is a deduction
to be made here, but it might not jump out at you. I'm going to answer the question
without making the deduction, and then with making the deduction. There will be a lesson
learned, I hope.
Before we look at the answers, we need to know what the question is asking. It is asking
for a pair of cities that can be connected. So we'll try each answer out and see if it can
work.
Answer A
Try connecting H and M.
Connections Cities: H M P T V
P
P
P
H and M
Go through the rules. M can only be used once, so we can cross M off. Keep on looking.
Rule 4 tells us that any city connected to H must also be connected to T. So M must be
connected to T. But we crossed M off; it can't be connected again. This isn't possible, so
we can eliminate this answer (and cross off the diagram too).
By the way, we might notice from this that H and M can never be connected.
Answer B
Try connecting H and V.
Connections Cities: H M P T V
P
P
P
H and V
H M
P(3)
T
V
H M
P(3)
T
V
H M
P(3)
T
V
Game Seven 167
Go through the rules again. Rule 4 tells us that V must be connected with T.
Connections Cities: H M P T V
P
P
P
H and V
V and T
Look through the rules again. No other rules obviously apply. We still need to figure out
the three cities to which P is connected. Let's try putting cities in and see if we can make it
work. We need to use each city (rule 1), so let's make sure we use cities that haven't been
used yet. Only M hasn't been used, so connect it to P. I put a star next to this to mark that
this is a decision I made, which wasn't forced on me by the rules. I'll use a dotted line
instead of a star in our alternate diagram to show a line that was optional.
Connections Cities: H M P T V
*P and M
P
P
H and V
V and T
I crossed M off because it can only be used once. Who else can we connect P with? Well,
how about H and T, since they are the first cities in alphabetical order that are still
available? This doesn't break any rules P is connected to both H and T, in accordance with
rule 4, and we know that P can't be connected to V if it is connected to T, but it isn't, so
we're OK.
Connections Cities: H M P T V
*P and M
P and H
P and T
H and V
V and T
This works, so this (answer B) is the answer.
OK, now what was the deduction we could have made? It was that P will have to be with M,
H, and T it has to be connected with three cities, and it can't be connected with V; if it is
connected with V, it can't be connected with T (rule 7), and thus it can't be connected with
H either (rule 5). This wouldn't fill all of P's spaces, so V is out, which only leaves M, H and
T.
Now, how much easier does that make this question? Not much you arrive at the same
place by just working through the answers. Often times you will look at an If-question and
feel that there is a deduction in there someplace; but if it doesn't come to you, just work
through the answers. That may not feel like the fastest way to answer the question, but it
sure beats staring at the page hoping for inspiration.
I never noticed this deduction when I first did the game; it wasn't until I started typing up
this chapter that I realized a deduction could be made here. Lesson? Even the best LSAT
takers often don't see or use deductions.
H M
P(3)
T
V
H M
P(3)
T
V
H M
P(3)
T
V
168 Game Seven
takers often don't see or use deductions.
Question 14
It's a "What can be true?" question, so try each answer out until one works.
Answer A
Let's test it out: connect M and P in a diagram and see if it works. Also, note that no one
else is connected with M and P.
Connections Cities: H M P T V
M and P
no more M and P
Notice that not only did I cross off M and P in my list of cities, I scrawled a little note to
myself in the connections column that I can't use M and P any more; I know that I
sometimes don't notice that I've crossed things off, so I make sure I can't forget what the
answer tells me to do (in the alternate diagram, I just crossed off both cities to note that I
couldn't connect them any more because it's harder to miss).
Let's see if we can make this work. Rule 1 tells us that every city must be connected at
least once (this, by the way, is the type of rule you are likely to remember without looking
at it when you do a game; see Intermediate Techniques). So we need to connect H, T,
and V with someone. We can't connect H and T (rule 3), nor H with M or P (because they
are crossed out), and H has got to be connected to something, so H and V have to be
connected. This tell us that T and V must be connected (rule 4).
Connections Cities: H M P T V
M and P
no more M and P
H and V
T and V
Look through the rules again; we're following all of them. This works, so this (answer A) is
the answer.
Notice this started as a difficult-seeming game, but we've just had three easy questions in
a row.
Question 15
Draw a new diagram and plug in what we've been told.
Connections Cities: H M P T V
T and P
no more P
Notice that I wrote "no more P"; question 15 tells us that P is connected only with T, but it
doesn't tell us that T isn't connected to anyone else (it doesn't say "P is the only city
H M
P
T
V
H M
P
T
V
H M
P
T
V
Game Seven 169
connected to T."). Look through the rules and see if any of them apply. Well, we've got to
connect H, M, and V to other cities (rule 1). Rule 7 also applies, because P and T are
connected, but it doesn't tell us anything we haven't already written down we already
know that P won't be connected with V.
See if we can find an answer that can be true.
Answer A
This answer means we don't get to connect T with anyone other than P. See if we can
connect up the cities that are left (H, M, and V). Connect H first (because it is first in the
alphabet). If H connects with M, M has to connect with T (rule 4), but only P connects to T
according to the answer. H can't connect with P because P is crossed out. H and T can't
connect (rule 3). And H and V can't connect because then V could have to connect with T,
but only P is connected with T here. H can't connect with anyone, so this answer is
impossible and we don't have to think about the other cities at all.
Answer B
This means V is only connected once. Note that in your diagram.
Connections Cities: H M P T V
one V
V and
Let's try this out. Who is going to be connected with V? Work it out; I'll wait.
OK, H won't work, because if H is connected with V, V also has to be connected with T (rule
4), but we want V to be connected only once. Try connecting V with M. Mark M with a dot
or star (or use a dotted line) because this is something we are trying out, which we don't
know has to happen. If the diagram doesn't work, we can try to change this.
Connections Cities: H M P T V
one V
V and *M
T and P
no more P
Go through the rules. M is connected only once (rule 2), so cross M out. Now we have to
connect H with someone. H can't be with M because M is crossed out. Can it be with P? No,
because we wrote down "no more P." H can't be with T (rule 3). So if V is connected with
M, there is no one to connect H to. Cross off this diagram.
H M
P
T
V
(1)
H M
P
T
V
(1)
170 Game Seven
Since the diagram didn't work, let's go back and change what we have starred. Who else
can we connect V with? H didn't work, M didn't work. P is out ("no more P"). Try V and T.
Don't star T, because this is the last person left it is forced on us by the rules (it's OK if
you do put a star there, because then you'll say, "Who can I try next?" and realize that no
one is left, so you'll arrive at the same conclusion).
Connections Cities: H M P T V
one V
V and T
T and P
no more P
Who goes with H? No one except M. Will that work? If M goes with H, M has to go with T
(rule 4), but M can only be used once (rule 2). This won't work. There is no one else to try
to connect V to. V can't be connected with just one city, so this answer is wrong.
Notice that we had to do a couple of diagrams to test out this answer. That happens, and it
can be slightly confusing, as you might lose track of what it is that you are trying to figure
out. That's why we put stars next to things we are trying out that we think may or may not
work. It lets us know what we can try to change. It's important to make these changes
systematically, in a certain order. Notice that I tried M before T it's easiest to try things
out alphabetically, or in numerical order. Once we have gone through the numbers or the
people, and none of them work, there is nothing else to try and we know that what we are
trying won't work. It's OK to glance back at the answer you are testing out to refresh your
memory.
Just think of how much more confusing this would be if you were trying to do it all in your
head.
Another thing: by this point you've probably noticed that M will never go with H you've
seen concrete examples of M not being able to go with H, and you understand the rules well
enough to see why it is impossible. So this is a useful deduction, and you can write it down
as a rule. See the section on Intermediate Techniques.
Answer C
Make a note that H is connected twice:
Connections Cities: H M P T V
T and P
no more P
H and
H and
The question is, who are these two cities going to be? By this time we know that M can't be
one of them. P can't be one of them that limitation is built into this question. So they
have to be T and V. But, going through the rules, we see H can't be connected with T (rule
3). This is clearly impossible. Cross out the answer and the diagram and move on.
H M
P
T
V
(1)
H(2) M
P
T
V
Game Seven 171
Answer D
T is already connected once, so note that it has to be connected again.
Connections Cities: H M P T V
T and P
no more P
T and
So, what is this mystery city that is connected with T? It can't be H (rule 3). Can it be M?
Try it out.
OK, finished? You should have seen that it won't work. Whoever you connect to H will have
to be connected to T, which will give T too many connections. So T can't be connected with
M. Let's try someone else; only V is left.
Connections Cities: H M P T V
T and P
no more P
T and V
Now we need to connect someone to H. It can't be M, P, or T (we're only using T twice,
remember?), so it must be V. I don't dot or star this connection, because this is forced
upon us by the rules. Go through the rules this seems OK so far, because V is already
connected to T.
Connections Cities: H M P T V
T and P
no more P
T and V
H and V
Now we just need to connect M to a city. It won't be H, P, or T. But M and V can be
connected. V is doing a lot of work for us, but there is no rule against it.
Connections Cities: H M P T V
T and P
no more P
T and V
H and V
M and V
This follows all the rules, so this (answer D) is the answer.
H M
P
T(2)
V
H M
P
T(2)
V
H M
P
T(2)
V
H M
P
T(2)
V
172 Game Seven
Question 17
Here we are told that one city is connected to four other cities. This means that the city in
question is connected to all the other cities. What city could this be? You work it out, and
work it out by looking at each city in turn.
OK, here's how I did it. We already know that H can't connect to everyone, because it can't
connect to either M or T. M clearly can't connect to more than one city, so M is out also.
Could it be P? Look through our rules. We can't have P connected to both T and V (rules 6
and 7), so P can't connect to all four others. T can't connect to H. So the only city left is V;
V must be connected to all four cities.
This is one of those questions that seems really difficult, but isn't. At first you panic you
say, "I don't know what city this could be!" But there are only five cities, and there is
nothing wrong with considering each of them in turn. If that seems like it is going to take
too long, then work through the answers. But here, if we are calm and systematic, it all
clicks.
This is our diagram so far:
Connections Cities: H M P T V
V and H
V and M
V and P
V and T
Notice that every city is connected to another city, so I don't need any more connections.
There might be others, but they don't have to be here. This is a "What must be true?"
question; we can eliminate any answer that doesn't have to be the case.
Answer A
Well, we can't connect H and M, so this is wrong.
Answer B
We don't need to connect any more cities than we already have; the diagram that we've
created could be the final diagram. So we don't have to connect H and P.
Answer C
Ahh, a connection between H and V is right in this diagram, so it has to happen. C is the
answer. We are done.
Question 13
Let's use our previous diagrams to eliminate any cities that we know can be connected.
Answer A
Well, we don't have to look any further. By this point in the game, having done as much
work as we have, we know that H and M cannot be connected. This (answer A) is the
answer.
Just to satisfy your curiosity, here's how to eliminate the other answers using previous
diagrams. Eliminate B with the diagram from 12B. Eliminate C with 12B or 14A. Eliminate
D with 15D or 17. Eliminate E with 15D.
H M
P
T
V
Game Seven 173
Question 16
This is a "What is the maximum number?" question. Look at our previous diagrams and
see what is the largest number of connections we have so far. The diagram for 12B has five
connections, so we know that A is wrong.
At this point on the LSAT, you really need to look at the clock. This is the last question in
the game, and it's a hard one to answer. I would be very tempted to skip this question,
even if I had over a minute left on this game. Just think of what you could do with that
time on a different game; it could be much more fruitfully employed elsewhere you might
not even finish this question in a minute, which would mean that that time will be wasted
when you move on. In many cases it's better to move on early than to start a hard
question that you know you are likely to end up skipping halfway through for time reasons.
On this actual LSAT, this would be the last game you did (you'll understand why when you
read the Timing section), so skipping this question would be something of a non-issue.
Still, the point is relevant to other games.
If you did skip this question, you would pick B, since five is the largest number of
connections you know is possible.
Well, let's try this out. Start with answer E, since it is the largest number of connections
you are given.
Answer E
We need to generate eight connections. We probably don't have any idea what they would
be, so let's go through our cities one by one and try to connect them as many times as
possible. Start with H who can it connect with?
H can only connect with P and V, so connect them (and cross H off because it can't connect
any more).
Connections Cities: H M P T V
H and P
H and V
Looking at the rules, we see that P and V will have to connect with T as well as H (from rule
4).
Connections Cities: H M P T V
H and P
H and V
P and T
V and T
H M
P
T
V
H M
P
T
V
174 Game Seven
Now let's move on to M. It can only connect to one city, so connect it to whomever you like
(as long as it isn't M). I like P. Since it doesn't matter who I connect M to, I don't bother to
mark this with a star; there's no reason to change this later on. Cross M off because it is
done.
Connections Cities: H M P T V
H and P
H and V
P and T
V and T
M and P
Now on to P. We want to connect it as much as possible. It's already connected with H, T,
and M. Can we connect it to V as well. Nope (rule 7).
Now on to T. It's already connected to P and V. It can't connect to H, and M is done, so
there is no one else for T to connect with.
How about V? Anyone else for it to connect with? It's already connected to H and T, M has
been used, and it can't connect to P because of rule 7. So this is it, we've got all of our
connections, and there are only five. We tried to get eight, but we couldnt make it, and
there's nothing here we can change. The answer is B.
Summary
What lessons have you learned from this game? Well, the lesson I want you to learn is that
no matter how odd they first seem, all games are really the same on the inside. It is
impossible for a game not to be a case of matching two or more types of things together.
All you have to do is spot the two types of things that the game is interested in and you are
in business. Our diagram wasn't terribly fancy or exciting, but it did the job and that is all
that matters.
I also want to point out a possible variation on this game. We can imagine a case where H
being connected to M is different from M being connected to H (e.g., one-way flights). This
doesn't make the game any easier or harder; it would just be something that we would
need to be aware of.
Any other lessons to be learned from this game? No, and that is the most important lesson
of all. You already had all the tools you needed to get these questions right. When
students do poorly on a game like this it is because they lack the confidence in themselves
to just work through the questions. Don't be afraid of games; don't try to approach
different games differently. You know how to answer questions on any game you might see
on the LSAT; all you have to do is actually apply your skills.
What Do I Do Next?
Take a break and then redo the game without notes. Do the game twice, once with each
type of diagram, and see how you like them. Make sure you are working through the rules
systematically one by one, in order.
Once you've done this game on your own and feel comfortable with it, go on to Game
Eight.
H M
P
T
V
Game Seven 175
Answer Key
11. A
12. B
13. A
14. A
15. D
16. B
17. C
176 Game Eight
Game omitted from electronic version of book.
Game Eight 177
Game Eight
(From LSAT 41)
This is one of the rare games where I want you to do something sort of special for the
diagram. I generally dislike teaching students new techniques if the techniques you
already know are capable of dealing with a situation on the test, why learn more? It just
makes things more complicated, and crowds your brain. But this is a case where a
somewhat unique diagram makes the game noticeably easier, the diagram is not hard to
learn, and it is very obvious when you should apply it. So here we go!
Step 1: Read the Setup/Draw a Diagram
Read through this game's setup. What makes this game different from other games we
have done? Well, the people are sitting in a circle; usually they are in a straight line.
Circles have the odd property that they don't have beginnings or ends, so everyone is next
to two people. Lines aren't like that. We want to build the properties of circles into our
diagram. We could just diagram this in a line, like we usually do, and remember that the
people on the ends are next to each other, but making circle diagrams is fun and easy.
There is one other thing we want to remember people sitting in circles sit across from
each other. The game tells us that two people are across from each other if they have three
people between them on either side. This isn't so shocking; there are eight people, so if I'm
sitting exactly opposite from you, half of the other six people should be on my left, and half
on my right. We also want to build this aspect of the circle into our diagram.
To remember who is across from whom, our diagram is going to have lines connecting each
pair of people across from each other. There are four pairs of people across from each
other, so we draw four connecting lines.
At the end of each line will sit a person.
Now we write down the names of the people, and our diagram is done.
One thing to notice is that we have no reason to number the seats the setup doesn't talk
about numbers at all. Should we put numbers in anyway? When in doubt about the setup,
look at the rules. They don't mention numbers either (for example, by talking about "the
fourth seat," or "the first seat"), so we won't add numbers to our diagram. This means that
no seat is any different from any other seat until we start putting people in.
F G H I K M O P
178 Game Eight
Step 2: Read the Rules
"Across" in the first rule means that there are three people between F and G, on either side,
so we include that when we write the rule.
F across from G (three between them)
The second rule is a little more complicated. It tells us about H's relation to both F and K.
It relates H to two different people, so I think we should break it into two rules to make it
simpler. One rule tells us that H is not next to F, and the other rule tells us that H is not
next to K. Notice that the order is not specified, so, for example, H can't be on F's right or
left.
We can write these rules about H, F, and K one of two ways:
H not next to F
H not next to K
or
no HF
no FH
no HK
no KH
I prefer the second way it spells out in a very easy to read way what will never happen.
The last rule tells us that I will be next to O, clockwise. We could write this in words, but
we would have to put in enough words to make sure we don't misunderstand it later:
I immediately clockwise from O
I grew up looking at digital watches, so it takes me a moment to visualize what clockwise
means. It is easier for me to write this rule in a way that visually represents what I will see
in the game. Remember, it is best if the way in which you write rules mimics the way the
people will appear in the diagram. I like to write this rule in this way:
The little arrows tell me that it will look one way on one side of the circle, and another way
on the other side. You don't need to write the rule in exactly this way, because you may
not need help thinking about clockwise in the same way I do; the point is that you should
find a way of writing rules that you find easy to understand, and that is responsive to how
you think.
OI
IO
or
Game Eight 179
Here is how our original diagram and list of rules looks:
1. F across from G (three between them)
2. no HF
3. no FH
4. no HK
5. no KH
6.
Step 3: Answer the Questions
Here is order in which you should do the questions:
Question Number Question Type
18 What can be true?
19 If
20 If
21 If
23 If
24 If
22 What is the minimum number?
Question 18
Each answer gives us the seating arrangement of half of the table four people next to
each other, in order. We have to test them out and see if they follow the rules. Since these
don't give us the seating arrangement of the entire table, we may have to draw a diagram
to test them; however, we should first check that the four they tell us about don't obviously
break any rules.
Answer A
This breaks rule 1 there are only two people between G and F.
Answer B
This doesn't explicitly break any of the rules, so we have to put this into a diagram and test
it out. Since none of the seats are different from any other seats (because they aren't
numbered), it doesn't matter where we put the people we are given, as long as we put
them in the right order. When two blanks in a game are identical to each other that is,
there are no given differences between them it doesn't matter if you put someone in one
or the other. For example, in Game Two, language Y has three spaces under it. They are
all the same all are under Y so it wouldn't matter if we put a researcher in the top,
OI
IO
or
F G H I K M O P
180 Game Eight
middle, or bottom one. Here, all the seats are seats, so we can put these people in any four
next to each other (keeping this order, of course). This is not a case of not knowing where
to put someone, so we don't need to put a little dot or star next to them. In this case, we
know any position will work they are all the same so there would be no point in putting
them someplace else later on.
I put H on top and the rest clockwise after H.
Now let's go through the rules. Rule 1 tells us F has to be across from G. But this is
impossible; none of the remaining seats are across from each other. This answer is wrong.
Answer C
This doesn't explicitly break any of the rules (it doesn't put O in, but there is room before I
for her to fit). Let's draw a diagram and test it out. Again, it doesn't matter where we put
anyone, as long as they are in the right order and next to each other, as the question tells
us.
Now let's go through the rules. Rule 1 tells us that F and G are across from each other. We
know where F is, so put G across from her.
H
O
I
K
F G H I K M O P
I
F
P
G M
F G H I K M O P
I
F
P
M
F G H I K M O P
Game Eight 181
Rules 2 and 3 tell us that H can't be next to F, but since F has no open spaces next to her,
we don't need to worry about that. 3 and 4 tell us that H can't be next to K, but we don't
know where K is yet, so we won't sweat that either. Rule 6 tells us that O will have to be in
that spot next to I. That leaves K and H in the last two spots. We don't know which goes
where, so put them in and mark them.
Since this follows all the rules, it (answer C) is the answer.
Question 19
The question tells us that H and O are next to G. But who is on what side? Look at the
rules and tell me.
You are right, it has to be H, then G, then O (and then I, from rule 6), in that order
clockwise. This is because O has to be on I's right. If O was on G's right, there would be no
room for I. We saw in the last question that we can just drop them anywhere in our
diagram, so let's do that.
I also put F across from G, since that is what rule 1 tells us. Let's go through the rest of the
rules. I know K can't be next to H, so I'll write that in too.
The question asks me which two people can both sit next to Peter. (I know that both of
them have to be next to Peter because it says "Each of whom." If it said "Either of whom,"
it would be asking for two people who can sit next to Peter, but not necessarily at the same
time; remember, its crucial to make sure you understand what the question is talking about
before you try to answer it. See the Glossary for a definition of "each" and "either.") I
don't know where P goes, but I know he'll have to go in one of the three blank spots. Let's
look at the answers.
I
O F
K* P
G
H*
M
F G H I K M O P
H
G
O
F I
F G H I K M O P
H
no K G
O
F I
F G H I K M O P
182 Game Eight
Answer A
Can P be next to both F and K? Well, we know where F is. There are blank spots on both
sides, but if P goes on the right, he'll be next to F and I, not K; if he goes on the left, K can't
be on his other side, as we have written in the diagram. So P won't be between F and K.
Answer B
Can P be next to both F and O? They are too far apart for P to be next to both of them.
This is wrong.
Answer C
This is also wrong; H and I are too far apart for P to be next to both of them.
Answer D
Well, I know where H is. Could P be between H and K? It looks possible, but let's try it out
to make sure. Put P next to H, and then put K next to P.
Since M is the only person left, I put him in the last space. Look through the rules; this
follows all of them. This (answer D) is the answer.
By this point I feel pretty confident that you can handle this game. You've seen how we
diagram it, and how we test the answers. Go ahead and try to answer the rest of the
questions. Once you've done that, work through the explanations that follow.
Question 20
We are told that G is not next to H. We don't know where G is, or where H is, so we can't
draw a diagram yet. Just write this down so we are ready to plug it in when we do test this
out.
no GH
no HG
This is similar to the last question; we are looking for two people who can be next to M. So
to test each answer, we put both people next to M and see if we can make the diagram
work.
H
P G
K O
F
M
I
F G H I K M O P
Game Eight 183
Answer A
Can F and H be next to M? I don't know what order they are supposed to go in, so I'll try
them in one order (mark them with dots or stars), and if that doesn't work I'll try them in
the other order (that is, I'll try FMH then HMF).
Now we go through the rules and see what we can discover. Rule 1 tells us where G goes.
The question tells us that G and H can't be next to each other, but that isn't a problem here.
Go through the rest of the rules. Rules 2 and 3 aren't an issue because F and H can't
possibly be next to each other. We know that K can't be next to H, so write that in.
We know that O and I have to be next to each other. Put them in someplace.
I didn't have to put O and I exactly there I could have put them one space over, counter-
clockwise. But I have to put them someplace; if this ends up causing problems, I'll move
them over. That's why I put the star next to O to tell me that if the diagram doesn't work
out, this is someone I can move.
F*
M
H*
F G H I K M O P
F*
M
H*
G
F G H I K M O P
F*
M
H*
G
no K
F G H I K M O P
F*
I M
O* H*
G
no K
F G H I K M O P
184 Game Eight
P and K are left; there is only one space where K can go, so we put him there, and P goes in
the last space.
Look through the rules. This follows all of them, so this (answer A) is the answer. Notice
that the diagram would still have worked if we'd put O one space over.
Question 21
We are told that M and O are next to each other. In what order are they? Look at the rules
and figure it out.
They have to be MO (then I), in that order, clockwise, because of rule 6. As I mentioned in
Intermediate Techniques, some rules stick in your head, and the OI rule sticks in my
head.
So I know that I'll have MOI. We can put these right into our diagram.
The question asks who must be next to F. We don't know where F goes, but the fact that
the question asks about her should make us check and see if there are any rules about her.
She has to be across from G, and there are only two spots left that are across from each
other the ones next to M and I. So F will have to go in one of those. With that hint, I'm
going to let you take a stab at this question. Look at my explanation when you are done.
This is a "What must be true?" question, so we test the answers by doing something other
than what they say. That is, we don't put the person in the answer next to F. Of course, if
an answer obviously can't be true, we can cross it off without testing it.
Answer A
Well, we know that H can't be next to F from rules 2 and 3, so we don't bother testing this
out. Something that can't be true isn't something that must be true.
F*
I M
O* H*
K
G
P
F G H I K M O P
M
O
I
F G H I K M O P
Game Eight 185
Answer B
We want to see if F could be someplace other than next to I. Well, we know that F and G
have to be across from each other, and there are only two open spots. Put G, rather than
F, next to I, and F across from him.
Now let's go through the rules and see if we can make this work. H can't be next to F, so
put her someplace else. Mark her with a dot or star so we know that we can try her
someplace else if this doesn't work.
Go through the rules again. Rules 4 and 5 tell us that K can't be next to H. But there are
no open spots that are not next to H. Does this mean that our diagram is totally wrong?
No go back and move the person we have starred (that's what the star tells us). The last
thing we did was to put H where she was. Can we put her someplace else? Not next to F
(rules 2 and 3), and not where she is, so how about next to G? That gives K room to go
next to F, and P plops in the middle.
Some of you may have been slapping your foreheads when I put H where I did before,
saying "Come on, Brian! She can't go there!" But I wanted to illustrate that sometimes you
try things and they don't work. That doesn't mean that your diagram is impossible;
sometimes you can change one thing and make it work. So, when a diagram breaks a rule,
go back to the last time when you had a choice of where to put something (which you
marked) and put it someplace else. If you run out of choices and the diagram still doesn't
work, then you know it breaks a rule. I know I've told you this many times before, but it's
so important that I'm going to keep on saying it.
Back to the question. This diagram works; this is a "What must be true?" question, so what
do we know? We know that the answer is wrong. See if you can use this diagram to
eliminate any other answers. The diagram has F next to K and M, which isn't different from
what C or D says, so we can't eliminate them, but it does show us that F doesn't have to be
M
F O
I
G
F G H I K M O P
M
F O
I
H* G
F G H I K M O P
M
F O
K I
P
H
G
F G H I K M O P
186 Game Eight
next to P. We can eliminate E, which leaves C and D. We only have to test out one more
answer and we'll have found the right one, either directly or by elimination.
Answer C
We got the previous diagram by putting F next to M, and we saw that K has to be next to F
in that situation; there's no reason to keep testing F there because we'll keep getting the
same result. Let's put F someplace else and try to not put K next to her. Where else can F
go? She can only go next to I (because she has to be across from G).
OK, now we don't want to put K next to F.
So where do we put K? Try putting him next to G. But then H can't go anywhere, because
H can't be next to K or F. Try putting him in the middle spot. But then H still can't go
anywhere. So this won't work. On a "What must be true?" question, when your diagram
won't work, the answer is correct. C is the answer.
Question 23
K is across from I; let's put that in the diagram someplace.
We put O in as well, since we probably remember she's on I's right. Now let's go through
the rules. F has to be across from G. There are a couple of spots F and G could go, so I'm
not going to put them anyplace. When you first set up the diagram on an If-question, you
only put in what has to be true (because you are setting up something that has to work for
every answer); when you test the answers, you can put things anyplace that they can go
(because you are only testing that particular answer). H can't be next to K, so I'll put that
in.
M
G O
I
F
F G H I K M O P
M
G O
I
no K
F
F G H I K M O P
K
I
O
F G H I K M O P
Game Eight 187
Let's try out the answers. This is a "What can't be true?" question, so we can eliminate any
answer that turns out to be possible. We test each answer by putting that person next to
O, as the question asks us to.
Answer A
Put F next to O, and put G across from her.
Notice that there is only one space where H can go (the only empty one that doesn't say
"no H" in it see how handy writing "no-such-and-such" can be?), so put her there.
M and P are all that are left, and we don't have any rules about them, so drop them in
wherever.
This works, so this is the wrong answer. Unfortunately, it doesn't help us eliminate any
other answers, since Ingrid (who is next to O) is not one of the answer choices. On we go.
K
no H no H
I
O
F G H I K M O P
K
no H no H
G F
I
O
F G H I K M O P
K
no H no H
G F
H
I
O
F G H I K M O P
K
M P
G F
H
I
O
F G H I K M O P
188 Game Eight
Answer B
Try putting G next to O.
Go through the rules. H can't be next to F (rules 2 and 3). But that leaves no space for H.
H has to go someplace, so this won't work. We can't change anything in this diagram
we've made no decisions so far, just followed rules so this is the right answer. Cross off
this diagram and circle answer B.
Question 24
Put K directly across from H, as the question tells us to, and go through the rules.
Rules 2 and 3 tell us that F can't be next to H (rules 4 and 5 tell us that K can't be next to H
either, but that is not a possible issue here).
So there are a few spaces F can go in. The rules don't tell us anything else useful, so let's
look at the answers.
The question wants the minimum number of spaces between G and K (with K clockwise
from G). So we try the smallest answer first (A), and work our way up from there.
Answer A
Our smallest answer is zero. Having zero spaces between G and K means that G and K are
next to each other. Putting G right next to K (with K clockwise from G) would put F right
across from G, next to H, and that won't do. This is wrong.
K
no H no H
F G
I
O
F G H I K M O P
H
K
F G H I K M O P
H
no F no F
K
F G H I K M O P
Game Eight 189
Answer B
Let's move G one space over counter-clockwise and see if we can have one space between
G and K. Now there is one space, clockwise, between G and K.
Now go through the rules. We have to put OI in someplace. But there are no two spaces
next to each other left. This won't work.
Answer C
Put two spaces between G and K (K on G's left).
Now let's put in O and I. There are two spots where they can go in the two empty spaces
on the left (between F and H) or on the right (between G and K). Try them wherever you
want. Then put P and M in the other two spaces.
This works, so we are done. C is the answer.
Question 22
We want the smallest number of spaces between I and M (with M clockwise from I). Look
at your previous diagrams.
The diagram from 19D has M directly to the left of I; there are zero spaces between them,
and M is clockwise from I. Since there is no number smaller than zero in the answers, this
is the minimum number. A is the answer. We are done.
Summary
This game teaches us how to deal with people sitting around in a circle. From time to time
you will be presented with other situations that you can't represent by putting things in a
H
no F no F
F G
K
F G H I K M O P
H
no F G
F
K
F G H I K M O P
H
P* G
M
*
O*
F
K
I
F G H I K M O P
190 Game Eight
straight line. For example, you might have houses facing each other along a street, or
people sitting in rows. In these cases you will have to draw a diagram that helps you
visualize the situation; draw something that is like a map of the situation, or a birds-eye
(overhead) view of the area. If people are sitting in rows, make rows in your diagram; if
houses are facing each other along a street, draw blanks across from each other. These
games are not difficult once you have seen one just make sure you understand the
situation they are describing, then draw a diagram that looks like it.
What Do I Do Next?
Take a break. Then look over your notes from this game and think about how we applied
the fundamental skills to it. When you feel like you understand the game, take another
break, and then do the game without notes. By this stage you should have the
fundamentals memorized so well that you can recall them without having to think much. If
you still struggle to recall the fundamental skills how to do questions, or what order to do
them in, or how to make diagrams you need to make up some flash cards and be working
on them.
Once you've done all that, here is what you will have accomplished: you will be conversant
with all the techniques that are most important to your logic games score. In addition, you
will be ready to handle the toughest, weirdest games on the test. At this point you will now
need to practice what you have learned and turn these skills into habits. Practice games
from the recommended list at the end of Game Three, and from the recommended list at
the end of this chapter (coming up in a second); do about an equal number from each list.
Make sure that when you practice you go through the steps I talk about in the Study Guide
make a short list of the techniques you want to practice, refer to it as you practice, and
review every game you do, trying to spot things you might want to do differently in the
future. If you don't remember what these techniques are (and you probably don't), go back
and reread that chapter. You can only ensure progress by studying properly, and the Study
Guide tells you how.
Once you feel comfortable with the techniques and the games, go on to Advanced
Techniques. You will then start working on timing.
You need at least two weeks to work on timing. If you are not comfortable with all these
techniques, but you have only two weeks left to work on games (either because the LSAT is
in two weeks, or you need to work on other aspects of the test), move on to Game Nine
(skip Advanced Techniques). Game Nine through Game Twelve come from the same
LSAT, LSAT 38. They are difficult games. I use them to illustrate some of the advanced
techniques, but I also show you how to do the games with just the fundamental techniques.
Work through those chapters and read the chapter on Timing; follow the instructions
therein.
Game Eight 191
Recommended Games List
I have grouped the games by similarity to each other. You won't be able to distribute the
games you do evenly between the two groups, since group 2 contains so many more
games, but you should at least do a few from group 1. I also recommend that if you are
doing games untimed, try to do games in LSATs from which you have already done games.
Once you start working on timing, you will want to be able to do complete LSAT games
sections and work on managing your timing through the entire section. When doing this it
is important to do games from the same section. Some games have many more questions
than others, and each section generally has a balance of long and short questions. Thus,
you'll want to have available for practice many games sections that you've never looked at.
If you have access to LSATs which are dated but not numbered, there is a chart (LSAT
Number to Date Conversion Chart) which gives the date each number corresponds to at
the end of the Introduction chapter.
Group One
LSAT
20
22
33
35
36
Game #
1
3
4
3
3
Group Two
LSAT
20
21
21
22
22
23
23
24
25
27
28
30
30
31
31
32
Game #
4
1
4
1
2
3
4
3
3
2
3
1
2
1
4
4
More Group
Two
LSAT
35
36
37
39
39
39
40
41
43
44
44
44
46
46
47
Game #
4
2
4
1
3
4
2
2
3
1
3
4
3
4
4
Answer Key
18. C
19. D
20. A
21. C
22. A
23. B
24. C
192 Advanced Techniques
Advanced Techniques
Who Should Learn These Techniques?
The following techniques can be helpful in many circumstances, but they are not necessary
to do well on the LSAT, and there are some people who should not bother learning them. If
you are still having trouble with the fundamental techniques I discuss in The
Fundamentals that is, if you often forget steps, or don't always feel comfortable going
through the steps don't bother reading this section; mastery of the basic techniques is
crucial for LSAT logic games success, and spending time acquiring that mastery will get you
more questions right than spending time learning these advanced techniques.
Let's say that you have mastered the fundamental skills I cover in The Fundamentals, but
your test date is a week or less away. Don't read this chapter. These skills take some time
to learn; if you get to the LSAT and have only half learned them, they may do you more
harm than good.
What if you have mastered the fundamentals, and have plenty of time before the LSAT, but
haven't mastered what I cover in Intermediate Techniques? Spend at least a few
practice sessions practicing the intermediate techniques. You don't need to feel completely
comfortable with them, but you need to be aware of what they are and how they work.
Learning several skills at once makes you slower at learning any of the skills.
Once you remember and feel comfortable applying all the fundamental skills, and are
familiar with the intermediate techniques, and if you have a couple of weeks before the test,
ask yourself if you have reached your logic games score goal (or are very close). If you are
already doing as well as you want to on the games, you may not need to learn any more
techniques. But, if you have ambitions that you have not yet met, read this chapter and
add these skills to your short list. Learn not just how to use these skills, but also when.
Reusing Previous Diagrams Equivalence
Now, raise your hand if you like to use previous diagrams. All of you? Great. Now, there
are only two problems that I know of with using previous diagrams: first, that we can't use
them on every question, and second, that they don't always eliminate as many answers as
we'd like. Well, this technique addresses that second issue.
We'd like to eliminate more answers using previous diagrams if we can. Sometimes, when
we look back at previous diagrams, we find that the situation we are looking for isn't in any
of them. Yet, when we test the situation out, we find that it could have happened, meaning
that it could have appeared in a diagram. So why didn't it? Why?!
Well, I'm not going to answer that question. But I am going to tell you how to get more use
out of the previous diagrams that you have. That'll be nice, right?
Here's how it works: some games have certain variable people/things that are equivalent.
Remember, the variable things are the ones we didn't build our diagram around; we write
them off to the side and put them in blanks when we can. In Game One, the horses are
variable. In Game Two, the researchers are variable. In Game Three, the birds are
variable. In Game Four, the cars and washes are variable. In Game Five, the medications
are variable. In Game Six, the employees are variable. In Game Seven, the cities are
variable. In Game Eight, the people are variable.
Advanced Techniques 193
are variable. In Game Six, the employees are variable. In Game Seven, the cities are
variable. In Game Eight, the people are variable.
When two things are equivalent, anyplace one can go, the other can go as well. Let's
imagine that X and Y are equivalent in some game. We know (because we have a previous
diagram) that X can go 1
st
. That tells us that Y can go 1
st
too, since the two are equivalent.
Or let's say that we have a previous diagram where X is 3
rd
and Y is 5
th
. Well, then we
know that X could be 5
th
and Y 3
rd
we can just switch them. This potentially doubles the
number of diagrams that we have (because you can switch the two in every diagram).
Not only can variable things be equivalent, definite things in a game can be equivalent.
The definite things are the ones you build the game around. In Game One, the numbers
are fixed. In Game Two, the languages are fixed. In Game Three, the in-forest and not-
in-forest groups are fixed. In Game Four through Game Six, the numbers are fixed. In
Game Seven, the connection group is fixed. In Game Eight, the table is fixed.
For example, if I have three fixed groups, A, B, and C, two of them might be equivalent.
Let's say A and B are equivalent; anyone who goes in A, then, could go in B, and vice versa.
I can look at any previous diagram and switch everyone in A and B, and that's a good
diagram too. When switching between equivalent fixed things in a diagram, you have to
switch every variable thing that goes in that fixed place with everyone in the other fixed
place. Why? Because you may have rules about one of the variable things; if you switch
only one of them, you are assuming that those variable things are equivalent. If they are
not, just switching one of them may break a rule. But you know that the entire collection of
them in that place works, so making the switch to an equivalent place all at once will work
as well.
Now, how do we know that two people or things or places are equivalent? If two
people/things/places don't have any rules about them, they are equivalent. If two
people/things/places have exactly the same rules about them, they are equivalent. In the
first case, neither person/thing/place is limited in any way, so there is no reason not to be
able to switch them. In the second case, the two people/things/places are limited in all the
same ways, so any way that works for one will work for the other (because the rules that
govern the two are the same).
Now, this means that spots in the diagram are rarely equivalent. Take a game where we
are putting things in order, from 1 to 5. Each spot is different from every other spot; 2 is
the only spot next to both 1 and 3, 3 is the only spot next to 2 and 4, and so on. So no two
of these spots are equivalent.
Let's look at some of our previous games and see where we can find equivalence. Look at
Game Two. We have researchers and languages. Are any of the researchers equivalent?
Sure L and P are equivalent. Neither can learn the same language as G, and that is the
only rule that governs either. G and H aren't equivalent because G can't learn the same
languages as L or P, but H could (we have no rule against it). Are any of the languages
equivalent? You tell me S and T are equivalent, because two researchers learn each, and
no other rules govern either of them. So any place L can go, P can go, and any group of
researchers who can learn S can learn T.
Now, does this actually help us on this game? Yes, on question 9. On question 9, we want
to see who can learn both S and Y at the same time. We look at previous diagrams, and
194 Advanced Techniques
switch the people in S with the people with T (because S and T are fixed, we have to switch
everyone at once). Here, for example, is the diagram from question 6, answer D:
R S T Y researchers: G H L P
L* H H H
G* P* L
P
We can switch everyone in S with everyone in T.
R S T Y researchers: G H L P
L* H H H
P G L
P
This tells us that we can have H and P in both S and Y; this eliminates one of the answers.
We can also switch P and L.
R S T Y researchers: G H L P
P H H H
L G L
P
This gives us H and L in both S and Y, eliminating another answer. See how this works?
In Game Three, none of the birds are equivalent they are all subject to different rules.
The two groups
in-forest and not-in-forest aren't equivalent, because they are subject to different rules as
well (for example, rule 1 tells us about H being in-forest, but we don't have an identical rule
about H being not-in-forest).
In Game Four, none of the people (Frank, Marquitta, and so forth) are equivalent.
Everyone has to be before or after different people. For example, F is the only person who
has to be after M; O and T both have to be after V, but O is also before M, so O and T aren't
equivalent. The washes aren't equivalent, because there can be only one p, whereas there
has to be at least one s, and there have to be at least two r's (since M is r and the car
before M is r). The positions aren't equivalent because each is before and after a different
number of positions.
In Game Five, none of the medicines are equivalent because each appears in different
rules. For example, F and G look equivalent, because one of them has to be 1
st
, but then F
is in rule 6 with K and rule 8 with M, and G isn't. The positions aren't equivalent because
they are numbered, and the groups aren't equivalent because there will be five medicines in
the "test" group but only two in the "not" group.
In Game Six, M and S look equivalent, but they aren't. You can't have an M or S attending
two of the same type of session if you switch an M with an S, you might inadvertently
switch them into sessions they already attend. The sessions aren't equivalent for the same
reasons that the employees aren't: if you switch the people in H and R, you might make
the same person attend two of the same kind of session. Of the days, days 1 and 2 are
equivalent because there are no rules about either. If you switch everyone on day 1 with
everyone on day 2, you will always get a good diagram (provided that the diagram you
started with was good).
Advanced Techniques 195
In Game Seven, no cities are equivalent because they are each subject to different rules.
In Game Eight, M and P are equivalent because there are no rules about either. We can
switch the two whenever we want. That ends up not helping too much because almost all of
the questions are If-questions, so we don't use previous diagrams (except as I talk about
below). One thing to point out: If-questions say that people have to go in certain places,
so this might make equivalent people temporarily not equivalent. For example, question 21
puts M in a certain place. Any diagram you make for that question has to have M in that
place, so you can't switch M and P in diagrams for this question. For example, let's say you
made a diagram for answer A, and you wanted to just switch M and P to make a diagram for
answer B. You couldn't do it, because it would violate what the If told you. On If-
questions, the condition laid down in the If, if it is about one of two equivalent things, will
make those equivalent things not equivalent for that question.
You can see that equivalence can get a little complicated. The key idea is that if two things
are governed by all the same rules (or by no rules), switch them (or everything associated
with them) at will, unless otherwise specified by the question you are doing. Here is my
recommendation on how to master this. Go back through games you have already done
and spot any equivalent things. Then go through your questions and previous diagrams and
see where you can use equivalence to answer questions. Make sure it gives you the right
answer; if it doesn't, you did something wrong, and you should go back and check.
Once you are comfortable using this concept on games you have already done, you are
ready to try it on a new game. It is only something to be thinking about in the later part of
the game, when you've answered all the If and "What can be true?" questions, and are
working with previous diagrams. It won't help you on every question, and it won't eliminate
the need to make test diagrams in general. But it can save you time here and there.
Some cautions: if you find that you spend more time thinking about what is equivalent, and
trying to rearrange previous diagrams, than you do making test diagrams, you are hurting
yourself. Students often spend a lot of time trying to take a previous diagram and totally
rearrange it to make it useful for an answer. This is often more complex than just creating
a diagram from scratch, and there is no guarantee that it will actually work. Making a test
diagram always gives you a diagram that is relevant to the answer you are testing. Don't
try to make multiple changes on a single previous diagram. If you can't get a helpful
diagram by making a single switch, you are probably wasting time looking at it. Move on to
another diagram, or test the answer out yourself. One more time making test diagrams
from scratch guarantees a useful diagram. Using previous diagrams is supposed to save
time, but if you are trying to make too many changes using equivalence, that defeats the
purpose.
Reusing Previous Diagrams on If-questions
I told you that you couldn't use previous diagrams to test answers on If-questions. Well, I
wasn't being entirely honest. The skill is a little complicated, and often not that useful, but
once in a while you can do it. Any previous diagram that matches the situation described in
the If part of the question can be used on that question. This includes diagrams from other
If-questions. For an example, take a look at Game Five. Question 10 has as its "if"
condition that I ranks third. We've done two questions before this, 6 and 7. In the diagram
for 6D, I is third, and in the diagram for 7C, I ranks third. We can use these diagrams to
help us on question 10.
196 Advanced Techniques
Use the diagrams as you would for a question that didn't have an "if." For example, if the
question says "If blah blah, what can't be true?", you can eliminate any answer that gives a
situation you see in the previous diagram. If the question says "If yadda yadda, what must
be true?", you can eliminate an answer if the previous diagram gives you a situation
different from what the answer describes. And if the question says "If such and such, what
can be true?", the previous diagram probably won't be helpful.
Let's see how this would work on Game Five, question 10. This is a "What can't be true?"
question, so we can eliminate any answer that appears in our previous diagrams. Neither of
our previous diagrams has M before H, so we can't eliminate A. Neither has K before G, so
we can't eliminate B. But the diagram for 7C has I before F, so we can eliminate C. The
diagram for 6D has H before M, so we can eliminate D. And 7C has G before K, so we can
eliminate E.
Now, why can we use these diagrams? Well, the "if" condition can be treated as a
temporary rule it is something that you have to obey when testing the answers to that
question. We still have to obey all the regular rules as well. If we find a previous diagram
that matches the situation described in the "if," we know that it conforms to this temporary
rule and all the regular rules (since every diagram follows all the given rules). Thus, it is
relevant to this If-question.
There are usually very few previous diagrams that can be used for a given If-question,
because most diagrams won't match the "if" condition. It's typically a waste of time to look
for these previous diagrams unless you've already done several questions. If you decide to
use previous diagrams, look for ones you can use on that question (ones that match the
situation described in the "if") before doing any other work for the question. Once you find
a diagram that you can use, look at all the answers and see if it eliminates any of them.
Then look for another diagram that you can use. If you run out of diagrams, make your
own diagram, plug in the "if" rule, go through the regular rules, and then test out the
remaining answers as normal.
Making Deductions Before You Look at the Questions Useful and Easy
Deductions
Sometimes two rules that you are given can be combined to make a third rule. That is, you
are told two things have to be true, and on the basis of that you can figure out that
something else, something you weren't told, also has to be true. This new thing that you
figured out is called a deduction. If you can figure some of these out before you look at
the questions, you can save yourself a bit of time.
Making deductions can be a difficult and time-consuming process. The time it takes is
typically wasted; I find that deductions rarely save more time in answering questions than it
takes to make them, and they almost never save more mental energy than they cost.
Everyone knows that you have very little time on the LSAT, but it is equally true that you
have very little mental energy to spare; a big part of getting a good LSAT score is
minimizing the energy you use on the test. That said, there is one type of deduction that is
easy to make (thus requiring a minimum expenditure of thought and energy), can be made
quickly, and is often very useful. So I am going to teach you to make it.
Advanced Techniques 197
Here is what to look out for: rules that tell you one person/thing is before another. These
are the kinds of rules we write using the word "before." Let's say we had the following rule:
X before Y
From this we know that X can't go in the last spot. Why not? You tell me. That's right,
because then there would be no place for Y (because Y goes after X). What else do we
know? We know that Y can't go 1
st
, because then there would be no place for X. These two
things are worth writing down.
But there is more. Games often contain several of this kind of rule, and often more than one
of them is about the same person. Count up the number of people who will go after X; for
each of them, we know X can't go in one more spot at the end. For example, if we know
that
X before Y
X before W
X before Z
Three people go after X. So X can't go last, can't go 2
nd
to last, and can't go 3
rd
to last. If
there were seven spots, X couldn't go 7
th
, 6
th
, or 5
th
. Why not? Because if X went 7
th
, there
would be no room for W, Y, or Z. If X went 6
th
, there would be room for one of these three,
but not for all of them. If X went 5
th
, there would be spaces for two of them, but not for all
three.
Likewise, we know that W, Y, and Z can't go 1
st
, because X has to be before each of them.
But one of them could go 2
nd
, because then we would have room for X.
Here's how we could put it in our original diagram:
1 2 3 4 5 6 7
no Y no X no X no X
no W
no Z
Now, it can get a little more complicated. Imagine the following set of rules:
A before B
B before C
This tells us that two people have to go after A; A is before B, who is before C. A must be
before C as well. We also know that C has to go after two people A and B; thus, it can't
be 1
st
or 2
nd
.
Once you've figured out that there are multiple people before or after someone, it is good to
write this down. If you know the order they have to go in (like in the A, B, C example),
write this down as a sequence of "before"s. If you don't know the exact order they will go
in (in the W, X, Y, Z example, W, Y, and Z are after X, but we don't know what order they
go in; W might be before X or after X, etc.), write it down in a way that tells you that you
don't know the exact order. So you might write something like:
A before B before C
198 Advanced Techniques
(I will often just add the C rule to where I wrote down the A rule, so as not to write a bunch
of similar rules)
X before W Y Z (what order?)
Now let's see how we could use this in games. In Game One, we know that M is before N.
So M can't be 6
th
and N can't be 1
st
. Not that useful. We don't know anything about K and
L, for example, because we don't know which is before the other (they can go in either
order). But look at Game Four. Here are the relevant rules:
V before O
V before T
M before F
O before M
Well, we've got V before O and T, but we don't know what order they go in, so just add in
the T to the O rule, like so:
V before O, T (what order?)
Now we've got O before M, so add this in; they are in a definite order, so we want to include
that in our rule. However, with the T in there, it all gets confusing. I'm not going to include
the order in the rule, because I still have the "O before M" rule to remind me of the order. I
know I'll confuse myself if I put it together, and I have all the information written down
someplace, so it doesn't have to be written in the most efficient way possible. Remember,
our goal is to make the rules clear and complete.
V before O, T, M (what order?)
O before M
But now we know that M is before F, so we have to put that in. I also combine it with the O
before M rule.
V before O, T, M, F (what order?)
O before M before F
There are only five spaces in this game. V has to have four people after her, so she can't
be, let's see, 5
th
, 4
th
, 3
rd
, or 2
nd
. That makes her 1
st
. O can't be 5
th
or 4
th
, so he has to be
2
nd
or 3
rd
. M can't be 5
th
. F can't be 1
st
or 2
nd
or 3
rd
(because V, O, and M are before him).
M can't be 1
st
or 2
nd
(because V and O are before her). See how useful that was?
Let's see this in a diagram:
1 2 3 4 5
V __ __ __ __
__ __ __ __ __
no F no F no O no O
no M no M
Advanced Techniques 199
Sometimes when people can only be in a couple of slots, it's better to show where they can
go, rather than where they can't go. Based on our above diagram, O can only go 2
nd
or 3
rd
,
F can only go 4
th
or 5
th
, and M can only go 3
rd
or 4
th
. It might be easier to write that than to
write all the "no this" and "no that." I like to use brackets, like this:
We don't have to draw these brackets when we draw diagrams to test answers. These
brackets are here for our reference, like new rules. We can see them when we look back at
the rules so we don't have to draw them in every time, just like we don't rewrite the rules.
Notice, though, that we did the game previously without making these deductions. We
made some of them (about V and M) as we went through the questions, and totally missed
the deduction about O (I never made that deduction myself until I was writing this book).
The point is that, while these deductions can be quite handy, you don't need to make them
initially. If they are important, the questions will force them on you. That said, if you can
get good at making them, and these deductions are straightforward and quick, you can
often save yourself some time.
So let me summarize. Look for rules that tell you who is before whom. If someone is
before or after multiple people, figure out who they are before and after and write this
down. Count the number of people before someone. For every person before them, they
can't go in one of the first spots (moving left to right). Count the number of people after
them. For every person after them, they can't go in one of the last spots (moving right to
left). Write all this down. Get used to this and it is quick, quick, quick. It is totally
memorizable and totally mechanical, just the way logic games should be.
Making Deductions Before You Look at the Questions General Theory
We've just discussed one type of deduction you can make before you look at the questions,
but there are others. I was very tempted not to include anything further in this book about
making deductions, but I knew that someone out there reading this book would hunt me
down if I didn't. So here you are.
Before I talk about how to make these deductions, I want to give one last warning. You
may think that I have belabored this point about how bad deductions can be, but I don't
think I have. Among the brightest and most ambitious students, a focus on making
deductions before answering the questions is the number one cause of bad LSAT scores.
This is because these students tend to be perfectionists they want to figure out everything
possible about the game before they do any questions. This is a waste of time, for all the
reasons I have already talked about: deductions rarely save you much time, and most
games either have few deductions or the deductions are very difficult to find, meaning that
the time spent looking for deductions often results in finding nothing. Even when a
deduction is made, the costs are often not worth the benefits. Among students who think
logic games are difficult and confusing, one of the primary causes of that confusion is trying
to make deductions without the help of the questions. The process of making deductions
1 2 3 4 5
V __ __ __ __
__ __ __ __ __
M
O F
200 Advanced Techniques
requires more thought and decision-making on the part of the test-taker than any other
aspect of the game, and it is hard to know where to start looking. Thinking and making
decisions with little guidance, under time pressure and in the presence of huge amounts of
information, results in confusion and possible mistakes. When you do questions, on the
other hand, the answers give you guidance as to what to test out, making things much
easier. Plus, thinking when doing questions is guaranteed to help you get questions right,
whereas thinking before doing the questions is not.
My concern is that students want to spend the most time learning what they find
challenging, rather than what will improve their scores the most. Making deductions before
looking at the questions is a challenge, and thus students overemphasize it, and don't learn
more important skills. So here is my first warning: don't try the following unless you have
mastered everything else in this book.
OK, if you are still reading, I'll assume that you have mastered everything else in this book.
If you haven't, shame on you for trying to sneak past me. Now, here's the second warning:
the most important part of making deductions is knowing when to stop. The deduction-
making process is a potentially vast pit of time wasting. If you know when not to make
deductions, then you won't go wrong.
Here is how to make deductions. Start by looking at the first rule. This rule will be about at
least one thing. Now, put your finger on this first rule, so you remember where you are.
Keeping your finger where it is (marking the first rule), skim down the list of rules and see if
there are any other rules about that thing, or any other aspect of this rule.
Once you have looked at every thing in that first rule, go to the second rule, and do the
same thing. Only look at rules under the rule you have your finger on (because you already
tried to put together the rule you're looking at with the rules above it when you had your
finger on them earlier).
Now, what if you find that you have more than one rule about the same thing? What do
you do? First, make sure that you are aware of all the rules about that thing. Are you?
Good. If you are working with a variable thing, use another finger, and point at each empty
spot in your diagram. Ask yourself, "Can this thing go here?" If it looks like it, look at the
next spot. If it would clearly break a rule (and you have no doubts about it), write down
that it can't go there. Go through every spot in this way. Once you have done this, go to
the next thing/rule. If the thing you are thinking about is a fixed entity, meaning that it
can't move (it is written in one place in your diagram), then try to put every variable entity
with it. Write down any that can't go under it. It is very important that you don't get too
complicated when working out your deductions. You shouldn't have to make an entire test
diagram to see if a thing can go in a given place; it should either be obvious, or figure out-
able in one or two steps. I'll give you an example soon. If you think that you might have a
deduction, but you start getting confused, or if it seems to take a lot of work to figure out,
skip it and move on. If you are not sure about a deduction you don't know if it has to be
true or not don't write it down. Oh, that's important, let me say it again: If you are not
sure about a deduction, don't write it down. And don't worry about it; move on. You lose
points by having false deductions, not by missing deductions.
So what it boils down to is this: you are looking through all the rules, one by one, for
things that are the subjects of multiple rules. We make sure to go through the rules in
order so that we don't miss anything, or get confused. If something is the subject of
multiple rules, try it in every spot in the diagram. If a position in the diagram is the subject
Advanced Techniques 201
of multiple rules, try to put every different thing in it. If something ends up not being
possible, write it down. With some practice, you can get quite good at this.
One more thing: when you look at "If then" rules to see who they are about, only look
at the "if" part. For example, let's say you have a rule that says "If X, then Y." You have
another rule that says "If A, then Y." These rules are not about the same person (Y),
because Y is not in the "if" part, so we don't try to put them together. For an explanation of
why this is, see Game Three.
Now, when do you not do this? If a game has a lot of rules, especially a lot of "If then"
rules, don't try to make deductions. It will take forever. Look at Game Three; there are
quite a few deductions to be made in this game, but making them takes longer than
answering all the questions. If figuring out the setup and the rules took you too long, don't
try to make deductions. Remember, answering the question takes the most time, and is
the most important step. A good rule of thumb is that you shouldn't spend more than two
minutes (maybe two and a half minutes) before answering the questions. If steps 1 and 2
take you two minutes, don't make deductions. If you don't understand the rules well, don't
make deductions. You are likely to confuse yourself; working through the questions is a
much easier way of understanding the game, because the questions will give you something
to work with either If conditions, or answers to test out. If you have looked at each of the
rules once, stop trying to make deductions. If you have looked at each of the rules, you
have made all the deductions that you are capable of making. Maybe you've missed
something, but you've missed it for a reason you aren't able to see it. If you were able to
see it, you would have.
OK, let's do an example. Look at Game Two. This is full of deductions. Here is our
diagram and the rules:
R S T Y researchers: G H L P
__ __ __ __
__ __ __
__
1. Each researcher can learn 1 to 3 languages.
2. If L learns a language, then G can't (learn that language).
3. If P learns a language, then G can't (learn that language).
4. If G learns a language, then L can't (learn that language).
5. If G learns a language, then P can't (learn that language).
6. If G learns a language, then H learns it (the same language).
7. If H doesn't learn a language, then G doesn't learn it (the same language).
Let's go through the rules. Rule 1 doesn't mention any particular researcher or language,
so we skip it. Rule 2 mentions L in the "if," but we don't have any other rule that has L in
the "if" part. Rule 2 mentions P in the "if" part, but no other rule does. Rule 4 is about G,
and so are 5 and 6. Bingo! We know that G has to go with H, but not with L or P. We are
aware of all the rules about G, so let's try to put G in all the spots.
Look at R first (be systematic). Can G go with R? No, because G has to go with H, but
there isn't any room under R. Can G go with S? Looks like it (there's room for H), so we
don't worry about it. Same for T. How about Y? Well, if G went with Y, then L and P
couldn't go there. But then there wouldn't be enough people to fill the three spaces. So G
can't go under Y. And, since there are only three people left (H, L, and P), they have to go
under Y.
202 Advanced Techniques
R S T Y researchers: G H L P
__ __ __ H
no G __ __ L
P
So how useful was that? We didn't make any of those deductions when we did the game,
and we didn't find the game that hard. In fact, I went through every games section since
LSAT 19, not letting myself make a single deduction of any kind prior to doing the
questions, and I found the sections do-able under normal LSAT time limits (35 minutes per
section). In only a couple of cases did I think that a game would have been easier with
deductions, and all of these cases were ones where the deductions were the easy kind I
talked about prior to this. But, if you want to try it out, here it is.
To practice these deductions, go back to games you have done before and work through
this process. Don't be sloppy go through the rules one at a time, like I say, and don't go
through them more than once. See how many deductions you find; if you find any, go
through the questions and see how they impact what you do. If you find the process not
worth the effort, forget it. But, if you find it worthwhile, and get comfortable with it, start
applying it to games in the future. Be careful to watch your time, though, and don't get
caught up in the deductions to the detriment of the questions.
What Do I Do Next?
We are going to do the entire games section from LSAT 38. I consider this games section
one of the most difficult I've ever seen. Every game in the section is challenging. We'll
employ all the techniques we've learned in this book, except the more complex type of
deductions. This will reinforce how the fundamentals help to make tough games easier, and
show us how to use these advanced techniques. Once you've seen how the new techniques
work, you can then start applying them, first to games you've done before, and then to new
games.
Game Nine 203
204 Game Nine
Game omitted from electronic version of book.
Game Nine 205
Game Nine
(From LSAT 38)
You should try to do this game on your own. When you are finished, read through my
explanation and see if there is anything you should have done differently.
Step 1: Read the Setup/Draw a Diagram
Based just on the setup, the game seems pretty straightforward. We are trying to figure
out what order the clowns come out of the car, so we are matching clowns to positions in
the order. We know more about the order than we do the clowns the order of the
numbers is unchanging (one is always before two) so we will build our diagram around
this ordering.
The game doesn't tell us how many positions there are. However, there are eight clowns,
and we are putting them in order. The game tells us that the clowns will get out one at a
time, so we know that there are eight positions, one clown per position. Since the clowns
are getting out in order, we assign a number to each position. That is, a clown will be first,
a clown will be second, and so forth. Our diagram ends up looking like this:
1 2 3 4 5 6 7 8 Clowns: Q R S T V W Y Z
__ __ __ __ __ __ __ __
Step 2: Read the Rules
Each rule talks about the order of the clowns. Some of the rules, such as the first one,
should be written as two rules for the sake of simplicity. Notice also that some rules say
"after" instead of "before." We translate "after" into "before" (if someone is after so-and-so,
you know that so-and-so is before them). We prefer "before" for two reasons: 1) we are
used to it, and 2) it mimics the order of the diagrams. If X is before Y, then X will be to the
left of Y in the diagram; X is also to the left of Y in "X is before Y," which makes for easy
reading.
1. V before Y
2. V before Q
3. Z before Q
4. T before V
5. R before T
6. V before S
7. R before W
The Deductions
Since we have all these "before"s, we can put them together as I discuss in Advanced
Techniques (the section on Useful and Easy Deductions). This assumes that we haven't
spent too much time writing down the setup and rules. If we had, we'd skip the deductions,
no matter how promising and helpful they looked. Let's go through this step by step.
As we work through the questions, I will put commentary in boxes on how you could have
answered the question without having made these deductions.
206 Game Nine
Look at rule 1 first. It is about V, and we have many other rules about V. We know that V
has to be before Y, and also before Q and S, so we add this to the first rule. We don't know
which will come first, Y, Q, or S (because there isn't a rule about it), so we note that.
1. V before Y & Q & S (what order?)
No rules tell us that Y or Q or S will be before anyone, so we have nothing more to add to
this rule. We know now that V can't be 8
th
, 7
th
, or 6
th
; remember, for every person that
comes after someone, that someone can't go in one of the last spots. Since three people
come after V, V can't go in any of the last three spots. We also know that Y, Q, and S can't
go 1
st
, because V has to be before them.
But we aren't done with the first rule. We still have more rules about V. Rule 4 tells us that
T has to be before V, and rule 5 tells us that R must be before T. Two people come before
V, so V can't be 1
st
or 2
nd
.
We can write all of this in our diagram, like so:
1 2 3 4 5 6 7 8 Clowns: Q R S T V W Y Z
__ __ __ __ __ __ __ __
no Y no V no V no V no V
no Q
no S
no V
Sometimes writing several "no so-and-so"s for the same person can make our diagram hard
to read. Instead, it may be easier to put a bracket around everyplace someone can't go
(e.g., put a bracket around 6, 7 and 8 and write "no V" under it), or to put a bracket around
the places they can go, if there are only a few of them. V can only go 3
rd
, 4
th
, or 5
th
, so it'll
be easier to write that than all these "no"s.
Now we move on. Rule 2 is about V, but we've already looked at V. It is also about Q. Do
we have any other rules about Q? Sure, we see that Z is before Q also. So Q can't be 1
st
or
2
nd
, because two people (V and Z) have to be before it, and Z can't be last.
The next rule is about Z. We don't have any rules about Z below this, so we move on (we
already combined this rule with any previous ones when we originally looked at those
previous ones).
The next rule is about T. T is before V. We already know that V has three people after it,
so T has to have these three plus V, which makes four. So T can't go in the last four
1 2 3 4 5 6 7 8 Clowns: Q R S T V W Y Z
__ __ __ __ __ __ __ __
no Y no Z
no Q V
no S
1 2 3 4 5 6 7 8 Clowns: Q R S T V W Y Z
__ __ __ __ __ __ __ __
no Y no Q no Z
no Q V
no S
Game Nine 207
spaces. Also, T is after R (rule 5), so it can't go 1
st
. This means that T can only go 2
nd
, 3
rd
,
or 4
th
. We write all this down; we add it to rule 4 (about T)
4. T before V before Y & Q & S (what order?)
and we put it in our diagram.
The next rule is about R. R is before T, who is before all these other people. R is also
before W (rule 7), but I don't know exactly where W will go in that list (it might be before T,
or after V, or anyplace). Since I already have rule 7, and I don't know where W goes after
R, it'll be too confusing to add it to rule 5. Here is what I do write down:
5. R before T before V before Y & Q & S (what order?)
R is before six people, including W, so it can't go in any of the last five spaces. This means
that it can only go in space 1 or 2. Put this in the diagram. We're running out of room
below the spaces, so I'll write it above.
The last two rules don't tell us anything new. But we've figured out quite a bit. And it
didn't take us too long; all we did was see how many people come before and after each
clown in the rules, and then quickly calculate where that clown could not go. If you go
through that process once more on your own, you'll see that it's faster to do than to
describe.
T
1 2 3 4 5 6 7 8 Clowns: Q R S T V W Y Z
__ __ __ __ __ __ __ __
no Y no Q no Z
no Q V
no S
R T
1 2 3 4 5 6 7 8 Clowns: Q R S T V W Y Z
__ __ __ __ __ __ __ __
no Y no Q no Z
no Q V
no S
208 Game Nine
Here is the final state of our original diagram and rules (note that we don't have to recopy
all the brackets and "no"s when we make diagrams to test answers; we have all the
brackets and "no"s in this diagram, so we can just refer back to it as we refer back to the
rules):
1. V before Y & Q & S (what order?)
2. V before Q
3. Z before Q
4. T before V before Y & Q & S (what order?)
5. R before T before V before Y & Q & S (what order?)
6. V before S
7. R before W
Step 3: Answer the Questions
Here's the order in which we do the questions:
Question Number Question Type
1 What can be true?
2 What can be true?
3 If
4 If
5 If
6 If
7 If
Question 1
Eliminate any answer whose list breaks a rule.
Answer A
This doesn't have R in spot 1 or 2, so this can't be right.
This answer also breaks the R before T rule (rule 5), in case you are curious as to how we'd
do this question if we hadn't made the deduction.
Answer B
This doesn't have T in spot 2, 3 or 4.
The answer breaks the V before Q rule (rule 2).
Answer C
This has R, T, and V in the right areas. But it breaks the Z before Q rule (rule 3).
This breaks a rule, but doesn't violate our deductions.
R T
1 2 3 4 5 6 7 8 Clowns: Q R S T V W Y Z
__ __ __ __ __ __ __ __
no Y no Q no Z
no Q V
no S
Game Nine 209
Answer D
This has R in spot 3.
This answer breaks the R before W rule (rule 7).
Answer E is the answer. Copy this answer down as a possible diagram:
1 2 3 4 5 6 7 8
R W T V Z S Y Q
Notice that we could have done question 1 without any deductions.
Question 2
This is a "What can be true?" question. Let's test the answers out.
Answer A
Draw a diagram; as I said above, you don't need to copy all the brackets and "no so-and-
so"s from the original diagram, since you have it all in the original diagram to refer to.
When you create new diagrams to test answers, you don't have to recopy all the deductions
from the original diagram; just make sure that you actually look back at that diagram when
you look back at the rules. It's easy to forget to look at your original diagram and end up
not using the deductions you made.
Put Y 2
nd
as the answer calls for:
1 2 3 4 5 6 7 8 Clowns: Q R S T V W Y Z
__ Y __ __ __ __ __ __
Go through the rules, looking at our original diagram (with all the brackets and "no"s) first.
I see that R has to be 1
st
or 2
nd
in our original diagram. Since Y is 2, R has to be 1.
1 2 3 4 5 6 7 8 Clowns: Q R S T V W Y Z
R Y __ __ __ __ __ __
Now we look through the rules. Rule 1 says that V has to be before Y, but there is no room
for V. So this isn't possible. Cross out the diagram and cross off the answer. Plus, we just
learned that Y can't ever go 2
nd
, so we can write that in our original diagram.
Here's how you do it without deductions. Put Y second. Rule 1 tells you that V is before Y,
so V goes first. Rule 4 tells you that T is before Y, but there is no room. This teaches us
that Y can't go 2
nd
or 1
st
(for similar reasons), which is something we can write down.
Answer B
This violates the deduction we made originally, that R has to be 1
st
or 2
nd
, so we don't
bother even diagramming it.
Here's how you do it without deductions. Put R third. Rule 5 tells you that R is before T, so
put T 4
th
, and mark it with a dot or star, because it might have gone later; rule 7 tells us
that R is before W, so put W 5
th
and mark it. Remember, every time you write something
down in a diagram, you go back through the rules, so go through the rules again. Rule 4
tells us that T is before V, so put V 6
th
and star it. Rule 1 tells you that V is before Y, so put
Y 7
th
; rule 2 tells you that V is before Q, so put Q 8
th
. But then rule 6 says V is before S,
and we are out of room. Can we move anyone around to make this work? No, the problem
is with the number of people, not their position. We are trying to fit six people in five
spaces, and this won't work.
210 Game Nine
and we are out of room. Can we move anyone around to make this work? No, the problem
is with the number of people, not their position. We are trying to fit six people in five
spaces, and this won't work.
Working through this answer teaches us that R can't go 3
rd
or later.
Answer C
Put Q fourth.
1 2 3 4 5 6 7 8 Clowns: Q R S T V W Y Z
__ __ __ Q __ __ __ __
Look at our diagram. We have multiple places V, T, and R can go (V can still go 3
rd
or 5
th
, T
can still go 2
nd
or 3
rd
, and R 1
st
or 2
nd
). Now go through the rules. Rules 2 and 3 tell you
that Z and V must be before Q. Put them in someplace, and mark them, since we decided
where to put them (note that I still prefer alphabetical order for V and Z).
1 2 3 4 5 6 7 8 Clowns: Q R S T V W Y Z
__ V* Z* Q* __ __ __ __
But wait, that gives us no room for R and T. If we move V or Z (our marked people) to
make room for R and T, we'll break rule 2 or 3. This won't work, so cross it off. We now
know that Q must be later than 4
th
. You might have been able to do this in your head, but I
like to see things in front of me; it makes it easier for me to think.
Do this without deductions: put Q fourth. As above, put Z and V in before it. Then you see
that T has to go before V, and R before T. But there is no room.
Answer D
Put S 5
th
.
1 2 3 4 5 6 7 8 Clowns: Q R S T V W Y Z
__ __ __ __ S __ __ __
We know from the brackets in our original diagram that V, T, and R will go before S (the
latest V could go is 5
th
, and S is sitting right there). So put them in. I like to put them in
spots 1, 2, and 3; not only are they the first available spots, but I also know that lots of
people will go after these three, so this gives them as much room as possible. There are
certain people in your game, the ones that have a lot of rules about them, to whom you
want to give as much leeway as possible. I put a star next to V, because I could have put it
someplace else (I don't star R and T, because my placement of V forced them to go 1 and
2).
1 2 3 4 5 6 7 8 Clowns: Q R S T V W Y Z
R T V* __ S __ __ __
Now we go through the rules. V is before Y, Q, and S, in no particular order. S is already
in, so put Y and Q in (mark them, since the order we choose is optional).
1 2 3 4 5 6 7 8 Clowns: Q R S T V W Y Z
R T V* Q* S Y* __ __
Game Nine 211
Uh oh, I see that Z has to be before Q (rule 3). So I move Q to the next available spot (7)
and put Z before it. Z has to go there (it's the only spot before Q), so I don't mark it.
1 2 3 4 5 6 7 8 Clowns: Q R S T V W Y Z
R T V* Z S Y* Q* __
W is left, so put it in the last spot.
1 2 3 4 5 6 7 8 Clowns: Q R S T V W Y Z
R T V* Z S Y* Q* W
This works, so this (answer D) is the answer.
The process for doing this without deductions is almost exactly what we just did, since we
didn't use our deductions much. Y goes 4
th
. V goes someplace before it. By this stage in
the game, we've already tested out seven answers; the main rules are probably starting to
stick in your head. You probably remember that R and T go towards the front, so you
automatically stick them before V; if you didn't remember that, your rules would quickly
force you to do it. Then you just start dropping people in where you can; the only rule that
you really have to watch out for is Z before Q (rule 3).
Here are the deductions we could have made while doing this question that we can add to
our original diagram: Y can't go 2
nd
, and Q can only go 5
th
, 6
th
, 7
th
, or 8
th
(so bracket these
spots).
Doing this question would have helped us make some of the deductions even if we hadn't
made any initially. The wrong answers tell us stuff that can't be true (see Intermediate
Techniques, the section on Making Deductions from Questions Part 1). From
answer A we learn that Y can't go 1
st
or 2
nd
. From B we learn that R is 1
st
or 2
nd
. From C
we learn that Q is 5
th
, 6
th
, 7
th
, or 8
th
.
Question 3
Before we make a diagram, let's consider if we should use our previous diagrams. In the
Advanced Techniques I told you that you can use previous diagrams on If-questions, and
this is an If-question. However, it is also a "What can be true?" question, and we don't use
previous diagrams on these. Sorry, but we are going to have to work this out.
Make a diagram and put Z seventh.
1 2 3 4 5 6 7 8 Clowns: Q R S T V W Y Z
__ __ __ __ __ __ Z __
Go through the rules. Rule 3 is about Z; it tells you that Z is before Q. So put Q 8
th
. No
other rule comes into play, because no other rule mentions Z.
1 2 3 4 5 6 7 8 Clowns: Q R S T V W Y Z
__ __ __ __ __ __ Z Q
This is a "What can be true?" question, so we test each answer out and see if we can make
it work.
212 Game Nine
Answer A
This is the sort of answer students are likely to jump on. They say to themselves, "Hey,
I've got a deduction that says R can be first or second, so of course R can be the second
clown. This is the answer."
That's wrong. We've got a deduction that says R can be 2
nd
, given the rules we have. This
If-question has limited the game even more; all the rules are going to apply, but R may be
forced to go someplace definite (other than spot 2) by Z being where it is. Don't jump to
conclusions work the answers out.
1 2 3 4 5 6 7 8 Clowns: Q R S T V W Y Z
__ R __ __ __ __ Z Q
Our version of rule 5 tells us that R has to be followed by T and V (in that order) and then Y,
Q, and S (in no particular order). So put these in, marking Y and S because, whatever
order we put them in, they might go in another order.
1 2 3 4 5 6 7 8 Clowns: Q R S T V W Y Z
__ R T V Y* S* Z Q
So far, so good, right? But we still have to look through the rules one more time. And the
last rule says that R is before W. But there is no place after R for W to go. This diagram
breaks the rules. Cross it off, and cross off this answer.
Here's how we do this without deductions. We put R 2
nd
. Then we go through the rules.
We know that R is before T, but we don't know where, after R, T will go. We put T 3
rd
and
mark it with a dot or a star, so we can move it later on if we have to. We put W after R as
well, and mark it. We go through the rules again. We put V after T, and then go through
the rules again. Eventually we realize that we are trying to fit five clowns (T, V, Y, S, and
W) into four spaces, and we can't do it. There is no point in moving the clowns that we
marked; the only direction we can move them is right, and that gives us even less room.
Answer B
Draw a new diagram based on the diagram for this question (with Z 7
th
and Q 8
th
), and put
T 4
th
.
1 2 3 4 5 6 7 8 Clowns: Q R S T V W Y Z
__ __ __ T __ __ Z Q
Now go through the rules. Rule 4 tells you that T is before V, Y, Q, and S. But there isn't
enough room for all of these clowns. This won't work.
Read the boxed explanation for answer A above; the procedure is essentially the same.
Answer C
Draw a new diagram (remember to put Z and Q 7
th
and 8
th
; on If-questions, keep the
diagram that you drew at the start of the question and recopy it for each answer).
1 2 3 4 5 6 7 8 Clowns: Q R S T V W Y Z
__ __ __ __ W __ Z Q
We have only one rule about W, that R has to be before it (rule 7). We want R to be first or
second (and we saw in answer A that it can't be 2
nd
), so put R 1
st
(spot 1 is generally a safe
Game Nine 213
bet for R, because he has to be followed by so many clowns). Then put V and T in, putting
them 2
nd
and 3
rd
since they are the first empty spaces (putting them in low-numbered
spaces is also more likely to work since they have to be followed by so many clowns).
1 2 3 4 5 6 7 8 Clowns: Q R S T V W Y Z
R T* V* __ W __ Z Q
Now we go through the rules. V has to be before Y, Q, and S. Q is already after it, so put
in Y and S; the order doesn't matter (as we noted when we wrote down the rule), so I don't
mark them because there will be no point in switching them.
1 2 3 4 5 6 7 8 Clowns: Q R S T V W Y Z
R T* V* Y W S Z Q
This follows the rules, so this (answer C) is the answer.
The procedure for doing this without deductions is the same as the procedure for doing it
with deductions; by this stage in the game you should have the sense that R, T, and V
should go in low-numbered spots, because they show up in so many rules, so your first
instinct should be to put them in first. Even if you didn't see that, you'd put them in low
spots because they are empty. After that, you just work through the remaining rules.
Question 4
Should we use previous diagrams on this question? It is an If-question, so we can; it is also
a "What must be true?" question, so previous diagrams would be helpful. We need a
previous diagram where T is 4
th
. Unfortunately, none exists. We are going to have to test
this out. Draw a diagram and put T 4
th
.
1 2 3 4 5 6 7 8 Clowns: Q R S T V W Y Z
__ __ __ T __ __ __ __
Before we look at our rules, look at what our original diagram tells us. We know that V can
only be 3
rd
, 4
th
, or 5
th
, but the rules tell us that V has to be after T, so it can only be 5
th
.
1 2 3 4 5 6 7 8 Clowns: Q R S T V W Y Z
__ __ __ T V __ __ __
Go through the rules again. Rule 1 tells us that V will be followed by Y, Q, and S, but we
don't know what order they will go in. Rule 5 tells us that R is before T, but we don't know
where. No other rules tell us about R or V, so that is all we can figure out.
Let's look at the answers; remember, this is a "What must be true?" question, so we test
the answers by doing something other than what they describe.
Answer A
Since this is a "What must be true?" question, let's not put R 1
st
. Put R 2
nd
(that's the only
other place it can go, according to our original diagram, so don't mark it with a star or dot).
1 2 3 4 5 6 7 8 Clowns: Q R S T V W Y Z
__ R __ T V __ __ __
214 Game Nine
Now go through the rules. We already know that V is before Y, Q, and S, so put them in.
1 2 3 4 5 6 7 8 Clowns: Q R S T V W Y Z
__ R __ T V Y* Q* S*
R is before W (rule 7), so put W 3
rd
.
1 2 3 4 5 6 7 8 Clowns: Q R S T V W Y Z
__ R W T V Y* Q* S*
All that is left is Z, so put him 1
st
.
1 2 3 4 5 6 7 8 Clowns: Q R S T V W Y Z
Z R W T V Y* Q* S*
This works. We can eliminate answer A. We can also eliminate answer B because Z doesn't
have to be 2
nd
. As we look through the answers to see if we can eliminate any others, we
see that D has to be the right answer V must be 5
th
, which we figured out in our initial
diagram for this question. So we are done.
This question is a bit harder without deductions, but it's still do-able. You probably won't
see that V has to go 5
th
, but you will see that V, Y, Q, and S have to go at the end (because
V is after T, and Y, Q and S are after V). When you test answer A, you'll put R 2
nd
, as we
did, not because you know he has to go there, but because it's the next available spot after
1 (or because question 2 had taught you R had to go 1
st
or 2
nd
). You'll mark R with a star.
After that, you put in V, Y, Q, and S (possibly with a little trial and error to get the right
order), and then W. That leaves Z, and you have a working diagram.
This diagram eliminates answers A, B, and E. So then you have to test out C, and you'll
realize that D is the right answer.
Question 5
Will previous diagrams be useful on this question? Potentially, because it is a "What can't
be true?" question, and we normally use previous diagrams on them. But, because it is an
If-question, we need a diagram that matches the condition in the "if." But none of our
previous diagrams have Q 5
th
, so we have to test this out by hand.
1 2 3 4 5 6 7 8 Clowns: Q R S T V W Y Z
__ __ __ __ Q __ __ __
Go through your rules. Rules 2, 4, and 5 tell us that R is before T, which is before V, which
is before Q. However, there are four spots open before Q, so we don't know exactly where
these three will go. We have to test out the answers. Since this is a "What can't be true?"
question, we simply try each answer and see if we can make it work.
Answer A
Put Z 1
st
; once you look back at your original diagram, you will see that this forces R to go
2
nd
.
1 2 3 4 5 6 7 8 Clowns: Q R S T V W Y Z
Z R __ __ Q __ __ __
Game Nine 215
Go through the rules. Rule 5 tells us that R is before T, which is before V, which is before
Q, so we now have the first five spots filled.
1 2 3 4 5 6 7 8 Clowns: Q R S T V W Y Z
Z R T V Q __ __ __
Now we have S, W, and Y left. We don't have rules relating one to the other, so we can put
them however we like.
1 2 3 4 5 6 7 8 Clowns: Q R S T V W Y Z
Z R T V Q Y S W
This shows answer A is possible, so we can eliminate this answer. It also shows that
answer E is possible, so we eliminate that answer as well.
On this question, even if we hadn't made our deductions about R, T, and V, our experience
with this game would show us that R, T, and V should go early on. Once we put them in the
earliest available spots, everything else works out well.
In addition, doing answer B on question 2 showed us that R can only be 1
st
or 2
nd
, even if
we hadn't deduced that before looking at the questions; thus, the R deduction is potentially
available to us even if we hadn't worked everything out beforehand.
Answer B
Copy the starting diagram for this question, with Q fifth, and then put T 2
nd
.
1 2 3 4 5 6 7 8 Clowns: Q R S T V W Y Z
__ T __ __ Q __ __ __
This forces R to go 1
st
(rule 5, or the brackets in our original diagram tell us this).
1 2 3 4 5 6 7 8 Clowns: Q R S T V W Y Z
R T __ __ Q __ __ __
Rule 1 is about V, so let's think about V now. It's a safe bet to put V 3
rd
, but we'll mark it
with a star so that we can move it if this doesn't work out.
1 2 3 4 5 6 7 8 Clowns: Q R S T V W Y Z
R T V* __ Q __ __ __
At this point we have S, W, Y, and Z left. We look through our rules. Z has to go before Q
(rule 3), and there is only one empty spot before Q.
1 2 3 4 5 6 7 8 Clowns: Q R S T V W Y Z
R T V* Z Q __ __ __
Now we have only S, W, and Y left. There are no rules that say one has to go before the
other, so we put them wherever we like (by the way, you might have noticed that S and Y
are equivalent (see Advanced Techniques) both have to go after V, but are not limited
by any other rules).
1 2 3 4 5 6 7 8 Clowns: Q R S T V W Y Z
R T V* Z Q S W Y
216 Game Nine
This shows us that answer B is possible, so we can eliminate this answer. And it shows us
that C is possible, so we can eliminate C. This leaves only D, so D is the answer.
The process for doing this answer without deductions is exactly the same as the procedure
we just went though.
Question 6
Can we use previous diagrams on this question? Although it's an If-question, it's also a
"What must be true?" question, so we normally would; do we have any previous diagrams
that have R in position 2? Look through your previous diagrams there's one! Question 4,
answer A (there's one more previous diagram that we can use, but one at a time).
OK, so we can use this previous diagram just the way we would for any "What must be
true?" question. If the diagram shows a scenario that is different than what an answer
describes, the answer is wrong.
Well, answer A is gone, because this diagram doesn't have S before T. Answer B gets
eliminated because T is not before W. Answer C stands, because W is before V. Answer D
stands, because Y is before Q. And E stands because Z is before W.
Alright, look for the next relevant diagram. We look for diagrams one at a time it doesn't
do you any good to spot three relevant diagrams at once, because you can only use them
one at a time. Spot one, use it, and then keep looking until you run out of diagrams. There
is another diagram we can use. The diagram for question 5, answer A has R 2
nd
. Let's see
if it can help us eliminate answers C, D, or E. W is not before V, so we eliminate C. Y is not
before Q, so D gets eliminated. And that's it, E is the answer.
Question 7
Because this is a "What can be true?" question, we can't use our previous diagrams. We
just have to test out the answers.
The "if" gives us the condition that V is before Z. Write this down.
V before Z
This eliminates one more place that V could go it can no longer go 5
th
, only 3
rd
or 4
th

because for every person who goes after V, V can't go in one of the last spots, and we have
now been given one more person who goes after V. But we don't know where V will go.
This "if" condition also affects T and R; since they are before V, they must be before Z as
well. Each of them can go in one less spot. This means that T can only go 2
nd
or 3
rd
, and R
can only go 1
st
. We can put this in our diagram.
Now let's look at the answers. This is a "What can be true?" question, so any answer that
breaks a rule is wrong.
Answer A
R can't be 2
nd
; this is obviously wrong.
1 2 3 4 5 6 7 8 Clowns: Q R S T V W Y Z
R __ __ __ __ __ __ __
T V
Game Nine 217
If you hadn't made the deductions, this would take some more work. You'd put R 2
nd
; then
you'd put T 3
rd
and V 4
th
you know that you want to put them as early as possible. Then
you go through the rules. You have to put Z, Y, Q, and S after V and W after R. This
means that you have to fit six people in five spots; it won't work, so this answer is wrong.
Answer B
You can't put T 4
th
, because we figured out when we started doing this question that T has
to be 2
nd
or 3
rd
(see the diagram above). This is obviously wrong.
Without deductions this one will be a little bit slower too. The rules tell you that V is after
T; then the rules tell you that you have to put in Z, Y, Q, and S after V (rules 1, 2, and 6,
and the "if" condition for this question); you soon realize that you don't have enough room.
Answer C
Put Q 4th.
Just looking at our diagram, we see that V has to be 3
rd
, because it's either 3
rd
or 4
th
and Q
is already 4
th
. This forces T to be 2
nd
, since T can only be 2
nd
or 3
rd
.
Now we look through the rules. Rule 3 tells us that Z has to be before Q, but there isn't any
room to put Z in. This won't work.
We also learned that Q couldn't be 4
th
from question 2. If we had written that down, we
wouldn't have had to test this out.
Put Q 4
th
, then go through the rules. V has to be before Q, as does Z (rules 2 and 3). This
leaves us no room for T, who has to be before V (rule 4), so this won't work.
Answer D
V can't be 5
th
, because V has to be 3
rd
or 4
th
. This is wrong.
V must be followed by Z (the "if" condition of this question), Y (rule 1), Q (rule 2), and S
(rule 6). If V is 5
th
, there are only three spaces after it, and we can't fit four people (Z, Y,
Q, and S) into three spaces.
So E has to be the answer.
Summary
What is the lesson of this game? This is a challenging game. Not because the rules are
hard to understand, or the setup is strange, but because you have to deal with a lot of
1 2 3 4 5 6 7 8 Clowns: Q R S T V W Y Z
R __ __ Q __ __ __ __
T V
1 2 3 4 5 6 7 8 Clowns: Q R S T V W Y Z
R T V Q __ __ __ __
T V
218 Game Nine
variables and answer a lot of questions. The questions don't make it easy on us we don't
get to reuse our previous diagrams much at all.
Now, anyone who masters the fundamentals of logical reasoning can get four or five of
these questions right in 8 minutes. And, in fact, if you follow my guidelines for managing
your time (see Timing), you would have almost 11 minutes to answer these questions
this was the longest game in it's section, meaning that you should do it last which would
most likely be enough time to get all the questions right.
But this game can be nerve-wracking. Even if you get a good enough feeling for the rules
that you realize you should generally put R, T, and V in the lowest-numbered spots possible,
it's hard to be sure that they can or can't go in certain places without working out the
possibilities. That requires time, and it requires effort, and these are commodities we'd
rather not use.
Making the deductions makes the game easier. The deductions give us no information we
didn't already have, nor do they radically change the way we approach the game. But they
give us a certain sense of surety, and they allow us to reuse the same thoughts over and
over. Once we know that R has to be first or second, we no longer have to figure out where
R goes. We can just recycle our thoughts, and we all know that recycling is a good policy.
So why not make these deductions. They are fairly straightforward to make. You know
that they are there to be made, because we have rules that say "before." You don't even
really have to catch every single deduction. All you have to do is count up how many
clowns go after each of the major players R, T, and V. It's easy to spot that R, T, and V
are the important ones because they are related, by rules, to so many different clowns (V
explicitly, in rules 1, 2, and 6, and R and T mostly implicitly, through being related to V). R,
T, and V should jump out at you as the clowns to consider; you count up the number of
clowns that must follow them, and you are in business. It is a systematic, rule-governed
process. You can memorize when to do it, you can memorize how to do it, and it is easily
practiced and mastered. Just the way I like it.
Notice, though, that if you keep your eyes open on "What can be true?", "What can't be
true?", and "What must be true?" questions (but not on If-questions), you can make many
of these deductions in the process.
What Do I Do Next?
Review your notes on this game. If you want to learn to make deductions, think about how
they are made. Relax for a while. Then do this game without any notes; if the deductions
give you a hard time, do it without them. See what skills you are still having trouble with.
Once you feel that you have learned what this game has to offer, go on to Game Ten.
Answer Key
1. E
2. D
3. C
4. D
5. D
6. E
7. E
Game Ten 219
220 Game Ten
Game omitted from electronic version of book.
Game Ten 221
Game Ten
(From LSAT 38)
Try this game on your own and then read my explanations.
Step 1: Read the Setup/Draw a Diagram
From what we are initially told, we are trying to figure out which of the six tasks is done by
each volunteer. We know that each volunteer will only demonstrate two tasks, but we know
next to nothing about the tasks. Thus, our diagram should be based around the volunteers,
because we know more about them. We write down the volunteers' names, and we put two
blanks under each, since each will demonstrate two tasks.
F G L tasks: H M P S T W
__ __ __
__ __ __
Step 2: Read the Rules
The first rule talks about the order in which the volunteers will demonstrate the tasks. But
up until now the game hasn't talked about putting the volunteers or the tasks in order. This
is something our diagram isn't built for. Quickly look through the other rules and see if
there are any more like this. There are: the next rule talks about the first and last tasks,
and the last rule talks about M being after T. This is a game about ordering the tasks, as
well as assigning volunteers to tasks. Our diagram is wrong.
For some of you (like me), when you first see the rules about order, you might think that by
"first task" the game means H, and by "last task" it means W. That is to say, maybe the
order of tasks refers to the order they were told to us in H is first, M is second, P is third,
etc. I thought this was possibly what they meant, but I wasn't sure. However, a quick look
at the questions tells us otherwise question 8 has the tasks all in a different order than
they are given to us. Also, the last rule has M coming after T, which is not the order in
which the tasks were initially given to us. This means that the tasks aren't already in the
right order we have to find the order.
Remember, if the setup confuses you, look at the rules to help figure the game out. If the
rules confuse you, glance at the questions.
So we have to go back and make a new diagram. Cross off the diagram we have. What are
we trying to figure out in this game? We want to put the tasks in order, and figure out who
does what task. So we have three types of things that we are trying to match up: order,
tasks, and volunteers. Which do we know more about? We know more about the order 1
is before 2, 2 is before 3, and so on. So we'll build our diagram around that. Since there
are six tasks, and none are done at the same time (it says "No two tasks will be
demonstrated concurrently.") we have to have first through sixth.
1 2 3 4 5 6
We want to figure out what task goes with each number. But we also want to figure out
what volunteer does each task. So we need two spaces under each number, one for tasks
and one for volunteers. We also need to remember that each volunteer goes twice; it's
easiest to write this as a rule.
222 Game Ten
1 2 3 4 5 6
tasks: __ __ __ __ __ __ H M P S T W
vol.: __ __ __ __ __ __ F G L
F, G, L used twice
Another handy way to write down that the volunteers will be used twice is to write two Fs,
two Gs, and two Ls, like this: F F G G L L. Then you can cross them off as they get used,
and you'll see that you have to use two of each.
Now, why didn't I just tell you this right out, instead of making us draw the wrong diagram?
There's a valuable lesson here. No one could have looked at that setup and known that
they had to put the tasks in order. Well, OK, not no one the fact that the setup told you
"No two tasks will be done concurrently" hints that we have to put the tasks in order. But
most people, most smart, good-at-the-LSAT people, would not have realized, just from
reading the setup, that they were supposed to put the tasks in order. And that's OK.
If you've learned what I've taught you, it doesn't take you very long to put together a
diagram. So realizing that your diagram is wrong isn't that big of a blow; just stay calm
and put together a new diagram. Not so hard, is it? No, it isn't. What we just experienced
is something that happens once in a while on the LSAT. It happens to me from time to
time, and it'll happen to you when you practice. You've got to accept that it is possible and
not worry about it. What I don't want you to do is worry about every setup, agonizing over
every possible scrap of meaning you might be able to get out of it. Take what they say
literally, and use it to put together a diagram. If your diagram isn't right, the rules will tell
you. Have confidence in your ability to understand what the LSAT is telling you, but don't
expect that you will be infallible. As long as you apply what you've learned from this book,
making mistakes won't be so bad, because they won't cost you a lot of time and you can
recover easily from them, just as we did here.
With that all said and done, let's go through the rules. The first one is a little odd, so let's
take it apart piece by piece.
You are told that F demonstrates "exactly one task" before G. What does "exactly one"
mean? It means one and only one. Frank will end up demonstrating two tasks. Exactly
one of these is before both Gs, the other isn't. It doesn't have to be after both Gs, but it
has to be after one G. So when we write this rule down, we want to make it clear that one
F is before both Gs, but not the other. So here we understand the rule by understanding
important LSAT words like exactly.
one F before both G (other F not before both Gs)
The next few rules are straightforward.
F not 1
F not 6
G not H
G not M
L not H
L not T
The last rule is also straightforward; it tells you that T is before M, and they are right next
to each other. Remember, even though the rule says "after," you rewrite it using "before."
Game Ten 223
There are several ways to write this rule. I like just writing TM, which, because I'm used to
doing it; I know means that T and M are next to each other, in that order. But you might
not be used to it, so there's nothing wrong with writing down a little more:
TM (next to each other, in this order)
Here is how our original diagram and rules turn out:
1 2 3 4 5 6
tasks: __ __ __ __ __ __ H M P S T W
vol.: __ __ __ __ __ __ F G L
1. F, G, L used twice
2. one F before both G (other F not before both Gs)
3. F not 1
4. F not 6
5. G not H
6. G not M
7. L not H
8. L not T
9. TM (next to each other, in this order)
The Deductions
There seem to be some deductions that we can make here. After all, we have the rule that
one F is before both Gs, which has a "before" in it, and we have the rule that T is before M.
So these are the types of deductions we know how to make.
We know that one F is before two Gs. So that F can't be 5
th
or 6
th
. But what about the
other F? Can it go 5
th
or 6
th
? Well, it can't go 6
th
because of rule 4. But can it go 5
th
? I
don't see any reason why not there are no other rules about the second F. So we can't
deduce that F won't go 5
th
, because that would mean that neither F could go there.
If this discussion confuses you, feel free to ignore it. As I did in the previous game, I will
give an analysis of how to look at each answer without having made these deductions in a
box after each answer. If we didn't use any deductions in evaluating a given answer, I
won't bother to have a separate, boxed explanation.
Sticking with the same rule, what about the Gs? Well, they both must go after F, so neither
of them can go 1
st
. We can write this in our diagram, or as a rule. I'm going to write it as a
rule, because I don't see us writing a lot of other deductions in the diagram. That means
that we won't have much reason to look at the diagram, and may forget to do so; if you
have only one or two deductions, writing them with the rules makes them easier to spot.
10. G not 1
We also have rule 9, which tell us that T is before M. So T can't go 6
th
and M can't go 1
st
.
11. T not 6
12. M not 1
And these are our deductions. Now, there is at least one other possible deduction, but I am
not going to tell you what it is yet. Why not? Two reasons: 1) The deduction either jumps
224 Game Ten
out at you or it doesn't; if it doesn't, I don't want you looking for deductions like it, because
that will slow you down; if it does jump out at you, you don't need me to tell you what it is.
2) We'll make the same deduction as we do the questions.
Step 3: Answer the Questions
Here's the order in which we should do the questions:
Question Number Question Type
8 What can be true?
10 If
11 If
13 What can be true?
9 What must be true?
12 What must be true?
Question 8
This is a "What can be true?" question, so we eliminate any answer that breaks a rule.
Since the answers spell everything out for us, we don't need to draw any test diagrams.
Answer A
This violates rule 3, "F not 1."
Answer B
This breaks rule 2, since both Fs are before both Gs the rule tells us that only one of the
Fs is before both Gs.
Answer C
F, G, and L are all used twice. Only one F is before both Gs. F is not 1 or 6. G doesn't do H
nor M. L doesn't demonstrate H or T. And T (threshing) is immediately before M. So this
follows all the rules. It is possible; copy it down so we will recognize it as a possible
diagram later on.
1 2 3 4 5 6
tasks: P S T M H W
vol.: L F G L F G
Question 10
Since this is a "What can be true?" question, we don't bother to use our previous diagrams.
We have to test the answers out. First draw a new diagram and put L fourth.
1 2 3 4 5 6
tasks: __ __ __ __ __ __ H M P S T W
vol.: __ __ __ L __ __ F G L
Game Ten 225
Now let's go through the rules. We have to use one more L. The rules about F don't seem
relevant, nor do the rules about G. Ahh, we know that position 4 can't be H or T (rule 8).
We can write that in.
1 2 3 4 5 6
tasks: __ __ __ __ __ __ H M P S T W
vol.: __ __ __ L __ __ F G L
no H
no T
Continuing through the rules, we see that T and M are next to each other, in that order (rule
9). Since T can't be 4, M can't be 5.
1 2 3 4 5 6
tasks: __ __ __ __ __ __ H M P S T W
vol.: __ __ __ L __ __ F G L
no H no M
no T
That is it for our rules. Let's look at the answers. This is a "What could be true?" question,
so we test each answer out and see if we can make it work. The question asks us where H
could be, so we put H in the place the answer tells us and try it out.
Answer A
Let's put H first.
1 2 3 4 5 6
tasks: H __ __ __ __ __ H M P S T W
vol.: __ __ __ L __ __ F G L
no H no M
no T
Now we go through the rules. Skim until we see a rule that is relevant; ahh, rule 6 talks
about H, as does rule 7. We know that H can't be with G or L. So G and L can't go 1
st
.
That leaves F to go first.
1 2 3 4 5 6
tasks: H __ __ __ __ __ H M P S T W
vol.: F __ __ L __ __ F G L
no H no M
no T
Go back through the rules. Rule 3 tells us that F can't go first. So this answer is wrong.
Cross off the diagram and eliminate the answer.
Now, before we go on, I want to point out a couple of things. Finding this answer was in no
way dependent on where L was, was it? So, no matter where L goes, can H go 1
st
? No, it
can't. This is a deduction. Also, no matter where L goes, who will go with H? G can't and L
can't, so F has to. We know that H and F go together, no matter what. So this answer
points us in the direction of two deductions. If you keep your eyes open for things that
have to be true as you go through the answers, you will pick things like this up.
226 Game Ten
Answer B
Let's try this out. We just learned that H goes with F, so put F 2
nd
as well. One thing to be
aware of in a game like this: when you have two types of blanks (for two types of things),
make sure you write the right type of thing in the right blank. For example, don't put L in
the "tasks" row, or you might think you have run out of spots for tasks.
1 2 3 4 5 6
tasks: __ H __ __ __ __ H M P S T W
vol.: __ F __ L __ __ F G L
no H no M
no T
Let's go through the rules. One F has to go before both Gs, so let's see where we can put
the Gs. We can't put either of them 1
st
, because then there would be no room for F to be
before them. Can we put one 3
rd
? Put it in there, but mark it because we don't know that it
has to go there.
1 2 3 4 5 6
tasks: __ H __ __ __ __ H M P S T W
vol.: __ F G* L __ __ F G L
no H no M
no T
Now, where do the other F and G go? It seems like they can go in either order all the rule
tells us is that the second F can't go before both Gs. We can't put the other F 1
st
, because
then two Fs would be before both Gs, and also rule 3 says F not 1. So put the F in the
next available spot, and G after, but mark them because we decided to put them there. We
also put L in the last spot left open, spot 1.
1 2 3 4 5 6
tasks: __ H __ __ __ __ H M P S T W
vol.: L F G* L F* G F G L
no H no M
no T
We continue through the rules. G can't go with H or M (rules 5 and 6). Since H is already
in, just note that M can't be 3
rd
or 6
th
.
1 2 3 4 5 6
tasks: __ H __ __ __ __ H M P S T W
vol.: L F G* L F* G F G L
no M no H no M no M
no T
L can't go with H or T (rules 7 and 8); again, only write about T, since H is already assigned
a spot.
1 2 3 4 5 6
tasks: __ H __ __ __ __ H M P S T W
vol.: L F G* L F* G F G L
no T no M no H no M no M
no T
Game Ten 227
T has to go immediately before M (rule 9). Where can these two go? Try every space until
you find a possible pair. They can't go 1
st
and 2
nd
, because H is already 2
nd
. They can go
3
rd
and 4
th
, because T can go 3
rd
and M can go 4
th
. In fact, they can't go anyplace else
you can't put T in 4 (because it says "no T" under 4), and you can't put T in 5, because M
can't go 6
th
.
1 2 3 4 5 6
tasks: __ H T M __ __ H M P S T W
vol.: L F G* L F* G F G L
no T no M no H no M no M
no T
We've got P, S, and W left. But we don't have any rules about these, so just put them
wherever.
1 2 3 4 5 6
tasks: P H T M S W H M P S T W
vol.: L F G* L F* G F G L
no T no M no H no M no M
no T
Alright, this works, so this (answer B) is the answer. Getting this is just a matter of going
through the rules one by one and writing everything down.
Question 11
Check your previous diagrams before you do anything else, since this is an If and a "What
must be true?" question. Do you have any diagrams that fit what the If tells you (G
demonstrates P, P is immediately before T, and F demonstrates T)? No, we don't, so we've
got to test this out.
They've given us a little block of people. G is before F, G demonstrates P, and F
demonstrates T, so we write this down. Here is the way I like to write it just the way it
will end up looking in our diagram, with volunteers on bottom and tasks on top:
P T
G F
Let's go through our rules. We know that one of the Fs has to be before both Gs (rule 2),
so the F we have here is not the first one, because it is after G. There must be an F before
this. We also know that F has to demonstrate H (we deduced that in question 10); the
second F is demonstrating T, so the first F must be the one that demonstrates H. Let's add
this on to what we have written down.
H P T
F before G F
Then rule 9 tells us that M has to follow T. So put the M in here:
H P T M
F before G F
228 Game Ten
Finally, one last look through the rules tells us that G can't learn M, so we can't have a G
immediately following the F. That might mean that the G comes before the F (and before
the other G) or after the F, but just not next to it.
H P T M
F before G F
no G
OK, that's a lot to have figured out. Let's work through the answers. This is a "What must
be true?" question, so we'll test the answers by doing something other than what they
describe.
Answer A
We don't want to put F 2
nd
. Let's try putting it someplace else. It can't go 1
st
(rule 3), so
try it 3
rd
(and star it, so we can move it if it doesn't work).
1 2 3 4 5 6
tasks: __ __ H __ __ __ H M P S T W
vol.: __ __ F* __ __ __ F G L
We have to have P, T, and M following the H/F pair. Since there are only three spots left,
they have to go in these spots. Put everything we have in that block in along with them
(including G, F, and the "no G"):
1 2 3 4 5 6
tasks: __ __ H P T M H M P S T W
vol.: __ __ F* G F __ F G L
no G
Now let's go through the rules. Rule 2 says that one F has to go before both Gs. We have
both Fs in, so both Gs have to go after the first F. But the only space open says "no G."
This won't work. The only thing we have starred is F, so that is the only thing we can
change about this diagram. We are always consistent about the way we move our marked
people; we always move them from left to right. We always put people in the earliest spot
(the leftmost spot) we can, so there is no reason to move them to the left they are
already as far to the left as possible. But F can't go any further to the right there already
is too little room to fit everyone in that has to go after F. So there is nothing we can do to
make this diagram work. Since this is a "What must be true?" question, what does this tell
us? It tells us that this (answer A) is the answer.
Question 13
This is a "What can be true?" question, so we test each answer out and see if we can make
it work.
Answer A
We know that H has to go with F; but rule 3 tells us that F can't go 1
st
. So H can't go 1
st
.
This is a wrong answer.
If we hadn't deduced that H and F go together, we could still do this answer. We put H first.
Rule 5 tells us that G can't go with H, so G can't go first. Rule 7 tell us that L can't go with
H, so L can't go first. That leaves F. But F can't go first.
Game Ten 229
Answer B
Let's try putting M 2
nd
.
1 2 3 4 5 6
tasks: __ M __ __ __ __ H M P S T W
vol.: __ __ __ __ __ __ F G L
Go through the rules. G can't be with M (rule 6).
1 2 3 4 5 6
tasks: __ M __ __ __ __ H M P S T W
vol.: __ __ __ __ __ __ F G L
no G
Keep going through the rules. T is right before M; that puts T first.
1 2 3 4 5 6
tasks: T M __ __ __ __ H M P S T W
vol.: __ __ __ __ __ __ F G L
no G
Since we've added to the diagram, we go back through the rules again. L can't go with T
(rule 8).
1 2 3 4 5 6
tasks: T M __ __ __ __ H M P S T W
vol.: __ __ __ __ __ __ F G L
no L no G
Rule 10 (one of our initial deductions) tells us that G can't go 1. Which leaves F for first.
But F can't go first (rule 3). So this won't work. Cross off the diagram and eliminate the
answer.
What if we hadn't initially deduced that G couldn't go 1
st
? We know that L can't go with T,
so L doesn't go first, and F can't go 1
st
. That leaves G to go 1
st
. But rule 2 tells us that one
F has to go before both Gs. If G is 1
st
, then there is no room for F to go before it. This tells
us that this is the wrong answer, and also makes us realize that G can't go first.
Answer C
We just saw that T can't be 1
st
, so eliminate this.
Answer D
This contradicts one of our initial deductions, that T can't go last.
If you hadn't made that deduction, you'd put T last and then you'd go through the rules.
When you got to rule 9 (T immediately before M), you'd see that there was no place to put
M, so this answer must be wrong.
Answer E must be the answer.
230 Game Ten
Question 9
By this stage in the game this is an easy question. We know answer A is right just by
looking at it, since we've already figured out that F and H have to go together.
If you hadn't realized that, here's how you'd do the question. It's a "What must be true?"
question, so if the answer describes a situation that isn't in every previous diagram, it is
wrong. Answer B gets eliminated by previous diagrams: the diagram for 8C doesn't have F
with M, nor does the diagram for 10A. C also gets eliminated by previous diagrams: 8C
doesn't have F with T, nor does 10A. The same thing happens with answer D. But answer
E is different; in each of our previous diagrams, one of the Gs demonstrates W.
Now, if you understand how equivalence works (see Advanced Techniques), you will
have realized that S and W are equivalent. So you can switch the S and the W in the
diagram for 8C (or 10A), and you will have G demonstrating T and S, instead of T and W.
Thus, G doesn't have to demonstrate W, and A is the right answer.
If you don't understand equivalence, you'd have to test out either A or E. You'd test out A
because it's first, and you'd soon realize that F has to demonstrate H; if it didn't then G or L
would have to, but they can't (from rules 5 and 7). So A is the right answer.
Question 12
Use previous diagrams before you do anything else. This is a "What must be true?"
question, so if we can find a previous diagram that has a situation different from what the
answer describes, the answer is wrong.
Each of our two previous diagrams has F in position 2, so we skip answer A. Neither of our
previous diagrams has G 4
th
, so we eliminate B. Both previous diagrams have G 6
th
, so we
skip C. Both have L 1
st
, so we skip D. Neither has L 2
nd
, so we eliminate E. We have to
test out A, C, and D.
Answer A
We don't want to put F 2
nd
. We can't put him 1
st
(rule 3), so put him 3
rd
, and mark him with
a dot or a star so we know that we had a choice.
1 2 3 4 5 6
tasks: __ __ __ __ __ __ H M P S T W
vol.: __ __ F* __ __ __ F G L
Now go through the rules. We know that F has to be before both Gs, but we can't have
both Fs before both Gs. We might have the second F between the Gs, or after the Gs. Let's
put him between the Gs, star him, and move him if we have to (as it turns out, we couldn't
have the F after the two Gs, because that would break rule 4, that F is not 6
th
; you might
not have thought of this, but if you actually tried to put F 6
th
, you would have realized that
this doesn't work when you looked back at the rules).
1 2 3 4 5 6
tasks: __ __ __ __ __ __ H M P S T W
vol.: __ __ F* G F* G F G L
Game Ten 231
This leaves the two Ls to go 1
st
and 2
nd
.
1 2 3 4 5 6
tasks: __ __ __ __ __ __ H M P S T W
vol.: L L F* G F* G F G L
Go through the rules. We know that G and L can't go with H, which reminds us that one of
the Fs has to demonstrate H. Give it to the first F, but star it since it didn't have to go
there.
1 2 3 4 5 6
tasks: __ __ H* __ __ __ H M P S T W
vol.: L L F* G F* G F G L
The next rule says G can't go with M, and later we see that M has to go after T. Where do T
and M go? They can't go 1
st
and 2
nd
, because L can't go with T; it looks like they can go 4
th
and 5
th
(and it turns out that they can't go 5
th
and 6
th
, because G can't go with M).
1 2 3 4 5 6
tasks: __ __ H* T M __ H M P S T W
vol.: L L F* G F* G F G L
We have P, S, and W left, and there are no rules about them, so put them wherever you'd
like.
1 2 3 4 5 6
tasks: P S H* T M W H M P S T W
vol.: L L F* G F* G F G L
This works, so we eliminate this answer. Unfortunately, this diagram doesn't help us
eliminate any other answers.
Answer C
G is 6
th
in the diagram we just made, so we can't eliminate this answer based on the work
we just did (and L is first, so we can't eliminate that one either). Or can we? We have the
star next to the F in position 5. That means we can move F over, making G not 6
th
! Except
when we check the rules, we see that F can't go 6
th
. So let's work this out the old fashioned
way.
We want G not to be 6
th
. Put a "not G" under 6 and let's try putting G someplace and see
where we can fit it. We start putting G as early as possible, and marking it with a star. If it
doesn't work, we move it one over and try again. We know that G can't be 1
st
(one of our
initial deductions, and something we also deduced from question 13, answer B). So we put
G 2
nd
.
1 2 3 4 5 6
tasks: __ __ __ __ __ __ H M P S T W
vol.: __ G* __ __ __ __ F G L
no G
232 Game Ten
Go through the rules. Rule 2 tells us that one F has to be before this G. That F will have to
go 1
st
. But the next rule tells us that F can't go 1
st
, so G can't be 2
nd
. So we look at what
we can change. G is starred, so move it over one.
1 2 3 4 5 6
tasks: __ __ __ __ __ __ H M P S T W
vol.: __ __ G* __ __ __ F G L
no G
Now we put one of the Fs in front of this G (but not in position 1).
1 2 3 4 5 6
tasks: __ __ __ __ __ __ H M P S T W
vol.: __ F G* __ __ __ F G L
no G
We know that we have to have an F and a G after this G. We don't want to G to go last, so
let's just put it in position 4 (that's the first available spot), and I'll put F in position 5 (the
next available spot). I'll star both of these, since they didn't have to go where I put them.
This also leaves the two Ls in 1 and 6 (don't star them, because they have to go there).
1 2 3 4 5 6
tasks: __ __ __ __ __ __ H M P S T W
vol.: L F G* G* F* L F G L
no G
Phew. Now we assign tasks. I'm going to put H with F, because I know that they go
together; I star it because H could have gone with the other F. I hope I don't have to go
back and rework this diagram, because we've got a lot of things we could change (if we did,
though, we'd probably have an idea of where the diagram went wrong, and we could
change the thing most relevant to that).
1 2 3 4 5 6
tasks: __ H* __ __ __ __ H M P S T W
vol.: L F G* G* F* L F G L
no G
Now go through the rules again. The Gs won't go with M (or H, but H is already put down),
and we've got to put in TM. Put TM in 4 and 5 or 5 and 6.
1 2 3 4 5 6
tasks: __ H* __ T M __ H M P S T W
vol.: L F G* G* F* L F G L
no G
Game Ten 233
This leaves P, S, and W, who can go anywhere.
1 2 3 4 5 6
tasks: P H* S T M W H M P S T W
vol.: L F G* G* F* L F G L
no G
This works, so the answer is wrong. This also shows us that L doesn't have to be 2
nd
, so E is
wrong. D is the answer.
Summary
This is a tough game, but if you keep a cool head, you can do it. The setup doesn't give
you enough information to draw a diagram, but the rules and questions do. The questions
really rely on your ability to test things out, but they don't require any amazing insights,
just a willingness to be methodical.
If you keep having to remind yourself what to do next in a game like this, you'll never finish
it. But, if you plug in what the answers say, look at the rules, put people in the first open
spots, and move marked people from left to right when diagrams don't work, all without
much hesitation, you'll do great.
What Do I Do Next?
Review your notes on this game. Relax for a while. Then do this game without any notes.
See what skills you are still having trouble with. Go back to the chapters that discuss those
skills and review them.
Once you feel that you have learned what you need to from this game, go on to Game
Eleven.
Answer Key
8. C
9. A
10. B
11. A
12. D
13. E
234 Game Eleven
Game omitted from electronic version of book.
Game Eleven 235
Game Eleven
(From LSAT 38)
Given what you already know, You should be able to handle this game except for question
16. Go ahead and work through the game on your own, and then take a look at what I
have here.
Step 1: Read the Setup/Draw a Diagram
Read the setup, spot what types of things are involved in this game, and figure out which
you know more about. Ready go!
That was fast. OK, we have two types of things in this game: applicants and positions. We
know more about the positions we know the number of applicants that get matched with
each position. So we will build our diagram around the positions. We write them down
horizontally, and put as many blanks under them as there are applicants that will be
matched to them management gets one; production, three; and sales, three.
M P S applicants: F G H I W X Y
__ __ __
__ __
__ __
I like a nice, straightforward diagram like this.
Alternately, we could use this diagram:
F G H I W X Y m
__ __ __ __ __ __ __ p p p
s s s
We assign each applicant a position and cross the positions off as they are used. Since
there is more than one p and s position, we have more than one of these to cross off. This
diagram is a little clunkier than the one we'll use, but it would work.
Step 2: Read the Rules
Here is how I wrote the rules; nothing fancy here. Did you do the contrapositive of the "If
then" rule about X?
H with Y
F not with G
If X in S, then W in P
If W not in P, X not in S
F in P
236 Game Eleven
I'm also going to put F in a blank under P, so I can see that he goes there. If it's possible to
put someone directly into a diagram, it's best to do so. This leaves only two open spots
under P. Here is how my rules and original diagram end up looking:
M P S applicants: F G H I W X Y
__ F __
__ __
__ __
1. H with Y
2. F not with G
3. If X in S, then W in P
4. If W not in P, X not in S
5. F in P
The Deductions
I don't have any rules that say who goes before whom, so I'm not going to make any of
that type of deduction here. I also don't have any people in multiple rules (well, F is in
rules 2 and 5, so we know that G can't be in P, but that isn't a very exciting deduction), so
the other type of deduction is out as well.
Step 3: Answer the Questions
Here is the proper order in which to do the questions:
Question Number Question Type
14 What can be true?
18 If
19 If
17 What can't be true?
15 Which is a complete and accurate list?
16 How many diagrams are possible?
Question 14
The fact that the question says "Which one could be a complete and accurate list" makes
this a "What can be true?" question.
Answer A
This follows rules 1 (H with Y) and 2 (F not with G), but breaks rule 3. X is in S, but W is
not in P.
Answer B
This has F in sales, not production, violating rule 5. This should have jumped right out at
you, since F is under P in our original diagram. This doesn't break any other rules, though.
Answer C
This breaks rule 1 H is not with Y.
Answer D
This breaks rule 1 also, because H and Y aren't together.
Game Eleven 237
Answer E must be the answer. Copy it down so you can use it as a previous diagram.
M P S
X F G
I H
W Y
Question 18
Since this is a "What can't be true?" question (even though it is also an If-question), the
first thing we do is look to see if we have a relevant previous diagram. But our single
previous diagram doesn't have F with X, as the "if" part of the question stipulates, so we
have to work this out.
Draw a new diagram and add what you are told. The question tells you that F is with X;
since we know F is in P, X must be in P as well.
M P S applicants: F G H I W X Y
__ F __
X __
__ __
Let's go through our rules. Rule 1 says that H and Y are together. Well, there is only one
place they could go S (because there is not enough room for both of them in either P or
M). So let's put them there. (If you hadn't realized that, you could still get this question
right; I'll explain how in the boxed text, at the end of the question.)
M P S applicants: F G H I W X Y
__ F H
X Y
__ __
Rule 2 says that F is not with G. So we can write "no G" under P.
M P S applicants: F G H I W X Y
__ F H
X Y
__ __
no G
Rule 3 says "If X in S," but S isn't in X, so we ignore this rule (see Game Three for more on
when to ignore "If then" rules). The same applies to 4 we don't know whether or not
W is in P, so we ignore this one. Our last rule tells us that F is in P, but we already knew
that. Since we added more stuff to the diagram, we go back through the rules, but nothing
new comes up. We're done looking at the rules. Let's check out the answers. The question
is an EXCEPT question (because it has the word "EXCEPT" in it). These questions describe
the wrong answers to us; here, we know the wrong answers will be things that can be true.
This means that the right answer can't be true this is a "What can't be true?" question
(see The Fundamentals for more on different ways questions can be worded).
238 Game Eleven
Answer A
Recopy our diagram and put G under S.
M P S applicants: F G H I W X Y
__ F H
X Y
__ G
no G
This leaves I and W to go in our last two spots. Look through the rules. Rule 5 says "If W
not in P, X is not in S." Well, if we put W in M, it won't be in P. That means that X can't be
in S. X isn't in S, so that's no problem. If we put W under P, rule 4 wouldn't apply (since it
tells you what happens if W isn't under P). And there are no rules about where I goes. So
either one of them can go either place. Let's just put them in alphabetical order (I going in
before W).
M P S applicants: F G H I W X Y
I F H
X Y
W G
no G
This follows the rules, so it is the wrong answer. It also helps you eliminate answer E, since
we see that W can be under P. If you are really canny, you'll see that we can eliminate D as
well we could have put W under M if we wanted to.
Answer B
We already figured out that H had to be under S before we tested the answers, so this can't
be true. This is the right answer.
If we hadn't seen earlier that H and Y had to go under S, we'd have to work out answer B.
You'd put H under P, and then you'd look at the rules. Rule 1 would tell you that H and Y
had to go together, and you'd see that there was no room for Y, so you'd know that this
couldn't work.
Question 19
Although this is an If-question, so it seems like we potentially can use previous diagrams,
it's also a "What can be true?" question; thus, we really can't.
The question doesn't tell us where X goes, but it tells us where X won't go, so we can write
that in our new diagram.
M P S applicants: F G H I W X Y
__ F __
__ __
__ __
no X
Game Eleven 239
Let's go through the rules. Rule 1 says H is with Y, but we don't know where they will go (I
guess we know that they won't go under M, but that's it). F is not with G, so we can write
"no G" under F.
M P S applicants: F G H I W X Y
__ F __
__ __
__ __
no X
no G
We don't know if rule 3 applies, because we don't know if X is in S or not, and we don't
know if rule 4 applies because we don't know about W. So we can't really figure anything
out here. Let's look at the answers; this is a "What can be true?" question, so we will test
out answers until one works.
Answer A
We don't even have to try this out, because we know that F can't be in sales; if F can't be in
sales, we can't have both F and H in sales (since "both" means "the two of them").
Answer B
Let's put both H and W in sales.
M P S applicants: F G H I W X Y
__ F H
__ W
__ __
no X
no G
Now go through the rules. Rule 1 tells us that H and Y go together, so put Y with H.
M P S applicants: F G H I W X Y
__ F H
__ W
__ Y
no X
no G
Rule 2 says that F is not with G. Well, if G isn't with F, where will it go? There's only one
place left that isn't with F, and that's M. So we put G there. It's easy to miss something
like this you just rush through the rules saying "Yeah, yeah, I already looked at that," and
boom, you miss something important. That's why you have to be systematic and really go
through each rule every time.
M P S applicants: F G H I W X Y
G F H
__ W
__ Y
no X
no G
240 Game Eleven
Now we only have two people left (I and X), and two spaces open (both under P). I and X
both have to go under P. But we have "no X" written under P. You may not remember why
you have it written down there, but there it is (you have it written because it's what the If
part of the question told you). So this won't work. Cross off the diagram and move on.
Answer C
Let's try putting both F and Y under P. Recopy the diagram for this question. F is already
under P, so put Y there as well. By this point I remember that H and Y go together without
having to look through the rules, so let's put H under P as well.
M P S applicants: F G H I W X Y
__ F __
H __
Y __
no X
no G
So far so good we haven't put X or G under P.
Look through the rules. We've already followed rule 1. Rule 2 tells us that G will be under
M or S, but we don't know where. Rule 3 says that if X is in S, then W is in P, but we don't
know if X is in S. Rule 4 says that if W is not in P, X is not in S. Well, can we have W in P?
No, because P is all filled up. So rule 4 applies here. X is not in S. If X is not in S, where is
it? It has to be in M, which leaves everyone else in S.
M P S applicants: F G H I W X Y
X F G
H I
Y W
no X
no G
Does this follow all the rules? Yes it does. So this (answer C) is the answer.
Question 17
Each of the answers is supposed to be two of the three people (hence "partial list") under S.
This is an EXCEPT question, so it describes the wrong answers to us. Since it says "could
be," we know that the right answer can't be an accurate partial list. We can use previous
diagrams and eliminate any answers that we know are possible.
We eliminate answer A based on the diagram for 19B (the one we just did), since that has G
and I in sales. We don't see G and X together in any of our previous diagrams, so skip that
one. We see G and Y together in 14E and 18A (although you should just see one of these
diagrams and stop looking), so we can eliminate C. We don't see H and W together in
sales, so skip answer D; nor do we see H and X. Remember, this is a "What can't be true?"
question. So we have to test out at least one of B, D, and E.
Game Eleven 241
Answer B
Let's try putting G and X together in sales.
M P S applicants: F G H I W X Y
__ F G
__ X
__ __
Let's go through the rules. Rule 1 says that H has to be with Y. Where will these two go?
Well, there's only one place with room enough for both of them P. So we put them there.
M P S applicants: F G H I W X Y
__ F G
H X
Y __
Rule 2 says that F and G aren't together, which is the case here, so that's fine so far. Rule
3 says that if X is in S, W has to be in P. Well, X is in S, so this is relevant. But W can't be
in P. Why not? Not enough room. So this won't work. Cross off the diagram and circle
this answer (answer B) (remember, we're looking for something that can't be true).
Question 15
This is a "Which is a complete and accurate list?" question. We want an answer that is a
complete and accurate list of people who cannot be in production. That means that if the
list contains people who can be in production, it is wrong. If the list leaves out someone
who can't be in production, it is wrong. We'll use our previous diagrams. If someone in the
answer is in a previous diagram under P, the answer is wrong.
Answer A
Well, we know that F has to be in P, so this is immediately wrong. We can also eliminate
any other answer with F in it (but none of the others has F, so that isn't too helpful).
Answer B
We know that G can't be in P (because it can't be with F). H and Y are in P in the diagram
for 19B, so this is wrong. We can also eliminate answer C because it has H and Y.
Answer D
Is G the only person who can't ever be in P? This looks good, since we know that G can't be
in P, but we don't know if the answer has left anyone else out. Look at E.
Answer E
Well, this answer leaves G out, so this has to be wrong. Answer D is the answer.
Question 16
We haven't done a question like this before, which isn't surprising, because they are pretty
rare. The question asks us under what conditions we could know exactly where everyone
goes. That means that the wrong answers will give us a situation where we don't know
exactly where everyone goes someone will have more than one possible place to be. So
we test out each answer and see if it forces everyone into certain positions.
We can use our previous diagrams to a certain extent if we see two previous diagrams
that fit the description in the answer, but have some other differences, we know that the
242 Game Eleven
answer doesn't completely determine what happens. But we don't often have two diagrams
that are both relevant to an answer, so we aren't going to count on this.
Answer A
We have two previous diagrams with F and W under P 14E and 18A. These diagrams are
not exactly the same: X and I are in different places. So having F and W both under P
doesn't completely determine what happens in the game. We can eliminate this answer.
Answer B
14E and 18A help us again both have G and Y in sales, but they aren't exactly the same in
every other way. We can eliminate this answer.
Answer C
We only have one diagram where I and W are under S (19B). Thus, we can't compare two
diagrams and see if they come out the same. Skip this answer.
None of our people or job categories are equivalent in this game they are all subject to
different rules (with the exception of I, who is subject to no rules). So we can't really
rearrange any of our previous diagrams (see Advanced Techniques).
Answer D
We only have one diagram (14E) where I and W are under P, so skip this answer too.
Answer E
We don't have any diagrams where I and X are under P.
So we have to test out at least one of C, D and E. Back to answer C. We test it by putting I
and W in sales and seeing if anything is left up to us, or if everything else is forced to
happen in a certain way.
M P S applicants: F G H I W X Y
__ F I
__ W
__ __
Rule 1 tells us that H and Y have to be together. So we have to put them under P (since
there is no place else that we could fit both of them). So far nothing is up to us.
M P S applicants: F G H I W X Y
__ F I
H W
Y __
Rule 2 tells us that G can't be with F, but since there is no room with F under P anyway, this
is no problem. We ignore rule 3, because we don't know if X is in S or not. Rule 4 says "If
W not in P," and W is indeed not in P (it's in S). So we know that X is not in S. There is no
place to put X except M. So we put him there; we have no choice, he has to go there.
M P S applicants: F G H I W X Y
X F I
H W
Y __
Game Eleven 243
This leaves G, who has to go under S.
M P S applicants: F G H I W X Y
X F I
H W
Y G
Everything that we put in was totally determined by the rules. So C is the answer.
Let's look at answer D. I want to show you how to eliminate an answer on one of these
questions without using previous diagrams. So let's draw a test diagram and, like D says,
put I and W under P.
M P S applicants: F G H I W X Y
__ F __
I __
W __
Now we go through the rules. H and Y have to be together, and they can't fit under M or P,
so we put them under S. So far we have no options.
M P S applicants: F G H I W X Y
__ F H_
I Y_
W __
Rule 3 says "If X in S," but we don't know if X is in S or not. Rule 4 says "If W not in P," but
this is not the case; W is in P. We've now run out of rules; it seems we can put G and X in
either spot. Just to be sure, put them in one way, and then the other.
M P S applicants: F G H I W X Y
G F H_
I Y_
W X_
M P S applicants: F G H I W X Y
X F H_
I Y_
W G_
Both of these follow all the rules; the only rule they might break is rule 4. In the first case,
rule 4 is relevant because X is in S. But rule 4 is followed, because W is also in P. In the
second case, rule 4 is irrelevant because X is not in S. Both of these diagrams work, so the
answer doesn't completely determine where everyone goes; there is more than one option.
Summary
Now we've seen a "How many diagrams are possible?" question, and you can probably
understand why we save them for the end. Nothing else in this game is new to us, which is
just how it's going to be on the LSAT everything you'll see on the test will be something
you already know how to handle. Doing well on the test is just a matter of showing up and
doing exactly what you've practiced so many times before; no need to be tricky, or think of
new and creative ways to approach questions.
244 Game Eleven
What Do I Do Next?
Review your notes on this game. Relax for a while. Then do this game without any notes.
See what skills you are still having trouble with. Go back to the chapters that discuss those
skills and review them. Once you feel that you are comfortable with this game and the
skills it tests, go on to Game Twelve.
Answer Key
14. E
15. D
16. C
17. B
18. B
19. C
Game Twelve 245
246 Game Twelve
Game omitted from electronic version of book.
Game Twelve 247
Game Twelve
(From LSAT 38)
Try this game out on your own. If you get confused, come back and read step 1, step 2,
and the explanation of the first question (question 20). Then work through the rest of the
questions on your own.
Step 1: Read the Setup/Draw a Diagram
What are the types of things we are matching together in this game? We have the pieces
(N, O, etc.), we have their order (first, second, etc.), and we have the instruments (fiddle,
lute, etc.). How do I know that we are concerned with order? The setup says that the
pieces are performed "successively (though not necessarily in that order)." "Successively"
means "in a row," and "not necessarily in that order" mentions order. If that wasn't
enough, you can glance at the rules and they talk about the order of the pieces (which are
next to which, and which is performed second).
The instruments are already matched with the pieces. F has to go with fiddle and lute, O
has to be with harp and mandolin, etc. How do we know this? The setup tells us what
pieces are performed with what instruments; that means that those pieces are performed
with those instruments. We don't need to worry about assigning pieces to instruments,
then. All we need to think about is the order the pieces are performed in.
So, which of the types of things do we know the most about? This is a toss-up. On the one
hand, I'm inclined to say that we know the most about the order first is next to second,
which is next to third, and so forth. On the other hand, we know quite a bit about the
pieces we know what instruments are assigned to which pieces. I have to admit that I
prefer to base the diagram around the order, because I've done that sort of thing before
and I'm comfortable with it. I know how a diagram like this, with 1, 2, 3, 4, and 5 in a row,
works. But, you could have based your diagram around the pieces. So I'm going to do this
game two different ways, in parallel. I will do the game with the first kind of diagram
based around the numbers 1, 2, 3, 4, and 5 and accompany the analysis of each step with
notes (in a box) on how to do it with the other sort of diagram.
So here is our diagram. I put three blanks under each number because I want to put a
piece and two instruments in with every number.
1 2 3 4 5
__ __ __ __ __
__ __ __ __ __
__ __ __ __ __
I also want to write down the names of the pieces, and which instruments go with which
pieces.
1 2 3 4 5 pieces: N O S T V
__ __ __ __ __ f h g f L
__ __ __ __ __ L m h g m
__ __ __ __ __
I wrote the names of the instruments in lowercase for your sake, so you could easily tell one
from the other (except I put L in uppercase because it looks too much like I in lower case).
248 Game Twelve
If we base our diagram around the pieces, then we write them down in order, and put a
blank under each where we'll put what number goes with the piece:
N O S T V
f h g f L
L m h g m
__ __ __ __ __
Step 2: Read the Rules
There are only two rules. The first one is a little complex. It says that each piece has to
have an instrument in common with one of the pieces before or after it. What does this
mean? In understanding odd rules, it's easiest to think of a specific example and see how
the rule would apply to it. Imagine N was 1
st
, for example. We know that N has to share
an instrument with the piece before it or after it. If N is 1
st
, there's no piece before it, so it
has to share an instrument with the piece after it, the 2
nd
piece. What instrument will this
be? Well, N has f and L, so it has to share f or L with the next piece. What piece could that
be? It could be T, which would share f with it, or V, which would share L with it.
Let's keep thinking. What if N was 2
nd
? Well, then it would have to share an instrument
with the piece before it, which would be the 1
st
piece, or the piece after it, which would be
the 3
rd
piece. So T or V would have to be 1
st
or 3
rd
. We could also have both T 1
st
and V
3
rd
, since a piece can share with both the piece before and the piece after it.
Does the rule make more sense now? Good. That took us a little while; was that wasted
time? No. You cannot answer the questions unless you understand the rules. Take as
much time as you need to make sure you know what the rules mean and how to apply
them. Here we can afford it because there are only two rules anyway.
Let's write down the first rule; I can't think of any fancy way to say it, so I'll just write it
plainly:
Each piece shares instrument with before OR after (or both)
The second rule is much simpler:
N or T is 2
Here is our original diagram and rules:
1 2 3 4 5 pieces: N O S T V
__ __ __ __ __ f h g f L
__ __ __ __ __ L m h g m
__ __ __ __ __
1. Each piece shares instrument with before OR after (or both)
2. N or T is 2
You go through the same thought process to figure out the rules if you diagram this game
the alternate way. You end up with very similar-looking rules. Here is the alternate original
diagram and rules:
Game Twelve 249
diagram and rules:
N O S T V
f h g f L
L m h g m
__ __ __ __ __
1. Each piece shares instrument with before OR after (or both)
2. N or T is 2
The Deductions
There are no deductions to make in this game we only have two rules to combine, neither
has "before" in it, and neither has anything in common with the other, as far as we can see.
Step 3: Answer the Questions
Here is the order in which you should do the questions:
Question Number Question Type
20 What can be true?
22 If
24 If
21 What can't be true?
23 What can't be true?
Question 20
This is a "What can be true?" question, so work through the answers, one by one, and see
which one doesn't break any rules.
Answer A
This breaks rule 2 N or T isn't 2
nd
. I'll be honest with you I checked out rule 2 first,
because it was simpler. I normally wouldn't do this, but I only had two rules, so I knew that
I wouldn't get confused looking at them out of order.
Answer B
This follows rule 2, so we have to see if it follows rule 1. We have to see who shares with
whom. S is performed with g and h, so it has to share one of these with the next piece, T.
T is performed with f and g, so S and T share. We don't need to worry about these two,
then, because they share with each other. Remember, a piece has to share with the one
before it or the one after, but doesn't have to share with both. So once we know that S and
T share, we don't need for T to share with O (although it is OK if they do).
Let's look at O. It has to share with either T or N. O is performed with h and m. T doesn't
have either, and N doesn't have either, so this rule is broken.
The answers to this question don't look like our alternate diagram. I took the answers and
put what they told me in our diagram. I put 1 under S, 2 under T, and so forth.
N O S T V
f h g f L
L m h g m
4 3 1 2 5_
250 Game Twelve
Then we go through the rules. Everyone has to share with someone numerically next to it.
We go from left to right just like we always do. That means that we start with N. It has to
share with either 3 or 5 (the pieces before and after it). It doesn't share with 3 (which is
O), so it has to share with 5. It does both have L. So now look at O, the next piece in
our diagram. It has to share with 2 or 4. It doesn't share with T, which is piece 2 neither
of them have any instruments in common. And we already know that it doesn't share with
N, which is piece 4. So this breaks the rules.
It is possible that at this point, if you had diagrammed the game in this way, you would be
feeling a little confused. That's OK, because of course there is an alternate way of
diagramming the game, which the answers give you just put the pieces in order, as each
answer does.
One nice thing about the way we approach games is that we don't spend a lot of time
putting together a diagram, which means that if we decide to change the way we diagram
the game, we have wasted barely any time at all. If you realized that this was a confusing
diagram for you, don't stick to it; switch to the other form that this question presents to
you. You end up having wasted only a few seconds.
Answer C
This follows rule 2, so we have to see if it follows rule 1. The first two pieces share f, so
look at the 3
rd
piece. O has to share with either N or V, the pieces next to it. It doesn't
share with N, but it shares m with V. So O and V are happy. This just leaves S. Since S is
last, it has to share with V. V and S don't share any instruments, so this breaks the rules.
We translate this into our alternate diagram:
N O S T V
f h g f L
L m h g m
2 3 5 1 4_
Let's go through the instruments one at a time, from left to right. N is 2
nd
, so it has to
share with the 1
st
or 3
rd
piece. It shares with the 1
st
. O is 3
rd
, so it has to share with the 2
nd
or 4
th
, and it shares with the 4
th
. S is 5
th
, so it has to share with the 4
th
, V. But the two
don't share anything, so this breaks a rule.
Answer D
This follows rule 2. Go through the pieces in order and see if they share appropriately.
T and N share f, so they are happy. Move on to S. S has to share with N or O. S and O
share h, so they are happy. Move on to V. V has to share with O, since there is no piece
after V. O and V share m. So everyone shares with one of the pieces before or after it; this
follows the two rules, so this is the answer. Write it down so we'll remember to use it as a
previous diagram.
1 2 3 4 5
T N S O V
f f g h L
g L h m m
Game Twelve 251
Translate the answer into our alternate diagram:
N O S T V
f h g f L
L m h g m
2 4 3 1 5_
N has to share with S (3
rd
) or T (1
st
). N and T share f, so they are happy. O has to share
with S (3
rd
) or V (5
th
). O shares with both of them. Lucky O. So that means O is happy.
We've already seen that T and N share, and V and O share, so everyone shares
appropriately. This is the answer.
Question 22
This is an If-question. At first glance it may seem like it is changing the rules now every
piece has to share with the piece after it. You may assume that this means they can't share
with the piece before it. You may also assume that the last piece won't share with anyone,
since it says "except the fifth." But these would be wrong assumptions.
When a question contradicts one of the rules, it will tell you that it is doing so. It will say
"Suppose the rule that blah blah is changed," or "If such and such a rule is removed" If
you think a question is contradicting a rule, but the question doesn't say that it is changing
or deleting one of the rules, you are wrong. In this case, the question adds a rule, but it
doesn't tell us that it is changing or getting rid of any of the existing rules. So pieces still
can share with the piece before them, they just have to share with the one after them. And
the last piece has to share instruments with the 4
th
piece, because that is the only piece
next to it. If the 4
th
and 5
th
didn't share, then the 5
th
would share with no one, and that
would break the first rule.
Write down this new rule:
Each shares with piece after
I have a gut feeling that there is some kind of deduction here, something that has to
happen. But it doesn't immediately hit me what it is, and I don't know how to make it. This
rule doesn't name any specific pieces; I have the feeling if I fooled around with the diagram
and pieces for long enough, I could figure something out. Should I do this? No. The
questions will give me concrete things to test out, which is much easier than just trying
random things and hoping I learn something. If it isn't clear how to apply an "if" situation,
work through the answers.
I'm also not going to check previous diagrams, both because this is a "What can be true?"
question and also because it would be too hard to figure out if their diagrams matched the
"if" part here.
This is a "What can be true?" question, so we'll test the situation each answer describes.
252 Game Twelve
Answer A
Draw a new diagram and put V 1
st
. Put V's instruments under it.
1 2 3 4 5 pieces: N O S T V
V __ __ __ __ f h g f L
L __ __ __ __ L m h g m
m __ __ __ __
V has to share with the piece after it, so whatever goes 2
nd
has to have L or m. The rules
tell me, though, that N or T will go 2
nd
. N has L, so it could share with V. T doesn't have L
or m, so it couldn't share with V. T can't go 2
nd
, then, so N has to.
1 2 3 4 5 pieces: N O S T V
V N __ __ __ f h g f L
L f __ __ __ L m h g m
m L __ __ __
Again, N has to share with the piece after it. So whatever goes 3
rd
has to have either f or L.
O doesn't have either and S doesn't have either. That leaves only T (since we've already
used V). T does share with N; since it is the only one left that does, it has to go 3
rd
.
1 2 3 4 5 pieces: N O S T V
V N T __ __ f h g f L
L f f __ __ L m h g m
m L g __ __
T has to share with whichever piece goes 4
th
; since T has f and g, the 4
th
piece has to have
one of them. O doesn't have either, but S does. That means S goes 4
th
and O goes 5
th
.
1 2 3 4 5 pieces: N O S T V
V N T S O f h g f L
L f f g h L m h g m
m L g h m
We aren't out of the woods yet. O has to share with S. And it does, so this works. This
(answer A) is our answer.
Let's do this with our other diagram. The steps we go through are the same; we just draw
it differently. We put a 1 under V.
N O S T V
f h g f L
L m h g m
__ __ __ __ 1_
This means that piece 2 has to have L or m; since only N or T can be 2, we know that N has
to be 2.
N O S T V
f h g f L
L m h g m
2 __ __ __ 1_
Game Twelve 253
Likewise, piece 3 has to have f or L, since each piece must share with the one after. There
is only one piece unused that has either T so it has to go 3.
N O S T V
f h g f L
L m h g m
2 __ __ 3 1_
The next piece has to have either f or g. Only O and S remain; only S can share with T, so
S is 4 and O is 5.
N O S T V
f h g f L
L m h g m
2 5 4 3 1_
Question 24
Since this is a "What can be true?" If-question, we don't check our previous diagrams.
Draw a new diagram and put S 5
th
.
1 2 3 4 5 pieces: N O S T V
__ __ __ __ S f h g f L
__ __ __ __ g L m h g m
__ __ __ __ h
Now go through the rules. We know that each piece must share with the one before or
after it. S doesn't have any piece after it, so it must share with the one before it. What
piece could that be? Well, S has g and h, so the piece before it has to have g or h. O has h
and T has g, and that's it. These are the only pieces that could go 4
th
.
1 2 3 4 5 pieces: N O S T V
__ __ __ __ S f h g f L
__ __ __ __ g L m h g m
__ __ __ __ h
O or T
We might be able to go further with this if T is 4
th
, then N has to go 2
nd
(rule 2). What
happens next? I don't know, and I don't want to work it out. When you set up your initial
diagram for an If-question, you want to figure out what has to happen, not what can
happen, because the diagram has to apply to every answer. Let's save our work for the
answers. This is a "What can be true?" question, so we'll test each out.
254 Game Twelve
Answer A
Put N in position 3.
1 2 3 4 5 pieces: N O S T V
__ __ N __ S f h g f L
__ __ f __ g L m h g m
__ __ L __ h
O or T
Now go through the rules. I'm going to look at rule 2 because rule 1 is so much harder to
apply, and we only have two rules. N or T has to go 2
nd
. Since I've put N 3
rd
, it can't go
2
nd
. That means that T will go 2
nd
.
1 2 3 4 5 pieces: N O S T V
__ T N __ S f h g f L
__ f f __ g L m h g m
__ g L __ h
O or T
As I am writing in the T in position 2, I see that O or T has to go 4
th
. Well, T certainly can't
be 4
th
, so O must be in position 4, which leaves V 1
st
.
1 2 3 4 5 pieces: N O S T V
V T N O S f h g f L
L f f h g L m h g m
m g L m h
O or T
Go through the rules. Each person must share with either the one before or the one after it.
But V doesn't share with T; this breaks the rules, so this is the wrong answer. Cross off the
diagram and cross off the answer.
By now you should see that the process for doing these answers with the alternate diagram
is exactly the same as doing them with our original type of diagram; we just write things
down a little differently. So we'd put a 3 under N; this would force a 2 to go under T, which
would force us to put a 4 under O, and a 5 under V. This breaks the rules in exactly the
same way.
Answer B
Copy down the initial diagram for this question and put O 3
rd
.
1 2 3 4 5 pieces: N O S T V
__ __ O __ S f h g f L
__ __ h __ g L m h g m
__ __ m __ h
O or T
Game Twelve 255
Now I see that T has to go 4
th
, since either O or T is 4
th
.
1 2 3 4 5 pieces: N O S T V
__ __ O T S f h g f L
__ __ h f g L m h g m
__ __ m g h
O or T
Go through the rules. Rule 2 tells us that N or T is 2
nd
; since T is 4
th
, it can't go 2
nd
. This
leaves N to go 2
nd
. That leaves only V, which must go in the last available spot (1).
1 2 3 4 5 pieces: N O S T V
V N O T S f h g f L
L f h f g L m h g m
m L m g h
O or T
Now go through the rules. Everyone has to share with someone before or after them. Does
that happen here? V shares with L, so that's good. O, though, doesn't share with anyone.
Poor O. This breaks the rules, so cross off the diagram and the answer.
Again, we go through the same process in our alternate diagram; a 3 goes under O, which
forces a 4 to go under T, a 2 under N, and a 1 under V. This breaks a rule, so it is wrong.
Answer C
Try putting T 4
th
.
1 2 3 4 5 pieces: N O S T V
__ __ __ T S f h g f L
__ __ __ f g L m h g m
__ __ __ g h
O or T
As we've been seeing, this forces N to go 2
nd
.
1 2 3 4 5 pieces: N O S T V
__ N __ T S f h g f L
__ f __ f g L m h g m
__ L __ g h
O or T
But now we have O and V left, and I don't know where they have to go. Let's just try
putting them in alphabetical order; I'll star them, so that if it doesn't work, we can switch
them later.
1 2 3 4 5 pieces: N O S T V
O* N V* T S f h g f L
h f L f g L m h g m
m L m g h
O or T
256 Game Twelve
Does this follow the rules? O doesn't share with N, so this doesn't work. Switch O and V.
1 2 3 4 5 pieces: N O S T V
V* N O* T S f h g f L
L f h f g L m h g m
m L m g h
O or T
This doesn't work either O doesn't share with either N or T. Switching O and V again
won't do me any good, so I can't make this work. This answer is wrong; cross off both it
and the diagram.
I'll diagram this to show you how starring works in this diagram. We put 4 under T. This
forces 2 to go under N. We have O and S left, and we don't know which will go 1
st
and
which 3
rd
. So we just try it out one way.
N O S T V
f h g f L
L m h g m
2 1* 5 4 3*
This won't work, because O doesn't share with N. So we switch O and V.
N O S T V
f h g f L
L m h g m
2 3* 5 4 1*
This doesn't work either, because O doesn't share with either N or T.
Answer D
Put V 1
st
.
1 2 3 4 5 pieces: N O S T V
V __ __ __ S f h g f L
L __ __ __ g L m h g m
m __ __ __ h
O or T
Let's go through the rules. N or T has to go 2
nd
, and the 1
st
spot must share with the 2
nd
.
Can I put N 2
nd
? Sure; it and V share L. Can I put T 2
nd
? No, T and V share nothing. So N
has to go 2
nd
.
1 2 3 4 5 pieces: N O S T V
V N __ __ S f h g f L
L f __ __ g L m h g m
m L __ __ h
O or T
This leaves O and T. I'm not sure which has to go where, so I'll try them in alphabetical
order first (starring them because this was my choice; if it doesn't work out, I'll be able to
move them).
Game Twelve 257
1 2 3 4 5 pieces: N O S T V
V N O* T* S f h g f L
L f h f g L m h g m
m L m g h
O or T
This doesn't work O doesn't share with anyone next to it. So switch O and T.
1 2 3 4 5 pieces: N O S T V
V N T* O* S f h g f L
L f f h g L m h g m
m L g m h
O or T
This works everyone shares with someone next to them. This (answer D) is the answer.
We do the same thing with our alternate diagram. We put 1 under V. This forces 2 to go
under N, since V has to share with someone, and either N or T is 2
nd
. This leaves O and T,
and we don't know which goes where. We put them in one way, and star them. If that
doesn't work, we try the other way. One of the two ways will work.
Question 21
Since this is a "What can't be true?" question, let's use our previous diagrams. We look for
previous diagrams to see what 3 and 4 share. We eliminate any answer that we see in a
previous diagram because this is a "What can't be true?" question.
The diagram for question 20D has S and O as 3 and 4. They share h, so I know that h can
be shared by 3 and 4. Cross off answer D.
The diagram for question 22A has T and S as 3 and 4. They share g, so I can cross off
answer B.
The diagram for 24D has O and T as 3 and 4, but they don't share anything. Darn it. They
don't have to share, because 3 could share with 2, and 4 with 5. Oh well, we'll have to
eliminate the other answers with test diagrams.
Answer A
How could 3 and 4 share f? Look for pieces that both have f. Only N and T both have f.
We don't know which will be 3 and which will be 4, so we'll put them in alphabetical order,
and star them.
1 2 3 4 5 pieces: N O S T V
__ __ N* T* __ f h g f L
__ __ f f __ L m h g m
__ __ L g __
Uh oh, this breaks rule 2 either N or T has to be 2. But if they are 3 and 4, neither can be
2. There's no point in switching them; we can't make this work. This (answer A) is the
answer.
Let's use our alternate diagram. We want 3 and 4 to share f, so we have to put 3 and 4
under pieces that both have f. Only N and T have f. We'd put a 3 under N and a 4 under T,
and star both because we might have put the 4 under N and the 3 under T. This means
that neither N nor T has a 2 under it, which breaks the second rule. Switching them won't
make a difference.
258 Game Twelve
that neither N nor T has a 2 under it, which breaks the second rule. Switching them won't
make a difference.
Question 23
This is a "What can't be true?" question, so look at previous diagrams to see which piece
can be 1
st
. Our previous diagrams only have T and V in position 1, so we eliminate answers
D and E.
Answer A
Put N first and see if we can make this work.
1 2 3 4 5 pieces: N O S T V
N __ __ __ __ f h g f L
f __ __ __ __ L m h g m
L __ __ __ __
By this point we probably remember that N or T has to go 2
nd
, so putting N first forces T to
go 2
nd
. If we didn't remember this, a quick look at the rules would tell us the same thing.
1 2 3 4 5 pieces: N O S T V
N T __ __ __ f h g f L
f f __ __ __ L m h g m
L g __ __ __
So far so good N and T share instruments, so this follows all the rules so far. We have O,
S, and V left, but we don't know where they will go. Let's just put them in alphabetical
order; we'll mark them with a star because they didn't have to go this way. If this doesn't
work, we'll change one of them.
1 2 3 4 5 pieces: N O S T V
N T O* S* V* f h g f L
f f h g L L m h g m
L g m h m
Does this work? No, because V doesn't share with anyone. So let's change things. We
always change things systematically; in general, we move things from left to right. We put
O 3
rd
, so let's move it over one. This will put S in position 3 instead. (Another way of
accomplishing the same thing is to see that we want V to share with someone; it could
share with O, so put O next to V.)
1 2 3 4 5 pieces: N O S T V
N T S* O* V* f h g f L
f f g h L L m h g m
L g h m m
This works, so we can eliminate answer A, as this is a "What can't be true?" question.
You'd do the same thing with your alternate diagram; by now you should see that it really
doesn't make a difference which diagram you select for this game. I'll talk about this at the
end of the section.
Game Twelve 259
Answer B
Put O 1
st
.
1 2 3 4 5 pieces: N O S T V
O __ __ __ __ f h g f L
h __ __ __ __ L m h g m
m __ __ __ __
Now, O has to share with someone next to it. Since O is 1
st
, only 2 is next to it, so whoever
goes 2
nd
has to have h or m. We know that either N or T goes 2
nd
. But neither of them has
h or m; they can't share with O. This is impossible, so this (answer B) is the answer.
Summary
I like this game. It seems like a very strange one that would require all sorts of odd
thinking and creativity, but you can get all the questions right in a decent amount of time
by just being systematic and using the skills you have learned in this book.
There are a couple of lessons I want you to take from this game:
First, it is crucial to understand the rules. Do what it takes to figure what they mean. Don't
jump to conclusions on your first reading of complex rules; really think about what the rules
say before you try to apply them. Often considering a specific example, which you make up
yourself, is the best way to get to the heart of a rule.
Second, don't get hung up making diagrams. We could have done this game with two very
different diagrams, but either would have worked. Now, this doesn't mean that the diagram
never matters, or that some diagrams aren't better than others. A diagram based around
the least changeable things in the game will be easier to work with than a diagram based
around the more changeable. For example, consider Game Four (the one with cars getting
washes in a certain order). If you had built your diagram around the cars, and tried to
assign each a number and a wash, the game would have been harder to do. Just harder,
though, not impossible. Setting up a game should generally be the quickest of the four
steps; think about what the different types of things in the game are, and which is most
definite. If you aren't sure, just pick a way of diagramming and use it. If it turns out to be
too hard, you can always change it later without having wasted much time; in the end, you
will save more time not worrying overmuch about your diagrams.
Finally, I want to talk again about test diagrams. I've said this before, but 90% of teaching
is repeating yourself (I just made that up I don't think it's true, but I like to have a maxim
for every occasion). Often you will go through the rules and just not be sure where to put
people. Don't freeze up. Put them someplace, and then mark that this was your decision.
You can always try them elsewhere later on. But be certain to be systematic about how you
try them out. If you try someone 2
nd
, then 4
th
, then 1
st
, you are just going to confuse
yourself. Move from left to right. Now, students are often nervous about just putting
people in places. They think, "What if this is the wrong place Won't I be wasting time?"
Or they wonder, "Will I have to try every possible arrangement?" Both these questions
have answers. Having to try more than one arrangement does take some time, this is true.
But figuring out, in your head, what the best place to put someone takes more time in the
long run. You may "waste" time on a single answer by putting the person someplace they
can't go, but in the long run you'll be faster because you won't have to think much about
how to test answers. In regard to the second question, the answer is "Not usually." You
usually won't have to test every possible arrangement either you'll find an arrangement
260 Game Twelve
that will work, so you're done, or you'll realize that no further changes will matter, and stop
testing. The point is that it is easier to see these things when you have something concrete
to work with than it is to see them in your head.
What Do I Do Next?
First, as you've been doing, do this game by yourself, with no notes. Done that? Then read
on.
I've taught you everything I know about how to do individual logic games. At this point you
can handle any logic game on any LSAT. Go ahead and start practicing them. You might
have noticed that I've left a couple of games out of the Recommended Games lists at the
end of Game Three and Game Eight. This was because those games were too difficult to
do at those stages, but now you should be able to handle them.
At this stage you should be doing all games under timed conditions. I'm going to explain
how to time yourself in the next section, Timing. Read that section before you do any
games. When you do a game under timed conditions, you are doing your best to treat the
game exactly like you would a game on the LSAT. When you time yourself, don't do
anything you wouldn't do on the real test. If you aren't certain how best to diagram the
game, don't fuss over it, just use some diagram. If you don't know how to do a question,
skip it. Don't try to make deductions if you don't have the time.
Now, after you do a game, when you are in the third step of studying (which is reviewing
what you have done; see the Study Guide), you can think about all this extra stuff what
would have been the best diagram, whether there were any deductions, exactly how you
should have done the questions you skipped. But if you do these things when you are
practicing, you'll be teaching yourself bad habits, and these habits might stick with you on
the real test.
Answer Key
20. D
21. A
22. A
23. B
24. D
Timing for the Section 261
Timing
(For the entire games section)
This section is about how to manage your time on the games section as a whole. If you
want to do each individual game as quickly as possible, you need to employ all the
techniques I've already taught you. Doing the questions in the right order, being
systematic, and knowing exactly what to do in every possible situation is crucial to speed;
skipping steps and cutting corners will speed you up, but only by making you really fast at
getting questions wrong. All that being said, we still need to talk about some techniques
that will help you deal with the entire section.
Do All Four Games
Before we go on and talk about timing strategies, we need to talk about goals. Your goal on
the LSAT is to answer as many questions correctly as you can in the time given. Our goal is
not to get a decent score, but a GOOD score. In fact, we won't settle for any less than they
best score we are able to get.
Now, many people sell themselves short. They decide that they are only going to do three
of the four games. They reason that every time they start a new game, they spend some
time setting the game up, writing down the rules, and making deductions. This is not time
spent answering questions. Doing four games requires doing more of this non-question
work than doing three games. Given that they don't have enough time to do all the
questions, they figure that it's better to do only three games, to maximize the amount of
time spent answering questions.
This is bad reasoning. First, as you have been seeing, it doesn't take that much time to get
through the setup and rules of a game; doing this for four games or three doesn't make a
huge difference. Second, the questions that take the most time for each game are the
hardest questions. These are also the questions that we are most likely to get wrong. You
can often get two questions right in the time it takes to answer one hard question and you
have a better chance of missing that hard question. One of the main reasons people run
out of time on the Games section is that they spend too much time working on hard
questions. There aren't that many of these questions, but they sure do eat up time. Doing
three games rather than four usually means that students spend more time working on
these three or four difficult questions; however, it also means that they miss out on five or
six other questions. That is a bad trade. These students who do three games often skip the
easiest game (just by accident), which means that they missed out on several easy points.
Finally, if you are planning to get in the mid- 160s and up, you can't afford to write off a
whole game's worth of questions.
The moral: do all four games.
Take Only Eight Minutes Per Game
If you want to do all four games, and we do, then you need to make sure that you have
enough time to do all four games. There is only one way to do that: make sure you don't
spend too much time on any one game. You have 35 minutes for the entire section. Split
among four games, that gives you 8 minutes and 45 seconds per game. We are going to
round that down to 8 minutes per game (I'll explain why in a second).
262 Timing for the Section
So, only spend 8 minutes per game. This rule is subject to a small exception which I'll get
to in a second. This is an important rule to stick to every extra minute you spend on one
game is going to come out of your time on the last game. Lose too many minutes out of
that last game and you aren't going to get very many points on it. There is only one way to
avoid this: when your time is up on a game, move on. I mean it. This isn't that bad if
you do the questions in the right order, you should have answered all the questions, or all
but the hardest, in 8 minutes. If you don't have enough time to do all of the questions, the
ones you skip will be the ones that are difficult and time-consuming. But, more likely, using
previous diagrams will allow you to get through all the questions in time.
Now, 8 minutes for each of four games only uses up 32 of our minutes. That's good it
gives us a little breathing room, and it helps us with the next strategy
Do the Games in the Right Order
Before you start doing any game, quickly glance through and count the number of questions
in each game. Do the games in order of the number of questions the game with the
fewest questions first, and the game with the most questions last. This strategy is
counterintuitive. Most test-takers do the game with the most questions first. They reason
that they may not have time to tackle every game, so they should make sure that, if they
miss out on a game, it isn't the one with the most questions. This is a strategy for people
intent on mediocrity. We aren't going to run out of time. Why not? Because we are going
to watch the clock, and move on when our 8 minutes are up.
So why do we save the game with the most questions for last? Well, what aspect of a game
takes us the most time? Think about it That's right, the questions. Which means that
what should be true about a game with more questions? It should take us longer. And a
game with fewer questions? It should be faster. This isn't always true a game with fewer
questions could be a hard one to figure out but it is true the great majority of the time.
At the beginning of the section, the first thing you should do is figure out the order in which
you are going to do the games. After you've done that, go from game to game and write at
the tops of the game the time at which you must finish with it. So, at the top of the first
game you will do, write whatever time it will be in 8 minutes; at the top of the second
game, write whatever time it will be in 16 minutes; at the top of the third game, write the
time it will be in 24 minutes; at the top of the fourth game, write the time it will be in 35
minutes (not 32 you get 35 minutes per section, so you have to finish with the last game
by the time 35 minutes are up). So, if you start the section at 10:33, you write 10:41,
10:49, 10:57, and 11:08 at the top of the games, from shortest to longest game
respectively. This makes it very easy to pace yourself on the games. When the time on
your watch is the same as the time on the top of the page, you must move on.
Here's a handy trick I learned when I was studying for the Bar exam: bring an old-
fashioned watch to the test. That'll look cool, and that's got to be worth something. But
wait, there's more to it. At the beginning of each section, you set the watch to noon (that's
why you use an old fashioned one they are really easy to set). You never have to do any
hard math during the section to figure out how much time you have left. When the hands
say "12:15," you've used 15 minutes.
The game with the fewest questions should take us the least amount of time. In fact, it
might take us less than 8 minutes. Which means that if we do it first, we will end up having
extra time for the other games. We don't need to think about this during the test, however.
Timing for the Section 263
That is to say, you don't ever need to say to yourself, "Oh, I only took 6 minutes on that
game, so I have 2 extra minutes for the next games." The times you have written on the
tops of each page will keep track of this for you. Let's say you were supposed to finish the
first game by 12:08 and the second by 12:16. You finish the first game at 12:06. This
means that you have 10 minutes before it is 12:16 and you have to move on; all you have
to do is make sure you move on by the time at the top of the page.
When we come to the last game, if we've been careful about watching the clock, we'll have
almost 3 extra minutes to play with (remember, 8 minutes per game, times four games,
equals 32 minutes, but the section is actually 35 minutes long). This is good, because this
game will probably take us longer than 8 minutes, since it has more questions. And, if we
end up with time left over, we can go back and answer any questions we skipped in
previous games.
Let me give a personal example of how this might work. I went and did all the games from
LSATs 19 to 41 to practice these strategies. I found that I rarely spent exactly 8 minutes
on a game. Generally, I would work through the shortest game (which typically has five
questions) in 5 minutes. This would give me 3 extra minutes for the other games. I
needed this; the other games sometimes took 9 or 10 minutes to work through, with the
last game sometimes taking up to 11 minutes. In every case where the test had a really
lengthy game, they always had a fast game to even it up. Other times, when the shortest
game wasn't so fast, the other games ended up not being so slow, either. The LSAT isn't
written to be impossible; if you stick to the strategies I discuss in here, you'll find that the
timing takes care of itself.
Now, what if I hadn't worked through the first game so fast? Well, I wouldn't have had
enough time to answer all the questions on the other games, but, since I did the questions
in the right order, I would have had to skip at most one question per game.
Can I Go Over These Time Limits?
People often feel that, if they are doing a really easy game, they should do all the questions,
even if it takes them more than 8 minutes. This is bad thinking. If you hit 8 minutes on a
game and haven't finished it, first, it isn't as easy as you think it is; and second, you have
only hard questions left, and doing one of them is likely to take as long as two questions on
another game. The only time you can go over 8 minutes on a game (except for the last
game) is if you hit 8 minutes and you are just finishing a question you have only one or
two more answers to look at.
Guessing
Make sure you fill in an answer for every question you skip, whether you skip it because you
are about to run out of time or because you don't know how to do it. I can't teach you how
to guess well if you don't know how to get the right answer, there isn't any way to
eliminate the wrong ones (since we usually get the right answer by eliminating the wrong
ones). The point of guessing isn't to get that question right; it's to give yourself a chance at
getting that question right, and to do it fast so that you have time to get other questions
right. Before you start the test, pick the answer you will put whenever you guess, and then
always guess that. I like B, which stands for Brian. Of course, don't pick that answer if
you've somehow eliminated it.
When you have a minute or less left in the section (on any section on the LSAT), go over
your answer sheet and make sure you've filled in an answer for every question. You aren't
264 Timing for the Section
allowed to write after the section ends, and if you are caught, many proctors will turn you in
for cheating.
Take Breaks
You are going to get tired during the games section. The only way to deal with this fatigue
is to take breaks. Take a break at least after finishing each game. Take enough time to
relax a little, catch your breath, move your torso and get blood flowing to your brain. This
is important time, so don't skimp on it.
You can take breaks more often than this. If you feel fuzzy-headed or confused or stressed
out, take a break. You can't afford not to.
Some Final Words
I can't emphasize enough that the key to speed is not in rushing. After a certain point, you
won't be able to read faster, or write more quickly, or go through rules more rapidly,
without losing accuracy. If you rush through these steps, you will miss questions. The only
way to speed up is to eliminate waste not to be fast and sloppy, but careful and efficient.
Wasted time is time you spend doing things that I didn't talk about in this book. Wondering
what you are supposed to do next is wasting time. Looking for extra deductions is wasted
time. Trying to figure out a question you don't understand is wasted time.
Memorize the approaches in this book. Visualize and practice them to the point where you
can explain them to anyone, in order, without notes; to the point where you use them on
practice tests without having to try to remember them. This will eliminate the waste of
wondering what to do. Then stick to these approaches and don't do anything else. Make
sure you read everything carefully and are methodical in writing out test diagrams and
working through answers. Watch the clock and don't break your timing rules. Take breaks
whenever you need them, and whenever you are supposed to. Do all of this and you will be
fast.
What Do I Do Next?
Now you are ready to study games under timed conditions. Every time you do a game from
here on out, time yourself. When your time is up, stop answering questions, and move on
to reviewing what you have done. Before you go on to do games you have never seen
before, do sets of games that you have already done. This will allow you to practice timing
without the pressure of having to figure out wholly new games. Once you feel somewhat
comfortable with how you are supposed to manage your time, start doing new games. You
don't need to completely master time management before you go on to new games that's
going to take a while but you want to be able to remember what you are supposed to do
and how it works without having to think before you try to apply these skills to new
situations.
Try to do games from the same section together, because the games in each section are
written to be done in 35 minutes together. That is, there is a balance of difficulty between
the games in the same section, so each games section averages out to be no more difficult
than any other games section, even if a single game is very easy or very hard.
You don't always need to do four games at a time, though. One thing you can do is to pick
an LSAT games section and do one game at a time. Do the games in the order you would
on the real test. Time yourself as you would on the LSAT, but stop when you either finish a
Timing for the Section 265
single game or run out of time on a game. After you finish each game, you can review it,
and then go on to the next game. You have to be very rigid with yourself when you do this;
often you will want to fudge and let yourself have extra time on a game. You can't do this
at first; you have no feeling for time, and won't be a good judge of when you should give
yourself extra time. Teach yourself what you can do in 8 minutes by stopping when those 8
minutes are up, even when you are in the middle of a question.
Soon you will reach a point where you want to do games in groups of four. Do this when
you no longer feel that you need to review each game immediately after you did it; that is,
when you start to have confidence that you know what you are doing. At this stage you can
give yourself a little extra time here and there, as I discussed above.
Read the next section at least a week or ten days before the LSAT.
266 Before the Test
Before the Test
Am I Ready for the LSAT?
There is an LSAT score that you need to get. Most likely this is the score you need to get
into your law school of choice, but maybe its a goal you've set for yourself out of pure
ambition. Getting this score is your sole reason for taking the test. If you aren't yet
capable of getting this score, you aren't ready to take the LSAT. How do you know if you
are capable of getting this score? It's easy. Sit down and take a sample LSAT; time
yourself exactly like the real thing. If you score within a couple of points of where you want
to be, you are ready. If you are way off, you aren't ready. A difference of a few points is
OK, because there are a couple of things you will do or learn between now and the real test
that you haven't done or learned yet; one of them is described below (and the rest are in
the next chapter).
Now, if you aren't ready for the test, you may have enough time to get ready. How much
time it will take depends on you. Look at the kinds of improvements you have made so far,
and how long it took you. See how much more you need to improve, and then ask yourself
if you are honestly capable of improving that much in the time you have left. If you have
less than a week left, and the improvement is more than a handful of points, the answer is
no.
If you aren't ready for the test, don't take it. Getting a score that won't get you into the law
school of your choice is a waste of your time and money. In addition, it can hurt your
chances of getting into that school at a later date. Even if you take the test again and do
better, many schools average multiple LSAT scores (call individual schools to find out if they
do). Even if they don't, the fact that you took the test more than once is reported to
schools; if you cancelled your score, this fact is reported (but not the score you would have
got). This won't help your chances of getting in; it won't necessarily hurt your chances,
either, but why take the risk if you can avoid it?
By the way, read the next section if you've made it all the way through this book, even if
you haven't decided to take the LSAT yet.
Make a Plan
At some point before the LSAT, you will be as ready as you are going to be for the logic
games section. You have practiced, turned skills into habits, and are confident that you can
get the score you want on the logic games section. This may come a week before the test;
it may come three days before the test. When you reach that point, I want you to do the
following:
If you are not totally confident of your LSAT logic games skills, but have made it through
this whole book and understood most everything, doing the following may help you become
confident.
Sit down, with no notes in front of you, and write down a step-by-step plan for the logic
games section. This plan should include every single thing that you are going to do (and
how you are going to do it) in the course of the section, in order. This plan is going to be
slightly different for different people. Everyone is going to use the fundamental techniques
I talk about in The Fundamentals. But some of you may not have made it to the
intermediate or advanced techniques, or may have made it through these sections and
Before the Test 267
decided not to use some or all of the techniques. That's OK; if you don't plan on using
some of these techniques, leave them out of your plan.
Some things will definitely happen in a certain order during the section. For example, you
are definitely going to start the section by deciding on the order in which you will do the
games. You are definitely going to write down the times at which you will finish each game
at the top of each page (see Timing). You will definitely take short breaks at the end of
each game. You are definitely going to think about each setup in a certain way, look at the
rules in a certain way, and do the questions in a certain order. Put all of the things that
definitely happen at certain intervals in your plan.
Other things will definitely happen at some point during the section, but you don't know
when. For example, you will definitely encounter a "What must be true?" question, but you
don't yet know when. You will definitely encounter an "If then" rule, and want to write
down its contrapositive, but you might not encounter one on any given game. Your plan
should also include how to recognize these types of situations, and what you will do when
they occur. Your plan must include how to go through the three steps you do on each game
reading the setup and making a diagram, writing down the rules, and answering the
questions. Your plan must talk about how to deal with every question type how to
evaluate the answers, when you can use previous diagrams (and how to do so), and what
order to do them in.
Some things may happen during the test. You may panic. You may realize that you aren't
sure how to diagram a game. The guy next to you may be very loud. Your plan should also
include how you will deal with these things (I'll talk about them in the During the Test
section).
To summarize your plan should include everything that can or will happen during the
games section, how to know that it is happening, and what to do when it does. In the end,
your plan will be an organized list of everything that you know about LSAT logic games, and
how to deal with any situation that might possibly occur during the test.
What is the point of this? There are several points. First, you may find that there are
things you can't remember. You'd better learn these things you don't get notes on the
LSAT. Memorize these and then sit down and write your whole plan all over again. Second,
writing this plan allows you to organize and review what you know about the test, which is
helpful for being prepared for the test. It's one thing to have all these techniques
somewhere in your mind; it's much more useful to have them all fitted together and ready
to go. Third, once you can write down your entire plan with no notes, you know that you
know everything. That's good; that's very good. At this point you are ready for the LSAT.
All you have to do is show up and let yourself do well.
268 During the Test
During the Test
In this chapter I will assume that you are ready to take the test (see Before the Test for
instructions on how to determine if you are ready).
Taking the LSAT can be scary and unpleasant. There are some negative things that are just
built into the nature of the test, which none of us can avoid. I'm going to tell you how to
deal with them here. There are also some bad things that happen to some of us during the
test, and I'm going to tell you how to deal with them.
Psychological Warfare
The LSAT messes with your head. No one can avoid this. You are going to be nervous; you
may even be scared. Now, there is nothing wrong with being nervous about the test it is
an inevitable and natural reaction. Everyone gets nervous, even the best test-takers. If
you think that you shouldn't be nervous, you are just going to scare yourself out even
more. Accept some nervousness; what you shouldn't accept is a lack of confidence. This
can negatively affect your performance. If you doubt yourself you may end up not applying
the techniques you have worked so hard to master; you may find yourself second-guessing
your diagrams, double-checking your work, looking twice at answers you know are wrong,
and so on. This is bad.
We are going to use a two pronged attack on our LSAT fear. First, we are going to try to
build our confidence. Second, we are going to ignore any remaining lack of confidence.
These skills are as important as any other aspect of studying, because at this point
confidence is all that stands between you and a great LSAT score. Take this as seriously as
you did learning everything else in this books this stuff takes commitment, effort, and
practice.
Building Confidence
The key to confidence is honesty. There are two reasons people lack confidence about a
task 1) they don't know what they are doing, or 2) they are lying to themselves. By this
stage in the book, and especially by the point when you have decided to take the test (as I
discuss in the previous chapter), you know what you are doing on the logic games section.
Have you read the previous chapter, Before the Test? No? Shame on you. Go read it. In
it I talk about putting together your LSAT logic games plan. This is a huge part of having
confidence on the test. If you know exactly what to do at every given moment of the test,
you don't need to worry about what to do. And if you know you know, you will feel good
about yourself. In addition, you have taken at least one sample LSAT and done as well as
you wanted. There are no accidental good scores on the LSAT. If you did well once, it is
because you are capable of doing that well again. If, at this point, you still lack confidence,
you are lying to yourself. Look back at that plan. You made that. You know all of that stuff
there is nothing the LSAT can throw at you that you are not ready for. Look at that
sample test score. You got that score. You can do it again. Doesn't that feel great? Yes, it
does! You are ready for this test. Stop lying to yourself and start feeling the confidence
you deserve.
There are some other ways to build confidence in addition to honesty. Music can be very
helpful I am big on singing. I find that dressing up helps; when I took the bar exam I
wore a three-piece suit every day of the test. Visualization is also useful. Think about
During the Test 269
times when you have been successful in the past; remember why you were successful (hint:
it's because you are wonderful) and how it felt. I also like talking to myself a few jokes, a
few compliments, and I start to feel good. Whenever you start to worry again, just close
your eyes and remember how great you are and how much you studied. This sort of thing
may be embarrassing, but it works. In fact, the fact that it is embarrassing makes it more
effective laughing at yourself distracts you from fear.
Acting Confident
If you can't be confident, at least act confident. It doesn't matter if you feel good during
the test, as long as you perform well.
Ask yourself, "What would a confident person do?", and do that. The most important thing
is this: a confident person believes that they know what they are doing. When confronted
with a difficult situation, they trust their skills and abilities, and do what they normally do,
confident that they will do well. This is what I want you to do on the test. One of the worst
things that people do during the LSAT is to change their approach. They start doing things
differently than they practiced at home. This is bad for two reasons: First, the way you
practiced at home is the best way you can take the test that's why you practiced it.
Second, you aren't thinking clearly on the LSAT; whatever new strategy you come up with is
not going to be a good one. Pretend that you have confidence in the skills you have
learned, and use them.
A confident person also believes that they knew what they were doing. They believe that
the work they have already done is correct. They don't double-check, they don't second-
guess, and they don't worry about the previous questions. This is crucial. Once you've
done a question, or made a deduction, or written down a rule, assume that it is right and
act accordingly either forgetting about it (if it's a question), or using it without questioning
it (if it's a deduction or diagram or rule). This takes practice, but it can be learned. Start
working on it today.
Of course, if you see something that explicitly contradicts work you've already done, that is
another story. I talk about what to do in these circumstances below. But until that
moment, don't question your previous work.
Having taken quite a few standardized tests myself, I have experienced most of what I talk
about here (and I've seen students of mine go through all of it, many, many times). A few
years ago I was taking the GRE. The GRE is computer adaptive. The questions are
supposed to get harder when you do well and easier if you do badly. I was doing the verbal
section, which is mostly vocabulary, and has a lot of antonym questions, where you have to
pick the answer with the opposite meaning. The vocabulary words seemed to be getting
easier and easier as I went along, which really worried me. Eventually I did an antonym
question where I knew I had picked the right answer, but the next question was
phenomenally easy. Suddenly I realized that I must be doing something wrong; the test
was getting demonstrably easier, which meant I must be getting questions wrong. I knew I
had picked the right antonym; if that wasn't the right answer, then maybe the questions
were not antonym questions. I had assumed they were, and had not even read the
instructions. Maybe the test had changed recently, and they were really synonym
questions!
If I wanted to salvage my score, I had to change my approach to the test and start doing
the questions as if they were synonym questions. But, before I did this, I stopped myself. I
said, "Brian, if you were one of your students, and they were in this situation, what would
you tell them? You'd say, 'Never change your strategy during the test. Always trust
yourself.'" I couldn't violate my own advice, even though I wanted to. I knew that it is
really hard to make good decisions during the test, and that you shouldn't trust your own
instincts. Instead, you follow your rules. You made these rules before the test, when you
were thinking clearly, so you have to trust that they are right. I stuck to my guns and kept
doing the questions as if they were antonyms.
270 During the Test
said, "Brian, if you were one of your students, and they were in this situation, what would
you tell them? You'd say, 'Never change your strategy during the test. Always trust
yourself.'" I couldn't violate my own advice, even though I wanted to. I knew that it is
really hard to make good decisions during the test, and that you shouldn't trust your own
instincts. Instead, you follow your rules. You made these rules before the test, when you
were thinking clearly, so you have to trust that they are right. I stuck to my guns and kept
doing the questions as if they were antonyms.
What happened? I got an 800 on that section, the highest score possible. They were
antonym questions, and I had been getting them right; I was just unable to judge how
difficult they were while I was in the middle of the test.
What's the moral of this story? Never change your approach to the test during the test, no
matter how much you want to. Follow the strategy you developed before the test (which
this book has taught you) and trust that you're doing things the right way. That's the only
way to get a good score, and it works.
Take Breaks
If you start to panic or lose confidence on the test, take a short break. Get your mind off
what you are doing, breathe deeply, don't think about anything. When you calm down,
remind yourself of that plan you put together. You know everything you need to know. It's
all there in your head. Relax, and let it happen. Worst case scenario, you have to skip the
question you are doing. No big deal.
A Trick or Two
Here are my two favorite confidence-building/faking tricks. They may not work for you, but
hopefully they will remind you of things that you do to build your confidence.
The first is that I love to bring a lot of pencils to the test. When I took the LSAT for law
school, I had maybe 40 pencils. Sure that gives me the advantage that I don't have to
worry about them getting dull or breaking, but the real point is that there is something
about seeing that huge pile of pencils that just makes me feel good. It's probably
ridiculous, but I feel like everyone is looking at me thinking, "Man, that guy brought 40
pencils. I only brought two. He must know 20 times what I know."
The second one is my absolute favorite. To understand it you have to have taken at least
one test before in a large group. Think back; when the teacher or proctor tells you to start,
what's the first thing you hear? You hear all the people in the room, in unison, opening
their books. So here's what I do: I wait until the room gets quiet again, and everyone is
busy starting the test (or the section), and then I open my book. The message I send is
that I'm so smart I don't need to start when they do. Now, no one probably notices but me,
but that's the important part. By acting like I feel confident, by putting forth that confident
image to others, even though they don't notice it, I put forth the confident image to myself.
During the Test 271
Imagine the Worst
Here are some of the scenarios students are concerned about, and how you should deal
with them.
What if I panic?
I talked about this above. Take a break. Then, after your break, remind yourself of your
plan, and the fact that you know everything that you need to know for this test.
What if I don't know how to deal with a certain question?
Skip it. Circle it and if you have time at the end do it then.
What if I don't know how to deal with a certain game?
Remember, you don't have to know how to make a great diagram for a game to do well at it
all you have to do is draw some diagram, understand the rules, and start answering
questions.
If the setup just doesn't make much sense, look at the rules. They will tell you what is
going on in the game. If they don't help, look at the questions. The questions will always
ask about the key issues in the game, so seeing what they are asking about will tell you
what the game is about. For example, if the questions ask who is before whom, you know
that ordering is an aspect of the game. If the questions ask who is in the same group as
whom, you know that there are groups, and they are important.
What if the game just flat-out makes no sense what if it might as well be written in a
foreign language? In all the times I've been asked this question, no student has ever given
me an example that they really didn't understand. Every example was a case where the
student didn't really read the whole setup, they just panicked halfway through. Calm down,
take a short break, and read what they are telling you. If it still doesn't make any sense,
save it for last. You'll have to change the times you are supposed to finish each game;
make sure you give yourself exactly 8 minutes on all the other games, and spend whatever
time is left on the game you are saving for last (write the adjusted times on the tops of
each game, as we discussed in Timing). Once you've done the other games, you'll feel a
lot better and most likely whatever psychological block is holding you back will evaporate.
If it still doesn't, at least try to answer some questions. I've never met a student who
couldn't answer at least one or two questions on any given game.
The most insidious thing about this concern is that you have it at all. You can figure out
every game there is. I promise you. Even if you think you can't, you really can. Just apply
what you know. This is not something you should be concerned with.
What if I realize that I missed a rule (or wrote a rule down wrong)?
Step one: relax. Step two: write the rule down (or write it down correctly). Step three:
circle (in your book, not on the bubble sheet) all the questions you have already done. Step
four: cross out any diagrams you have made so far, so that you don't use them on future
questions (since they might be wrong). Step five: ignore any deductions you have made
up until this point. Step six: continue with the game as normal, except don't use previous
diagrams from the questions you have circled. Do not go back and redo questions you have
already done. Do not. Now, if you finish all the other questions and you have time left
over, go back and do the ones you circled. If not, go on to the next game.
272 During the Test
Why not redo the questions? Well, the questions you have already done might be right.
After all, the answers agree with most of the rules. It could be that the rule you missed, or
the one you wrote down wrong, didn't apply to the answers you did, or that it wouldn't have
made a difference if it did apply. This is often the case. So you have a decent chance of
getting the questions you have done right without any more work. But you have next to no
chance of getting the questions you haven't done right if you don't do them.
Every time I have missed a rule or written it down wrong, and I've done it a few times, I
have gotten right most of the questions I'd done before realizing my mistake. Think about
it if there are five rules, and you missed one, you've got 80% of them right. Those four
rules that you have will push your answers towards correctness, even without the rule you
missed. If you go back and redo those questions, you won't have time to do some of the
questions at the end of the game. If you had, let's say, a 50/50 chance of getting right the
ones that you have already done, you only have a one in five (20%) chance of getting
questions right that you skip because you ran out of time. This means that, by re-doing
questions, you gain 50% more chance of being right, in exchange for losing 75% chance of
being right on other questions. That is a bad trade.
What if I wrote the wrong diagram?
If your diagram is false it doesn't actually show what happens in the game treat it as if
you wrote a rule down wrong (see above). If you just realize that you could have used a
different and better diagram, change to it now and use it on the remaining questions, but
don't redo any questions you've already done. You did them, they're right.
That's the most important point assume that what you've already done is right. If you
worry about the previous questions, you aren't going to focus on what you are doing, and
you are going to make silly mistakes. If you are honest with yourself, you will realize that
using a "bad" diagram still allows you to get questions right, just not as quickly. We saw
that in this book when I showed how to do certain games with alternate diagrams. So any
questions you've already done are right; forget about them.
What if I have to go to the bathroom?
How bad? Can you hold it to the end of the section without being significantly distracted?
Logic games is probably the most time-sensitive of all the sections losing time from this
section makes a bigger difference than the other sections, since you get faster as you get
further into a game (given previous diagrams). So if you can wait and then go to the
bathroom during another section, you should.
But if you find that you are being really distracted by having to go, go. It's better to lose a
little bit of time than to be distracted for the rest of the section. Losing time may cost you a
few questions, but being distracted may cost you the rest of the section.
What if somebody is making noise and it is bothering me?
First, take a short break. Chances are that they aren't making that much noise, and if it's
distracting you it is because you are tired, and thus easily distractible. But if, after your
break, you are still distracted, raise your hand and ask the proctor to ask them to be quiet.
Do it as soon as you can; otherwise the distraction will just build as you get more and more
annoyed.
This completes the book. Assuming that you have learned everything in here, you now
know everything I know about logic games. You too are a big fat genius.
Glossary 273
Glossary
Underlined words in definitions refer to other words in the Glossary. Boldfaced words give
chapter titles.
Analytical Reasoning The official name for logic games. See Introduction.
and Same as both "X and Y have some trait" means that each of these two things, X and
Y, have the trait. If one of them didn't have the trait, this would be contradicted. See
"If then" Rules and Negations.
at least "At least X" means X or more. It would be contradicted if there were less than X.
See "If then" Rules and Negations.
both Same as and. "Both X and Y have some trait" means that each of these two things, X
and Y, have the trait. If one of them didn't have the trait, this would be contradicted.
See "If then" Rules and Negations.
contrapositive Every "If then" rule has a contrapositive. This is formed by switching the
"if" and "then" parts, and making both negative. "If X, then Y" becomes "If not Y, then
not X." See "If then" Rules and Negations.
deduction Something you figure out must be true based on the rules of the game.
Deductions can be made before you do the questions (see Advanced Techniques) or
as you do questions (see Intermediate Techniques).
definite (thing/person) Some of the types of things in the game change less than others.
The definite things are those that change less. Your diagram consists of the definite
things written down in a row. See also variable. See The Fundamentals.
diagram A way of visually representing the information in a game. Every game has you
matching one or more types of things; your diagram should have the more definite of
these things written in a line, with blanks underneath for the variable things that will go
with them. See The Fundamentals.
each Same as putting "and" between all items (like both). "Each of X and Y are A" means
"Both X and Y are A;" "Each of A, B, and C are happy" means "A and B and C are
happy."
either "Either of X and Y are A" means "X or Y are A." See or.
equivalent Two people or things in a game are equivalent if they are subject to all the same
rules, or to no rules at all. Equivalent variable things can be switched with each other
whenever you want. You can switch all the variable things associated with two definite
things (but you must switch all of them). See Advanced Techniques.
exactly "Exactly X" means X, no more, no less. It would be contradicted if there were more
or less than X. "Exactly the same" means no differences of any type. See "If
then" Rules and Negations.
EXCEPT question A question containing the word EXCEPT. These questions describe the
wrong answers, whereas questions typically tell you something about the right answer.
For example, "All of the following can be true EXCEPT" tells you that the wrong answers
can be true, so the right answer can't be true. See The Fundamentals.
"How many diagrams are possible?" question The hardest type of question. There are
three versions. The first asks how many diagrams are possible, generally under a
certain condition (given in an "if;" see If-question). The correct answer is the number of
diagrams differing in at least one way from each other that can be generated (given the
condition). Test the answers by making a test diagram and putting every variable thing
in every place they can possibly be. A second version asks for how many of the things
in the game can some quality be determined. To find this out, see how many of the
things can only be put in the diagram one way. If an entity can be put in more than one
position, or be matched with more than one other thing, do not count it. The third
274 Glossary
version of this question asks what information would allow you to determine the
situation of every thing in the game. The answer will be information such that only one
diagram can be drawn incorporating it. To test the answers, put in the information
given and see if you can draw multiple different diagrams that follow the rules. If you
can, eliminate the answer. If not, it is the correct answer. See The Fundamentals.
If-question A question that puts a further limit on the game. This limitation is for this
question only. If-questions generally begin with the word "if," but "suppose" also means
"if." Whenever we come across an If-question, we first copy our original diagram. Then
we add the constraint given in the "if." Then we go through the rules and add anything
to our diagram that has to be true based on what we have been told. We use this
diagram when testing the answers. If-questions are generally easier than other
questions, so we do them first. See The Fundamentals. Using previous diagrams on
If-questions is different than on other questions see Advanced Techniques.
"If then" rule A rule that takes the form "If X, then Y." It tells you that every time X is
the case, Y has to be the case. However, if Y is the case, X may or may not be the case.
"If then" rules always have a contrapositive, which you should write down. See "If
then" Rules and Negations.
logic games One of the sections of the LSAT, and the subject of this book. Officially called
Analytical Reasoning. See Introduction.
long list The list of everything you want to learn in logic games. As you come across new
ideas, information, or techniques in this book, add them to your long list. See How to
Study for LSAT Logic Games.
mark (with a dot or star) If you are not certain where some thing has to go in a test
diagram, when you put it someplace, you put a little mark (such as a dot or a star) next
to it. This tells you that it could have gone someplace else. If the diagram ends up not
working, you can move your marked things someplace else. It is best to put things as
far to the left as you can initially, and when you move them, move them to the right.
See The Fundamentals.
neither "Neither X nor Y have some trait" means that X doesn't have the trait and Y doesn't
have the trait. It would be contradicted if one (or both) of them had the trait. See "If
then" Rules and Negations.
or "X or Y has some trait" means that one (or both) of the two has the trait. It would be
contradicted if neither had the trait. See "If then" Rules and Negations.
original diagram The diagram you create during step one (the setup), before you look at
any questions. Copy this diagram down every time you need to test something out.
See The Fundamentals.
previous diagram A diagram that you made while testing an answer to a question before
the one you are currently doing. Since you cross off any diagram you make that breaks
a rule, all non-crossed off previous diagrams give a possible way the game could end up.
These diagrams can be used to eliminate answers on different question types. See The
Fundamentals.
question There are seven types of questions (in order of difficulty): What can be true?,
What can't be true?, What must be true?, What can be false?, Which is a complete and
accurate list?, Which is the maximum/minimum number of?, and How many diagrams
are possible? See The Fundamentals.
rule Rules tell you what can and can't happen in a game. They are listed under the setup.
See The Fundamentals.
setup Every game starts with a paragraph describing the situation of the game, and the
things involved in the game. This is the setup. The setup tells you how to diagram the
game. See The Fundamentals.
short list The list of skills you are currently most focused on learning. Each item on the list
should be an instruction to yourself that you can follow when practicing. You should
constantly refer to the list while studying to ensure that you practice the techniques on
Glossary 275
it. Your short list will change periodically as you improve or spot new problems. See
How to Study for LSAT Logic Games.
test (an answer) To test an answer we draw a diagram and see if it breaks the rules. We
recopy our original diagram unless it is an If-question. To test an answer, you must fill
every empty spot in the diagram until you break a rule or all spots are filled. We always
cross off any test diagrams that break the rules. See The Fundamentals.
variable (thing/person) Some of the types of things in the game change less than others.
The variable things are those that change more. Your diagram should contain a blank
under each definite thing for each variable things that will go with it; the variable things
should also be written off to the side for quick reference. See The Fundamentals.
What can be false? question The fourth-easiest (third-hardest) type of question. The
correct answer does not have to be true; it may either be possibly true, or it may be
definitely false. The wrong answers must be true. Test the answers as you would on a
What must be true? question; either look at previous diagrams for situations other than
what the answer describes, or make a test diagram that differs from what the answer
describes. If you have a previous diagram, or can make a test diagram, which differs
from what the answer describes, the answer is correct. If not, the answer is wrong.
See The Fundamentals.
What can be true? question The easiest type of question. The correct answer follows all the
rules. The wrong answers break the rules. Test the answers by making a test diagram
containing the situation the answer describes. If the diagram breaks the rules, it is
wrong; if it follows the rules, it is right. We don't use previous diagrams on these
questions. See The Fundamentals.
What can't be true? question The second-easiest type of question. The correct answer
breaks a rule, and the wrong answers follow all the rules. Use previous diagrams to
eliminate answers before you test the answers. If a previous diagram contains the
situation described by the answer, the answer is wrong. To test the answers, make a
diagram containing the situation described by the answer. If you can fill all the spaces
in the diagram without breaking a rule, the answer is wrong. See The Fundamentals.
What must be true? question The third-easiest type of question. The correct answer is a
situation that occurs in any possible diagram. Use previous diagrams to eliminate
answers before testing answers. If a previous diagram contains a situation other than
that described in the answer, the answer is wrong. To test an answer, make a diagram
that does not contain the situation described in the answer. If you can fill all the spaces
in the diagram without breaking a rule, the answer is wrong. If you can't, the answer is
right. See The Fundamentals.
Which is a complete and accurate list? question The fifth-easiest (third-hardest) type of
question. The correct answer contains all the things that fit the description in the
question; any answer which leaves an item out, or contains an item that doesn't belong,
is wrong. Use previous diagrams to eliminate answers before testing them. Previous
diagrams can tell you what things fit the description in the question; eliminate any
answers that don't contain these. To test answers, test each thing in the list to see if it
can fit the given description; if it can't, eliminate the answer. If one answer contains
more items than another, test only the items that are not in one list. See The
Fundamentals.
Which is the maximum/minimum number of? question The second most difficult question.
The correct answer gives the largest or smallest number of things fitting a certain
description. Check previous diagrams to find the largest or smallest number; eliminate
any answer smaller/larger than this. Test answers by starting with the largest/smallest
and creating a diagram with that number of items fitting the description in it. If this
won't work, move to the next largest/smallest answer. See The Fundamentals; see
also Game Three for an example.
Visit WWW.BIGFATGENIUS.COM for
! Updates on this book
! Additional study materials
! Information on our test preparation services
Questions or comments about this book?
Have an LSAT story you'd like to share?
We welcome all feedback.
Email us:
Logicgames, the at sign, bigfatgenius dot com

You might also like